You are on page 1of 217

15 Mock test

cOMBINED
pHYSICS, Chemistry AND Mathematics

Team Prabhat

Prabhat
Prakashan
 Information contained in the work has been obtained by Prabhat Paperbacks, from sources believed to be reliable. However,
neither Prabhat Paperbacks nor its authors guarantee the accuracy or completeness of any information published herein,
and neither Prabhat Paperbacks nor its authors shall be responsible for any errors, omissions, or damages arising out
of use of this information. This work is published with the understanding that Prabhat Paperbacks and its authors are
supplying information but are not attempting to render engineering or other professional services.
 No part of this publication may be reproduced, stored in a retrieval system, or transmitted in any form or by any means,
electronic, mechanical, photocopying, recording, scanning, or otherwise, without prior written permission of the publisher.
 All disputes are subject to Delhi jurisdiction only.

Publisher
Prabhat Prakashan
4/19 Asaf Ali Road, New Delhi–110 002
Ph. 23289555 • 23289666 • 23289777 • Helpline/ 7827007777
e-mail: prabhatbooks@gmail.com • Website: www.prabhatexam.com

© Reserved

Edition
2021

Price
Two Hundred Eighty Five Rupees

ISBN ISBN 978-93-5322-575-9

Printed at
Nakshatra Art, Delhi

IIT-JEE JEE MAIN AND aDVANCED


15 Mock test
cOMBINED
pHYSICS, Chemistry AND Mathematics
by Team Prabhat

Published by Prabhat Prakashan


4/19 Asaf Ali Road, New Delhi–110 002

ISBN ISBN 978-93-5322-575-9

` 285.00
CONTENTS
l Mock Test – 1 Physics....................................................................................................................................1-18
l Mock Test – 2 Physics..................................................................................................................................19-32
l Mock Test – 3 Physics..................................................................................................................................33-44
l Mock Test – 4 Physics..................................................................................................................................45-58
l Mock Test – 5 Physics..................................................................................................................................59-72
l Mock Test – 1 Chemistry.............................................................................................................................73-91
l Mock Test – 2 Chemistry...........................................................................................................................92-102
l Mock Test – 3 Chemistry.........................................................................................................................103-113
l Mock Test – 4 Chemistry.........................................................................................................................114-124
l Mock Test – 5 Chemistry.........................................................................................................................125-134
l Mock Test – 1 Mathematics.....................................................................................................................135-154
l Mock Test – 2 Mathematics.....................................................................................................................155-168
l Mock Test – 3 Mathematics.....................................................................................................................169-182
l Mock Test – 4 Mathematics.....................................................................................................................183-196
l Mock Test – 5 Mathematics.....................................................................................................................197-212
Mock Test-1
Test Booklet Code
1

A Mock Test “JEE-Main”


Do not open this Test Booklet until you are asked to do so.
Read carefully the Instructions on the Back Cover of this Test Booklet.

Important Instructions:
1. Immediately fill in the particulars on this page of the Test Booklet with Blue/Black Ball Point Pen. Use of pencil is strictly
prohibited.
2. The Answer Sheet is kept inside this Test Booklet. When you are directed to open the Test Booklet, take out the Answer
Sheet and fill in the particulars carefully.
3. The test is of 3 hours duration.
4. The Test Booklet consists of 75 questions. The maximum marks are 300.
5. There are three parts in the question paper A, B, C consisting of, Physics, Chemistry and Mathematics having 25
questions in each part of equal weightage.
6. Each question is allotted 4 (four) marks for each correct response. For MCQs - 4 Marks will be awarded for every correct
answer and 1 Mark will be deducted for every incorrect answer.
7. For answer with numeric value - 4 Marks will be awarded for every correct answer and 0 Mark will be deducted for every
incorrect answer.
8. Use Blue/Black Ball Point Pen only for writing particulars/ marking responses on Side-1 and Side-2 of the Answer Sheet.
Use of pencil is strictly prohibited.
9. No candidates is allowed to carry any textual material, printed or written, bits of papers, pager, mobile phone, any
electronic device, etc., except the Admit Card inside the examination hall/room.
10. Rough work is to be done on the space provided for this purpose in the Test Booklet only. This space is given at the
bottom of each page and at the end of the booklet.
11. On completion of the test, the candidate must hand over the Answer Sheet to the Invigilator on duty in the Room / Hall.
However, the candidates are allowed to take away this Test Booklet with them.
12. The CODE for this Booklet is A. Make sure that the CODE printed on Side-2 of the Answer Sheet and also tally the serial
number of the Test Booklet and Answer Sheet are the same as that on this booklet. In case of discrepancy, the candidate
should immediately report the matter to the invigilator for replacement of both the Test Booklet and the Answer Sheet.
13. Do not fold or make any stray marks on the Answer Sheet.

Name of the Candidate (in Capital letters):


Roll Number : in figures
in words
Examination Centre Number:
Name of Examination Centre (in Capital letters):
Candidate’s Signature: Invigilator’s Signature:
2 Physics
Read the following instructions carefully:
1. The candidates should fill in the required particulars on the Test Booklet and Answer Sheet (Side-1) with
Blue/Black Ball Point Pen.
2. For writing/marking particulars on Side-2 of the Answer Sheet, use Blue/Black Ball Point Pen only.
3. The candidates should not write their Roll Numbers anywhere else (except in the specified space) on the Test
Booklet/Answer Sheet.
4. Out of the four options given for each question, only one option is the correct answer.
5. For each incorrect response, one-fourth (¼) of the total marks allotted to the question would be deducted for
MCQs and No deduction for numeric questions from the total score. No deduction from the total score,
however, will be made if no response is indicated for an item in the Answer Sheet.
6. Handle the Test Booklet and Answer Sheet with care, as under no circumstances (except for discrepancy in
Test Booklet Code and Answer Sheet Code), another set will be provided.
7. The candidates are not allowed to do any rough work or writing work on the Answer Sheet. All
calculations/writing work are to be done in the space provided for this purpose in the Test Booklet itself,
marked ‘Space for Rough Work’. This space is given at the bottom of each page and at the end of the booklet.
8. On completion of the test, the candidates must hand over the Answer Sheet to the Invigilator on duty in the
Room/Hall. However, the candidates are allowed to take away this Test Booklet with them.
9. Each candidate must show on demand his/her Admit Card to the Invigilator.
10. No candidate, without special permission of the Superintendent or Invigilator, should leave his/her seat.
11. The candidates should not leave the Examination Hall without handing over their Answer Sheet to the
Invigilator on duty and sign the Attendance Sheet again. Cases where a candidate has not signed the Attendance
Sheet a second time will be deemed not to have handed over the Answer Sheet and dealt with as an unfair
means case. The candidates are also required to put their left hand THUMB impression in the space
provided in the Attendance Sheet.
12. Use of Electronic/Manual Calculator and any Electronic Item like mobile phone, pager etc. is prohibited
13. The candidates are governed by all Rules and Regulations of the JAB/Board with regard to their conduct in the
Examination Hall. All cases of unfair means will be dealt with as per Rules and Regulations of the JAB/Board.
14. No part of the Test Booklet and Answer Sheet shall be detached under any circumstances.
15. Candidates are not allowed to carry any textual material, printed or written, bits of papers, pager,
mobile phone, electronic device or any other material except the Admit Card inside the examination
hall/room.
Mock Test-1 3
JEE-MAIN: PHYSICS MOCK TEST-1

SECTION 1 (Multiple Choice Question) following shows the relative nature of the liquid columns
in the two tubes?
1. If a particle moves from point P (2,3,5) to point Q (3,4,5).
A B A B
Its displacement vector be:
a. b.
a. iˆ + ˆj + 10 kˆ b. iˆ + ˆj + 5kˆ
c. ˆi + ˆj d. 2iˆ + 4 ˆj + 6kˆ A B A B
c. d.
2. Long horizontal rod has a bead which can slide along its
length and is initially placed at a distance L from one end
A of the rod. The rod is set in angular motion about A with 7. Two rods of different materials having coefficients of
a constant angular acceleration α. If the coefficient of linear expansion α1 , α 2 and Young’s
friction between the rod and bead is µ, and gravity is moduli Y1 , Y2 respectively are fixed between two rigid
neglected, then the time after which the bead starts massive walls. The rods are heated such that they undergo
slipping is: the same increase in temperature. There is no bending of
µ µ the rods. If α1 : α 2 = 2 : 3, the thermal stresses developed
a. α b. α in the two rods are equal provided Y1 : Y2 is equal to:
1 a. 2 : 3 b. 1 : 1
c. µα d. infinitesimal c. 3 : 2 d. 4 : 9

3. A wind-powered generator converts wind energy into 8. Three liquids of equal volumes are thoroughly mixed. If
electric energy. Assume that the generator converts a their specific heats are s1 , s2 , s3 and their temperature
fixed fraction of the wind energy intercepted by its blades θ1 , θ 2 ,θ3 and their densities d1 , d 2 , d 3 respectively then
into electrical energy. For wind speed v, the electrical
the final temperature of the mixture is:
power output will be proportional to
a. v b. v2 s1 θ1 + s2 θ 2 + s3 θ3 d1 s1 θ1 + d2 s2 θ2 + d3 s3 θ3
a. b.
c. v 3
d. v4 d1 s1 + d 2 s2 + d3 s3 d1 s1 + d2 s2 + d3 s3

4. A spherical hollow is made in a lead sphere of radius R, d1 s1 θ1 + d 2 s2 θ 2 + d3 s3 θ3 d1 θ1 + d 2 θ 2 + d3 θ3


such that its surface touches the outside surface of lead c. d.
d1 θ1 + d 2 θ 2 + d3 θ3 s1 θ1 + s2 θ 2 + s3 θ3
sphere and passes through the center. What is the shift in
the center of mass of lead sphere due to the hollowing? 9. Two monatomic ideal gases 1 and 2 of molecular masses
R R m1 and m2 respectively are enclosed in separate containers
a. b.
7 14 kept at the same temperature. The ratio of the speed of
R sound in gas 1 to the gas 2 is given by:
c. d. R
2
m1 m2
5. The bob of a simple pendulum executes simple harmonic a. b.
m2 m1
motion in water with a period t, while the period of
oscillation of the bob is t0 in air. Neglect frictional force c. m1 d. m 2
m2 m1
of water and given that the density of the bob is
(4 / 3) × 1000 kg/m3. What relationship is true between t 10. A gas is compressed adiabatically till its temperature is
doubled. The ratio of its find volume to initial volume will
and t0 ?
be:
a. t = t0 b. t = t0 / 2
1 1
a. b. more than
c. t = 2t0 d. t = 4t0 2 2
6. A capillary tube A is dipped in water. Another identical 1
c. less than d. between 1and 2
tube B is dipped in a soap-water solution. Which of the 2
4 Physics
11. A black body is at a temperature of 2880 K. The energy of 1
a. b. 1
radiation emitted by this body with wavelength between 2
499 nm and 500 nm is U1 , between 999 nm and 1000 nm 2
c. d. 2
is U 2 and between 1499 nm and 1500 nm is U 3 . The 3

Wien constant, b = 2.88 × 106 nm-K. Then, 16. Double slit interference experiment is carried out with
monochromatic light and interference fringes are observed. If
a. U1 = 0 b. U 3 = 0
now monochromatic light is replaced by white light, what
c. U1 > U 2 d. U 2 > U1
change is expected in interference pattern?
12. In moving from A to B along an electric field line, the a. no change
electric field does 6.4 × 10−19J of work on an electron. If b. pattern disappears
φ1 , φ2 are equipotential surfaces, then the potential difference c. white and dark fringes are observed throughout the
pattern
(VC − VA ) is
d. a few coloured fringes are observed on either side of
a. – 4V b. 4V
central white fringe
c. Zero d. 64 V
17. The electric field of an electromagnetic wave in free space
13. Force acting on a charged particle kept between the plates
of a charged condenser is F. If one of the plates of the is given by E = 10 cos (107 t + kx ) ˆj V/m, Where, t and x are
condenser is removed, the force acting on the same in seconds and metres respectively. It can be inferred that
particle becomes: (a) the wavelength λ is 188.4 m
F (b) the wave number k is 0.33 rad/m
a. 0 b.
2 (c) the wave amplitude is 10 V/m
c. F d. 2F (d) the wave is propagating along + x direction
Which one of the following pairs of statements is correct?
14. An electric cable of copper has just one wire of radius
a. (c) and (d) b. (a) and (b)
9 mm. Its resistance is 50 Ω. This single copper wire of
c. (b) and (c) d. (a) and (c)
cable is replaced by 6 different well insulated copper
wires, each of radius 3 mm. The total resistance of the 18. Light of wavelength λ strikes a photosensitive surface
cable will now be equal to: and electrons are ejected with kinetic energy E. If the
a. 7.5 Ω b. 45 Ω kinetic energy is to be increased to 2E, the wavelength
c. 90 Ω d. 270 Ω must be changed to λ ′ where:
15. An infinitely long conductor PQR is bent to form a right λ
a. λ ′ = b. λ ′ = 2λ
angle as shown. A current I flows through PQR. The 2
magnetic field due to this current at the point M is H1. λ
c. < λ′< λ d. λ ′ = λ
Now another infinitely long straight conductor QS is 2
connected at Q so that the current is I/2 in QR as well as in
19. The ionisation potential of hydrogen atom is 13.6 V. The
QS, The current in PQ remaining unchanged. The
energy required to remove an electron in the n = 2 state of
magnetic field at M is now H 2. .The ratio H1 / H 2 is given
the hydrogen atom is
by
a. 27.2 eV b. 13.6 eV
M
c. 6.8 eV d. 3.4 eV

20. Hydrogen bomb is based on which of the following


I 90o
phenomenon?
–∞ +∞
P Q 90o S a. Nuclear fission
b. Nuclear fusion
R c. Radioactive decay
–∞
d. None of these
Mock Test-1 5
SECTION 2 (Numeric Value Question) smaller body, the intensity of gravitational field will be
zero:
21. A cube has a side of length 1.2 × 10 −2 m. Calculate its
1 1
volume: a. m b. m
9 10
a. 1.7 × 10 −6 m 3 b. 1.73 × 10 −6 m 3
1 10
c. 1.70 × 10 −6 m 3 d. 1.732 × 10 −6 m 3 c. m d. m
11 11
22. A body of mass 2 kg is released at the top of a smooth 24. Due to a small magnet intensity at a distance x in the end
inclined plane having inclination 30°. It takes 3 seconds to on position is 9 gauss. What will be the intensity at a
reach the bottom. If the angle of inclination is doubled x
distance on broad side on position?
keeping same height (i.e., made 60° ), what will be the 2
a. 9 gauss b. 4 gauss
time taken?
c. 36 gauss d. 4.5 gauss
a. 3 sec b. 3 sec 25. When both source and observe approach each other with a
c. 3 3 sec d. 1.5 sec velocity equal to the half the velocity of sound the change
in frequency of sound as detected by the listener is:
23. There are two bodies of masses 100 kg and 10000 kg a. 0 b. 25 %
separated by a distance 1m. At what distance from the c. 50% d. 150%

Space for rough work


6 Physics

JEE ADVANCE PAPER-I

Time 3 Hours. Max. Marks 180 (60 for Physics)


Read the Instructions Carefully

Question Paper Format and Marking Scheme:


1. The question paper has three parts: Physics, Chemistry and Mathematics. Each part has three sections.
2. Section 1 contains 6 multiple choice questions with one or more than one correct option.
Marking Scheme: +4 if only (all) the correct option(s) is (are) chosen, +3 if all the four options are correct but only three
options are chosen, +2 if three or more options are correct but only two options are chosen, both of the options must be
correct, +1 if two or more options are correct buy only one option is chosen and it must be correct, -2 (in all other cases).
3. Section 2 contains 8 Numerical value answer type questions. The answer to each question is a single digit integer ranging
from 0 to 9 (both inclusive).
Marking Scheme: +3 if only correct numerical value is given.
4. Section 3 contains 2 Paragraph based questions (2 paragraphs, each having 2 MCQs with one correct answer only)
Marking Scheme: +3 if only the correct option is selected, -1 in all other cases.
Note:
Possible response Section 1 Section 2 Section 3
Not attempted 0 0 0
Partial Correct +1 for each correct option selected 0 0
Correct +4 +3 3
incorrect -2 0 -1

SECTION 1 (Maximum Marks: 24) acting on the conductor, then the correct statement(s)
MCQs with one or more than one correct answer is(are)
y
1. A transparent thin film of uniform thickness and refractive R R
index n1 = 1.4 is coated on the convex spherical surface of π/6 π/4
radius R at one end of a long solid glass cylinder of L R R L x
refractive index n2 = 1.5, as shown in the figure. Rays of 
a. If B is along zˆ, F ∝ ( L + R)
light parallel to the axis of the cylinder traversing through 
the film from air to glass get focused at distance f1 from b. If B is along xˆ, F = 0

the film, while rays of light traversing from glass to air get c. If B is along yˆ , F ∝ ( L + R)
focused at distance f 2 from the film. Then 
d. If B is along zˆ, F = 0
n1
3. Planck’s constant h, speed of light c and gravitational
Air n2
constant G are used to form a unit of length L and a unit of
mass M. Then the correct option(s) is(are)
a. M ∝ c b. M ∝ G
a. | f1 | = 3R b. | f1 | = 2.8 R c. L ∝ h d. L ∝ G
c. | f 2 | = 2 R d. | f 2 | = 1.4 R 4. Two independent harmonic oscillators of equal mass are
2. A conductor (shown in the figure) carrying constant oscillating about the origin with angular frequencies ω1
current I is kept in the x-y plane in a uniform magnetic and ω2 and have total energies E1 and E2 , respectively.

field B. If F is the magnitude of the total magnetic force The variations of their momenta p with positions x are
Mock Test-1 7
a a SECTION 2 (Maximum Marks: 24)
shown in the figures. If = n 2 and = n, then the correct
b R Numerical value answer type questions
equation(s) is(are)
P P 7. A small filament is at the centre of a hollow glass sphere
Energy = E1 Energy = E2
of inner and outer radii 8 cm and 9 cm respectively. The
b refractive index of glass is 1.50. Calculate the position of
x x
a R the image of the filament when viewed from outside the
sphere.
OA = 8 cm
ω Glass OB = 9 cm
a. E1ω1 = E2ω2 b. 2 = n
2
m = 1.50
ω1
E1 E2 O A B
c. ω1ω 2 = n 2 d. = Air
ω1 ω2
5. Inner and outer radii of a spool are r and R respectively.
A thread is wound over its inner surface and spool is 8. An infinitely long uniform line charge distribution of
placed over a rough horizontal surface. Thread is pulled charge per unit length λ lies parallel to the y-axis in the
by a force F as shown in figure. In case of pure rolling, 3
which of the following statements are false? y-z plane at z = a (see figure). If the magnitude of the
2
flux of the electric field through the rectangular surface
F
R ABCD lying in the x-y plane with its center at the origin is
α
r λL
(ε 0 = permittivity of free space), then the value of n
nε 0
is
z
a. Thread unwinds, spool rotates anticlockwise and
friction acts leftwards
D L
b. Thread winds, spool rotates clockwise and friction acts C
3
leftwards a
a
2
c. Thread winds, spool moves to the right and friction acts O
y
rightwards
d. Thread winds, spool moves to the right and friction
A B
does not come into existence
x
6. Two ideal batteries of emf V1 and V2 and three resistances
9. For an atom of an ion having single electron, the
R1 , R2 and R3 are connected as shown in the figure. The
wavelength observed λ1 = 2 are units and λ3 = 3 units
current in resistance R2 would be zero if
figure. The value of missing wavelength λ2 is
V1
R1 n3 orbit
R2 λ1 λ3

n2 orbit
λ2
V2 n1 orbit
R3
10. Consider an elliptically shaped rail PQ in the vertical
a. V1 = V2 and R1 = R2 = R3 plane with OP = 3 m and OQ = 4 m. A block of mass
b. V1 = V2 and R1 = 2 R2 = R3 1 kg is pulled along the rail from P to Q with a force of
c. V1 = 2V2 and 2 R1 = 2 R2 = R3 18 N, which is always parallel to line PQ (see the given
d. 2V1 = V2 and 2 R1 = R2 = R3 figure). Assuming no frictional losses, the kinetic energy
8 Physics
of the block when it’s reaches Q in (n × 10) joules. The SECTION 3 (Maximum Marks: 12)
−2 2 Paragraph based questions (2 paragraphs, each having
value of n is (take acceleration due to gravity 10 ms )
2 MCQs with one correct answer only)
Q
Paragraph for Question No. 15 to 16
Light guidance in an optical fiber can be understood by
4m
considering a structure comprising of thin solid glass cylinder
90° of refractive index n1 surrounded by a medium of lower
O 3m P refractive index n2 . The light guidance in the structure takes
11. Two identical uniform discs roll without slipping on two
place due to successive total internal reflections at the interface
different surfaces AB and CD (see figure) starting at A and
of the media n1 and n2 as shown in the figure. All rays with
C with linear speeds v1 and v2 , respectively, and always
the angle of incidence i less than a particular value im are
remain in contact with the surfaces. If they reach B and D
with the same linear speed and v1 = 3 m / s, then v2 in m/s confined in the medium of refractive index n1 . The numerical
is ( g = 10 m / s 2 ) aperture (NA) of the structure is defined as sin im .

A v1 = 3m / s C
v2
n1 > n2
Air
30 m 27 m Cladding n2
B D
θ Core
i n1

12. Two spherical stars A and B emit blackbody radiation. The


radius of A is 400 times that of B and A emits 104 times
15. For two structures namely S1 with n1 = 45 / 4 and
the power emitted from B. The ratio (λ A / λB ) of their
n2 = 3 / 2, and S 2 with n1 = 8 / 5 and n2 = 7 / 5 and taking
wavelengths λA and λB at which the peaks occur in their
the refractive index of water to be 4/3 and that of air to be
respective radiation curves is
13. The half life of a freshly prepared radioactive sample is 2 h. 1, the correct option(s) is(are)
If the sample emits radiation of intensity, which is 16 times a. NA of S1 immersed in water is the same as that of S 2
the permissible safe level, then the minimum time taken 16
after which it would be possible to work safely with source immersed in a liquid of refractive index
3 15
is
6
14. A Young’s double slit interference arrangement with slits b. NA of S1 immersed in liquid of refractive index
15
S1 and S 2 is immersed in water (refractive index = 4/3) as
is the same as that of S 2 immersed in water
shown in the figure. The positions of maxima on the
surface of water are given by x 2 = p 2 m 2 λ 2 − d 2 , where λ c. NA of S 1 placed in air is the same as that of S 2

is the wavelength of light in air (refractive index = 1), 2d 4


immersed in liquid of refractive index .
is the separation between the slits and m is an integer. The 15
value of p is d. NA of S1 placed in air is the same as that of S 2 placed
in water
S1
d 16. If two structures of same cross-sectional area, but different
x Air
numerical apertures NA1 and NA2 ( NA2 < NA1 ) are joined
d
water longitudinally, the numerical aperture of the combined
S2
structure is
Mock Test-1 9
NA1 NA2 Two points K and M are symmetrically located on the
a. b. NA1 + NA2
NA1 + NA2 opposite faces parallel to the x-y plane (see figure). V1
c. NA1 d. NA2 and V2 are the potential differences between K and M in
strips 1 and 2, respectively. Then, for a given current I
Paragraph for Question No. 17 to 18 flowing through them in a given magnetic field strength
In a thin rectangular metallic strip a constant current I flows B, the correct statement(s) is(are)
along the positive x-direction, as shown in the figure. The
a. If w1 = w2 and d1 = 2d 2 , then V2 = 2V1
length, width and thickness of the strip are ℓ, w and d,
 b. If w1 = w2 and d1 = 2d 2 , then V2 = V1
respectively. A uniform magnetic field B is applied on the
c. If w1 = 2w2 and d1 = d 2 , then V2 = 2V1
strip along the positive y-direction. Due to this, the charge
carriers experience a net deflection along the z-direction. d. If w1 = 2w2 and d1 = d 2 , then V2 = V1
This results in accumulation of charge carriers on the surface 18. Consider two different metallic strips (1 and 2) of same
PQRS and appearance of equal and opposite charges on the
dimensions (lengths ℓ, width w and thickness d) with
face opposite to PQRS. A potential difference along the z-
carrier densities n1 and n2, respectively. Strip 1 is
direction is thus developed. Charge accumulation continues
until the magnetic force is balanced by the electric force. The placed in magnetic field B1 and strip 2 is placed in
current is assumed to be uniformly distributed on the cross- magnetic field B2, both along positive y-directions.
section of the strip and carried by electrons. Then V1 and V2 are the potential differences developed
l between K and M in strips 1 and 2, respectively.
y
•K
I W I Assuming that the current I is the same for both the
x
S R
•M z strips, the correct option(s) is(are)
d
P Q a. If B1 = B2 and n1 = 2n2 , then V2 = 2V1
17. Consider two different metallic strips (1 and 2) of the b. If B1 = B2 and n1 = 2n2 , then V2 = V1
same material. Their lengths are the same, widths are
c. If B1 = 2B2 and n1 = n2 , then V2 = 0.5V1
w1 and w2 and thicknesses are d1 and d2 , respectively.
d. If B1 = 2 B2 and n1 = n2 , then V2 = V1
Space for rough work
10 Physics

JEE ADVANCE PAPER-II

Time 3 Hours. Max. Marks 180 (60 for Physics)


Read the Instructions Carefully

Question Paper Format and Marking Scheme:


1. The question paper has three parts: Physics, Chemistry and Mathematics. Each part has three sections.
2. Section 1 contains 6 MCQs with one or more than one correct answer
Marking Scheme: +4 if only (all) the correct option(s) is (are) chosen, +3 if all the four options are correct but only three
options are chosen, +2 if three or more options are correct but only two options are chosen, both of the options must be
correct, +1 if two or more options are correct buy only one option is chosen and it must be correct, -2 (in all other cases)
3. Section 2 contains 8 Numerical value answer type questions.
Marking Scheme: +3 if only correct numerical value is given
4. Section 3 contains 4 Matching type questions with 4 options.
Marking Scheme: +3 if only the correct option is selected, -1 in all other cases
Note:
Possible response Section 1 Section 2 Section 3
Not attempted 0 0 0
Partial Correct +1 for each correct option selected 0 0
Correct +3 +4 +3
Incorrect -1 -2 0

SECTION 1 (Maximum Marks: 24) in equilibrium with the sphere P in L1 and sphere Q in L2
MCQs with one or more than one correct answer and the string being taut (see figure). If sphere P alone in

1. A fission reaction is given by L2 has terminal velocity VP and Q alone in L1 has

236
92 U 
→ 14054 Xe + 94
38Sr + x + y, where x and y are two
terminal velocity VQ , then
236
particles. Considering 92 U to at rest, the kinetic energies
L1
P
of the products are denoted by K Xe , KSr , K x (2MeV) and
K y (2MeV), respectively. Let the binding energies per L2
Q
236
nucleon of 92 U, 140
54 Xe and
94
38 Sr be 7.5 MeV, 8.5 MeV  
| VP | η1 | VP | η 2
and 8.5 MeV respectively. Considering different a.  = b.  =
| VQ | η 2 | VQ | η1
conservation laws, the correct option(s) is(are)
   
a. x = n, y = n, KSr = 129 MeV, K Xe = 86 MeV c. VP ⋅VQ > 0 d. VP ⋅VQ < 0
b. x = p, y = e − , K Sr = 129 MeV, K Xe = 86 MeV 3. In terms of potential difference V, electric current I,
c. x = p, y = n, KSr = 129 MeV, K Xe = 86 MeV permittivity ε 0 , permeability µ0 and speed of light c, the
d. x = n, y = n, KSr = 86 MeV, K Xe = 129 MeV dimensionally correct equation(s) is(are)
a. µ 0 I 2 = ε 0V 2 b. ε 0 I = µ0V
2. Two spheres P and Q of equal radii have densities ρ1 and
c. I = ε 0 cV d. µ0 cI = ε 0V
ρ 2 , respectively. The spheres are connected by a mass
less string and placed in liquids L1 and L2 of densities σ 1 4. Consider a uniform spherical charge distribution of radius
R1 centred at the origin O. In this distribution, a spherical
and σ 2 and viscosities η1 and η2 , respectively. They float
Mock Test-1 11
cavity of radius R2, centred at P with distance 8. A large spherical mass M is fixed at one position and two
OP = a = R1 − R2 (see figure) is made. If the electric field identical point masses m are kept on a line passing
   through the centre of M (see figure). The point masses are
inside the cavity at position r is E (r ), then the correct
connected by a rigid mass less rod of length ℓ and this
statement(s) is(are) assembly is free to move along the line connecting them.
R2
P All three masses interact only through their mutual
a
R1 gravitational interaction. When the point mass nearer to M
O is at a distance r = 3ℓ from M, the tension in the rod is
 M 
zero for m = k   . The value of k is
  288 
a. E is uniform, its magnitude is independent of R2 but its
 M m m
direction depends on r

b. E is uniform, its magnitude depends on R2 and its r ℓ

direction depends on r
 9. The energy of a system as a function of time t is given as
c. E is uniform, its magnitude is independent of a but its

direction depends on a E (t ) = A 2 exp( − α t ), where α = 0.2 s −1 . The measurement
 of A has an error of 1.25%. If the error in the
d. E is uniform and both its magnitude and direction
 measurement of time is 1.50%, the percentage error in the
depend on a
value of E (t ) at t = 5 s is
5. In plotting stress versus strain curves for two materials P
and Q, a student by mistake puts strain on the y-axis and 10. The densities of two solid spheres A and B of the same
stress on the x-axis as shown in the figure. Then the r 
radii R vary with radial distance r as ρ A ( r ) = k   and
correct statement(s) is(are) R
5
r
ρ B (r ) = k  , respectively, where k is a constant. The
R
Strain

P
moments of inertia of the individual spheres about axes
Q passing through their centres are I A and I B , respectively.
Stress
IB n
a. P has more tensile strength than Q If = , the value of n is
I A 10
b. P is more ductile than Q
c. P is more brittle than Q 11. Four harmonic waves of equal frequencies and equal
d. The Young’s modulus of P is more than that of Q π 2π
intensities I0 have phase angles 0, , and π. When
6. A spherical body of radius R consists of a fluid of constant 3 3
density and is in equilibrium under its own gravity. If P(r) they are superposed, the intensity of the resulting wave is
is the pressure at r(r < R), then the correct option(s) is(are) nI 0 . The value of n is
P(r = 3R / 4) 63
a. P(r = 0) = 0 b. = 12. For a radioactive material, its activity A and rate of change
P(r = 2 R / 3) 80
dN dA
P(r = 3R / 5) 16 P (r = R / 2) 20 of its activity R are defined as A = − and R = − ,
c. = d. = dt dt
P (r = 2 R / 5) 21 P(r = R / 3) 27
where N(t) is the number of nuclei at time t. Two
SECTION 2 (Maximum Marks: 24)
radioactive sources P (mean life τ ) and Q (mean life 2τ )
Numerical value answer type questions
have the same activity at t = 0. Their rates of change of
7. An electron in an excited state of Li2+ ion has angular
activities at t = 2τ are R P and RQ , respectively. If
momentum 3h / 2π . The de Broglie wavelength of the
electron in this state is pπ a0 (where a0 is the Bohr RP n
= , then the value of n is
RQ e
radius). The value of p is
12 Physics
13. A monochromatic beam of light is incident at 60° on one (D) Ratio of the 4. 5 : 27
face of an equilateral prism of refractive index n and wavelengths of first
emerges from the opposite face making an angle line of Lyman series
θ ( n ) with the normal (see the figure). For n = 3 the and first line of
Balmer series is

value of θ is 60° and = m. The value of m is 5. 0
dn
a. A→ 1; B→ 2, 4; C→3; D→ 4
b. A→3; B→1, 2; C→1; D→ 2
c. A→4; B→3, 4; C→2; D→ 1
60° θ d. A→2; B→2, 3; C→4; D→ 3
16. You are given many resistances, capacitors and inductors.
These are connected to a variable DC voltage source
(the first two circuits) or an AC voltage source of 50 Hz
14. The figure shows a portion of an electric circuit. Resistors frequency (the next three circuits) in different ways as
are known and are indicated on the diagram and the shown in Column II. When a current I (steady state for DC
voltmeters are identical. If the voltmeters V1 and V2 read or r.m.s. for AC) flows through the circuit, the
7.5 V and 5.0 V respectively, find the reading of the volt- corresponding voltage V1 and V2. (indicated in circuits) are
meter V3. related as shown in Column I.
V1
R1 = 9
Column I Column II
(A) I ≠ 0, V1 is 1.
V1 V2

V2 proportional to I 6 mH 3 µF
R2 = 14

V3
V
R3 = 24
(B) I ≠ 0, V2 > V1 2. V1 V2
SECTION 3 (Maximum Marks: 12)
6 mH 2Ω
Matching type questions with 4 options
V
15. Match the statement of Column with those in
Column II: (C) V1 = 0, V2 = V 3. V1 V2

Column I Column II 6 mH 2Ω
(A) In any Bohr orbit of 1
1. −
the hydrogen atom, 2 V
the ratio of kinetic
(D) I ≠ 0, V2 is 4. V1 V2
energy to potential
energy of the electron proportional to I 6 mH 3µF
is
(B) The ratio of the 2. 2 V
kinetic energy to the
5. V1 V2
total energy of an
electron in a Bohr 1 kΩ 3µF
orbit is
(C) In the lowest energy V
h
3.
level of hydrogen 2π a. A→3, 4; B→2, 3, 4; C→1, 2; D→2, 3, 4
atom, the electron has b. A→1, 2; B→1, 2, 3; C→3, 4; D→1, 3 ,4
the angular c. A→2, 4; B→2, 3, 4; C→2, 3; D→1, 2, 3
momentum d. A→1, 3; B→1, 3, 4; C→1, 3; D→2, 3, 4
Mock Test-1 13

17. The gravitational field intensity E of earth at any point is a. A→1; B→2; C→3; D→4
defined as the gravitational force per unit mass at that b. A→4; B→3; C→2; D→1
point. If varies from place to place. The variation is shown c. A→4; B→1; C→3; D→2
  d. A→3; B→2; C→1; D→4
in column II with form of position r v s the E in the form
 18. A person in a lift is holding a water jar, which has a small
of graphs. The variation of r is given in column I.
hole at the lower end of its side. When the lift is at rest, the
Choose the correct form of graphs for the corresponding water jet coming out of the hole hits the floor of the lift at a

variations of r . distance d of 1.2 m from the person. In the following, state
Column I Column II of the lift’s motion is given in Column I and the distance
  where the water jet hits the floor of the lift is given in
(A) Position r of body R rE
1. O Column II. Match the statements from Column I with those
measured from
in Column II and select the correct using the code given
surface earth
 below the lists.
upward E
Column I Column II
 
(B) Position r of body R rE (A) Lift is accelerating 1. d = 1.2 m
O
measured from 2. vertically up.
surface of earth (B) Lift is accelerating 2. d > 1.2 m

along diameter to E vertically down with
opposite point on an acceleration less
surface of earth than the gravitational
  acceleration.
(C) Position r measured r
O
3. (C) Lift is moving 3. d < 1.2 m
from center of earth
vertically up with
to any point
 constant speed.
E
(D) Lift is falling freely. 4. No water leaks out of
 
(D) Position r measured R rE the jar
4. O
from center of a. A→1; B→1; C→1; D→4
hollow sphere to b. A→4; B→3; C→2; D→1

any point E c. A→4; B→1; C→3; D→2
d. A→3; B→2; C→1; D→4

Space for rough work


14 Physics
ANSWER & SOLUTIONS M 7M
m2 = M − = = mass of remaining sphere
JEE-Main 8 8
Choosing the centre of big sphere as the origin,
1. 2. 3. 4. 5. 6. 7. 8. 9. 10.
m x + m2 x2
c a c b c b c b b b X CM = 1 1
11. 12. 13. 14. 15. 16. 17. 18. 19. 20. m1 + m2
d b b a c d d c d b
0=
( M / 8 ) × ( R / 2 ) + ( 7 M / 8 ) × x2
21. 22. 23. 24. 25. M
a b c c c −R
Solving, we get x2 =
 14
1. (c) Displacement vector r = ∆xiˆ + ∆yjˆ + ∆zkˆ
5. (c) In air g eff = g
= (3 − 2)iˆ + (4 − 3) ˆj + (5 − 5)kˆ = iˆ + ˆj
y Buoyant force d g
In water, g eff = g – =g− w g=
Mass db 4
F cos 60o


F
t
60o Now t = 2π , Hence 2t0 = t
g eff
x
F sin 60o
6. (b) Soap solution has lower surface tension as compared
2. (a) Tangential force (Ft) of the bead to pure water so h is less for soap solution.
will be given by the normal reaction
Ft 7. (c) F1 = F2
(N), while centripetal force (Fc) is A L
Fc Y1 r2 3
provided by friction (Fr). The bead ⇒ Y1 A α1∆θ = Y2 A α 2 ∆θ ⇒ = =
starts sliding when the centripetal Y2 r1 2
force is just equal to the limiting Ft is inwards 8. (b) Let 0°C be the reference temperature for zero heat,
friction. Therefore, Ft = ma = mα L = N then initial heat energy = final heat energy
∴ Limiting value of friction ( f r ) max = µ N = µ mα L . . . (i) m1s1θ1 + m2 s2θ 2 + m3 s3θ3 = ( m1s1 + m2 s2 + m3 s3 )
Angular velocity at time t is ω = α t m1s1θ1 + m2 s2θ 2 + m3 s3θ3
⇒ θθ =
∴ Centripetal force at time t will be m1s1 + m2 s2 + m3 s3
Fc = mL ω 2 = mL α 2 t 2 . . . (ii) Vd1s1θ1 + Vd 2 s2θ 2 + Vd 3 s3θ3 d s θ + d 2 s2θ 2 + d 3 s3θ 3
= = 11 1
µ Vd1s1 + Vd 2 s2 + Vd 3 s3 d1s1 + d 2 s2 + d 3 s3
Equation (i) and (ii), we get t =
α
γ RT 1
9. (b) Speed of sound in a gas is given by v = ,v ∝
µ
For t > , F > ( f r )max i.e. , the bead starts sliding. M M
α c
v1 M2 m2
In the figure Ft is perpendicular to the paper inwards. ∴ = =
  v2 M1 m1
3. (c) Power = F i v = Fv
Here,γγ =
Cp 5 for both the gases  5
 dm   d ( ρ × volume )  =  γ monoatomic = 
F = v  = v  CV 3  3
 dt   dt  γ −1
V  T1
 d ( volume ) 
γ −1 γ −1
10. (b) T1 V1 = T2 V2 ⇒ 2  =
= ρv   = ρ v ( Av ) = ρ Av
2
 V1  T2
 dt  γ −1 1−γ

∴ Power P = ρ Av3 or P ∝ v 3 V1  T1   T  1 1
or =  = 1  = 1−γ
>
4
V2  T2  ( 2)
 2T2  2
4. (b) Let ρ be the density of lead. Then M = π R3ρ =
3 11. (d) Wien’s displacement law is λmT = b (b = Wien’s
= mass of total sphere constant)
3
4 R M  b 2.88 × 106 nm-K
m1 = π   ρ = mass of removed part =   ∴ λm = =
3 2  8  T 2880K
Mock Test-1 15
∴ λ = 1000 nm y-direction. As E is varying with x and t, hence
Energy distribution with wavelength will be as follows: propagation of electromagnetic wave takes place along –x
axis. Thus statement (d) is wrong. Comparing the relation,
E = 10 cos(107 t + kx) with standard equation of
electromagnetic wave
2π  2πυ 2π 
E = E0 cos (υ t + x) = E0 cos  t+ x
1000
1499
1500

λ 
499
599
999

λ  λ
U1 We have, E0 = 10V / m. Thus statement (c) is correct.
From the fig. it is clear that (In fact U2 is maximum)
U2
2πυ 2π × (3 × 108 )
12. (b) Work done by the field W = q (−dV ) = −e(VA − VB ) = 107 or = 107
λ λ
= e(VB − VA ) = e(VC − VA ) (∵ VB = VC) 22
−19
or λ = 60π = 60 × ≈ 188.4 m
W 6.4 × 10 7
⇒ (VC − VA ) = = = 44V
V
e 1.6 × 10−19 Thus, statement (a) is correct.
13. (b) In the electric field between plates of parallel plate 2π 2π 1
⇒ k= = = = 0.033 rad
rad / m
m
σ λ 60π 30
capacitor E =
ε0 Thus, statement (b) is wrong.
qσ hc
∴ F = qE = . . . (i) 18. (c) E = = −W . . . (i)
ε0 λ
When one plate is removed, the electric field becomes hc
2E = = −W . . . (ii)
σ λ′
E′ =
2ε 0 hc
−W
σ Dividing (ii) by (i) 2 = λ ′ =
2hc
− 2W =
hc
−W
∴ F ′ = qE′ = . . . (ii)
2ε 0 hc
−W
λ λ′
λ
F′ 1 F
= ⇒ F′ = hc 2hc 1 2 W
F 2 2 ⇒ = −W ⇒ = −
λ′ λ λ ′ λ hc
14. (a) Resistance of 9 mm cable = 5 Ω
1 λ
1 1 ⇒ λ′ = ⇒ λ′ > but less than λ
As R ∝ or R ∝ 2 ; resistance of 3 mm 2 W 2
A r −
λ hc
Cable 9 × 5 = 45 Ω
19. (d) Energy required to remove electron in the n = 2
In second case 6 wires are connected in parallel, so total
13.6
45 state = + 2 = +3.4 eV
eV
resistance of cables = = 7 ⋅ 5Ω (2)
6
15. (c) Magnetic field at any point lying on the current 20. (b) Hydrogen bomb is based on nuclear fusion.
carrying straight conductor is zero. Here
21. (a) V = l 3 = (1.2 × 10−2 m)3 = 1.728 × 10 −6 m 3
H1 = Magnetic field at M due to current in PQ.
∵ Length (l) has two significant figures, the volume (V) will
H2 = Magnetic field at M due to QR + magnetic field at M
also have two significant figures. Therefore, the correct
due to QS + magnetic field at M due to PQ
answer is V = lV = 1.7 × 10 −6 m 3
H 3 H 2
= 0 + 1 + H 1 = H1 ⇒ 1 =
2 2 H2 3 h
22. (b) a = g sin θ , l =
16. (d) If monochromatic light is replaced by white light, a sin θ
few coloured fringes are seen on either side of a central h 1 1
⇒ = g sin θt 2 ⇒ t ∝
white fringe. sin θ 2 sin θ
17. (d) Electric field of an electromagnetic wave in free space t22 sin θ11 1/ 2 1 t 3
 ⇒ =
= = = / t2 = 1 = = 3 s
is given by E = 10cos(107 t + kx) ˆj which is acting along t1 sin θ22 3/2 3 3 3
16 Physics
GM 1
=
GM 2  a  ω2
23. (c) ⇒  = = n2
(1 − x )  b  ω1
2 2
x
Planet
100 10, 000 2 1 E1 E2 E1 E2
r2 r1
⇒ = P ⇒ = ⇒ =
x 2 (1 − x )2 ω12 a 2 ω22 a 2 ω1 ω2

1 x2 1 x2 1 5. (a, c, d) Since, the spool rolls over the horizontal surface,


or = ⇒ = ⇒ x= m
100 (1 − x ) 2
10 1 − x 11 therefore, instantaneous axis of rotation passes through the
point of contact of spool with the horizontal surface.
2M About the instantaneous axis of rotation, moment
24. (c) In C.G.S. Baxial = 9 = . . . (i)
x3 produced by F is clockwise. Therefore, the spool rotates
M 8M clockwise. In that case acceleration will be rightward and
Bequaterial = = . . . (ii)
x
3
x3 thread will wind. If rotational motion of spool is
2 considered about its own axis then resultant moment on it
 
must be clockwise. But moment produced by the force F
From equation (i) and (ii) Bequaterial = 36 gauss.
is anticlockwise and its magnitude is equal to F.r,
vr v/2 3 Hence, moment produced by the friction (about its own
25. (c) n′ = n + n =n+ n= n axis) must be clockwise and its magnitude must be greater
r v 2
% change in frequency than F.r. It is possible only when friction acts leftwards.
Therefore, option (b) is correct.
n′ − n  n′ 
= ×100% =  − 1 ×100%
n n  R1 (V1 + V2 )
6. (a, b, d) V1 = ⇒ V1 R3 = V2 R1
3  R1 + R3
=  − 1 ×100% = 50%
2  R3 (V1 + V2 )
⇒ V2 = ⇒ V2 R1 = V2 R3
R1 + R3
JEE Advance Paper -I
7. (9) For refraction at the first surface, u = −8 cm,
1. 2. 3. 4. 5. 6. 7. 8. 9. 10.
c a,b,c a,c,d b,d a,c,d a,b,d 9 6 6 5 R1 = −8 cm, µ1 = 1, µ 2 = 1.5
11. 12. 13. 14. 15. 16. 17. 18. µ2 µ1 µ 2 − µ1
7 2 8 5 a,c d a,d a,c ⇒ − =
v u R1
1.5 1.4 − 1 1.5 − 1.4 1.5 1 0.5
1. (c) For air to glass = + ∴ f1 = 3 R ⇒ + = ⇒ v ' = −8 cm
f1 R R v ' 8 −8
1 1.4 − 1.5 1 − 1.4 It means due to the first surface the image is formed at the
For glass to air. = + ∴ f2 = 2R
f2 −R −R centre.
  For the second surface u = −9 cm, µ1 = 1.5, R2 = −9 cm
2. (a, b, c) F = 2 I ( L + R)[iˆ × B]
2(L + R) m2 m1 m2 − m1
⇒ − =
3. −1
(a, c, d) h ≡ [ ML T ], c ≡ [ LT ], G ≡ [ M L T ]
2 −1 −1 3 −2 v u R2
1 1.9 1 − 1.5
hc hG ⇒ + = ⇒ v = −9 cm
⇒ M∝ ,L ∝ v 5 −9
G c3
Thus, the final image is formed at the centre of the sphere.
1
4. (b, d) For first oscillator E1 = mω12 a 2 8. (6) From the figure θ = 60°
2
Line change
and p = mv = mω1a = b
a 1 θ
⇒ = . . .(i) 3
a
b mω1 2
For second oscillator a Rectangular
surface
1
E2 = mω22 R2 , and mω2 = 1 . . .(ii) 360°
2 So, No. of rectangular surfaces used to form a m = =6
60°
Mock Test-1 17
9. (6) As is clear from figure. E1 = E2 + E3 for the combined structure and hence, correct option is d.
hc hc hc 17. (a, d) I1 = I 2
= +
λ1 λ2 λ3 ⇒ neA1v1 = neA2 v2
1 1
1 1 1 1 ⇒ d1 w1v1 = d 2 w2 v2
∴ = − = − = = 6 units
λ2 λ1 λ3 2 3 6
Now, potential difference developed across MK, V = Bvw
10. (5) Using work energy theorem Wmg + WF = ∆KE V1 v1w1 d 2
⇒ = = and hence correct choice is a & d.
−mgh + Fd = ∆KE −1×10 × 4 + 18(5) = ∆KE V2 v2 w2 d1
⇒ ∆KE = 50 ∴ n = 5 18. (a, c) As I1 = I 2
11. (7) Kinetic energy of a pure rolling disc having velocity of ⇒ n1w1d1v1 = n2 w2 d 2 v2
1 2 1  mR  v 2
3 2
V2 B2v2 w2  B2 w2   n1w1d1  B2 n1
centre of mas v = mv +   = mv2 Now, = =  =
2 2  2  R2 4 V1 B2v1w1  B1w1   n2 w2 d2  B1n2
3 3
So, m(3)2 + mg (30) = m(v2 ) 2 + mg (27) JEE Advance Paper -II
4 4
∴ v2 = 7 m / s 1. 2. 3. 4. 5. 6. 7. 8. 9. 10.
a a,d a,c d a,b b,c 2 7 4 6
 dQ  4  dQ 
 = 10   (400 R) TA = 10 ( R TB )
12. (2)  2 4 4 2 4 11. 12. 13. 14. 15. 16. 17. 18.
 dt  A  dt  B 3 2 2 3 a a d a
λ A TB 1. (a) Q value of reaction
So, 2TA = TB and = =2
λB TA = (140 + 94) × 8.5 − 236 × 7.5 = 219 MeV
13. (8) Here, T = 2 h, t = ? So, Total kinetic energy of Xe and Sr = 219 − 2 − 2 = 215 MeV
To work safely with the sample, its activity must be So, by conservation of momentum, energy, mass and
charge, only option a. is correct.
1
reduced to .
16 2. (a, d) From the given conditions, ρ1 < σ 1 < σ 2 < ρ 2
n 4
N 1 1 1 From equilibrium, σ 1 + < σ 2 = ρ1 + ρ 2
From =  = =  ∴ n=4
N 0  2  16  2 
2  ρ −σ2  2  ρ2 − σ1 
Vp =  1  g and VQ =  g
⇒ t = n T = 2 × 4 = 8h 9  η2  9  η1 

4 | VP | η1  
14. (3) For maxima, d 2 + x 2 − d 2 + x 2 = mλ , m is an So,  = and VP .VQ < 0
3 | VQ | η1
integer
3. (a, c) BI ℓc ≡ VI
So, x 2 = 9m 2λ 2 − d 2 ⇒ p = 3
⇒ µ0 I 2 c ≡ VI
15. (a, c) θ ≥ c n2
⇒ µ0 Ic = V ⇒ µ 02 I 2 c 2 = V 2
⇒ 90° − r ≥ c θ
nm
r n1 ⇒ µ0 I 2 = ε 0V 2 ⇒ ε 0 cV = I
⇒ sin(90° − r ) ≥ c i

⇒ cos r ≥ sin c  ρ 


4. (d) E = C1C2
sin i ni n 3ε 0
Using = and sin = 2
sin r nm n3 C1 ⇒ Centre of sphere and
C2 ⇒ centre of cavity.
n12 − n22
We get, sin 2 im = stress
nm2 5. (a, b) Y =
strain ρ
Putting values, we get, correct options as a & c
1 strain
16. (d) For total internal reflection to take place in both ⇒ =
Y stress
structures, the numerical aperture should be the least one
18 Physics
1 1 r1 + r2 = 60° . . .(ii)
⇒ > ⇒ YP < YQ
YP YQ Snell’s Law on 2 surface: n sin r2 = sin θ
nd

 r2  Using equation (i) and (ii) n sin(60° − r1 ) = sin θ


6. (b, c) P (r ) = K 1 − 2 
 R   3 1 
nh 3h ⇒ n  cos r1 − sin r1  = sin θ
7. (2) mvr = =  2 2 
2π 2π

de-Broglie Wavelength λ =
h
mv

d  3

dn  4
( ) 
4n 2 − 3 − 1  = cos θ


dn
2π r 2π a0 (3) 2 dθ
= = = 2π a0 For θ = 60° and n = 3 ⇒ =2
3 3 z Li dn

8. (7) For m closer to M;


GMm Gm2
− 2 = ma . . .(i) V1V2 ( R2 − R1 )
14. (3) V3 = = 3.0 V
9ℓ 2 ℓ V1 ( R3 − R1 ) − V2 ( R3 − R2 )
Gm2 GMm
and for the other m: + = ma . . .(ii) 15. (a) A→ 1; B→ 2, 4; C→ 3; D→ 4
ℓ2 16ℓ 2
From both the equations, k = 7 16. (a) A→ 3, 4; B→ 2, 3, 4; C→ 1, 2; D→ 2, 3, 4
2 − at
9. (4) E (T ) = A e 17. (d) Gravitational field intensity at any point outside due to
⇒ 2 − at
dE = −α A e dt + 2 AdAe − at   GM GM
earth is given by E = g = = 2
Putting the values for maximum error, ( R + h) 2 r
dE 4  1
⇒ = ⇒ % error = 3. where h is height from surface or E ∝ 2 but – ve in sign.
E 100 r
2 Graph is parabolic.
10. (6) I = ∫ ρ 4π r 2 r 2 dr
3 Inside earth g decreases are r increases form surface of
⇒ I A ∝ ∫ ( r )(r 2 )( r 2 ) dr earth.
  x g 1
⇒ I B ∝ ∫ ( r 5 )( r 2 )( r 2 ) dr E = g 1 +  = g − x −E ∝ 2
 R R r
IB 6
∴ = or E ∝ x is straight line decreasing to 0 at centre of earth and
I A 10 then increases in magnitude but – ve as directed towards
11. (3) First and fourth wave interfere destructively. So from the centre always.
the interference of 2nd and 3rd wave only, 1
 2π π  At any point outside E ∝ 2 (parabolic graph)
⇒ I net = I 0 + I 0 + 2 I 0 I 0 cos  −  = 3I 0 ⇒ n = 3 r
 3 3 Inside earth E ∝ r (linear graph)
−λ t
1 1 R ( A λ )e p
In hollow sphere inside E = 0 graph along r-axis; outside
12. (2) λ P = ; λQ = ⇒ P = 0 P −λ t
τ 2τ RQ A0 λ Q e Q

1
R 2 as if whole mass lies at centre E ∝ decreasing
At t = 2τ ; P = r2
RQ e
parabolic graph after sudden rise at surface of earth.
3
13. (2) Snell’s Law on 1st surface: = n sin r1 18. (a) A→1; B→1; C→1; D→4
2
In A, B, C no horizontal velocity is imparted to falling
3
sin r1 = . . .(i) water, so d remains same.
2n
In D, since its free fall, aeff = 0
3 4n 2 − 3
⇒ cos r1 = 1 − 2 =
4n 2n ∴ Liquid won’t fall with respect to lift.

  
Mock Test-2 19
JEE-MAIN: PHYSICS MOCK TEST-2

SECTION 1 (Multiple Choice Question) 5. The total energy of a particle executing simple harmonic
αZ motion is:
α −
1. In the relation p = e kθ
p is pressure, Z is distance, k
β a. ∝ x b. ∝ x2
is Boltzmann constant and θ is the temperature. The c. independent x d. ∝ x1/2
dimensional formula of α will be: 6. Two satellites A and B go around a planet P in circular
−2 2
a. [ MLT ] b. [ML T] orbits having radii 4R and R respectively. If the speed of
c. [ML T ] 0 −1
d. [M 0 L2 T −1 ] satellite A is 3v, the speed of satellite B will be:
a. 12 v b. 6 v
2. A small block is shot into each of the four tracks as shown
4 3
below. Each of the tracks rises to the same height. The c. v d. v
3 2
speed with which the block enters the track is the same in
all cases. At the highest point of the track, the normal 7. The following diagram shows three soap bubbles A, B and
reaction is maximum in C prepared by blowing the capillary tube fitted with stop
cocks, S1, S2 and S3. With stop cock S closed and stop
cocks S1, S2 and S3 opened.
v v
a. b. C
S
S1 S2
S3
v v A B
c. d.
3. A particle, which is constrained to move along x-axis, is a. B will start collapsing with volumes of A and C
subjected to a force in the same direction which varies increasing
with the distance x of the particle from the origin as b. C will start collapsing with volumes of A and B
F ( x ) = − kx + ax 3 . Here, k and a are positive constants. increasing
For x ≥ 0, the functional form of the potential energy c. C and A both will start collapsing with the volume of B
U(x) of the particle is increasing
U(x) U(x) d. Volumes of A, B and C will becomes equal at
equilibrium
a. x b. x 8. A steel wire of length 20 cm and uniform cross-section
1 mm2 is tied rigidly at both the ends. The temperature of
the wire is altered from 40°C to 20°C. What is the
U(x) U(x) magnitude of force developed in the wire? (Coefficient of
linear expansion for steel, α = 1.1 × 10–5 /°C and Y for
c. x d. x steel is 2.0 × 1011 N/m 2 ).
a. 2.2 × 106 N b. 16 N
4. A carpenter has constructed a toy as c. 8 N d. 44 N
shown in the adjoining figure. If the O2 2R
9. The intensity level of two waves of same frequency in a
density of the material of the sphere given medium are 20 dB and 60 dB. Then the ratio of
is 12 times that of cone, the position their amplitudes is:
of the center of mass of the toy is 2R a. 1 : 4 b. 1 : 16
given by. O1 c. 1 : 104 d. 1 : 100
a. at a distance of 2R from O
O
b. at a distance of 3R from O 10. The specific heat of a substance varies as (2t2 + t) ×10–3
2R
c. at a distance of 4R from O cal/g °C. What is the amount of heat required to raise the
d. at a distance of 5R from O temperature of 100 g of substance through 20°C to 40°C.
20 Physics
a. 37.9 kilocal b. 3.79 kilocal 17. The intensity ratio at a point of observation due to two
c. 379 kilocal d. 82 cal coherent waves is 100 : 1. The ratio between their
11. A real gas behaves like an ideal gas if its amplitudes is:
a. pressure and temperature are both high a. 1 : 1 b. 1 : 10
c. 1 : 100 d. 10 : 1
b. pressure and temperature are both low
c. pressure is high and temperature is low 18. An electromagnetic wave propagating along north has its
d. pressure is low and temperature is high electric field vector upwards. Its magnetic field vector
points towards
12. The plates of a parallel plate capacitor of capacity of 50µF
a. north b. east
are charged by a battery to a potential of 100 volt. The
c. west d. downwards
battery remains connected the plates are separated from
each other so that the distance between them is doubled. 19. The ionisation energy of 10 times ionised sodium atom is
How much is the energy spent by battery in doing so? a. 13.6 eV b. 13.6 × 11 eV
a. 25 × 10–2 J b. –12.5 × 10–2 J 13.6
–2
c. eV d. 13.6 × (11)2 eV
c. – 25 × 10 J d. 12.5 × 10–2 J 11
13. The magnetic moment produced in a substance of 1gm is 20. The example of nuclear fusion is
6 × 10–7 ampere-metre2. If its density is 5 gm/cm3, then a. Formation of barium and krypton from uranium
b. Formation of helium from hydrogen
the intensity of magnetisation in A/m will be
c. Formation of plutonium 235 from uranium 235
a. 8.3 × 106 b. 3.0
d. Formation of water from hydrogen and oxygen
c. 1.2 × 10–7 d. 3 × 10–6
14. An ionized gas contains both positive and negative ions. If SECTION 2 (Numeric Value Question)
it is subjected simultaneously to an electric field along the 21. A force of 5 N acts on a particle along a direction making
+x direction and a magnetic field along the +z direction, an angle of 60° with vertical. Its vertical component be:
then a. 10 N b. 3 N
a. Positive ions deflect towards +y direction and negative c. 4 N d. 2.5 N
ions towards –y direction 22. A neutral water molecule (H2O) in it's vapor state has an
b. All ions deflect towards +y direction electric dipole moment of magnitude 6.4 ×10–30 C–m.
c. All ions deflect towards –y direction How far apart are the molecules centres of positive and
d. Positive ions deflect towards –y direction and negative negative charge?
ions towards +y direction a. 4 m b. 4 mm
c. 4 µm d. 4 pm
15. A hundred turns of insulated copper wire are wrapped
around an iron cylinder of area 1 × 10–3 m2 are connected 23. A cell of e.m.f. 1.5 V having a finite internal resistance is
to a resistor. The total resistance in the circuit is 10 ohms. connected to a load resistance of 2 Ω. . For maximum
If the longitudinal magnetic induction in the iron changes power transfer, the internal resistance of the cell in ohms
from 1 Wb m–2, in one direction to 1 Wb m −2 in the should be:
opposite direction, how much charge flows through the a. 4 b. 0.5
circuit? c. 2 d. None of these
a. 2 × 10–2 C b. 2 × 10–3 C 24. Two plane mirrors are at 45° to each other. If an object is
c. 2 × 10–4 C d. 2 × 10–5 C placed between them, then the number of images will be
a. 5 b. 9
16. A resistor R, an inductor L and a capacitor C are
c. 7 d. 8
connected in series to an oscillator of frequency n. If the
resonant frequency is nr, then the current lags behind 25. A photon of energy 8 eV is incident on a metal surface of
Threshold frequency 1.6×1015Hz. The kinetic energy of the
voltage, when
photoelectrons emitted (in eV) (Take h = 6 × 10 −34 J − s)
a. n = 0 b. n < nr
c. n = nr d. n > nr a. 1.6 b. 6
c. 2 d. 1.2
Mock Test-2 21
JEE ADVANCE PAPER-I

SECTION 1 (Maximum Marks: 24) a. θ = 45 °


MCQs with one or more than one correct answer b. θ > 45 ° and a frictional force acts on the block towards P.
1. A piece of wire is bent in the shape of a parabola y = kx 2
c. θ < 45 ° and a frictional force acts on the block towards
(y-axis vertical) with a bead of mass m on it. The bead can d. a frictional force acts on the block towards
slide on the wire without friction. It stays at the lowest
point of the parabola when the wire is at rest. The wire is 5. In the figure, a ladder of mass m is shown leaning against
now accelerated parallel to the x-axis with a constant a wall. It is in static equilibrium making an angle θ with
acceleration a. The distance of the new equilibrium the horizontal floor. The coefficient of friction between
position of the bead, where the bead can stays at rest with
the wall and the ladder is µ1 and that between the floor
respect to the wire, from the y-axis is
and the ladder is µ2. The normal reaction of the wall on
a a 2a a
a. b. c. d. the ladder is N1 and that of the floor is N2. If the ladder is
gk 2 gk gk 4 gk
about to slip, then
2. A block of mass m is placed on a surface with a vertical
µ11
x3
cross-section given by y = . If the coefficient of friction
6
is 0.5, the maximum height above the ground at which the
block can be placed without slipping is:
θ
1 1
a. m b. m µ2
3 2
1 2
c. m d. m mg
6 3 a. µ1 = 0, µ 2 ≠ 0 and N2 tan θ =
2
3. A ball moves over a fixed track as shown in the figure.
mg
From A to B the ball rolls without slipping. Surface BC is b. µ1 ≠ 0, µ 2 = 0 and N1 tan θ =
2
frictionless. KA, KB and KC are kinetic energies of the ball
at A, B and C, respectively. Then mg
c. µ1 ≠ 0, µ 2 ≠ 0 and N1 =
C 1 + µ1 µ 2
A
mg
hc d. µ1 = 0, µ 2 ≠ 0 and N1 tan θ =
hA 2
B
6. Two small particles of equal masses start moving in
a. hA > hC ; K B > K C b. hA > hC ; K C > K A
opposite directions from a point A in a horizontal circular
c. hA = hC ; K B = K A d. hA < hC ; K B > K C orbit. Their tangential velocities are v and 2v
respectively, as shown in the figure. Between collisions,
4. A small block of mass of 0.1 kg lies on a fixed inclined
plane PQ which makes an angle θ with the horizontal. A the particles move with constant speeds. After making
horizontal force of 1 N acts on the block through its centre how many elastic collisions, other than that at A, these
of mass as shown in the figure. The block remains two particles will again reach the point A?
stationary if (take g = 10 m/s2). v A 2v
Q

1N

θ a. 4 b. 3
O P
c. 2 d. 1
22 Physics
SECTION 2 (Maximum Marks: 24) 12. A cylindrical vessel of height 500 mm has an orifice
Numerical value answer type questions (small hole) at its bottom. The orifice is initially closed
and water is filled in it up to height H. Now the top is
7. The densities of two solid spheres A and B of the same completely sealed with a cap and the orifice at the bottom
is opened. Some water comes out from the orifice and the
radii R vary with radial distance r as ρ A (r ) = k  r  and
 R  water level in the vessel becomes steady with height of
5 water column being 200 mm. Find the fall in height
r
ρ B (r ) = k  , respectively, where k is a constant. (in mm) of water level due to opening of the orifice.
R
The moments of inertia of the individual spheres about [Take atmospheric pressure = 1.0 × 105 N / m 2 , density of
axes passing through their centres are IA and IB, water = 1000 kg/m3 and g = 10 m/s2. Neglect any effect of
IB n surface tension.]
respectively. If = , the value of n is
I A 10 13. Consider two solid spheres P and Q each of density 8 gm

8. Gravitational acceleration on the surface of a planet is cm−3 and diameters 1 cm and 0.5 cm, respectively. Sphere

6 P is dropped into a liquid of density 0.8 gm cm−3 and


g, where g is the gravitational acceleration on the viscosity η = 3 poiseulles. Sphere Q is dropped into a
11
surface of the earth. The average mass density of the liquid of density 1.6 gm cm −3 and viscosity η = 2
1 poiseulles. The ratio of the terminal velocities of P and Q
planet is times that of the earth. If the escape speed on
3 is:
the surface of the earth is taken to be 11 kms–1, the escape
14. A 0.1 kg mass is suspended from a wire of negligible
speed on the surface of the planet in kms–1 will be
mass. The length of the wire is 1m and its cross-sectional
area is 4.9 × 10−7 m 2 . If the mass is pulled a little in the
9. A bullet is fired vertically upwards with velocity v from
the surface of a spherical planet. When it reaches its vertically downward direction and released, it performs
maximum height, its acceleration due to the planet’s simple harmonic motion of angular frequency 140 rad

gravity is 1/ 4th of its value at the surface of the planet. If s −1. If the Young’s modulus of the material of the wire is
n × 109 Nm −1 , the value of n is
the escape velocity from the planet is vesc = v N , then the
value of N is (ignore energy loss due to atmosphere) SECTION 3 (Maximum Marks: 12)

10. The ends Q and R of two thin wires, PQ and RS, are Paragraph based questions (2 paragraphs, each having 2
soldered (joined) together. Initially each of the wires has a MCQs with one correct answer only)
length of 1m at 10°C. Now the end P is maintained at Paragraph for Question No. 15 to 16
10°C, while the end S is heated and maintained at A small block of mass 1 kg is released from rest at the top of a
400 °C. The system is thermally insulated from its rough track. The track is circular arc of radius 40 m. The block
surroundings. If the thermal conductivity of wire PQ is slides along the track without toppling and a frictional force
twice that of the wire RS and the coefficient of linear acts on it in the direction opposite to the instantaneous velocity.
thermal expansion of PQ is 1.2 ×10−5 K −1 , the change in The work done in overcoming the friction upto the point Q, as
length of the wire PQ is: shown in the figure, below, is 150 J. (Take the acceleration due
to gravity, g = 10 m / s −2 ).
11. Two soap bubbles A and B are kept in a closed chamber
where the air is maintained at pressure 8 N/m2. The radii
R
of bubbles A and B are 2 cm and 4 cm, respectively. P
30°
Surface tension of the soap-water used to make bubbles is
n
0.04 N/m. Find the ratio B , where nA and nB are the
nA Q R

number of moles of air in bubbles A and B, respectively.


[Neglect the effect of gravity.]
Mock Test-2 23
15. The speed of the block when it reaches the point Q is Paragraph for Question No. 17 to 18
a. 5 ms −1 b. 10 ms − 1 Two waves y1 = A cos(0.5 π x − 100π t ) y2 = A cos
c. 10 3 m s − 1 d. 20 ms −1 (0.4 π x − 92π t ) are travelling in a pipe placed along x-axis.
17. Find the number of times intensity is maximum in time
16. The magnitude of the normal reaction that acts on the interval of 1 sec
block at the point Q is a. 4 b. 6
a. 7.5 N c. 8 d. 10
b. 8.6 N
18. Find wave velocity of louder sound
c. 11.5 N
a. 100 m/s b. 192 m/s
d. 22.5 N
c. 200 m/s d. 96 m/s

Space for rough work


24 Physics

JEE ADVANCE PAPER-II


SECTION 1 (Maximum Marks: 24) horizontal circular path about vertical axis. The maximum
MCQs with one or more than one correct answer tension that the string can bear is 324 N. The maximum
possible value of angular velocity of ball (in rad/s)
1. A person trying to lose weight by burning fat lifts a mass is
of 10 kg upto a height of 1 m 1000 times. Assume that the
potential energy lost each time he lowers the mass is
dissipated. How much fat will he use up considering the
L
work done only when the weight is lifted up? Fat supplies
3.8×10 7 J of energy per kg which is converted to
mechanical energy with a 20% efficiency rate (Take m
g = 9.8 ms−2 ) :
a. 2.45×10 –3 kg b. 6.45×10 –3 kg a. 9 b. 18
c. 27 d. 36
c. 9.89×10 –3 kg d. 12.89 ×10 –3 kg
6. A small mass m is attached to a massless string whose
2. If the resultant of all the external forces acting on a system other end is fixed at P as shown in the figure. The mass is
of particles is zero, then from an inertial frame, one can undergoing circular motion in the x-y plane with centre at
surely say that O and constant angular speed ω . If the angular
a. linear momentum of the system does not change in momentum of the system, calculated about O and P are
 
time denoted by LO and LP respectively, then.
b. kinetic energy of the system does not change in time z
c. angular momentum of the system does not change in P
time
d. potential energy of the system does not change in time
O m
3. A point mass of 1 kg collides elastically with a stationary
point mass of 5 kg. After their collision, the 1 kg mass
 
reverses its direction and moves with a speed of 2 a. LO and LP do not vary with time
ms −1 . Which of the following statement(s) is (are) correct  
b. LO varies with time while LP remains constant.
for the system of these two masses?  
a. Total momentum of the system is 3 kg ms −1 c. LO remains constant while LP varies with time
 
b. Momentum of 5 kg mass after collision is 4 kg ms −1 d. LO and LP both vary with time.
c. Kinetic energy of the centre of mass is 0.75 J SECTION 2 (Maximum Marks: 24)
d. Total kinetic energy of the system is 4 J Numerical value answer type questions
4. A solid sphere of mass M, radius R and having moment of
7. A body sends waves 500 mm long through medium A and
inertia about an axis passing through the centre of mass as
0.25 m long in medium B. If velocity of waves in medium
I, is recast into a disc of thickness t, whose moment of
A is 16 m/s, what is the velocity (in m/s) of waves in
inertia about an axis passing through its edge and
medium B?
perpendicular to its plane remains I. Then, radius of the
8. A light pointer fixed to one prong of a tuning fork touches
disc will be gently a smoked vertical plate. The fork is set vibrating
2R 2 and the plate is allowed to fall freely. Two complete
a. b. R
15 15 oscillations are traced when the plate falls through 40 cm.
4R R What is the frequency (in Hz) of the tuning fork?
c. d.
15 4 9. A water tank is 20 m deep. If the water barometer reads
5. A ball of mass (m) 0.5 kg is attached to the end of a string 10 m at that place, then what is the pressure at the bottom
having length (L) 0.5 m. The ball is rotated on a of the tank in atmosphere?
Mock Test-2 25
−4
10. There is a soap bubble of radius 2.4 × 10 m in air cylinder SECTION 3 (Maximum Marks: 12)

which is originally at the pressure of 105 N/m 2 . The air in Matching type questions with 4 options
the cylinder is now compressed isothermally until the 15. Work is defined as dot product of force and displacement
radius of the bubble is halved. The pressure of air in the  
W = ∫ dW = ∫ F ⋅ dS . It is a scalar quantity. The total work
cylinder now becomes n ×105 N/m 2 . The surface tension
done will depend on the displacement and the force,
of soap film is 0.08Nm –1 . Find the integer value of n. which may be constant or variable.
Thus in different situations of variable force applied in
11. A length of wire carries a steady current I. It is bent first
column I, the final expression for work done can be
to form a circular plane coil of one turn. The same length
expressed as in column II.
is now best more sharply to give double loop of smaller
Column I Column II
radius. If the same current I is passed, the ratio of the
(A) Force constant in magnitude acts 1
magnitude of magnetic field at the centre with its first 1. kx 2 cos θ
at constant angle θ with direction 2
value is. of motion
12. A current 1 amp is flowing in the sides of an equilateral (B) Force constant in magnitude acts F
2. sin θθ
triangle of side 4⋅5 ×10−2 m. the magnetic field at the at angle θ which varies as θ = kx K

centroid of the triangle in the unit of (10 −5 T) is. (C) Force varies with distance x as 3. Fx sin θ
F = k x but angle θ is constant
13. There are two infinite long parallel straight current (D) Force is constant in magnitude 4. zero
carrying wires, A and B separated by a distance r (Fig.) but changes in direction with
A B changing angle and always acts
along radius of circular path
I
a. A→1, B→2, C→3, D→4 b. A→3, B→2, C→1, D→4
P Q
c. A→4, B→3, C→2, D→1 d. A→2, B→1, C→4, D→3
r
16. Four charges Q1 , Q2 , Q3 and Q4 of same magnitude are fixed
I
r along the x-axis at x = −2a, − a, + a and +2a respectively.
A positive charge q is placed on the positive y-axis at a
distance b > 0 . Four options of the signs of these charges
The current in each wire is I. The ratio of magnitude of
magnetic field at points P and Q when points P and Q lie are given in column I. the direction of the forces on the
in the plane of wires is charge q is given in column II. Match column I with
column II and select the correct answer using the code
14. A length of wire carries a steady current I. It is bent first given below the lists.
to form a circular plane coil of one turn. The same length +q (0, b)
is now bent more sharply to give double loop of smaller
radius. If the same current I is passed, the ratio of the
magnitude of magnetic field at the centre with its first
value is Q1 Q2 Q3 Q4
(−2a,0) (−a,0) (+ a,0) (+2a,0)
Column I Column II
P a (A) Q1 , Q2 , Q3 , Q4 all positive 1. + x
O
(B) Q1 , Q2 positive; Q3 , Q4 negative 2. − x
(C) Q1 , Q4 positive; Q2 , Q3 negative 3. + y
2a (D) Q1 , Q3 positive; Q2 , Q4 negative 4. − y
26 Physics
a. A→3, B→1, C→4, D→2 c. A→2, B→3, C→4, D→1
b. A→4, B→2, C→3, D→1 d. A→3, B→4, C→1, D→2
c. A→3, B→1, C→2, D→4 18. To determine specific heat of different substances we use
d. A→4, B→2, C→1, D→3 different types of calorimeters. Can you match the type of
17. The vibration of body can be under various types of colorimeter used named in column I to the substance
forces. The vibration are classified mentioned in column I whose specific heat is determined by corresponding
under the conditions mentioned in column II. Match the calorimeter in column II.
type of vibrations in column I with conditions in Column I Column II
column II. (A) Regnault’s 1. To determine specific
Column I Column II calorimeter heat of solids at very
(A) Free vibrations 1. A body vibrating in low temperatures
viscous medium (B) Joly’s differential 2. To determine specific
(B) Forced vibrations 2. A body vibrating under calorimeter heat of solids or gases
its natural restoring force at constant pressures
(C) Resonant vibrations 3. A body vibrating under (C) Calender and 3. Used to measure specific
the influence of another Barmer’s calorimeter heat at constant volume
vibrating body (D) Nernst’s vacuum 4. Specific heat of liquids
(D) Damped vibrations 4. A body vibrating with calorimeter and gases at constant
its natural frequency pressure
under the influence of a. A→1, B→2, C→3, D→4
another vibrating body
b. A→2, B→3, C→4, D→1
of same frequency
c. A→3, B→4, C→1, D→2
a. A→1, B→2, C→3, D→4
d. A→4, B→3, C→2, D→1
b. A→4, B→3, C→2, D→1

Space for rough work


Mock Test-2 27
ANSWER & SOLUTIONS 2k
U ( x ) = Negative for x >
JEE-Main a
From the given function we can see that
1. 2. 3. 4. 5. 6. 7. 8. 9. 10.
F = 0 at x = 0 i.e., slope of U – x graph is zero at x = 0.
a a d c c b c d d a
11. 12. 13. 14. 15. 16. 17. 18. 19. 20. 4. (c) If the density of cone be ρ, then its mass will be
1 16
m1 = π ( 2 R ) ( 4 R ) ρ = π R 3 ρ
2
d a b c a d d b d c
3 3
21. 22. 23. 24. 25.
h 4R
d d c c c And its centre of mass will be at a height = =R
4 4
[GM] [M −1L3 T −2 M]  kθ  From O on the line of symmetry, i.e., y1 = R
1. (a) [V] = = = [L2 T −2 ] ⇒ [ α] =  
[r ] [L] Z Similarly the mass of the sphere
α  4
m2 = π R3 (12 ρ ) = 16π R 3 ρ = 3m1
Further [ p] =  β  3
 
And its centre of mass will be at its centre O 2 , i.e.,
α   kθ 
∴ [β] =  p  =  Zp  y2 = 4 R + R = 5R (from O).
   
y
Dimensions of k θ are that to energy.
2R O2 m2 O2
[Kθ] [ML2 T −2 ]
Hence, [ α ] = = = [MLT −2 ] C
[ z] [L]
2. (a) Since, the block rises to the same heights in all the
four cases, from conservation of energy, speed of the block 4R
at highest point will be same in all four cases. Say it is v0 O1 m1 O1
2
mv
Equation of motion will be N + mg = 0 O
O
R 2R
mv02 Now treating the sphere and cone as point masses with
or N = − mg
R their masses concentrated at their centres of mass
R (the radius of curvature) in first case is minimum. respectively and taking the line of symmetry as y-axis
Therefore, normal reaction N will be maximum in first with origin at O, the centre of mass of the toy is given by
case. m y + m2 y2 m1 × R + 3m1 × 5 R
YCM = 1 1 = = 4R
v0 m1 + m2 m1 + 3m1
i.e., centre of mass of the toy is at a distance 4R from O on
the line of symmetry, i.e., at the apex of the cone.
N + mg 5. (c) The total energy of a particle in simple harmonic
Note: In the question it should be mentioned that all the motion is constant.
four tracks are frictionless. Otherwise, v0 will be different GM
6. (b) Orbital speed of a satellite v0 =
in different tracks. r
dU vB r 4R
3. (d) F = − ⇒ = A = =2
dx vA rB R
x

or U ( x) = − (−kx + ax ) dx vB = 2v A = 2 × ( 3v ) = 6v
3
∴ dU = − F . dx ⇒
0

kx 2 ax 4 7. (c) Excess pressure inside soap bubble is inversely


U ( x) = −
2 4 proportional to the radius of bubble, i.e., ∆P ∝ 1/r′. This
2k means that bubbles A and C posses greater pressure inside
U ( x ) = 0 at x = 0 and x =
a it than B. So the air will move from A and C, towards B.
28 Physics
8. (d) F = Y A α∆θ
∆ dφ nAdB 100 × 1× 10 × 2 −3
15. (a) dQ = = = = 2 × 10−2C
= ( 2.0 × 10 11
) × (1×10 ) × (1.1×10 ) ( 40 − 20) = 44 N
−6 −5 R R 10
16. (d) The current will lag behind the voltage when reactance
I of inductance is more than the reactance of condenser.
9. (d) L2 − L1 = 10log10 2
I1 1 1
Thus, ω L > or ω >
I2 ωC LC
⇒ 60 − 20 = 10log10
I1 1
or n> or n > nr
2π LC
I2 a2
= 104 ⇒ 22 = 104 where, nr = resonant frequency.
I1 a1
a2 a1 I1 100 10
⇒ = 100 ⇒ a1 : a2 = 1 : 100 17. (d) I ∝ a 2 ⇒ = = =
a1 a2 I2 1 1

10. (a) dQ = mc dt 18. (b) The direction of propagation of electromagnetic wave


 
t2 is given by the direction of ( E × B ). Here, the em wave is
Q = ∫ mc dt propagating along north. The electric field vector is acting
t1
upwards, so the magnetic field vector will point towards
t2 = 40 40
−1  2t t2 
3
= ∫ 100 × ( 2t 2
+ t ) × 10 −3
dt = 10  + 
east.
t1 = 20  3 2  20
19. (d) (Eion)Na = Z 2 ( Eion ) H = (11) 213.6 eV
2 1 2
= 10 −1  ( 40 ) − ( 20 )  + ( 40 ) − ( 20 ) 
3 3
20. (c) Fast neutrons can escape from the reaction. So as to
3   2 
proceed the chain reaction. Slow neutrons are best.
= 3.79 ×103 cal = 3.79 kilocal
21. (d) The component of force in vertical direction
11. (d) A real gas behaves like an ideal gas at low pressure y
and high temperature.
F cos 60o


F
12. (a) When separation between the plates is doubled the 60 o

capacitance becomes one half i.e., C ′ = 25 µFF


x
F sin 60o
Energy spent by battery = qV
1
= ( C ′V )V = C ′V 2 = F cos θ = F cos 60° = 5 ×
= 2.5 N
2
= 25 ×10−6 × (100 ) = 25 ×10−2 J
2
22. (d) There are 10 electrons and 10 protons in a neutral
water molecule.
M M
13. (b) I = = , So it's dipole moment is p = q ( 2l ) = 10e(2l )
V mass/density
Hence length of the dipole i.e,. distance between centres
Given mass = 1gm = 10−3 kg of positive and negative charges is
5 ×10−3 kg p 6.4 × 10 −20
And density = 5gm / cm3 = −2 3
= 5 ×103 kg / m3 2l = = = 4 × 10 −12 m = 44 pm
(10 ) m 3 10e 10 × 1.6 × 10 −19

6 × 10 −7
× 5 × 10 3 23. (c) For maximum power Rext = Rint = 2 Ω
Hence I = −3
=3
10 360°
24. (c) n = −1 = 7
14. (c) As the electric field is switched on, positive ion will 45°
start to move along positive x-direction and negative ion
25. (c) W = hv0 = 6 ×10−34 ×1⋅ 6 ×1015 joule
along negative x-direction. Current associated with motion
of both types of ions is along positive x-direction. 6 × 10−34 × 1 ⋅ 6 × 1015
= eV = = 6 eV
According to Fleming's left hand rule force on both types 1 ⋅ 6 × 10−19
of ions will be along negative y-direction. ∴ Ek = hv − W = 88eV eV−–6 eV==22eV
6 eV eV
Mock Test-2 29
JEE Advance Paper -I If θ > 45° ; mg sin θ > F cos θ
1. 2. 3. 4. 5. 6. 7. 8. 9. 10. ∴ Friction acts towards Q.
b c a,b,c a,c c,d c 6 3 2 7 If θ < 45 ° F cos θ > mg sin θ
11. 12. 13. 14. 15. 16. 17. 18. ∴ Friction acts towards P.
6 6 3 4 b a a c
5. (c, d) Condition of translational equilibrium N1 = µ2 N 2
a mg
1. (b) tan θ = y
⇒ N 2 = µ1 N1 = Mg , Solving N2 =
g 1 + µ1µ2
N cos θ
dy N
µ2 mg
tan θ = = 2kx θ ⇒ N1 = Applying torque equation about corner (left)
dx N
1 + µ1 µ2
ma sin θ
a
⇒ x= θ x point on the floor
2 gk ma

⇒ mg cos θ = N1ℓ sin θ + µ1 N1ℓ cos θ
2. (c) mg sin θ = µ s mg cos θ [when partied is just balanced] 2
dy  x 2  1 − µ1µ 2
⇒ tan θ = us ⇒ tan θ = = Solving tan θ =
dx  2  2µ 2

6. (c) Velocity will exchange after each collision


V A 2V
fs
N
Mg cos θθ
Mg sin θ 2π / 3 2π / 3
2V V
st 2π / 3
1 collision nd
2 collision
x2 1
∴ = 0.5 ⇒ x = 1 ⇒ y = V 2V
2 6
2
7. (6) I = ∫ ρ 4π r 2 r 2 dr
3. (a, b, c) EA = mghA + K A 3
⇒ EB = K B ⇒ I A ∝ ∫ ( r )( r 2 )( r 2 ) dr
⇒ EC = mghC + K C ⇒ I B ∝ ∫ ( r 5 )( r 2 )( r 2 ) dr
Using conservation of energy E A = EB = EC IB 6
∴ =
⇒ K B > KC I A 10
⇒ KB > K A
g' 6 ρ' 2
K − KA 8. (3) = ; =
⇒ Mg (hA − hC ) + ( K A − K C ) = 0 ⇒ hA − hC = C g 11 ρ 3
Mg
4. (a, c) f = 0 (θ = 45°) R' 3 6
Hence, =
R 22
So, for block to be at rest F cos θ = mg sin θ
V 'esc R '2 ρ ' 3
1 ⇒ ∝ = ⇒ v 'esc = 3km/s.
LHS F cos θ = 1 × cos 45° = Vsec R2 ρ 11
2
1 1 9. (2) At height R from the surface of planet acceleration due
RHS mg sin θ = 0.1 × 10 × =
2 2 1
to planet’s gravity is th in comparison to the value at the
F cos θ 4
surface
F F GMm 1 2 GMm GMn 1 2
So, − + mv = − and − + mvesc = 0
θ R 2 R+R R 2
mg
∴ vesc = v 2
{LHS = RHS)
30 Physics
10. (1) 2
v
1m Q, R 1m S
⇒ 200 − 150 =
P 2
10°C 140°C ⇒ v = 10 m/s
400°C
2 k , α1 k ,α 2
mv 2
d ℓ = dxα1 (θ − 10) ∆ℓ = ∫ d ℓ 16. (a) N − mg cos 60° =
R
 θ − 10  130 5
∵  = ⇒ θ = 10 + 130x ⇒ N = 5+ = 7.5 N
x  1 2
1
x2 17. (a) f1 − f 2 = 4 s −1
∆ℓ = ∫ (130 x)α1dx ∆ℓ = 130α1
0
2
1 18. (c) v1 = v2 = 200 m
m/s
/s
∆ℓ = 130 × 1.2 × 10−5 × = 0.78 mm ≃ 1
2
4T JEE Advance Paper -II
11. (6) PA = P0 + = 16N / m2
RA 1. 2. 3. 4. 5. 6. 7. 8. 9. 10.
d a a,c a d c 8 7 3 8
A B
11. 12. 13. 14. 15. 16. 17. 18.
RA RB R
2 cm 4 4 8 4 b a c a
4 cm
8 N / m2
1. (d) Let m mass of fat is used.
3 1
4T nB PB  RB  m(3.8 ×107 ) = 10(9.8)(1)(1000)
⇒ PB = P0 + = 12N / m2 ⇒ =   =6 5
RB n A PA  R A 
9.8 × 5
12. (6) P = P0 − ρ gh = 98 × 10 3 N / m 2 m= = 12.89 × 10 −3 kg
3.8 ×103
⇒ 0 0 = PV
PV 2. (a) Linear momentum remains constant if net external
force on the system of particles is zero.

500 mm 3. (a, c) By conservation of linear momentum


H 500 mm u = 5v − 2 . . .(i)
200 mm By Newton’s experimental law of collision
u =v+2 . . .(ii)
⇒ 105 [ A(500 − H )] = 98 × 103 [ A(500 − 200)]
5 kg
1 kg
⇒ H = 206 mm u
Level fall = 206 – 200 = 6 mm before collision

13. (3) 6πη rv + ρ LVg = ρ 0Vg 1 kg 5 kg


V
VP (ρ PVP − ρ LVP ) g 6πηQ rQ
= × after collision
VQ 6πηP rP ( ρQVQ − ρ LVQ )
Using (i) and (ii) we have v = 1m/s and u = 3 m/s
r 3 (8 − 0.8) rQ .ηQ
= P × 3 Kinetic energy of the centre of mass
ηP .rP (8 − 1.6) rQ
1
2 = msystem v 2cm = 0.75J
 r   ηQ   7.2  7.2 2 2
=  P  ×  ×  = 4× × =3
 rQ   ηP   6.4  6.4 3 2 3 2R
4. (a) MR 2 = Mr 2 ⇒ r =
5 2 15
14. (4) ω = YA
mL
5. (d) mωmax
2
r = Tmax
15. (b) Using work energy theorem
1 2 Tmax 324
mg R sin 30° + W f = mv ⇒ ωmax = = = 1296 = 36 rad/s
2 mr 0.5 × 0.5
Mock Test-2 31

6. (c) LO = (mω r 2 )kˆ 4S  8S 
P2 = P′ + =  P′ + 
( R / 2)  R 

θ LO  Using Boyle’s law, we have, P1 V1 = PV
2 2
LP
 4S   8S  V1
or P+ V1 =  P′ + 
 R   R 8
 4S 1  8S 
Direction of LP varies with time as its direction will be or P+ =  P′ + 
R 9 R
perpendicular to string i.e. changing with time.
  24 S 24 + 0.08
LO = Constant , LP = Variable or P′ = 8P + = 8 ×105 + = 8.08 ×105 = n ×105
R 2.4 ×10−4
∴ n = 0.08 ≈ 8.
7. (8) Here, λ1 = 500 mm = 0.5m
λ2 = 0.25 m;υ1 = 16 m/s,υ2 = ? 11. (4) When wires is taken in the form of one turn circular
l
As frequency of body is fixed, say n, therefore, coil, then length, l = 2π r or r = , n = 1 Magnetic field
υ1 = λ1 n;υ2 = λ2n 2π
induction at the centre of circular coil due to current I is
υ 2 λ2 0.25 1
⇒ = = = µ0 2π nI µ0 2π × 1× I µ0π I
υ1 λ1 0.5 2 B= = =
4π r 4π (l / 2π ) l
υ1 16
⇒ υ2 = = = 8m/s When wire is taken in the form of double loop, then
υ2 2
I
l = 2 × 2π r1 or r1 = and n = 2
8. (7) Here, .m = 2, h = 40cm, n = ? 4π
Time taken by the plate to fall down, ∴ Magnetic field induction at the centre of the circular coil.
2h 2 × 40 2 µ0 2π × n × I µ0 2π × 2 × I πI
t= = = sec B1 = = = 4 × µ0
g 980 7 4π r1 4π (l / 4π ) l
m 2 B1
Frequency of fork, n = = = 7Hz. ∴ =4
t 2/7 B

9. (3) The pressur at the bottom, P = P0 + h ρ g 12. (4) Refer Fig.


A
Where, p0 = atmospheric pressure, therefore,
I
hρ g
P =1+ atmosphere
P0
O
20 ρ g I
= 1+ = 3 atmosphere.
10 ρ g 60° 60°
r
C
B D
10. (8) Here, p = 105 N/m 2 , R = 2.4 × 10 −4 m; S = 0.08 N/m I
a
Initial pressure inside the soap bubble in air cylinder, The magnetic field induction at the centriod O due to
4S current I through one side BC of the triangle will be
P1 = P + .
R µ0 I
B1 = (sin θ1 + sin θ 2 )
Let V1 be the initial volume of the soap bubble; 4π r
4 It will be acting perpendicular to the plane of triangle
V1 = π R3
3 upwards. Total magnetic field induction at O due to
After compression, volume of the soap bubble; current through all the three sides of the triangle will be
4 R V
3 3µ 0 I
V2 = π   = 1 B = 3B1 = [sin θ1 + sin θ 2 ]
3 2 8 4π π
BD a/2
If is the air pressure after compression in the cylinder, Here, I = 1 A, θ1 = 60° = θ 2 and r = OD = =
then pressure inside the bubble is tan 60° 3
32 Physics
r r r r
a −2
4 ⋅ 5 × 10 ∴ Displacements S = x while angle θ between F and x is
= 2
= m.
3 2 3 constant
−7 1 dθ dθ
∴ B = 3 × 10 × × [sin 60° + sin 60°] θ = kx, hence = K or dx =
(4 ⋅ 5 × 10 −2 / 2 3) dx k
r uur
On solving, B = 4 × 10 −5 T. ∴ ∫ dW = ∫ F ⋅ dS
13. (8) Magnetic field at P due to currents in two wires will be r uur dθ F
acting perpendicular to the plane of wires, upwards and is = ∫ F ⋅ dx = ∫ F ⋅ cos θ ⋅ == sin θ
K K
given by. r uur uur uur
W = ∫ dW = ∫ F ⋅ dx = ∫ Kx ⋅ dx
µ0 2 I µ 2I 2µ I
BP = + 0 = 0
4π ( r / 2 ) 4π ( r / 2 ) πr 1
= K ∫ x dx cos θ == Kx 2 cos θ
2
Magnetic field at Q due to current in A is perpendicular to r r
the plane of wire upwards and due to current in B is F ⊥ x at every point
r uur
perpendicular to the plane of wire downwards and is given ∴ dW = F ⋅ dx = 0
µ0 2 I µ 0 2 I µ 0 I Q θ = 90° = Fdx cos90° = 0
by BQ = + =
4π 2r 4π r 4π r
16. (a) A→3; B→1; C→4; D→2
BP ( 2µ0 I / π r )
∴ = = 8.
BQ ( µ0 I / 4π r ) 17. (c) A→2, B→3, C→4, D→1
When a body is disturbed from its mean position and left
14. (4) When wire is taken in the form of one turn circular to vibrate under restoring force, e.g., A tunning force
coil, then length l = 2π r struck with rubber pad, vibrations are called free or
l natural vibrations. When a body is mode to vibrate with
or r= , n =1
2π another vibrating body placed nearby, e.g., stem of a
Magnetic field induction at the centre of circular coil due vibrating tunning fork when pressed on top of sonometer
to current I is makes the sonometer wire to vibrate having forced
µ0 2π nI µ0 2π × 1× I µ0π I vibrations. When a body is forced to vibrate by another
B= = =
4π r 4π ( l / 2π ) l vibrating body the vibrations are forced on other body.
But if frequency of forced vibrations is equal to the
When wire is taken in the form of double loop, then
natural frequency of forced body then vibrations of forced
l
l = 2 × 2π r1 or r1 = and n = 2 body are called resonant vibrations. When a body is
4π vibrating under the action of viscous force, gradually the
∴ Magnetic field induction at the centre of the circular coil, amplitude of vibration decreases because energy of
µ0 2π × n × I µ0 2π × 2 × I πI vibrating body is dissipated as work done against viscous
B1 = = = 4 × µ0
4π r1 4π ( l / 4π ) l force in the form of heat etc. and body gradually stops
B1 vibrating. Such vibratious are called damped vibrations.
⇒ = 4.
B 18. (a) A→1, B→2, C→3, D→4
15. (b) A→3, B→2, C→1, D→4 The particular names of calorimeters are after the names
r r of scientists who designed them for specific heat
W = F ⋅ S = Fx cos θ
measurements in different conditions.
Mock Test-3 33
JEE-MAIN: PHYSICS MOCK TEST-3
SECTION 1 (Multiple Choice Question) 7. A solid sphere of radius R made of material of bulk
ur
1. If A = 2iˆ + 4 ˆj − 5 kˆ the direction of cosines of the vector modulus K is surrounded by a liquid in a cylindrical
ur container. A massless piston of area A floats on the
A are:
surface of the liquid when a mass m is placed on the
2 4 −5 1 2 3
a. , and b. , and piston to compress the liquid, the fractional change in the
45 45 45 45 45 45
δR
4 4 3 2 5 radius of the sphere, is:
c. , 0 and d. , and R
45 45 45 45 45
mg mg
a. b.
2. A particle is projected vertically upwards and it is at a AK 3 AK
height h after 2 seconds and again after 10 seconds. The mg mg
c. d.
height h is: A 3 AK
a. 196 m b. 98 m
8. The wavelength of light observed on the earth, from a
c. 9.8 m d. 19.8 m
moving star is found to decreases by 0.05%. Relative to
3. An insect crawls up a hemispherical the earth, the star is:
surface very slowly (see the figure). a. moving away with a velocity of 1.5 ×105 m/s
α
The coefficient of friction between
b. coming closer with a velocity of 1.5 ×105 m/s
the surface and the insect is 1/3. If
the line joining the centre of the hemispherical surface to c. moving away with a velocity of 1.5 ×104 m/s
the insect makes an angle α is given d. coming closer with a velocity of 1.5 ×104 m/s
a. cot α = 3 b. tan α = 3
9. An ideal gas expands isothermally from a volume V1 to
c. sec α = 3 d. cosecα = 3
V2 and then compressed to original volume V1
4. An ideal spring with spring constant k is hung from the
adiabatically. Initial pressure is p1 and final pressure is
ceiling and a block of mass M is attached to its lower end.
The mass is released with the spring initially unscratched. p3 . The total work done is W. Then,
Then the maximum extension in the spring is a. p3 > p1 ,W > 0 b. p3 < p1 ,W < 0
4Mg 2Mg c. p3 > p1 ,W < 0 d. p3 = p1 ,W = 0
a. b.
k k
Mg Mg 10. At NTP one mole of diatomic gas is compressed
c. d. adiabatically to half of its volume ( γ = 1.40). The work
k 2k
done on the gas will be ( 2 )
0.4
5. A particle executes simple harmonic motion between = 1.40 :
x = − A and x = + A. The time taken for it to go from a. 1280 J b. 1610 J
0 to A/2 is T1 and to go from A/2 to A is T2 . Then: c. 1792 J d. 2025 J
a. T1 < T2 b. T1 > T2 11. Two identical conducting rods are first connected
c. T1 = T2 d. T1 = 2T2 independently to two vessels, one containing water at
100°C and the other containing ice at 0ºC. In the second
6. A large number of liquid drops each of radius r coalesce
case, the rods are joined end to end connected to the same
to form a single drop of radius R. The energy released in
the process is converted into the kinetic energy of the big vessels. Let q1 and q2 gram per second be the rate of
drop so formed. The speed of the big is (Given surface melting of ice in the two cases respectively. The ratio
tension of liquid is T, density of liquid is ρ) q1 / q2 is
T 1 1  2T  1 1  1 2
a. − b. − a. b.
ρ  r R  ρ  r R  2 1
4 1
4T  1 1  6T  1 1  c. d.
c. − d. − 1 4
ρ  r R  ρ  r R 
34 Physics
12. The energy density (energy per unit volume) in an electric 19. The symbol given in figure represents
field caused by a point charge falls off with the distance E C
from the point charge as:
a. 1/ r b. 1/ r 2
B
c. 1/ r 3 d. 1/ r 4 a. NPN transistor
b. PNP transistor
13. The needle of a deflection galvanometer shows a
c. Forward biased PN junction diode
deflection of 60° due to a short bar magnet at a certain
d. Reverse biased NP junction diode
distance in tan A position. If the distance is doubled, the
deflection is SECTION 2 (Numeric Value Question)
 3  3 20. For television broadcasting, the frequency employed is
a. sin −1   b. cos −1  
 8   8  normally
a. 30–300 MHz b. 30–300 GHz
 3  3
c. tan −1   d. cot −1   c. 30–300 kHz d. 30–300 Hz
 8   8 
21. The potential difference V and current i flowing through
1
14 If power factor is in a series RL circuit R = 100 Ω . an appliance in an ac circuit are given by
2
V = 5cos ωt volt, i = 5sin ωt amp and power dissipated in
ac mains is used then L is
the appliance is
3
a. henry b. π henry a. 0 W b. 10 W
π
c. 5 W d. 2.5 W
π
c. henry d. None of these 22. A wire has a mass (03. ± 0.003) g, radius (0.5 ± 0.005) mm
2
and length (6 ± 0.06) cm. The maximum percentage error
15. The amplitude ratio of two superposing waves is 2 : 1.
in the measurement of its density is:
The ratio of maximum and minimum intensities is:
a. 1 b. 2
a. 1 : 1 b. 2 : 1
c. 3 d. 4
c. 4 : 1 d. 9 : 1
23. An aircraft with a wing-span of 40 m flies with a speed of
16. The electric and the magnetic field, associated with an
1080 km h–1 in the eastward direction at a constant
e.m. wave, propagating along the + z-axis, can be
altitude in the northern hemisphere, where the vertical
represented by
r r r r component of earth's magnetic field is 8.3 × 10−4 T. Then
a.  E = E0iˆ, B = B0 ˆj  b.  E = E0 kˆ, B = B0iˆ 
  the e.m.f. that develops between the tips of the wings is:
r r r r a. 0.5 V b. 0.35 V
c.  E = E0 ˆj , B = B0iˆ  d.  E = E0 ˆj , B = B0 kˆ 
  c. 1 V d. 2.1 V
17. If the wavelength of the first line of the Balmer series of 24. If an observer is walking away from the plane mirror with
hydrogen is 6561 Å, the wavelength of the second line of 3 m/sec. Then the velocity of the image with respect to
the series should be observer will be
a. 13122 Å b. 3280 Å a. 6 m/sec b. −6 m / sec
c. 4860 Å d. 2187 Å c. 12 m / sec d. 3 m / sec
11 11
18. In the nuclear reaction 6 C →5 B + β + X, what does
+
25. The eccentricity of earth’s orbit is 0.0167. The ratio of its
X stand for maximum speed in its orbit to its minimum speed is:
a. An electron b. A proton a. 2.507 b. 1.033
c. A neutron d. A neutrino c. 8.324 d. 1.000
Space for Rough Work
Mock Test-3 35

JEE ADVANCE PAPER-I


SECTION 1 (Maximum Marks: 24) length L and diameter 2d. The way these wires are
MCQs with one or more than one correct answer connected is given in the options. How much time in
minutes will it take to raise the temperature of the same
1. In a large building, there are 15 bulbs of 40 W, 5 bulbs of
amount of water by 40 K?
100 W, 5 fans of 80 W and 1 heater of 1 kW. The voltage
a. 4 if wires are in parallel
of the electric mains is 220 V. The minimum capacity of
b. 2 if wires are in series
the main fuse of the building will be:
c. 1 if wires are in series
a. 12 A b. 14 A
d. 0.5 if wires are in parallel
c. 8 A d. 10 A
SECTION 2 (Maximum Marks: 24)
2. When 5V potential difference is applied across a wire of
length 0.1 m, the drift speed of electrons is Numerical value answer type questions

2.5 × 10 −4 ms −1. If the electron density in the wire is 7. A current 1 amp is flowing in the sides of an equilateral
−3
8 ×10 m , the resistivity of the material is close to:
28 triangle of side 4.5 ×10−2 m . The magnetic field at the

a. 1.6 × 10 −8 Ω m b. 1.6 × 10 −7 Ω m centroid of the triangle in the units of (10−5 T ) is

c. 1.6 × 10 −6 Ω m d. 1.6 ×10−5 Ω m 8. If the maximum values of signal and carrier waves are 4 V
3. A galvanometer having a coil resistance of 100 Ω gives a and 5V respectively, the percentage of amplitude
full scale deflection, when a current of 1 mA is passed modulation is a × 10%. What is the value of a ?
through it. The value of the resistance, which can convert 9. A signal wave of frequency 4.5 kHz is modulated with a
this galvanometer into ammeter giving a full scale carrier wave of frequency 3.45 MHz. The bandwidth of
deflection for a current of 10 A, is: FM wave is kHz is
10. What is the maximum usuable frequency (in MHz) for
a. 0.01 Ω b. 2 Ω
E-layer of atmosphere having critical frequency 4 MHz,
c. 0.1 Ω d. 3 Ω
when the angle of incidence is 60°?
4. The temperature dependence of resistances of Cu and 11. A T.V. Tower has a height 100 m. In order to triple its
undoped Si in the temperature range 300–400 K, is best coverage range, the height of tower to be increased is
described by: a × 10 2 m. What is the integer value of a?
a. Linear increase for Cu, linear increase for Si
12. A microwave telephone link operating at the central
b. Linear increase for Cu, exponential increase for Si
frequency of 10 GHz has been established. If 2% of this is
c. Linear increase for Cu, exponential decrease for Si
available for microwave communication channel and each
d. Linear decrease for Cu, linear decrease for Si
telephone is allotted a bandwidth of 8 kHz, the number of
5. Two ideal batteries of emf V1 and V2 and three resistances telephone channels which can be simultaneously granted
R1, R2 and are R3 connected as shown in the figure. The is 2.5 × 10 a. What is the integer value of a ?
current in resistance R2 would be 13. A uniform circular disc of mass 1.5 kg and radius is
V1
zero if R1 initially at rest on a horizontal frictionless surface. Three
a. V1 = V2 and R1 = R2 = R3 R2 forces of equal magnitude F = 0.5 N are applied
b. V1 = V2 and R1 = 2R2 = R3 simultaneously along the three sides of an equilateral
triangle XYZ with its vertices on the perimeter of the disc
c. V1 = 2V2 and 2R1 = 2R2 = R3 (see figure). One second after applying the forces, the
R3 V2
d. 2V1 = V2 and 2R1 = R2 = R3 angular speed of the disc in rad s –1 is
F
6. Heater of an electric kettle is made of a wire of length L X
and diameter d. It takes 4 minutes to raise the temperature
of 0.5 kg water by 40 K. This heater is replaced by a new O
Y F
heater having two wires of the same material, each of Z
F
36 Physics
14. A horizontal circular platform of radius 0.5 m and mass 16. Consider two different metallic strips (1 and 2) of same
0.45 kg is free to rotate about its axis. Two mass less dimensions (lengths ℓ , width w and thickness d) with
spring toyguns, each carrying a steel ball of mass 0.05 kg carrier densities n1 and n2, respectively. Strip 1 is placed
are attached to the platform at a distance 0.25 m from the in magnetic field B1 and strip 2 is placed in magnetic field
centre on its either sides along its diameter (see figure). B2, both along positive y-directions. Then V1 and V2 are
Each gun simultaneously fires the balls horizontally and the potential differences developed between K and M in
perpendicular to the diameter in opposite directions. After strips 1 and 2, respectively. Assuming that the current I is
leaving the platform, the balls have the same for both the strips, the correct option(s) is(are)
horizontal speed of 9 ms −1 with a. If B1 = B2 and n1 = 2n2 , then V2 = 2V1
respect to the ground. The
b. If B1 = B2 and n1 = 2n2 , then V2 = V1
rotational speed of the platform in
c. If B1 = 2 B2 and n1 = n2 , then V2 = 0.5V1
rad s –1 after the balls leave the platform is
d. If B1 = 2 B2 and n1 = n2 , then V2 = V1
SECTION 3 (Maximum Marks: 12)
Paragraph for Question No. 17 to 18
Paragraph based questions (2 paragraphs, each having 2
The capacitor of capacitance C can be charged (with the help of
MCQs with one correct answer only)
a resistance R) by a voltage
Paragraph for Question No. 15 to 16 source V, by closing switch S1 V
In a thin rectangular metallic strip a constant current I flows R
while keeping switch S 2 open. S1
along the positive x-direction, as shown in the figure. The C
length, width and thickness of the strip are l, w and d, The capacitor can be
 connected in series with an
respectively. A uniform magnetic field B is applied on the L S2
strip along the positive y-direction. Due to this, the charge inductor ‘L’ by closing switch
carriers experience a net deflection along the z-direction. This S 2 and opening S1
results in accumulation of charge carriers on the surface PQRS 17. Initially, the capacitor was uncharged. Now, switch S1 is
and appearance of equal and opposite charges on the face closed and S2 is kept open. If time constant of this circuit
opposite to PQRS. A potential difference along the z-direction is τ, then
is thus developed. Charge accumulation continues until the
a. after time interval τ, charge on the capacitor is CV/2
magnetic force is balanced by the electric force. The current is
b. after time interval 2τ , charge on the capacitor is
assumed to be uniformly distributed on the cross-section of the
strip and carried by electrons. CV (1 − e −2 )
l y c. the work done by the voltage source will be half of the
I
•K heat dissipated when the capacitor is fully charged
W I
x
S R d. after time interval 2τ , charge on the capacitor is
d •M z
CV (1 − e −1 )
P Q
15. Consider two different metallic strips (1 and 2) of the 18. After the capacitor gets fully charged, S1 is opened and S2
same material. Their lengths are the same, widths are w1 is closed so that the inductor is connected in series with
and w2 and thicknesses are d1 and d2, respectively. Two the capacitor. Then
points K and M are symmetrically located on the opposite a. at t = 0, energy stored in the circuit is purely in the
faces parallel to the x-y plane (see figure). V1 and V2 are form of magnetic energy
the potential differences between K and M in strips 1 and b. at any time t > 0, current in the circuit is in the same
2, respectively. Then, for a given current I flowing
direction
through them in a given magnetic field strength B, the
c. at t > 0, there is no exchange of energy between the
correct statement(s) is(are)
inductor and capacitor
a. If w1 = w2 and d1 = 2d2, then V2 = 2V1
d. at any time t > 0, instantaneous current in the circuit
b. If w1 = w2 and d1 = 2d2, then V2 = V1
c. If w1 = 2w2 and d1 = d2, then V2 = 2V1 may V
C
d. If w1 = 2 w2 and d1 = d 2 , then V2 = V1 L
Mock Test-3 37
JEE ADVANCE PAPER-II
SECTION 1 (Maximum Marks: 24) µ 0 NI b µ 0 NI  b + a 
a. In   b. In  
MCQs with one or more than one correct answer 2(b − a )  a  2(b − a )  b – a 

1. The figure shows certain wire segments joined together to µ 0 NI  b  µ 0 NI  b+ a 


c. In   d. In  
form a coplanar loop. The loop is placed in a 2b  a  2b  b – a 
perpendicular magnetic field in the direction going into
4. A loop carrying current I lies in the x-y plane as shown
the plane of the figure. The magnitude of the field
in the figure. The unit vector k̂ is coming out of the
increases with time. I 1 and I 2 are the currents in the
plane of the paper. The magnetic moment of the current
segments ab and cd. Then, loop is
y
c d
a b
I
a x

a. I1 > I 2
b. I1 < I 2 π 
a. a 2 Ikˆ b.  + 1 a 2 Ikˆ
2 
c. I 1 is in the direction ba and I 2 is in the direction cd
π 
d. I1 is in the direction ab and I 2 is in the direction dc c. −  + 1 a 2 Ikˆ d. (2π + 1) a 2 Ikˆ
2 
2. A thin flexible wire of length L is connected to two 5. An infinitely long hollow conducting cylinder with inner
adjacent fixed points and carries a current I in the radius R/2 and outer radius R carries a uniform current
clockwise direction, as shown in the figure. When the density along its length. The magnitude of the magnetic
system is put in a uniform magnetic field of strength B 
field, | B | as a function of the radial distance r from the
going into the plane of the paper, the wire takes the shape
of a circle. The tension in the wire is axis is best represented by.

 
B B
a. b.
r r
R/2 R R/2 R

IBL IBL IBL


a. IBL b. c. d.
π 2π 4π 

B B
3. Along insulated copper wire is closely wound as a spiral c. d.
of 'N' turns. The spiral has inner radius 'a' and outer radius r r
'b'. The spiral lies in the X-Y plane and a steady current 'I' R/2 R R/2 R

flows through the wire. The Z component of the magnetic 6. A circular loop of radius 0.3 cm lies parallel to a much
field at the center of the spiral is
bigger circular loop of radius 20 cm. The centre of the
Y
small loop is on the axis of the bigger loop. The distance
between their centres is 15 cm. If a current of 2.0 A flows
I a
X
through the smaller loop, then the flux linked with bigger
b
loop is
a. 9.1 × 10 −11 weber b. 6 × 10 −11 weber
c. 3.3 × 10 −11 weber d. 6.6 × 10 −9 weber
38 Physics
SECTION 2 (Maximum Marks: 24) Column I Column II
Numerical value answer type questions (A) Boltzmann Constant 1. [ML2 T −1 ]

12
(B) Coefficient of viscosity 2. [ML−1T −1 ]
7. The isotope 5 B having a mass 12.014 u undergoes
(C) Plank Constant 3. [MLT −3 K −1 ]
β - decay to 12
6 C. 126 C has an excited state of the nucleus
(D) Thermal conductivity 4. [ML2 T −2 K −1 ]
( 126 C* ) at 4.041 MeV above its grounds state. If
12 12
a. A-4; B-1, 4; C- 4; D-2
5 B decays to 6 C* , the maximum kinetic energy of the
b. A-1; B- 1,4; C-4; D-4
β - particle in units of MeV is: c. A-4; B-1,4; C-4; D-2
( 1u = 931.5MeV / c2,, where c is the speed of light in d. A-1; B- 1,4; C-2; D-4
vacuum)
16. Column I give a list of possible set of parameters
8. A binary star consists of two stars A (mass 2.2Ms) and B measured in some experiments. The variations of the
(mass 11Ms), where Ms is the mass of the sun. They are parameters in the form of graphs are shown in Column II.
separated by distance d and are rotating about their centre Match the set of parameters given in Column I with the
of mass, which is stationary. The ratio of the total angular graph given in Column II.
momentum of the binary star to the angular momentum of Column I Column II
star B about the centre of mass is (A) Potential energy of a 1. y
simple pendulum (y axis)
9. Two homogeneous spheres A and B of masses m and 2m
as a function of
having radii 2a and a respectively are placed in contact.
displacement (x axis) O x
The ratio of distance of c.m. from first sphere to the
distance of c.m. from second sphere is: (B) Displacement(y axis) as a 2. y
function of time (x axis)
10. A non-uniform thin rod of length L is placed along X-axis
for a one dimensional
so that one of its ends is at the origin. The linear mass
motion at zero or constant O x
density of rod is λ = λ0 x ⋅ The centre of mass of rod
acceleration when the
divides the length of the rod in the ratio: body is moving along the
11. A sphere of mass 5 kg and diameter 2 m rotates about a positive x-direction
tangent. What is its moment of inertia? (C) Range of a projectile 3. y
(y-axis) as a function of its
12. A uniform rod of length 1 m and mass 0.5 kg rotates at
velocity (x-axis) when
angular speed of 6 rad/sec about one of its ends. What is
projected at a fixed angle O x
the KE of the rod?

13. A particle performing uniform circular motion has angular


(D) The square of the time 5. y
momentum L. When its angular velocity is doubled and
period (y-axis) of a simple
KE is also doubled, the new angular momentum becomes
pendulum as a function of
x times. What is x?
its length (x -axis) O x
14. The fundamental frequency of a closed organ pipe is equal
to first overtone frequency of an open organ pipe. If the a. A-4; B- 1,4; C- 4; D- 2
length of the open pipe is 36 cm, what is the length b. A-1; B- 1,4; C- 4; D- 4
(in cm) of the closed pipe? c. A-2; B- 1,4; C- 4; D- 2
d. A-1; B- 1,4; C- 2; D- 4
SECTION 3 (Maximum Marks: 12)
Matching type questions with 4 options 17. Motion is defined as rate of change of position. In the
motions described in column I, match the facts about that
15. Match Column I with Column II and select the correct
motion in column II for the changes in position vector.
answer using the codes given below the Columns:
Mock Test-3 39
Column I Column II of kinetic energy to
(A) Magnitude only 1. A coin dropped from potential energy of the
roof of house electron is
(B) Direction only 2. A coin thrown at any (B) The ratio of the kinetic 2. 2
angle with horizontal energy to the total energy
(C) Both in magnitude and 3. A coin held in your of an electron in a Bohr
direction hand orbit is
(D) Remains invariant 4. A coin in rotated in (C) In the lowest energy level h
3.
circular path with of hydrogen atom, the 2π
variable speed electron has the angular
a. A-1; B-2; C-3; D-4 momentum
b. A-4; B-2; C-3; D-1 (D) Ratio of the wavelengths 4. 5 : 27
c. A-1; B-4; C-2; D-3 of first line of Lyman
d. A-3; B-2; C-4; D-1 series and first line of
Balmer series is
20. Match the statement of Column with those in Column II:
a. A-1; B-3,4; C-4; D-2,3
Column I Column II
b. A-1; B-3,4; C-4; D-3
(A) In any Bohr orbit of the 1
1. − c. A-2; B-3,4; C-1; D-3
hydrogen atom, the ratio 2
d. A-1; B-2,4; C-3; D-4

Space for Rough Work


40 Physics
ANSWER & SOLUTIONS 6. (d) Work done, W = T ∆A
JEE-Main or W = T [(4π R 2 ) n − 4π R 2 ] = T .4π [nr 2 − R 2 ]
4 4 1
1. 2. 3. 4. 5. 6. 7. 8. 9. 10. Where n × π r 3 = π R 3 and W = mυ 2
3 3 2
a b a b b d b b c c
1 2
11. 12. 13. 14. 15. 16. 17. 18. 19. 20. ∴ mυ = T .4π (nr 2 − R 2 )
c d c a d a c d a a 2
1 4 3 2
21. 22. 23. 24. 25. or × π R ρυ = T .4π (nr 2 − R 2 )
a d c a b 2 3
 6T  nr 2 R 2  6T  nr 2 1  6T  1 1 
(a) A = 2ˆi + 4 ˆj − 5 kˆ or υ=  3 − 3 =  − = −
ρ  r R 
1.
ρ R R  ρ  nr 3 R 
∴ | A | = (2) 2 + (4 ) 2 + (−5 ) 2 = 45
∆P mg / A ∆V mg
7. (b) = = . . .(i)
2 4 −5 ∆V / V ∆V / V V AK
∴ cos α = , cos β = , cos γ =
45 45 45 4
But for a sphere V = πR 2
3
1 2
2. (b) h = ut − gt 4
2 Differentiating δV = π3 R 2 (δR )
3
1 1
h = u × 2 − g × (10 ) = u × 10 − g × (10 )
2 2
⇒ 4
2 2 π(3R 2 )δR
δV 3 3δR
⇒ 2u − 2 g = 10u − 50 g ⇒ u = 6g ⇒ = =
V 4 3 R
πR
∴ h = 2u − 2 g = 2 × 6 g − 2 g = 10 g = 98 m 3
3δV mg δR mg
3. (a) Equilibrium of insect gives ∴ From (i) = ⇒ =
R AK R 3 AK
µN α 0.05 v
N 8. (b) ∆λ = λ , As ∆λ = λ
100 c
α
0.05 v
mg cos α
mg sin α ⇒ λ = λ or v = 5 × 10 −4 c
mg 100 c
N = mg cos α . . . (i) or v = 5 × 10 −4 × 3 × 108 = 1.5 × 105 m/s
µ N = mg sin α . . . (ii) As λ decreases, the star is approaching the observer.
1 9. (c) Slope of adiabatic process p
From equation (i) and (ii) we get cot α = C
=3 at a given state (p, V, T) is
µ
more than the slope of A
4. (b) Let x be the maximum B
isothermal process. The
extension of the spring. From corresponding p-V graph for V
k
conservation of mechanical energy the two processes is as shown
decrease in gravitational potential in figure. In the figure, AB is isothermal and BC is
v=0 M
energy adiabatic.
x
= increase in elastic potential energy v=0 M WAB = positive (as volume is increasing) and
1 2 2 Mg WBC = negative (as volume is decreasing) plus,
∴ Mg x = kx or x =
2 k
WBC > WAB , as area under p-V graph gives the work done.
5. (b) For block P, friction will provide the necessary
restoring force. Hence, WAB + WBC = W < 0

fmax = mω2 A with ω 2 = k = k From the graph itself, it is clear that p3 > p1 .

m + m 2m Hence, the correct option is
 k  kA Note: At point B, slope of adiabatic (process BC) is
Hence, f max = m A=
 2m  2 greater than the slope of isothermal (process AB).
Mock Test-3 41
γ 17. (c) The wavelength of spectral line in Balmer series is
V 
10. (c) P1 V1γ = P2 V2γ ⇒ P2 = P1  1  1 1 1
 V2  given by = R 2 − 2 
λ 2 n 
γ −1
R 1   V1  
For first line of Balmer series, n = 3
work = ( P2 V2 − P1 V1 ) =  P1   − 1
1− γ 1 − γ   V2    1 1  5R
 ⇒
1
= R 2 − 2  = ; For second line n = 4.
1 λ1  2 3  36
× 105 × 22.4 ×10−3 × ( 2 ) − 1
0.40
=
1 − 1.40   1 1 1  3R
⇒ = R 2 − 2  =
=
1
× 22.4 ×102 [1.32 − 1] = 1792 J λ2  2 4  16
0.40 λ2 20 20
∴ = ⇒ λ1 = × 6561 = 4860 Å
dQ  dm  Temperature difference  dm  λ1 27 27
11. (c) = L  or = L 
dt  dt  Thermal resistance  dt 
18. (d) 6 C 11 → 5 B 11 + β+ + γ because β+ = 1 e 0
dm 1 1
or ∝ ⇒q∝
dt Thermal resistance R 19. (a) The base is always thin
In the first case rods are in parallel and thermal resistance
20. (a) VHF (Very High Frequency) band having frequency
R range 30 MHz to 300 MHz is typically used for TV and
is while in second case rods are in series and thermal
2 radar transmission.
resistance is 2R.
 π
q1 2R 4 21. (a) V = 5cos ωt = 5sin  ωt +  ⇒ I = 2sin ωt
⇒ = =  2
q2 R / 2 1
π π
1 1 ∴ φ= ⇒ P = Erms I rsm cos =0 W
12. (d) Energy u ∝ E2 and u ∝ for a point charge; so u ∝ 4 2 2
r2 r
m
13. (c) For short bar magnet in tan A-position 22. (d) Density ρ =
π r2L
µ0 2M ∆ρ
= H tan θ . . . (i)  ∆m ∆r ∆L 
4π d 3 ∴ × 100 =  +2 +  ×100
ρ  m r L 
When distance is doubled, then new deflection θ ′ is given by
After substituting the values, we get the maximum
µ0 2 M
= H tan θ ′ . . . (ii) percentage error in density = 4%
4π (2 d )3
23. (c) L = 40 m, v = 1080 km,
tan θ ′′ 1
∴ = h–1 = 300 m sec−1 and B = 8.3 × 10−4 T
tan θ 8
tan θ tan 60° 3  3  ⇒ e = Blv = 8.3 ×10−4 × 40 × 300 = 1 V
⇒ tan θ ′ = = = ⇒ θ ′ = tan –1  
8 8 8  8  24. (a) Relative velocity of image w.r.t. object
1 = 3 – (–3) = 6 m/sec
14. (a) cos φ = ⇒ φ = 60°
2
–3m/sec 3m/sec
ωL 3
tan 60° = ⇒L= H O
R π I

I max ( a1 + a2 )  2 + 1  9
2 2

15. (d) = =  = 25. (b) Angular momentum of satellite about center of earth
I min ( a1 − a2 )2  2 − 1  1
remains constant, i.e., mvr = constant
16. (a) In the given question, the electromagnetic wave is 1 v r
⇒ v∝ ⇒ 2 = 1
propagating along + z axis. In e.m. wave, the electric field 2 v1 r2
 
( E) and magnetic field ( B ) are perpendicular to each other Speed is maximum at position 2,
and also perpendicular to the direction of propagation of vmax r1 (1 + ε ) a 1 + ε 1 + 0.0167
∴ = = = = = 1.033
wave, vmin r2 (1 − ε ) a 1 − ε 1 − 0.0167
 
So, E = E0 iˆ and B = B0 ˆj.
42 Physics
JEE Advance Paper -I A

1. 2. 3. 4. 5. 6. 7. 8. 9. 10. I
a d a c a,b,d b,d 4 8 9 8
O
11. 12. 13. 14. 15. 16. 17. 18.
I
8 4 2 4 a,d a,c b d 60° 60°
r
B C
1 1 1 I D
1. (a) = + a
R e q R1 R2
It will be acting perpendicular to the plane of triangle
V2 V2 V2 upwards. Total magnetic field induction at O due to
⇒ P= = + + .....
R e q R1 R2 current through all the three sides of the triangle will be
= 15 × 40 + 5 × 100 + 80 × 5 + 1× 1000 = 2500 w
W 3µ0 I
B = 3B1 = sin θ11++sin
[sin sin θ2 2]
⇒ P = VI 4π r
Here, I = 1 A, θ1 = 60° = θ2
250 125
⇒ P = 2500 = 220 I ⇒ I = = BD a/2 a 4.5 × 10 −2
22 11 And r = OD = = = = m
tan 60° 3 2 3 2 3
i i
2. (d) υd = ∴ = υd ⋅ ne 1
Ane A ∴ B = 3 × 10−7 × × [sin 60° + sin 60°]
V V
( 4.5 ×10 −2
/2 3 )
⇒ i= = ∴ρ= V
R ρ ℓ i On solving, B = 4 × 10–5 T.

A A
8. (8) Here, Am = 4 V ; Ac = 5 V
V
⇒ ρ= Percentage of modulation,
υd ⋅ ne ℓ
Am 4
5 µ= × 100 = × 100 = 80% = a × 10%. ⇒ a = 8
⇒ ρ= Ac 5
2.5 ×10−4 × 8 ×1028 ×1.6 ×10−19 × 0.1
2 1 9. (9) Here, vs = 4.5kHz; vc = 3.45MHz = 3450 kHz
= = × 10−4 = 1.6 ×10−5 Ω m
8 × 1.6 × 10−4 6.4 Bandwidth = 2 vs = 2 × 4.5 = 9.0 kHz.
ig G 10. (8) Here, vc = 4 MHz, i = 90°
3. (a) S = Here ig = 10 −3 A
1 − ig
Maximum usuable frequency = vc sec i
⇒ G = 102 Ω, I = 10 A ⇒ S = 10−2 Ω = 4 × sec60° = 4 × 2 = 8MHz
4. (c) For conductor (Cu) resistance increases linearly and
11. (8) d ' = 2h ' R = 3d = 3 2hR
for semiconductor resistance decreases Exponentially in
given temperature range. or h ' = 9 h = 9 ×100 = 900 m
Increase in height of tower = 900 − 100 = 800 = a × 10 2
R (V + V2 )
5. (a, b, d) V1 = 1 1 ⇒ V1 R3 = V2 R1 ⇒ a =8
R1 + R3
R3 (V1 + V2 ) 12. (4) Microwave frequency used in telephone link = 10 GHz
⇒ V2 = ⇒ V2 R1 = V2 R3
R1 + R3 = 10 × 109 Hz = 1010 Hz
Frequency available for microwave communication
V2 V2 V2 2
6. (b, d) H = 4= t1 t2 = 2% of 10 GHz = ×1010 = 2 × 108 Hz
R R / 2 R /8 100
⇒ t1 = 2 min. ⇒ t2 = 0.5 min. Bandwidth of each telephone channel = 8 kHz

7. (4) Refer Fig., The magnetic field induction at the = 8 × 103 Hz


centroid O due to current I through one side BC of the Number of microwave telephone channels
µ0 I 2 × 108
triangle will be B1 = . (sin 1θ+1 + sin2 θ2) = = 2.5 × 10 4 = 2.5 × 10 a ∴ a = 4
4π r 8 × 103
Mock Test-3 43
13. (2) τ = Iα dθ
F 2. (c) 2T sin = BiRdθ (for θ small)
⇒ 3FR sin 30° = Iα 2
T cos (dθ / 2) T cos(dθ / 2)
MR 2 ⇒ Tdθ = BiRdθ
R 30° T T
⇒ I =
2 BiL
⇒ T = BiR =
R sin 30° 2T
F sin(dθ / 2)
⇒ α=2 2π
F

⇒ ω = ω0 + αt ⇒ ω = 2 rad / s b
µ0 IN µ0 IN b
dr µ IN b
(a) ∫
2(b − a) ∫
3. dr = = 0 ln  
14. (4) Since net torque about centre of rotation is zero, so we a 2r (b − a) a r 2(b − a)  a 
can apply conservation of angular momentum of the 4. (b) M = IA
system about centre of disc Li = L f 2
a
π  a
0 = Iω + 2mv (r / 2); comparing magnitude
A = 4 ×   + a2
2
⇒ a a
2 a
 0.45 × 0.5 × 0.5  0.5
∴   ω = 0.05 × 9 × ×2
π a2  a2  2  π  2
 2  2 = 2× + a2 = π   + a  + 1 a
∴ ω=4 4  2 2 
π 
15. (a, d) I1 = I2 ∴ M =  + 1 a 2 Ikˆ
 2 
⇒ 1 1 = neA2v2 ⇒ d1w1v1 = d2 w2v2
neAv
5. (d) Let current be I.
Now, potential difference developed across MK
I
V = Bvw Current density J =
 R2 
V1 v1w1 d2 π  R2 − 
⇒ = = and hence, correct choice is a & d.  4  R/2
V2 v2 w2 d1 Amperian
I 4I r
⇒ J= = R loop
16. (a, c) As I1 = I2 n1w1d1v1 = n2 w2d2v2 3R 2 3π R 2
π
4
V2 B2v2 w2  B2 w2   n1w1d1  B2 n1
Now, = =  =  R R
V1 B2v1w1  B1w1   n2 w2 d2  B1n2 Field for  r ≤  B = 0 For < r ≤ R
 2 2
∴ Correct options are a & c. Applying Ampere’s Law B.2π r
17. (b) Q = Q0 (1 − e − t /τ ) 4I  R 2  4µ I  R2 
= µ0 .π  r 2 −  = 02  r 2 − 
Q = CV (1 − e−t /τ ) after time interval 2τ . 3π R 
2
4  3R  4 
2 µ0 I  R2  R
18. (d) q = Q0 cos ω t B=  r −  ⇒ At r = , B = 0
3π R 2  4r  2
dq
⇒ i=− = Q0ω sin ωt R
dt ∴ At r = filed is continuous.
2
C
⇒ ikax = CωV = V From the above expression as r increases B increases.
L
For r ≥ R B.2π r = µ 0 I
µ0 I 1
JEE Advance Paper -II ⇒ B= ∴B∝
2π r r
1. 2. 3. 4. 5. 6. 7. 8. 9. 10.
µ0 I
d c a b d a 9 6 2 2/3 ⇒ At r = R, B=
2π R
11. 12. 13. 14. 15. 16. 17. 18.
From inside expression at r = R
7 3 1 9 c a c d
2µ0 I  R 2  µ0 I
1. (d) According to Lenz’s Law, current will be in ⇒ B= .  R − =
3π R2  4R  2π R
anticlockwise sense as magnetic field is increasing into
the plane of paper. This proves the continuity in the graph at r = R. From the
above only correct option is (d).
44 Physics
6. (a) First we find the mutual inductance of the assembly.
2
11. (7) Here, m = 5 kg; r = = 1m ; I = ?
r 2
R Moment of inertia of the sphere about a tangent to the
7 7
I sphere, I = mr 2 = × 5 × 12 = 7kgm2 ⋅
5 5
Let current I flow through the larger loop.
12. (3) Here, l = 1 m, m = 0.5 kg, ω = 6 rad/sec
The strength of induction at the smaller loop
1 2 1  ml 2  2 1 0 ⋅ 5 × 12
µ0 IR2 KE = Iω =  ω = × × 62 = 3 J ⋅
= 2 2 3  2 3
2( R 2 + d 2 )3 / d
13. (1) Here, L1 = I1ω1 , When ω2 = 2ω1 ,
µ0πIR 2 r 2
∴ Flux through smaller loop = 1 1 
2( R 2 + d 2 )3 / 2 and KE = I 2ω22 = 2  I1ω12 
2  2 
µ0 R 2 r 2
∴ Mutual inductance = 1 1 1
2( R 2 + d 2 )3 / 2 I 2 (2ω1 ) 2 = 2 × I1ω12 ; I 2 = I1
2 2 2
µ 0πR 2 r 2 1 L
∴ Flux linked through coil = .i ∴ L2 = I 2ω2 = I1 × 2ω1 = I1ω1 = L1 ⇒ x = 2 = 1 ⋅
2( R 2 + d 2 ) 3 / 2 2 L1
4π × 10−7 × π × 10−2 × 9 × 10−6 × 2 14. (9) Let l1 be the length of closed pipe l2 = 36cm= length
=
2(4 ×10 −2 + 2.25 ×10−2 )3 / 2 of open pipe

=
16 10 × 18
× × (10 +3−15 ) = 9.1× 10 −11 weber Fundamental frequency of closed pipe n1 = υ
2 15.625 4l1
12 Frequency of first overtone of open pipe,
7. (9) 12
→ C + β− +γ −
B  6
υ υ υ
5

Q = (m5 B12 − m612C * )c 2 = [m5 B12 − (m6C 12 + ∆m)]C 2 n2 = 2 × = = As n1 = n2


2l1 l2 36
= (m5 B12 − m6C12 )C 2 − ∆mC 2 = 0.014 × 931 − 4.041 = 9 υ υ 36
∴ = ;4l1 = 36 ⇒ l1 = = 9cm.
Ltotal m r 2 4l1 36 4
8. (6) = +11 1
2
LB mr 2 2 15. (c) A-4; B-1, 4; C- 4; D-2
9. (2) Assuming that masses of the two spheres are 3
A. KE = KT ⇒ [ ML2T −2 ] = K ′[ K ] ⇒ K ′ = [ ML2T −2 K −1 ]
concentrated at their centres A and B and taking x = 0 at A, 2
Fig., we get m B. F = 6πη rv ⇒ [ MLT −2 ] = η [ L ] [ LT −1 ] ⇒ η [ ML−1T −1 ]
2m
m × 0 + 2m × 3a C h
xcm = = 2a = AC A C. E = hf ⇒ [ ML2T −2 ] = ⇒ h = [ ML2T −1 ]
( m + 2m ) 2a a B [T ]
AC 2a dQ K ′A(∆T ) [ ML2T −2 ] k[ L2 ] [ K ′]
CB = a ∴ = = 2⋅ D. = ⇒ =
CB a dt ∆x [T ] [ L]
10. (2/3) As shown in Fig., mass of small element of length dx
⇒ K ′ = [ MLT −3 K −1 ]
is dm = λ dx = λ0 x dx dm
x 16. (a) A-4; B- 1,4; C- 4; D- 2
L L

∫ xdm ∫ x(λ xdx ) 0 dx


17. (c) c. A-1; B-4; C-2; D-3
When coin is dropped if falls is straight line where
⇒ xcm = 0
= 0 L
∫ dm
L
direction is same but magnitude changes with change in
∫ λ xdx
0
0
position. When a coin is rotated in circular path direction
goes on changing but radius remains constant. In
[ x 3 / 3]0L L3 / 3 2 L
xcm = = = projectile motion both direction and magnitude of r
[ x 2 / 2]0L L2 / 2 3
changes. A coin held in your hand has constant position
2L 2L L x 2L / 3 1 vector and hence it is invariant.
If x1 = , x2 = L − = ⇒ 1 = = ⋅
3 3 3 x2 L/3 2 18. (d) A-1, B-2,4, C-3, D-4
  
Mock Test-4 45
JEE-MAIN: PHYSICS MOCK TEST-4
SECTION 1 (Multiple Choice Question) π 4 π 
a. MR 2 b.  −  MR 2
1. Which of the following sets have different dimensions? 12 3 4
a. Pressure, Young’s modulus, Stress  8 10π  4 π 
c.  −  MR
2
d.  −  MR 2
b. Emf, Potential difference, Electric potential  3 16  3 6
c. Heat, Work done, Energy
6. The radius of earth is 6400 km and g = 10 m/s2. In order
d. Dipole moment, Electric flux, Electric field
that a body of 5 kg weighs zero at the equator, the angular
2. The 100 coplanar forces each equal to 10 N act on a body. speed of earth in radian/sec is:
π 1 1
Each force makes angle with the preceding force. a. b.
50 80 400
What is the resultant of the forces? 1 1
a. 1000 N b. 500 N c. d.
800 1600
c. 250 N d. Zero
7. The lower end of a capillary tube of radius r is placed
3 A string of negligible mass going over a clamped pulley vertically in water. Then with the rise of water in the
of mass m supports a block of mass M as shown in the capillary, heat evolved is:
figure. The force on the pulley by the clamp is given by
π 2 r 2h 2 π 2 r 2 h2 dg
a. + dg b. +
m J 2J
π r h dg
2 2
π r 2 h 2dg
c. − d. −
2J J
M 8. A wire of length L and cross-sectional area A is made of a
material of Young’s modulus Y. If the wire is stretched by
a. 2 Mg b. 2 mg the amount x, the work done is:
c. ( M + m) 2 + m 2 g d. ( ( M + m) 2 + M 2 ) g YAx 2 YAx 2
a. b.
2L L
4. A simple pendulum is oscillating without damping. When
YAx
the displacement of the bob is less than maximum, its c. d. Yax 2 L
 2L
acceleration vector a is correctly shown in
9. Which of the following graphs correctly represent the
a. b.
dV / dp

a variation of β ==− with p for an ideal gas at
V
 constant temperature?
a
c. d.
a. b.

a
 p p
a

5. Four holes of radius R are cut form of a thin square plate


of side 4R and mass M. The moment of inertia of the c. d.
remaining portion about z-axis is:
p p
y
10. Two rods of different materials and identical
cross-sectional area are joined face to face at one end and
x
their free ends are fixed to the rigid walls. If the temperature
of the surroundings is increased by 30°C, the magnitude of
the displacement of the joint of rods is: (length of the
46 Physics
rods l1 = l2 = 1 unit, ratio of their Young’s a. tan −1 ( 3 / 2) b. tan −1 ( 3)
modulii, Y1 / Y2 = 2 ; coefficients of linear expansion are α1 c. tan −1 ( 3 / 2) d. tan −1 (2 / 3)
and α2)
15. Two straight long conductors AOB and COD are
a. 5(α 2 − a1 ) b. 10(α1 − a2 )
perpendicular to each other and carry currents i1 and i2 .
c. 10(α 2 − 2a1 ) d. 5(2α1 − a2 ) The magnitude of the magnetic induction at a point P at a
11 A steel ball of mass m1 = 1 kg moving with velocity distance a from the point O in a direction perpendicular to
50 ms–1 collides with another stationary steel ball of mass the plane ACBD is
m2 = 0.200 kg. During the collision, their internal µ0 µ0
a. (i1 + i2 ) b. (i1 − i2 )
energies change equally. If T1 and T2 are rise in 2π a 2π a
temperature of balls m1 and m2 respectively, specific heat µ0 2 µ I1 I 2
c. ( I1 + I 22 )1/ 2 d. 0
of steel = 0.105 and mechanical equivalent of heat J = 4.2 2π a 2π a ( I1 + I 2 )
J/cal, then: 16. A circular coil and a bar magnet placed nearby are made
3 3
a. T1 = 7.0×10 °C, T2 = 1.4×10 °C, to move in the same direction. The coil covers a distance
b. T1 = 1.4 ×103 °C, T2 = 7.0×103 °C, of 1 m in 0.5 sec and the magnet a distance of 2 m in
1 sec. The induced e.m.f. produced in the coil:
c. T1 = 1.4 °C, T2 = 7.0 °C, y
d. T1 = 7.0 °C, T2 = 1.4 °C, A B

12. Three identical dipoles are arranged as shown below. x


 1 
What will be the net electric field at P  k = ?
 4πε 0 
C D

–Q → +Q
a. Zero
p
b. 1 V
c. 0.5 V
x
–Q +Q d. Cannot be determined from the given information
P
x →
17. Following figure shows an ac generator connected to a
x p "box" through a pair of terminals. The box contains
possible R, L, C or their combination, whose elements and
–Q →
p +Q arrangements are not known to us. Measurements outside
the box reveals that e = 75 sin(sin ωt ) volt, i = 1.5 sin
k. p 2kp
a. b. (ωt + 45°) amp, then, the wrong statement is
x3 x3
2 kp
c. Zero d. ~ ?
x3
13. The area of the plates of a parallel plate capacitor is A and
a. There must be a capacitor in the box
the distance between the plates is 10 mm. There are two
b. There must be an inductor in the box
dielectric sheets in it, one of dielectric constant 10 and
c. There must be a resistance in the box
thickness 6 mm and the other of dielectric constant 5 and
d. The power factor is 0.707
thickness 4 mm. The capacity of the capacitor is:
12 2 18. The focal length of a concave mirror is f and the distance
a. ε 0A b. ε 0 A from the object to the principle focus is x. The ratio of the
35 3
5000 size of the image to the size of the object is
c. ε0A d. 1500 ε 0 A f +x f
7 a. b.
14. If a magnet is suspended at an angle 30o to the magnetic f x
meridian, it makes an angle of 45o with the horizontal. The f f2
c. d.
real dip is x x2
Mock Test-4 47
19. The energy of incident photon is 12.375 eV while the 23. A sound wave of wavelength 32 cm enters the tube at S as
energy of scattered photon is 9.375 eV. Then the kinetic shown in the figure. Then the smallest radius r so that a
energy of recoil electron is minimum of sound is heard at detector D is.
a. 3 eV b. less than 3 eV
c. more than 3 eV d. 21.75 eV
r
20. In a transistor circuit shown here the base current is
S D
35 µA. The value of the resistor Rb is
E C a. 7cm b. 14 cm
c.21cm d. 28 cm
B
24. Three rods made of the same material and having the
Rb RL
9V same cross-section have been joined as shown in the
figure. Each rod is of the same length. The left and right
a. 123.5 kΩ b. 257 k Ω ends are kept at 0°C and 90ºC respectively. The
c. 380.05 k Ω d. None of these
temperature of junction of the three rods will be
SECTION 2 (Numeric Value Question) 90°C

21. A wooden block of mass 10 g is dropped from the top of a 0°C


cliff 100 m high. Simultaneously a bullet of mass 10 g is
fired from the foot of the cliff upwards with a velocity of
100 m/s. The bullet and the wooden block will meet, each 90°C
other, after a time:
a. 45°C b. 60°C
a. 10 s b. 0.5 s
c. 30° C d. 20° C
c. 1 s d. 7 s
25. Six resistance of 6 ohm each are connected to form a
22. The displacement of a particle varies according to the
hexagon. The resistance between any two adjacent
relation x = 4(cos πt + sin πt ). The amplitude of the
terminals is:
particle is:
a. 8 b. – 4 a. 6 Ω b. 36 Ω

c. 4 d. 4 5 c. 5 Ω d. 1 ohm

Space for Rough Work


48 Physics

JEE ADVANCE PAPER-I

SECTION 1 (Maximum Marks: 24)


MCQs with one or more than one correct answer

1. A glass tube of uniform internal radius r has a valve d


α
separating the two identical ends. Initially, the valve is in
a tightly closed position. End 1 has a hemispherical soap
bubble of radius r. End 2 has sub-hemispherical soap d
bubble as shown in figure. Just after opening the valve,
1 + tan α 1 + sin α
a. b.
1 − tan α 1 − cos α
1 + sin α 1 + cos α
c. d.
1 − sin α 1 − cos α
2 1
a. air from end 1 flows towards end 2. No change in the 5. On heating water, bubbles being formed at the bottom of
volume of the soap bubbles the vessel detaches and rise. Take the bubbles to be
b. air from end 1 flows towards end 2. Volume of the soap spheres of radius R and making a circular contact of radius
bubble at end 1 decreases r with the bottom of the vessel. If r << R, and the surface
c. no changes occurs tension of water is T, value of r just before bubbles
d. air from end 2 flows towards end 1. volume of the soap detaches is: (density of water is ρw )
bubble at end 1 increases

2. A thin uniform cylinder shell, closed at both ends is


partially filled with water. It is floating vertically in water R
in half-submerged state. If ρc is the relative density of the
material of the shell with respect to water, then the correct 2r
statement is that the shell is.
ρw g 3ρ w g
a. more than half-filled if ρc is less than 0.5 a. R 2 b. R 2
T T
b. more than half-filled if ρc is less than 1.0
c. half-filled if ρc is more than 0.5 2ρw g ρw g
c. R 2 d. R 2
3T 6T
d. less than half- filled if ρc is less than 0.5
6. Two solid spheres A and B of equal volumes but of
3. An open glass tube is immersed in mercury in such a way
that a length of 8 cm extends above the mercury level. The different densities d A and d B are connected by a string.
open end of the tube is then closed and sealed and the tube They are fully immersed in a fluid of density d F . They
is raised vertically up by additional 46 cm. What will be get arranged into an equilibrium state as shown in the
length of the air column above mercury in the tube now? figure with a tension in the string. The arrangement is
(Atmospheric pressure = 76 cm of Hg) possible only if
a. 38 cm b. 6 cm
c. 16 cm d. 22 cm
A
4. There is a circular tube in a vertical plane. Two liquids
which do not mix and of densities d1 and d2 are filled in
B
the tube. Each liquid subtends 90° angle at centre. Radius
joining their interface makes an angle α with vertical.
a. d A < d F b. d B > d F
d
Ratio 1 is c. d A > d F d. d A + d B = 2d F
d2
Mock Test-4 49
SECTION 2 (Maximum Marks: 24) SECTION 3 (Maximum Marks: 12)
Numerical value answer type questions Paragraph based questions (2 paragraphs, each having 2
MCQs with one correct answer only)
7. A 25 watt bulb, which is producing monochromatic light
of wavelength 6600 Å is used to illuminated a metal Paragraph for Question No. 15 to 16
surface. If the surface has 3% efficiency for photoelectric The general motion of a rigid body can be considered to be a
effect, then the photoelectric current produced in deci combination of (i) a motion of its centre of mass about an axis,
ampere is (use h = 6.6 × 10 −34 Js) : and (ii) its motion about an instantaneous axis passing through
the centre of mass. These axes need not be stationary. Consider,
8. de-Broglie wavelength associated with an electron for example, a thin uniform disc welded (rigidly fixed)
accelerated through a potential difference 4 V is λ1 . When horizontally at its rim to a massless stick, as shown in the
accelerating voltage is decreased by 3 V, its de-Broglie figure.
λ1 ω
wavelength is λ2 . The ratio is
λ2
P P Q
Q y
9. What is the energy in eV that should be added to an
electron of energy 2 eV to reduce its de-Broglie
x
wavelength from 1 nm to 0.5 nm?
When the disc-stick system is rotated about the origin on a
10. The angular momentum of electron in 2nd orbit of horizontal frictionless plane with angular speed ω , the motion
hydrogen atom is L. The angular momentum of electron in at any instant can be taken as a combination of (i) a rotation of
4th orbit of hydrogen atom is n L where n = ? the centre of mass of the disc about the z-axis, and (ii) a
rotating of the disc through an instantaneous vertical axis
11. For an atom of an ion having single electron, the
passing through its centre of mass (as is seen from the changed
wavelength observed λ1 = 2 are units and λ3 = 3 units
orientation of points P and Q). Both these motions have the
figure. The value of missing wavelength λ2 is same angular speed ω in this case.
n3 orbit Now consider two similar systems as shown in the figure: Case
λ3
(a) the disc with its face vertical and parallel to x-z plane; Case
λ1
(b) the disc with its face making an angle of 45° with x-y plane
n2 orbit and its horizontal diameter parallel to x-axis. In both the cases,
λ2 the disc is welded at point P, and the systems are rotated with
n1 orbit constant angular speed ω about the z-axis.
z z
Q
12. How many different wavelength may be observed in the ω ω Q
spectrum from a hydrogen sample if the atoms are excited
y 45° y
to 3rd excited state? P P

13. Light waves from two coherent sources having intensities x x


I and 2I cross each other at a point with a phase diff. of
15. Which of the following statements regarding the angular
60º. What is the resultant intensity at the point? speed about the instantaneous axis (passing through the
centre of mass) is correct?
14. A pn-junction diode can withstand currents upto 10 mA.
a. It is 2ω for both the cases
When it is forward biased, the potential drop across it is
1.0 V. Assuming that this potential drop is independent of ω
b. It is ω for case a.; and for case b.
the current, find the maximum voltage of the battery used 2
c. It is ω for case a.; and ω for case b.
to forward bias the diode when a resistance of is
d. It is ω for both the cases
connected in series with the diode.
50 Physics

16. Which of the following statements about the instantaneous Where vrot is the velocity of the particle in the rotating frame of
axis (passing through the centre of mass) is correct? 
reference and r is the position vector of the particle with
a. It is vertical for both the cases a. and b.
respect to the centre of the disc.
b. It is vertical for case a.; and is at 45°to the x-z plane
Now consider a smooth slot along a diameter of a disc of radius
and lies in the plane of the disc for case b.
R rotating counter-clockwise with a constant angular speed ω
c. It is horizontal for case a.; and is at 45° to the x-z plane
and is normal to the plane of the disc for case b. about its vertical axis through its center. We assign a coordinate
d. It is vertical for case a.; and is at 45° to the x-z plane system with the origin at the center of the disc, the x-axis along
and is normal to the plane of the disc for case b. the slot, the y-axis perpendicular to the slot and the z-axis along

Paragraph for Question No. 17 to 18 the rotation axis (ω = ω kˆ ). A small block of mass m is gently

A frame of reference that is accelerated with respect to an placed in the slot at r = ( R / 2)iˆ at t = 0 and is constrained to
inertial frame of reference is called a non-inertial frame of move only along the slot.
reference. A coordinate system fixed on a circular disc rotating
17. The distance r of the block at time t is:
about a fixed axis with a constant angular velocity ω is an
R R
example of a non-inertial frame of reference. a. cos 2ωt b. ( e 2ωt + e −2ωt )
2 4
ω R R ωt
c. cos ωt d. (e + e − ω t )
2 4
R
18. The net reaction of the disc on the block is :
m
R/2 a. mω 2 R sin ω tjˆ − mgkˆ

 b. − mω 2 R cos ω tjˆ − mgk 6ˆ


The relationship between the force Frot experienced by a
1
particle of mass m moving on the rotating disc and the force c. mω 2 R(eωt − e −ωt ) ˆj + mgkˆ
 2
Fin experienced by the particle in an inertial frame of reference 1
       d. mω 2 R (e 2ω t − e −2ωt ) ˆj + mgkˆ
is, Frot = Fin + 2m (vrot × ω ) + m (ω × r ) × ω , 2

Space for Rough Work


Mock Test-4 51

JEE ADVANCE PAPER-II


SECTION 1 (Maximum Marks: 24)
 
a. F1 = F2 = 0
MCQs with one or more than one correct answer  
b. F1 is radially inwards and F2 is radially outwards
1. One end of a horizontal thick copper wire of length 2L  
c. F1 is radially inwards and F2 = 0
and radius 2R is welded to an end of another horizontal  
thin copper wire of length L and radius R. When the d. F1 is radially outwards and F2 = 0
arrangement is stretched by applying forces at two ends, 6. Consider a spherical shell of radius R at temperature T.
the ratio of the elongation in the thin wire to that in the The black body radiation inside it can be considered as an
thick wire is ideal gas of photons with internal energy per unit volume
a. 0.25 b. 0.50 U 1U 
u= ∝ T 4 and pressure P =   . If the shell now
c. 2.00 d. 4.00 V 3 V 
2. The pressure that has to be applied to the ends of a steel undergoes an adiabatic expansion the relation between T
wire of length 10 cm to keep its length constant when its and R is:
temperature is raised by 100°C is: (For steel Young’s a. T ∝ e − R b. T ∝ e −3R
modulus 2 × 1011 Nm–2 and coefficient of thermal 1 1
c. T ∝ d. T ∝
expansion is 1.1 × 10–5 K–1 ) R R3
a. 2.2×107 Pa b. 2.2×106 Pa
SECTION 2 (Maximum Marks: 24)
c. 2.2×108 Pa d. 2.2×109 Pa
Numerical value answer type questions
3. A pendulum made of a uniform wire of cross-sectional
area A has time period T. When an additional mass M is 7. An electron in an excited state of Li2+ ion has angular
added to its bob, the time period changes to TM . If the momentum 3 h / 2π . The de-Broglie wavelength of the
Young's modulus of the material of the wire is Y then 1/Y electron in this state is pπa0 (where a0 is the Bohr radius).
is equal to: (g = gravitational acceleration) The value of p is

 T  2  A  T 2  Mg 8. Consider a hydrogen atom with its electron in the nth


a.  M  − 1 b.  M  − 1 orbital. An electromagnetic radiation of wavelength 90 nm
 T   Mg  T   A
is used to ionize the atom. If the kinetic energy of the
  T 2  A   T 2  A ejected electron is 10.4 eV, then the value of n is (hc =
c. 1 −  M   d. 1 −   
  T   Mg   TM   Mg 1242 eV nm)
9. A hydrogen atom in its ground state is irradiated by light
4. Function x = A sin 2 ωt + B cos 2 ωt + C sin ωt cos ωt
of wavelength 970 Å. Taking hc/ e = 1.23 7 × 1 0 − 6 eV m and
represents SHM. the ground state energy of hydrogen atom as –13.6 eV, the
number of lines present in the emission spectrum is:
θ
L 10. To determine the half life of a radioactive element, a
dN (t )
student plots a graph of ℓ n versus t.
I I dt
a. For any value of A, B and C (except C = 0) 6
5
ℓn|dN(t)/dt|

b. If A = – B; C = 2B, amplitude = | B 2 |
4
c. If A = B; C = 0 3
d. If A = B; C = 2B amplitude = |B| 2
5. Two coaxial solenoids of different radii carry current I in 1
 2 3 4 5 6 7 8
the same direction. Let F1 be the magnetic force on the Years
 Here dN(t)/dt is the rate of radioactive decay at time t. If
inner solenoid due to the outer one and F2 be the magnetic
the number of radioactive nuclei of this element decreases
force on the outer solenoid due to the inner one. Then: by a factor of p after 4.16 years, the value of p is
52 Physics
11. The activity of a freshly prepared radioactive sample is (D) If different masses have m1
10 10
disintegrations per second, whose mean life is same momentum, the ratio 4. m
2

10 s. The mass of an atom of this radioisotope is 10−25 kg.


9 of their kinetic energy is
a. A-1, B-2, C-3, D-4 b. A-2, B-1, C-4, D-3
kg. The mass (in mg) of the radioactive sample is
c. A-4, B-3, C-2, D-1 d. A-3, B-2, C-1, D-4
12. A freshly prepared sample of a radioisotope of half-life
16. Waves are classified on the basis of frequency range.
1386s has activity 103 disintegrations per second.
Column I mentions the type of waves while column II
Given that ln 2 = 0.693, the fraction of the initial number
gives the corresponding frequency range. Can you match
of nuclei (expressed in nearest integer percentage)
the proper options?
that will decay in the first 80s after preparation of the
Column I Column II
sample is
(A) Infrasonic 1. Objects having velocity
13. A nuclear power plant supplying electrical power to a more than velocity of
village uses a radioactive material of half life T years as sound in air at 0°C
the fuel. The amount of fuel at the beginning is such that (B) Audible 2. Frequency < 20 Hz
the total power requirement of the village is 12.5 % of the (C) Ultrasonics 3. Frequency > 20,000 Hz
electrical power available from the plant at that time. If (D) Supersonics 4. Frequency lies between
the plant is able to meet the total power needs of the 20 Hz to 20, 000 Hz
village for a maximum period of nT years, then the value a. A-4, B-3, C-2, D-1 b. A-2, B-4, C-3, D-1
of n is c. A-4, B-4, C-1, D-2 d. A-3, B-2, C-1, D-4
14. For a radioactive material, its activity A and rate of change 17. Various electromagnetic waves are given in column I and
dN dA various frequency ranges in column II
of its activity R are defined as A = − and R = − ,
dt dt Column I Column II
where N(t) is the number of nuclei at time t. Two (A) Radiowaves 1. 1×1016 to 3×10 21 Hz
radioactive sources P (mean life τ ) and Q (mean life 2τ ) (B) γ-rays 2. 1×109 to 3×1011 Hz
have the same activity at t = 0. Their rates of change of (C) Microwaves 3. 3×1018 to 5×1022 Hz
activities at t = 2τ are RP and RQ , respectively. If
(D) X-rays 4. 5×105 to 109 Hz
RP n
= , then the value of n is a. A-1, B-2, C-3, D-4 b. A-2, B-3, C-4, D-1
RQ e
c. A-3, B-4, C-1, D-2 d. A-4, B-3, C-2, D-1
SECTION 3 (Maximum Marks: 12) 18. In each of the following questions, match column I and
Matching type questions with 4 options column II, and select the correct match out of four given
15. There is a definite relation between velocity, mass and choices.
Column I Column II
acceleration of body to know about the work done by the
(A) Transducer 1. Range of frequencies over
forces applied on body. Column I has some statements
which communication
showing some relations which are related with some system works
statements in Column II. Match the correct options. (B) Attenuation 2. A device that has input in
Column I Column II electrical form or provides
(A) If kinetic energy is K the 1. zero output in electrical form
momentum P is 0 (C) Range 3. Loss of strength of a
(B) If momentum p is zero the 2. 2 mk signal during propagation.
kinetic energy is 0 (D) Bandwidth 4. Argest distance between
(C) If different masses have m2 transmitter and receiver.
3.
same kinetic energy the m1 a. A-,2 B-4, C-1, D-3 b. A-4, B-3, C-1, D-2
ratio of their momenta is 0 c. A-1, B-2, C-3, D-4 d. A-2, B-3, C-4, D-1
Mock Test-4 53
ANSWERS & SOLUTIONS ∴ Moment of inertia of the remaining portion,
JEE-Main I = I square − 4I hole
 mR 2 
1. 2. 3. 4. 5. 6. 7. 8. 9. 10.
=
M
12
(16 R 2 + 16 R 2 ) − 4 
2
+ m ( 2 R 2 )
d d d c c c b a a c  
11. 12. 13. 14. 15. 16. 17. 18. 19. 20. 8  8 10π 
c c c a c a b b a b = MR 2 − 10mR 2 =  −  MR
2

3  3 16 
21. 22. 23. 24. 25.
c d b b c 6. (c) g ′ = g − Rω 2 (at equator)

1. (d) Dipole moment = (charge) × (distance) Given g ′ = 0 ⇒ g = Rω 2


Electric flux = (electric field) × (area) g 10 1
⇒ ω= = = rad/s
π R 6400 × 103 800
2. (d) Total angle = 100 × = 2π
50 7. (b) When the tube is placed vertically in water, water rises
So, all the force will pass through one point and all forces 2T cos θ
will be balanced. i.e. their resultant will be zero. through height h given by h =
rdg
3. (d) Free body diagram of pulley is shown in figure. Upward force = 2πr ×T cos θ
T = Mg F Work done by this force in raising water column through
Mg height h is given by ∆W = (2πrT cos θ)h
 rhdg 
= (2πrh cos θ)T = (2πrh cos θ)   = πr h dg
2 2

R  2 cos θ 
mg T=Mg Mg + mg However, the increase in potential energy ∆E p of the raised
Pulley is in equilibrium under four forces. Three forces as h
water column = mg , where m is the mass of the raised
shown in figure and the fourth, which is equal and 2
column of water.
opposite to the resultant of these three forces, is the force
∴ m = π r 2 hd
applied by the clamp on the pulley (say F).
 gh  π r h dg
2 2

Resultant R of these three forces is R = ( (M + m)2 + M 2 )g So, ∆E p = (π r 2 hd )   =


 2  2
Therefore, the force F is equal and opposite to R as shown π r 2 h 2 dg
Further, ∆W − ∆E p =
in figure. 2
∴ F = ( ( M + m) + M ) g .
2 2 The part (∆W − ∆E p ) is used in doing work against viscous
 forces and frictional forces between water and glass
4. (c) Net acceleration a of the bob in position
A
surface and appears as heat.
∆W − ∆E p π r 2h 2 dg
 So, Heat released =

an J 2J
a B 1 1
 8. (a) Energy stored, U = × (force)×(extension) = Fx
at 2 2
B has two components. FL YAx
 As Y = ⇒ F=
(a) an = radial acceleration (towards BA) Ax L
 1  YA 1 YAx 2
(b) at = tangential acceleration (perpendicular to BA) 
∴ U=  x x =
 2 L  2 L
Therefore, direction of a is correctly shown in option (c).
dV / dp
5. (c) M = Mass of the square plate before cutting the holes 9. (a) β == − = compressibility of gas
V
 M  2 πM
Mass of one hole m =  2 
πR = =
1 1
= under isothermal
 16 R  16 and β =
Bulk modulus of elasticity p
54 Physics
conditions. Thus, β versus p graph will be a rectangular BV
14. (a) Let the real dip be φ, then tan φ =
hyperbola. BH
KA (θ1 − θ 2 ) For apparent dip,
10. (c) Applying, Q =
l BVV BV 2 BV
=
tan φ ′ = = =
temperature difference across the rods are 20°C and 10°C BH cos ββ BH cos 30° 3 BH
respectively.
2 −1  3 
∴ Displacement of the joint = α 2t2 − α1t1 or tan 45° = .tan φ or φ = tan  
3  2 
= 10 α 2 − 20α1 = 10 (α 2 − 2α1 )
15. (c) At P : Bnet = B12 + B22
1 1
11. (c) ⋅ m1v 2 = JJ mm11ss ∆θ
∆ 11 B2
2 2
P B1
( 50 )
2
v2 a
⇒ ∆θ
∆ 1=
= == = 1.4 °C A
44Js 4 × 4.2 × 103 × 0.105 i1
C D
m1 s ∆θ1 = m2 s ∆θ 2 O i2
B
m 1
⇒ ∆θ 2 = 1 ∆θ1 = ×1.4 = 7°C
µ0
2 2
m2 0.2  µ 2i   µ 2i 
=  0 1  + 0 2  = (i12 + i22 )1/ 2
12. (c) Point P lies at equatorial positions of dipole 1 and 2  4π a   4π a  2π a
and axial position of dipole 3. Hence field at P
2
–Q 1 +Q 16. (a) Speed of the magnet v1 = = 2m
m/s
/s
1
–Q +Q 1
P Speed of the coil v2 = = 2 m/s
m/s
0.5
E2 E1 E3 3
Relative speed between coil and magnet is zero, so there
–Q 2 +Q is no induced emf in the coil.
k. p
Due to dipole 1 E1 =
(towards left)
x3
N S
k. p
Due to dipole 2 E2 = 2 (towards left) v1
x
v2
k .(2 p )
Due to dipole 3 E3 = (towards right)
x3 17. (b) Since voltage is lagging behind the current, so there
So, Net field at P will be zero. must be no inductor in the box.
13. (c) The two capacitor C1 and C2 are connected in series I f I f
18. (b) = ; where u = f + x ∴ =−
Kε A K ε A 6 mm 4 mm O f −u O x
C1 = 1 0 , C2 = 2 0
d1 d2
19. (a) Kinetic energy of recoil electron, Ek = hv − hv′
1 1 1 d1 d2
= + = , = 12.375 − 9.375 = 3eV
C C1 C2 K1 ε 0 A K 2 ε 0 A
9
1  d1 d 2  20. (b) Vb = ib Rb ⇒ Rb = = 257 kkΩ

= + 35 ×10−6
ε 0 A  K1 K 2 
21. (c) s1 + s2 = 100
1  6 ×10−3 4 × 10−3 
= +
ε 0 A  10 5 
 10 mm

1 2 1
gt + ut − gt 2 = 100 m
2 2
1 14 ×10−3
=
ε 0 A 10 ⇒ ut = 100 m ⇒ t =
100 100
= == 1ss
u 100
5000
This gives C = ε0 A
7
Mock Test-4 55
22. (d) x = 4(cos πt + sin πt ) ρliwuidVshell  M  ρliquid
⇒ M shell = ⇒  shell  =
 1 1  2  Vshell  2
4 2 cos π t + sin π t 
 2 2  ρliquid 1
⇒ σ shell = ⇒ R.d = .
 π π   π 2 2
= 4 2 sin cos π t + cos sin π t  = 4 2 sin  π t + 
 2 2   4 1
If R.d < , Then the cylinder should be more than half
Standard equation of displacement is x = a sin (ωt + φ ) 2
filled so that it is half submerged and floating.
Comparing the given equation with standard equation
a=4 2 3. (a) P′(54 − h) = P0 8 (Boyel’s Law)

λ P0 8
23. (b) Path difference (πr − 2r) = =
32
= 16 ⇒ P′ = ⇒ P′ + ρ gh = P0
2 2 54 − h
8 P0
2l l ⇒ + ρ gh = P0
54 − h
 46 − h 
2Y θ Y ⇒ ρ gh =   P0 ( P0 = ρ g (76))
θ1 θ2  54 − h 

16 (54-h)
⇒ r= = 14cm P0 P’
π−2 54 cm
24. (b) Let θ be the temperature of the junction (say B). h
Thermal resistance of all the three rods is equal. Rate of
heat flow through AB + Rate of heat flow through CB =  46 − h 
⇒ h=  (76) ⇒ 54h − h2 = 76 × 46 − 76h
Rate of heat flow through BD  54 − h 
A
90°C ⇒ h2 − 130h + 76 × 46 = 0 ⇒ ( h − 38) ( h − 92) = 0
D B ⇒ h = 38,92 ⇒ h = 38cm

0°C Not possible
90°C
4. (a) Pressure at 0 level is same from both sides.
C

90°− θ 90°− θ θ − 0
∴ + = , Here, R =Thermal resistance
R R R α
α
∴ 3θ = 180°, or θ = 60°C d
α
25. (c) Resistance ( 6 × 5) = 30 Ω and 6 Ω are in parallel; so
0
30 × 6
equivalent resistance R = =5Ω d1 (1 − sin α ) = d1 (1 − cos α ) + d 2 [cos α + sin α ]
30 + 6
JEE Advance Paper -I d1 sin α + cos α
⇒ =
d 2 cos α − sin α
1. 2. 3. 4. 5. 6. 7. 8. 9. 10.
b a a a c a,b,d 4 2 6 2
2ρw g
11. 12. 13. 14. 15. 16. 17. 18. 5. (c) R 2 , The downward force on the bubble due to
3T
6 6 4 2 d a d c
2π Tr 2
surface tension = 2πr.T sin θ =
1. (b) P1 = pressure just inside the bubble at the end R
4T R
2 = P0 +
R θ
2. (a) If water is half filled M shell g = ρ ( ∆V ) displaced g

V 
⇒ ( ∆V ) =  shell  r
 2  θ
δT
56 Physics
The upward buoyant force exceeds, the surface tension Amplitude R of resultant wave is
force then the bubble detaches. R = a 2 + b 2 + 2 ab cos φ
2ρw g
∴ 4 π R 3 ρ w g = 2π Tr ⇒ r = R 2
2
R 2 = a 2 + b2 + 2 ab cos φ
3 R 3T
As intensity ∝ (amplitude)2
6. (a, b, d) A 2dF Vg ∴ Resultant intensity, I R = I1 + I 2 + 2 I1 I 2 cos φ

(d A + d B )Vg = I + 2 I + 2 I × 2 I × cos 60°


B = 3 I + I 2 = I (3 + 2) = 4.414 I

7. (4) No. of photons emitted per second by 25 watt source 14. (2) See figure
Vd VR
will be
E Eλ 25 × (6600 ×10−10 ) 25 R
n= = = =
hc / λ hc (6.6 × 10−34 ) × (3 ×108 ) 3 × 10−19 I

Current, I = 3% of number of photoelectrons emitted per V

sec × charge of electron


Voltage drop across diode is Vd = 1.0V
V
3 3 25
= ne × ×1.6 ×10−19 = 0.4A = 4 deciampere Voltage drop across R is VR = RI
100 100 3 ×10−19
4 1 = 100 ×10 × 10−3 = 1.0V
8. (2) de-Broglie wavelength, λ = or λ ∝
2 mqV V Vmax = Vd + VR = 1.0 + 1.0 == 2.0V
2.0V
15. (d) It is ω for both case.
λ2 V1 4V
∴ = = =2 ω k̂
λ1 V2 4V − 3V

−ω (ℓ + d cos 45°)iˆ
h h2 1
9. (6) As λ = or EK = or EK ∝ 2
2mEK 2 mλ 2
λ
2 2 −ω ℓiˆ
E K1 λ   0.5  1
∴ = 2 =  =4 P d cos 45°
EK2 λ
 1  1  As angular velocity of a rigid body about any point should
or EK1 = 4 EK1 = 4 × 2 eV == 88 eV   
eV be same. v = vPC + vCO
   
Increase in energy = EK2 − EK1 = 8 − 2 = 6eV ⇒ vPO = rPC + ω '+ vCO
d ˆ ˆ   d 
 h   h  −ω Liˆ = − (k + j ) × ω + −  ω L + .45°  iˆ
10. (2) L = 2   and L ' = 4   2 2  2 
 2π   2π  
⇒ rp = ℓˆj
∴ L ' = 2 L = n L, where n = 2
  d  d 
11. (6) As is clear from figure. E1 = E2 + E3 ⇒ rc =  ℓ +  cos 45°   ˆj +  sin 45°  kˆ
  2    2 
hc hc hc
= +  d  d 
λ1 λ2 λ3 ⇒ rPC = −  cos 45°  ˆj −  sin 45°  kˆ
2  2 
1 1 1 1 1 1
∴ = − = − = = 6 units   d 
λ2 λ1 λ3 2 3 6 ⇒ vCO = −  ℓ + cos 45°  ωiˆ
 2 
12. (6) 3rd excited state means n = 4. No. of different ωd ˆ d ˆ  
⇒ i =− (i + j ) × ω ' ⇒ ω ' = ω kˆ
n( n − 1) 4(4 − 1) 2 2 2 2
wavelength observed N = = =6
2 2
16. (a) V = ω ℓ 2 + r 2 TOP View
13. (4) Here, I1 = I
 2V sin θ  ω ℓ 2 + r 2 r
W = = =ω
I 2 = 2 I ,φ = 60°  2r  ℓ + r2
r 2
Mock Test-4 57
Here there is a vrel of P′ and Q′ (not for P, Q) (vrel)P’Q’ will JEE Advance Paper -II
be in horizontal plane 1. 2. 3. 4. 5. 6. 7. 8. 9. 10.
c c a a,b,c a a,b 2 2 6 8
p1
ℓ+r 2 11. 12. 13. 14. 15. 16. 17. 18.
1 4 3 2 b b d d
θ θ ω0 ℓ 2 + r 2
ℓ2 + r2 F
θ 1. (c) F

⇒ ω must be along vertical. Same situation will be there for 2L, 2R


L, R

P as mentioned ω should be same for both cases.


π 4R 2 x π R2 y
⇒ k1 = , k2 =
17. (d) mrω 2 = ma 2L L
y k
vdv ⇒ F = k1 x = k2 y ⇒ = 1 = 2
⇒ a = rω2 ⇒ = rω 2 x k2
dr
v r
R2 F ∆ℓ ∆ℓ
⇒ = α∆T
⇒ ∫ vdv = ω ∫ rdr ⇒ v = ω r2 − 2. (c) =Y
2

0 R/ 2
4 A ℓ ℓ
F
= Y α∆T
v t
dr ⇒ P=
∫ R2
= ∫ ω dt . . .(i) A
r2 −
R/2 0

4 = 2 ×1011 × 1.1 × 10−5 × 100 = 2.2 × 108 Pa/ 2

R ℓ1 ℓ
Assume: r = sec θ 3. (a) T = 2π , TM = 2π 2
2 g g
R ℓ1 ℓ
⇒ dr = sec θ tan θd θ T 2 = 4π 2 ⋅ , TM2 = 2π 2 2
2 g g
R
sec θ tan θd θ t TM2 ℓ 2
2 ⇒ = x
⇒ ∫ R2
= ∫ ω dt
0
T 2 ℓ1
tan θ
2

4 TM2 ℓ ℓ −ℓ
⇒ 2
−1 = 2 −1 = 2 1
 2r  T ℓ1 ℓ1
4r 2 − R 2
⇒ ω t = ℓn  +  Mg ℓ1
 R R  ⇒ γ= ⋅
A (ℓ 2 − ℓ 1 )
R ωt
⇒ r= [ e + e −ω t ]
A  TM  
2
4 1 A (ℓ 2 − ℓ1 )
⇒ = ⋅ =   − 1
  γ Mg ℓ1 Mg  T  
18. (c) Frot = Fin + 2m(vrot iˆ) × ωkˆ + m(ωkˆ × riˆ) × ωkˆ
 A B C
mrω 2iˆ = Fin + 2 mvrotω ( − ˆj ) + mω 2 riˆ 4. (a, b, c) x = (1 − cos 2ωt ) + (1 + cos 2ωt ) + sin 2ωt
2 2 2

Fin = 2mvrω ˆj . . .(i) C
For A = 0, B = 0 x = sin 2ωt
R ωt −ωt ω (kˆ) y ( ˆj ) 2
r= e + e  For A = − B and C = 2 B x = B cos 2ω t + B sin 2ω t
4
dr R Amplitude = | B 2 |
= vr = ωeωt − ωe−ωt  r vr
dt 4 For A = B; C = 0 x=A

 Rω ωt −ωt ˆ Hence this is not correct option
Fin = 2m e − e  ω j
4  For A = B, C = 2 B x = B + B sin 2ωt
Also reaction is due to disc surface then
It is also represents SHM.
 mRω 2 ωt (a) Solenoid consists of circular current loops which are
Freaction = e − e −ωt  ˆj + mgkˆ 5.
2  placed in uniform field, so magnetic fore on both
solenoids = 0.
58 Physics
– λt − λt
F1 12. (4) f = (1 − e ) = 1− e ≈ 1 − (1 − λ t ) = λ t
⇒ f = 0.04 . Hence % decay ≈ 4%
B=0
t

13. (3) A =  1 
T
B2
A0  2 
x x x x x x x x x x x x Where, A0 is the initial activity of the radioactive material

x x x x x x x x x x x x and A is the activity at t.


t
 
⇒ F1 = F2 = 0 12.5  1  T
So, =  ∴ t = 3T .
100  2 
1U 
6. (c) P =  ∝T
4
1 1
3 V  14. (2)λλpP = ; λQ =
τ 2τ
For an ideal gas PV = nRT
RP ( A λ ) e− λ P t R 2
nR′T ⇒ = 0 P −λQ t , At t = 2τ; P =
∴ P= ( R′ -molar gas constant) RQ A0 λ Q e RQ e
V
15. (b) A-2, B-1, C-4, D-3
nR′
∴ T ∝ e− R ⇒ ∝T3 1 2 p2
V Kinetic energy K = mv or, K = or p = 2mK
1 1 1 2 2m
1
∴ ∝T3 ⇒ ∝T3 ∴ T3 ∝ 3 ⇒ T∝ If K = 0, p = 0 as above
V 4π 3 R R
R
3 For different mass m1 and m2 if E1 = E2
nh 3h p12 p2 p m1
7. (2) mvr = = de-Broglie wavelength = 2 or 1 =
2π 2π 2m1 2m2 p2 m2
h 2π r 2π a0 (3)2
λ= = = = 2π a0 E1 p 2 2m m
mv 3 3 z Li If p1 = p2 for different masses. = 1 × 2 = 2
E2 2m1 p2 m1
8. (2) E photon = Eionize atom + Ekinetic energy
16. (b) A-2, B-4, C-3, D-1
1242 13.6 Waves having frequency less than minimum audible
⇒ = 2 + 10.4 . From this, n = 2
90 n frequency are infrasonics.
9. (6) [Here we are assuming that in original paper atom Waves of frequency between 20Hz to 20,000Hz are called
 1 1  12370 audible waves. Waves having frequency > 20,000Hz
⇒ atoms] 13.6  2 − 2  =
1 n  970 are ultrasonic waves. Supersonic objects are those which
⇒ n 2 = 16 ⇒ n = 4 have velocity more than velocity of sound.
4×3 17. (d) A-4, B-3, C-2, D-1
No. of lines = 4 C 2 = =6
2 The frequency ranges of various waves are as under :
−λt
10. (8) N = N0e Radiowaves; 5 ×105 to 109 Hz;;
⇒ ℓn | dN / dt | ℓn( N 0 λ ) − λ t γ-rays; 3 × 1018 to 5 ×1022 Hz
1 Microwaves; 1× 109 to 3 × 1011 Hz;;
From graph, λ = per year
2 X-rays; 1× 1016 to 3 ×1021 Hz..
0.693 18. (d) A-2, B-3, C-4, D-1
⇒ t1/ 2 = = 1.386 year
1/ 2 A transducer is a device that has input in electrical form or
⇒ 4.16 yrs =3t1/ 2 ∴ p = 8 provides output in electrical form. Attenuation is loss of
strength of a signal during propagation. Range is largest
A
11. (1) A = λ N ⇒ N = = Aτ distance between transmitter and receiver. Bandwidth is
λ
range of frequencies over which communication system
So M = mN = mAτ = 10−25 × 1010 × 109 = 10−6 kg = 1 mg works.
  
Mock Test-5 59
JEE-MAIN: PHYSICS MOCK TEST-5

SECTION 1 (Multiple Choice Question) 5. A mass M is suspended from a spring of negligible mass.
The spring is pulled a little and then released, so that the
1. Frequency is the function of density ( ρ ) , length (a) and
mass executes SHM of time period T. If the mass is
surface tension (T ) . Then its value is
increased by m, the time period becomes 5T/3. The ratio
a. k ρ 1/ 2 a 3/ 2 / T b. k ρ 3/ 2 a 3/ 2 / T of m/M is
1/ 2 3/ 2
c. k ρ a / T
3/ 4 1/ 2 1/ 2
d. k ρ a / T
3/ 2 5 3 16 28
a. b. c. d.
3 5 9 9
2. With respect to a rectangular cartesian coordinate system, 6. The gravitational potential difference between the surface
r r
three vectors are expressed as: a = 4iˆ − ˆj , b = −3iˆ + 2 ˆj of a planet and a point 10 m above is 4.0 J/kg. The
r gravitational field in this region, assumed uniform is
and c = − kˆ where iˆ, ˆj , kˆ are unit vectors, along the X, Y
a. 0.025 N/kg b. 0.40 N/kg
and Z-axis respectively. The unit vectors r̂ along the c. 40 N/kg d. 4.0 N/kg
direction of sum of these vector is: 7. 1000 drops of water all of same size join together to form
1 ˆ ˆ ˆ 1 ˆ ˆ ˆ a single drop and the energy released raises the
a. rˆ = (i + j − k ) b. rˆ = (i + j − k )
3 2 temperature of the drop. Given the T is the surface tension
1 1 ˆ ˆ ˆ of water. r the radius of each small drop, ρ the density of
c. rˆ = (iˆ − ˆj + kˆ) d. rˆ = (i + j + k )
3 2 liquid, J the mechanical equivalent of heat. What is the
3. If W1 , W2 and W3 represent the work done in moving a rise in temperature?
a. T / Jr b. 10T / Jr
particle from A to B along three different paths 1, 2 and 3
respectively (as shown) in the gravitational field of a c. 100T / Jr d. None of these
point mass m. Find the correct relation between W1 , W2 8. A simple pendulum is made by attaching a 1 kg bob to
and W3 . 5 m long copper wire. Its period is T. now if 1 kg bob is
replaced by 10 kg bob, the period of oscillations:
B a. remains T
b. becomes greater than T
1 2 c. becomes less than T
3
d. any of above answer depends on locality
A
9. On which of the given scales of temperature, the temperature
a. W1 > W2 > W3 b. W1 = W2 = W3 is inverse negative?
c. W1 < W2 < W3 d. W2 > W1 > W3 a. Celsius b. Fahrenheit
4. The moment of inertia of a uniform rod of length 2l and c. Reaumur d. Kelvin
mass m about an axis xx′ passing through its center and 10. An ideal gas is initially at temperature T and volume V. Its
inclined at an angle α is: volume is increased by ∆V due to an increase in
x temperature ∆T , pressure remaining constant. The quantity
δ = ∆V / V ∆T varies with temperature is:
A α B
C
a. b.
x′
2 ml 2 T T
a. ml sin 2 α b. sin 2 α T T
3 12
ml 2 ml 2
c. cos 2 α d. cos 2 α c. d.
6 2
T T
T T
60 Physics
11. A lead ball moving with velocity V strikes a wall and µ0 M µ0 M
a. ( 2) 3 b. ( 3) 3
stops. If 50T of its energy is converted into heat, then 4π d 4π d
what will be the increases in temperature (specific heat of
 2µ  M µ0 M
lead is s)? c.  0  3 d. ( 5) 3
 π d 4π d
2V 2 V2
a. b.
Js 4 Js 16. The figure shows four wire loops, with edge lengths of
2 2 either L or 2L. All four loops will move through a region
V s V s 
c. d. of uniform magnetic field B (directed out of the page) at
J 2J
the same constant velocity. Rank the four loops according
12. The plots of intensity versus wavelength for three black to the maximum magnitude of the e.m.f. induced as they
bodies at temperatures T1 , T2 and T3 respectively are as move through the field, greatest first:
shown. Their temperatures are such that • • • • •

I • • • • •
T3 • • • • •
a b
T1 T2 • • • • •
c d
a. (ec = ed ) < (ea = eb ) b. (ec = ed ) > (ea = eb )
c. ec > ed > eb > ea d. ec < ed < eb < ea
λ
a. T1 > T2 > T3 b. T1 > T3 > T2 17. A resistance is connected to an AC source. The average
power dissipation in the resistance is
c. T2 > T3 > T1 d. T3 > T2 > T1
a. 50% of the peak power
13. The electric field in a region surrounding the origin is b. 25% of the peak power
uniform and along the x-axis. A small circle is drawn with c. 41.4% of the peak power
the centre at the origin cutting the axes at points A, B, C, D d. equal to peak power
having co-ordinates (a, 0), (0, a), (– a, 0), (0, – a); 18. When a point source of monochromatic light is at a
respectively as shown in figure then potential in minimum distance of 0.2 m from a photoelectric cell, the cut-off
at the point voltage and the saturation current are 0.6 V and 18 mA

B E respectively. If the same source is placed 0.6 m away from
the photoelectric cell, then
C a. the stopping potential will be 0.2 V
b. the stopping potential will be 0.6 V
D
c. the saturation current will be 6 mA
a. A b. B d. the saturation current will be 18 mA
c. C d. D
19. A beam of fast moving alpha particles were directed
14. A spherical capacitor consists of two concentric spherical towards a thin film of gold. The parts A ′, B ′ and C ′ of the
conductors of inner one of radius R1 maintained at
transmitted and reflected beams corresponding to the
potential V1 and the outer one of radius R2 at potential V2 . incident parts A, B and C of the beam, are shown in the
The potential centre ( R2 > x > R1 ) is: adjoining diagram. The number of alpha particles in

V1 − V2 V1 R1 ( R2 − x ) + V2 R2 ( x − R1 )
a. ( x − R1 ) b. B′
R2 − R1 ( R2 − R1 ) x B
A A′
Vx V +V
c. V1 + 2 d. 1 2 x C
R1 − R1 R2 + R1 C′
15. Two identical short bar magnets, each having magnetic a. B′ will be minimum and in C′ maximum
moment M, are placed a distance of 2d apart with axes b. A′ will be maximum and in B′ minimum
perpendicular to each other in a horizontal plane. The c. A′ will be minimum and in B′ maximum
magnetic induction at a point midway between them is d. C′ will be minimum and in B′ maximum
Mock Test-5 61
20. Energy released in the fission of a single 92 U 235
nucleus a. 1.29 km/sec b. 2.58 km/sec
c. 3.87 km/sec d. 5.16 km/sec
is 200 MeV. The fission rate of a 92 U 235 fuelled reactor
24. A battery of 10 cells each of e.m.f. E = 1.5 V, and internal
operating at a power level of 5W is
resistance 0.5Ω has 1 cell wrongly connected. It is being
a. 1 . 56 × 10 +10 s −1 b. 1 . 56 × 10 +11 s −1
charged by 220V power supply with an external resistance
c. 1 . 56 × 10 +16 s −1 d. 1 . 56 × 10 +17 s −1
of 47 Ω in series. The potential difference across the
SECTION 2 (Numeric Value Question) battery is:
21. A train of 150 m in length is going towards north direction a. 32 V b. 8 V
at a speed of 10 m/s. A parrot flies at a speed of 5 m/s c. 180 V d. 188 V
towards south direction parallel to the railway track. Then 25. A cell is connected between the points A and C of a
the time taken by the parrot to cross the train is equal to: circular conductor ABCD of centre O with angle
a. 12 sec b. 8 sec A OC = 60°. If B1 and B2 are the magnitudes of the
c. 10 sec d. 15 sec magnetic fields at O due to the currents in ABC and ADC
22. What is the maximum value of the force F such that the B
respectively, the ratio 1 is
block shown in the arrangement, does not move? B2
F m = 3 kg i1
1
60° µ= B
300o
2 3
O

a. 20 N b. 10 N 60o
c. 12 N d. 15 N A C

D
23. When an astronaut in a rocket reaches near the moon, he 1A i2
sends a radiowave of frequency 5000 MHz towards the
moon. He finds that the reflected wave from the moon has a. 0.2 b. 6
a frequency 86 kHz more than the real frequency. The c. 1 d. 5
velocity of the rocket with respect to the moon is:

Space for Rough Work


62 Physics

JEE ADVANCE PAPER-I

SECTION 1 (Maximum Marks: 24) 4. A 2µF capacitor is charged as shown in figure. The
MCQs with one or more than one correct answer percentage of its stored energy dissipated after the switch
S is turned to position 2 is
1. Given, 1 2
R1 = 1 Ω C1 = 2 µ
µFF s
R2 = 2 Ω C 2 = 4 µF
F
C1 V
2µF 8µF
C1 C2

R2 C2 R1 R2
R1 C1 C2 a. 0% b. 20% c. 75% d. 80%
V
R2
5. In the given circuit, a charge of +80 µF is given to the
+ V –
R2 V upper plate of the 4µF capacitor. Then in the steady state,
(I) (II) (III) the charge on the upper plate of the 3µF capacitor is.
The time constants (in µS) for the circuits I, II, III are
respectively +80 µC 4 µF
a. 18, 8 / 9, 4 b. 18, 4, 8 / 9
c. 4, 8 / 9, 18 d. 8 / 9, 18, 4 2 µF 3 µF

2. A circuit is connected as shown in the figure with the


switch S open. When the switch is closed the total amount
of charge that flows from Y to X is a. +32 µC b. +40 µC
3µF 6µF c. +48 µC d. +80 µC
X
6. Two capacitors C1 and C2 are charged to 120 V and 200 V
S
respectively. It is found that by connecting them together
3Ω 6Ω the potential on each one can be made zero. Then
Y a. 5C1 = 3C 2 b. 3C1 = 5C 2
9V c. 3C1 + 5C 2 = 0 d. 9C1 = 4C 2
+ –
a. 0 b. 54 µC SECTION 2 (Maximum Marks: 24)
c. 27 µC d. 81 µC Numerical value answer type questions
3. A parallel plate capacitor C with plates of unit area and
separation d is filled with a liquid of dielectric constant 7. The magnetic flux of φ (in weber) in a closed circuit of
resistance 3 ohm varies with time t (in second) according
d
K = 2. The level of liquid is initially. Suppose the
to the equation φ = 2 t 2 − 10t + 3. What will be the
3
liquid level decreases at a constant speed V , the time magnetic of induced current at t = 0.25 s ?
constant as a function of time t is 8. An a.c. generator gives an output voltage of E = 170 sin
56⋅52t. What is the frequency of alternating voltage
C
d
produced?
d R
3 9. A condenser of capacitance 0.144 pH is used in a
transmitter to transmit at wavelength λ. If inductance of
1
6ε 0 R (15d + 9Vt )ε 0 R mH is used for resonance, what is the value of λ?
a. b. π2
5d + 3Vt 2d 2 − 3dVt − 9V 2t 2
6ε 0 R (15d − 9Vt )ε 0 R 10. In the series RLC circuit as shown in Fig., what would be
c. d. the ammeter reading?
5d − 3Vt 2d 2 + 3dVt − 9V 2t 2
Mock Test-5 63
200 V 200 V 15. The piston is now pulled out slowly and held at a distance
2L from the top. The pressure in the cylinder between its
top and the piston will then be
R = 500 Ω L P0
C a. P0 b.
A 2
100 V, 50 Hz P0 Mg P0 Mg
~ c. + d. −
2 π R2 2 π R2
11. An a.c. source of frequency 1000 Hz is connected to a coil 16. While the piston is at a distance 2L from the top, the hole
200 at the top is sealed. The piston is then released, to a
of mH and negligible resistance. If effective current
π position where it can stay in equilibrium. In this condition,
through the coil is 7.5 mA, what is the voltage across the the distance of the piston from the top is
coil?  2 P0π R 2   P π R 2 − Mg 
a.   (2 L ) b.  0  (2 L )
 π R P0 + Mg   π R P0
2 2
12. Two alternating currents are given by I1 = I 0 sin ( ωt − φ ) 

and I 2 = I 0 cos (ωt + φ ) . What is the ratio of virtual values  P π R 2 + Mg   P0π R 2 


c.  0  (2 L ) d.   (2 L )
 π R P0  π R P0 − Mg 
2 2
of the two currents? 

13. A silver sphere of radius 1 cm and work function 4.7 eV is


Paragraph for Question No. 17 to 18
suspended from an insulating thread in free-space. It is
The nuclear charge (Ze) is non-uniformly distributed within a
under continuous illumination of 20 nm wavelength light.
nucleus of radius R. The charge density ρ ( r ) [charge per unit
As photoelectrons are emitted, the sphere gets charged
volume] is dependent only on the radial distance r from the
and acquires a potential. The maximum number of photo-
centre of the nucleus as shown in figure. The electric field is
A × 10Z (where 1 <
electrons emitted from the sphere is A
only along the radial direction.
A < 10). The value of Z is
ρ (r )
14. When a piece of metal is illuminated by monochromatic
light of wavelength λ then the stopping potential for d

photoelectric current is 3V0. When the same surface is


illustrated by light of wavelength 2λ, then stopping
o
potential becomes the V0 value of Threshold wavelength a R r
for photoelectric current is aλ, where the value of a is 17. The electric field at r = R is
a. independent of a
SECTION 3 (Maximum Marks: 12)
b. directly proportional to a
Paragraph based questions (2 paragraphs, each having 2
c. directly proportional to a 2
MCQs with one correct answer only)
d. inversely proportional to a
Paragraph for Question No. 15 to 16 2
A fixed thermally conducting cylinder 18. For a = 0, the value of d (maximum value of ρ as shown
L
has a radius R and height L0 . The in the figure) is
cylinder is open at its bottom and has a 3Ze 3Ze
a. b.
small hole at its top. A piston of mass
L0 4π R3 π R3
M is held at a distance L from the top 4Ze Ze
c. d.
surface, as shown in the figure. The 3π R3 3π R3
Piston
atmospheric pressure is P0 .
Space for Rough Work
64 Physics

JEE ADVANCE PAPER-II

SECTION 1 (Maximum Marks: 24) 4. Figure shows three resistor configurations R1, R2, and R3
MCQs with one or more than one correct answer connected to 3V battery. If the power dissipated by the
configuration R1, R2, and R3 is P1, P2, and P3, respectively,
1. A parallel plate capacitor has a dielectric slab of dielectric
then
constant K between its plates that covers 1/3 of the area of
1Ω 1Ω
its plates, as shown in the figure. The total capacitance of 1
1 +
the capacitor is C while that of the portion with dielectric + 1 1 3V
1 +
3V –
in between is C1. When the capacitor is charged, the plate 1 1 3V 1Ω 1Ω
– E2 –
area covered by the dielectric gets charge Q1 and the rest 1 1 1 1Ω
of the area gets charge Q2. The electric field in the
R1 R2 R3
dielectric is E1 and that in the other portion is E2. Choose
a. P1 > P2 > P3 b. P1 > P3 > P2
the correct option/options, ignoring edge effects.
E E 1 c. P2 > P1 > P3 d. P3 > P2 > P1
a. 1 = 1 b. 1 =
E2 E2 K 5. Incandescent bulbs are designed by keeping in mind that
Q 3 C 2+ K the resistance of their filament increases with the increase
c. 1 = d. =
Q2 K C1 K in temperature. If at room temperature, 100 W, 60 W and
40 W bulbs have filament resistances R100, R60 and R40,
2. A parallel plate capacitor having plates of area S and plate
respectively, the relation between these resistances is
separation d, has capacitance C1 in air. When two
1 1 1
dielectrics of different relative permittivities (ε1 = 2 and a. = + b. R100 = R40 + R60
R100 R40 R60
ε2 = 4) are introduced between the two plates as shown in
1 1 1
C2 c. R100 > R60 > R40 d. > >
the figure, the capacitance becomes C 2 . The ratio is R100 R60 R40
C1
d/2 6. To verify Ohm’s law, a student is provided with a test
resistor RT, a high resistance R1, a small resistance R2, two
identical galvanometers G1 and G2, and a variable voltage
S/2 source V. The correct circuit to carry out the experiment is
ε2
G1 G1
S/2
R2 R1
ε1 G2 G2
d a. RT R1 b. RT R2
a. 6/5 b. 5/3 c. 7/5 d. 7/3
3. Consider a cylindrical element as shown in the figure. V V

Current flowing the through element is I and resistivity of G1 G1


R1 R2
material of the cylinder is ρ . Choose the correct option out G2 G2
RT RT
the following c. d.
A R2 R1
B
C
4r I 2r V V

ℓ./ 2 ℓ./ 2
SECTION 2 (Maximum Marks: 24)
Numerical value answer type questions
a. Power loss in first half is four times the power loss in
second half 7. Image of an object approaching a convex mirror of radius
b. Voltage drop in first half is twice of voltage drop in of curvature 20 m along its optical axis is observed to
second half 25 50
move from m to m in 30 seconds. What is the
c. Current density in both halves are equal 3 7
d. Electric field in both halves is equal speed of the object in km per hour?
Mock Test-5 65
8. Water (with refractive index = 4/3) in a tank is 18 cm 12. A silver sphere of radius 1 cm and work function 4.7 eV is
7 suspended from an insulating thread in free space. It is
deep. Oil of refractive index lies on water making a
4 under continuous illumination of 200 nm wavelength
convex surface of radius of curvature ‘R = 6 cm’ as shown. light. As photoelectrons are emitted, the sphere gets
Consider oil to act as a thin lens. An object ‘S’ is placed charged and acquires a potential. The maximum number
24 cm above water surface. The location of its image is at of photoelectrons emitted from the sphere is A × 10 z
‘x’ cm above the bottom of the tank. Then ‘x’ is (where 1 < A < 10). The value of 'Z' is
S
R = 6 cm 13. A proton is fired from very far away towards a nucleus
µ = 1.0
with charge Q = 120 e, where e is the electronic charge. It
µ = 7/4 makes a closest approach of 10 fm to the nucleus. The de
Broglie wavelength (in units of fm) of the proton at its
µ = 4/3 start is: (take the proton mass,
1
9. Consider a concave mirror and a convex lens (refractive m p (5 / 3) × 10 −27 kg; h / e = 4.2 × 10−15 J .s / C; = 9 ×109
4πε 0
index = 1.5) of focal length 10 cm each, separated by a m / F ;1 fm = 10 −15 m)
distance of 50 cm in air (refractive index = 1) as shown in
the figure. An object is placed at a distance of 15 cm from 14. The work functions of silver and sodium are 4.6 eV and
the mirror. Its erect image formed by this combination has 2.3 eV, respectively. The ratio of the slope of the stopping
magnification M 1 . When the set-up is kept in a medium of potential versus frequency plot for silver to that of sodium
is
refractive index 7/6, the magnification becomes M 2 . The
M2 SECTION 3 (Maximum Marks: 12)
magnitude is
M1 Matching type questions with 4 options

15. Some laws / processes are given in Column I. Match these


with the physical phenomena given in Column II.
15 cm Column I Column II
(A) Intensity of light 1. Radius of aperture (R)
50 cm received by lens
10. A monochromatic beam of light is incident at 60° on one (B) Angular magnification 2. Dispersion of lens
face of an equilateral prism of refractive index n and (C) Length of telescope 3. Focal length fa , fe
emerges from the opposite face making an angle θ (n) (D) Sharpness of image 4. Spherical aberration
with the normal (see the figure). For n = 3 the value of a. A-1; B-3; C-3; D-1,2,3
dθ b. A-1,2,3; B-4; C-2,3, D-1
θ is 60° and = m. The value of m is c. A-1; B-3; C-1,2,3; D-2
dn
d. A-2,3; B-1; C-4; D-3
16. Study of properties of matter helps the student to know
60° θ some phenomenon having important applications of the
principles learnt by him in his daily life. Can you match
some of the phenomenon you observe with the principles
you have studied.
11. An α-particle and a proton are accelerated from rest by a Column I Column II
potential difference of 100 V. After this, their de Broglie (A) A cricketer spins his ball 1. Surface tension
λp while bowling to change
wavelengths are λα and λ p respectively. The ratio , to
λα the direction and
the nearest integer, is momentum of ball is
66 Physics
according to principle of (D) A sensitive magnetic 4. Soft iron of high
(B) A painter askes you for 2. Gravitational pull instrument can be shielded permeability
primers coating on walls very effectively from
before panting to outside magnetic fields by
remove sewage of water placing it inside a box of
through bricks in walls a. A-4; B-3; C-2; D-1 b. A-2; B-4; C-3; D-1
(C) Formation of stars in 3. Bernoulli’s pull c. A-1; B-2; C-3; D-4 d. A-4; B-1; C-3; D-2
universe
18. Column I defines the type of modulus or coefficient of
(D) Ships are asked to 4. Capillarity
elasticity. Column II gives the type of corresponding
spread oil on sea surface
during his tides modulus. Match the definition with proper type of
a. A-1; B-2; C-3; D-4 b. A-4; B-3; C-2; D-1 elasticity.
c. A-3; B-2; C-1; D-4 d. A-3; B-4; C-2; D-1 Column I Column II
(A) Ratio of longitudinal 1. Modulus of Rigidly
17. Match the statement of Column I with those in Column II:
or tensile stress to
Column I Column II
longitudinal strain
(A) Magnetic field intensity is 1. The spin motion
(B) Ratio of normal or 2. Poisson’s ratio
defined as of electron
hydrostatic stress to
(B) Which of the following, 2. Steel volumetric strain
the most suitable material
(C) Ratio of lateral strain 3. Bulk modulus
for making permanent
to longitudinal strain
magnet is
(D) Ratio of tangential 4. Young’s modulus
(C) In the case of bar magnet, 3. Run continuously
stress to shear strain
lines of magnetic through the bar
a. A-1; B-2; C-3; D-4 b. A-4; B-3; C-2; D-1
induction and outside
c. A-3; B-2; C-1; D-4 d. A-2; B-1; C-4; D-3
Space for Rough Work
Mock Test-5 67
ANSWER & SOLUTIONS ∆Vg 4
6. (b) f g = = = 0.4 N/kg
JEE-Main ∆r 10
4 3 4
1. 2. 3. 4. 5. 6. 7. 8. 9. 10. 7. (d) π R = n × π r3
d a b a c b d b d c 3 3
11. 12. 13. 14. 15. 16. 17. 18. 19. 20. R = ( n)1/ 3 r = 10r
b b a b d b a b b b
Decrease in surface area ∆A = n(4π r 2 ) − 4π R 2
21. 22. 23. 24. 25.
c a d a c
= 1000(4π r 2 ) − 4π (10 r ) 2
= 4π r 2 [1000 − 100] = 900 × 4π r 2
v [ LT −1 ]
1. (d) Velocity gradient = = = [T −1 ]
x [ L] 900× 4πr 2 T
Energy released = ∆A × T == cal
J
V [ ML2T −3 A−1 ]
Potential gradient = = = [ MLT −3 A−1 ] Let θ be the rise in temperature;’ then
x [ L]
4 900 × 4πr 2 T
E [ ML2T 2 ] 1000× πr 3 ×1× d θ =
Energy gradient = = = [ MLT −2 ] 3 J
x [ L]
2.7T
P [ ML−1T −2 ] ∴ dθ =
Pressure gradient = = = [ ML−2T −2 ] Jr
x [ L]
    l
2. (a) r = a + b + c = 4iˆ − ˆj − 3iˆ + 2 ˆj − kˆ = iˆ + ˆj − kˆ 8. (b) For simple pendulum T = 2π when weight
g

r iˆ + ˆj − kˆ iˆ + ˆj − kˆ suspended on elastic wire is increased, it length increases,
rˆ = = =
|r| 1 + 1 + ( −1)
2 2 2
3 so time period increases.

3. (b) Gravitational field is a conservative force field. In a 9. (d) Zero Kelvin = –237 °C (absolute temperature). As no
conservative force field work done is path independent matter can attain this temperature, hence temperature can
∴ W1 = W2 = W 3 never be negative on Kelvin scale.
x =+ l
10. (c) For an ideal gas : pV = nRT
4. (a) The desired moment of inertia is I = ∫
x =− l
dI
For p = constant p ∆ V = nT ∆ T
+l
m  ml 2 ∆V nR nR V
= ∫  dx  ( x sin α ) =
2
sin 2 α ∴ = = =
∆T p nRT T
−l  2l  3 T
V
∆V 1 1
x sin α ∴ = or δ =
α V ∆T T T

x
Therefore, δ is inversely proportional to temperature T.
dx
i.e., when T increases, δ decreases and vice-versa.
Hence, δ − T graph will be a rectangular hyperbola as
shown in the above figure.
5. (c) m1 = M , T1 = T
1 1 2
5T 11. (b) ⋅ mv = J ms ∆∆θ
⇒ m2 = M + m, T2 = 2 2
3
v2
T1 2π m1 / K m1 M ⇒ ∆θ =
⇒ = = = 4 Js
T2 2π m2 / K m2 M +m
2 12. (b) Wien’s displacement law for a perfectly black body is
M + m  T2   5T / 3  25
or, =  = = λmT = constant = Wien’s constant b
M  T1   T  9
Here, λm is the minimum wavelength corresponding to
m 16
⇒ = maximum intensity I.
M 9
68 Physics
1 19. (b) Because atom is hollow and whole mass of atom is
or λm ∝
T concentrated in a small centre called nucleus.
From the figure ( λm )1 < ( λm )3 < ( λm )2 Energy
20. (b) 200 MeV = 200 × 10 6 × 1 .6 × 10 −19 J
= 200
Therefore, T1 > T3 > T2 Fission
5
13. (a) In the direction of electric field, potential decreases. Fission rate = = 1 . 56 × 10 11 fission/sec.
200 MeV
14. (b) Let q1 and q2 be charges on inner and outer spherical
21. (c) Relative velocity of parrot and train
conductors.
= 10 + 5 = 15 m/s /
1  q1 q2 
Then V1 =  +  . . .(i) s 150
4πε 0  R1 R2  ∴ Time = = = 10 s
v 15
1 q1 + q2
Also V2 = . . .(ii) 22. (a) Free body diagram (FBD) of the block (shown by a
4πε 0 R2
dot) is shown in figure. For vertical equilibrium of the
1  q1 q2  block
Then potential at point P, V =  +  . . .(iii)
4πε 0  x R2  N Vertical

Solving (i) and (ii), q1 and q2 may be found. If these are f


F cos 60° Horizontal
substituted in (iii), we get
V1R1 ( R2 − x ) + V2 R2 ( x − R1 )
V= mg + F sin 60°
( R2 − R1 ) x F
N = mg + F sin 60° = 3 g + 3
2
15. (d) At point P net magnetic field Bnet = B12 + B22
For no motion, force of friction or f ≥ F cos 60°
2
1  3F  F
1 d d N ⇒ µ N ≥ F cos 60° or  3g +
 ≥
P B1 2 3 2  2
N
F
S B2 S or g ≥
2
µ0 2M µ M or F ≤ 2 g or 20N
where B1 = . 3 and B2 = 0 . 3
4π d 4π d Therefore, maximum value of F is 20 N.
µ 5M
⇒ Bnet = 0 . 3 23. (d) Here change in frequency takes place in two steps
4π d
2v
16. (b) Emf induces across the length of the wire which cuts ∴ ∆v = v
c
the magnetic field.
v
(Length of c = Length d) > (Length of a = b). ⇒ 86 × 103 = 2 × 5000 ×106
c
So, (ec = ed ) > ( ea = eb )
86 ×103
⇒ v=
17. (a) Peak power, P0 = V0 I 0 1010
V0 I 0 c = 86 × 10 −7 × 3 × 108 m/s
Average power dissipation, Pav = cos φ
2 = 2.58 ×103 m/s
For a resistive circuit φ = 0
24. (a) Current in circuit
⇒ cos φ = 1
220 − ( 9 − 1) ×1 ⋅ 5 220 − 12
V0 I 0 P0 i= = =4A
⇒ Pav = = = 50% of P0 47 + 0 ⋅ 5 ×10 52
2 2 ∴ Potential difference across battery which is being charged
18. (b) By changing distance the intensity change but
V = E + ir
frequency remains same; so stopping potential remains
= ( 8 × 1 ⋅ 5 ) + 4 × ( 0 ⋅ 5 × 10 ) = 32 V
same.
Mock Test-5 69
µ 0 θi i1 l2 θ22 1
(d) U i = × (2)V 2
25. (c) B = ⇒ B ∝θi (but = = ) 4.
4π r i2 l1 θ11 2
1 (2) × (8)
i1 ⇒ ∆U loss = × ×V 2
1 2 10
300o
O ∆U loss 4
⇒ ηlost = = = 80%
60o Ui 5

i2 2 5. (c) Charge on capacitor of 3 µ F


3
1A ⇒ Q = 80 = 48µ C
2+3
B1 θ i
⇒ = 1. 1 6. (b) Potential = 0 on connecting them together i.e., Q = 0
B2 θ 2 i2
i.e. C1V1 = C2V2 [capacitance is positive but they are
B1 θ θ
So, = 1 × 2 ⇒ B1 = B2 connected with opposite polarity]
B2 θ 2 θ1
⇒ 120 C1 = 200 C2
JEE Advance Paper-I ⇒ 3C1 = 5C 2
1. 2. 3. 4. 5. 6. 7. 8. 9. 10.
7. (3) Here, R = 3 Ω
d c a d c b 3 9 7 2
⇒ φ = 2 t 2 − 10t + 3
11. 12. 13. 14. 15. 16. 17. 18.
3 1 7 4 a d a b I = ? t = 0.25 g
dφ d
1. (d) τ 1 = 8 / 9 µµ S ⇒ e= = − (2t 2 − 10t + 3)
dt dt
⇒ τ 2 = 18 µµ S ⇒ τ 3 = 4 µµ S ⇒ e = – 4 t + 10
At t = 0.25 s, e = −4 × 0.25 + 10 = 9 volt
2. (c) 27 µ C
e 9
3µF 6µF ⇒ I= = =3A
X R 3
+18µC +18µC 8. (9) Here, E = 170sin 56.52t
S Compare with the standard form of equation of alternating
3Ω 6Ω emf
1A Y E = E0 sin ωt
9V
⇒ ω = 56.52
+ –
Initial charge distribution (when switch S is open) ⇒ 2π v = 56.52
3µF 6µF 56.52 56.52
+9µC X +18µC ⇒ v= = = 9 Hz.
2π 2 × 3.14
+9µC +36µC
S
9. (7) Here, C = 0.144 pF = 0.144 ×10−12 FF
+27µC 1 10−3
⇒ L= mH = H
3Ω Y 6Ω π2 π2
9V
1
⇒ υ=
Final charge distribution (when switch S is closed) 2π LC
2ε 0 ε0 v
. ⇒ λ= = υ.2π LC
d 2d υ
− vt + vt
3. (a) Cequivalent = 3 3
10 −3
2ε 0 ε0 = 3 × 108 × 2π × 0.144 × 10−12
+ π2
d 2d
− vt + vt
3 3 1.2 ×10−8
⇒ λ = 3 × 108 × 2π ×
∴ τ = Cequivalent R π
70 Physics
λ = 7.2 m 16. (d) Mg + P(π R ) = P0π R 2 2

The correct answer is 7 m.


⇒ P0 (2Lπ R 2 ) = P( xπ R 2 )
10. (2) As VL = VC = 200V
1 1 = PV
( PV 2 2 for isothermal process)

∴ The circuit is series resonance circuit Z = R  P0π R 2 


Eυ Ev 100 ⇒ x=  (2 L )
 π R P0 − Mg 
2
⇒ Iυ = = = = 2A
Z R 50
17. (a) independent of a
11. (3) Here, v = 1000 Hz, R
d
200 0.2 18. (b) q = ∫ ( R − x )4π x 2 dx = Ze
L= mH = H 2
π π 0

3Ze
Iυ = 7.5mA = 7.5 × 10−3 A ⇒ d=
π R3
Eυ = ?
As resistance of coil is negligible, JEE Advance Paper-II
∴ Eυ = Iυ ( X L ) = Iυ ( 2π vL )
1. 2. 3. 4. 5. 6. 7. 8. 9. 10.
 −3 0.2  a,d d b c d c 3 2 7 2
= 7.5 ×10  2π ×1000 × = 3V
 π  11. 12. 13. 14. 15. 16. 17. 18.

Iυ1 3 7 7 1 a d c b
I0 / 2
12. (1) = =1
Iυ2 I0 / 2 1. (a, d) As E = V/d
Phase difference has no impact on virtual value of current. E1 / E2 = 1 (both parts have common potential difference)
eV,, λ = 200 nm.
13. (7) Here, r = 1 cm = 10 −2 m; φ0 = 4.7 eV Assume C0 be the capacitance without dielectric for
Let Vs be the stopping potential. According to Einstein’s whole capacitor.
photoelectric equation (using hc = 1240 eV nm), we have C 0 2C 0
⇒ k + =C
hc 1240eV 3 3
eVs − φ0 = − 4.7eV
λ 200nm C 2+k Q k
⇒ + ⇒ 1 + .
= 6.2eV − 4.7eV = 1.5eV = 1.5eV C1 k Q2 2
∴ Vs = 1.5
S
4ε 0
The sphere will stop emitting photoelectrons, when the
2. (d) C10 = 2 = 4ε 0 S
potential on its surface becomes 1.5 V. Let n be the no. of d /2 d
photoelectrons emitted from sphere. C10 C20
The charge on sphere, q = ne, Vs = 1.5V
Vs × r × 4πε 0 (1.5) × (10 −2 ) 1
∴ n= = −19
× = 1.05 × 107
e 1.6 × 10 (9 × 109 )
C30
As per questions, A × 10 Z = 1.05 × 10 7 or Z = 7
2ε 0 S ε S
⇒ C20 = , C30 0
hc hc d d
3eV0 0 =
14. (4) 3eV − φ0 and eV
eV00 = − φ0
λ 2λ 1 1 1 d  1
⇒ = + = 1 + 2 
hc  1  hf hc
hc C10 C10 C10 2ε 0 S
'
 
2eV00 = 1 −  =
or 2eV or = 4λ
λ  2  2λ eV
eV0 0
4ε 0 S
⇒ C10' =
If λ0 is the stopping wavelength, then eV
eV00 = hc / λ0 3d
hc 7ε 0 S
or λ0 = = 4λ ⇒ C2 = C30 + C10' =
eV0 3d
15. (a) Po C2 7
⇒ =
C1 3
Mock Test-5 71
R1 A1 4 M2 M m2 M ℓ 2
3. (b) = = So = =7
R2 A2 1 M1 M m1 M ℓ1
P1 I 2 R1 4
⇒ = =
P2 I 2 R2 1 3
10. (2) Snell’s Law on 1st surface: = n sin r1
V1 IR1 4 J 1 2
⇒ = = ⇒ 1 =
V2 IR2 1 J2 4 3
sin r1 = . . .(i)
2n
2
V
4. (c) P = 3 4n 2 − 3
R ⇒ cos r1 = 1 − =
4n 2 2n
⇒ R1 = 1 Ω, R2 = 1/ 2 Ω, R3 = 2 Ω
r1 + r2 = 60° . . .(ii)
∴ P2 > P1 > P3 nd
Snell’s Law on 2 surface: n sin r2 = sin θ
5. (d) Power ∝1/ R Using equation (i) and (ii) n sin(60° − r1 ) = sin θ
6. (c) G1 is acting as voltmeter and G 2 is acting as ammeter.  3 1 
n  cos r1 − sin r1  = sin θ
7. (3) For v1 =
50
m, u1 = −25m  2 2 
7

⇒ v2 =
25
m, u2 = −50m
d  3

dn  4
( )

4n2 − 3 − 1  = cos θ


dn
3
25 18 For θ = 60 ° and n = 3
Speed of object = × = 3 kmph.
30 5 dθ
⇒ =2
7 dn
−1
7 1
8. (2) − = 4 1 2
4V1 −24 6 11. (3) mv = qV
2
⇒ V1 = 21 cm h
⇒ λ=
4/3 7/4 mv
⇒ − =0
V2 21 ⇒ λ = 8 ≃ 3.
⇒ V2 = 16 cm hc
12. (7) Stopping potential = −W
⇒ x = 18 − 16 = 2 cm λ
= 6.2 eV − 4.7 eV = 1.5 eV
9. (7) Image by mirror is formed at 30 cm from mirror at its
right and finally by the combination it is formed at 20 cm Kq
⇒ V = = 1.5
on right of the lens. So in air medium, magnification by r
7 1.5 × 10 −2
lens is unity. In second medium, µ = , focal length of ⇒ n= = 1.05 × 107
6 9 × 109 × 1.6 × 10 −19
the lens is given by, ⇒ Z =7
 1 1  1 2 1 (120)e2
1 (1.5 − 1)  −  13. (7) mv∞ = [By Conservation of energy]
10 =  R1 R2  2 4πε 0 (10 fm)
1  1.5  1 1  Assuming the nucleus to be considerably massive, we can
 − 1  − 
f  7 / 6   R1 R2  disregard its motion.
∴ Let momentum of proton be p = mv∞
35
⇒ f = cm
2 p2 1 (120)e2
∴ =
So, in second medium, final image is formed at 140 cm to the 2m p 4πε 0 (10 × 10−15 )
right of the lens. Second medium does not change the
5 120 × e 2
magnification by mirror. ∴ p = 9 × 109 × 2 × × 10 −27 ×
3 10 −14
72 Physics
⇒ p = 30 × 120 × 10 9 − 27 +14
×e 2 Primers contain materials which make obtuse angle of
contact due to their surface tension and do not allow
∴ p = 3600 × 10 −4 × e 2 capillary rise of water in pours of bricks.
∴ p = 60 × 10 −2 × e Interstellar dust particles attract each other by
h 42 × 10−15 42 gravitational pull to build a star.
h
∴ λ= = = = × 10−15 m Oil has smaller surface tension than water and is lighter
p e × 60 × 105 60 × 10−2 6
than water so it spreads move over the surface of water to
∴ λ = 7fm decrease height of tides.
14. (1) Slope of graph is h/e = constant ⇒ 1 17. (c) A-1; B-2; C-3; D-4
15. (a) A→1, B→3, C→3, D→1,2,3 18. (b) A-4; B-3; C-2; D-1
A, C and D in case of concave mirror convex lens image F l Longitudinal stress
can be real, virtual, diminished magnified or of same size. Y= × = = Young’s modulus
A ∆l Longitudinal strain
B is case of convex mirror image is always virtual (for
F V Normal stress
real object) K= × = = Bulk modulus
A ∆V Volumetric strain
16. (d) A-3; B-4; C-2; D-1 = Volume elasticity
According to Bernoulli’s theorem the spinning of ball Lateral strain
along with horizontal throw will change the velocity of Poisson’s ratio =
Longitudinal strain
streamlines of air above and below the ball which will
∆D
change the pressure on ball above and below it which ∆D l
cause change in its direction of motion. This is called ⇒ σ= D = ×
∆l D ∆l
magnus effect. l

  
Mock Test-1
Test Booklet code 73

A Mock Test 1 “JEE-Main”


Do not open this Test Booklet until you are asked to do so.
Read carefully the Instructions on the Back Cover of this Test Booklet.

Important Instructions:
1. Immediately fill in the particulars on this page of the Test Booklet with Blue/Black Ball Point Pen. Use of pencil is strictly
prohibited.
2. The Answer Sheet is kept inside this Test Booklet. When you are directed to open the Test Booklet, take out the Answer
Sheet and fill in the particulars carefully.
3. The test is of 3 hours duration.
4. The Test Booklet consists of 90 questions. The maximum marks are 360.
5. There are three parts in the question paper A, B, C consisting of, Physics, Chemistry and Mathematics having 30
questions in each part of equal weightage. Each question is allotted 4 (four) marks for each correct response.
6. Candidates will be awarded marks as stated above in instruction No. 5 for correct response of each question. 1/4 (one
fourth) marks will be deducted for indicating incorrect response of each question. No deduction from the total score will
be made if no response is indicated for an item in the answer sheet.
7. There is only one correct response for each question. Filling up more than one response in each question will be treated as
wrong response and marks for wrong response will be deducted accordingly as per instruction 6 above.
8. Use Blue/Black Ball Point Pen only for writing particulars/ marking responses on Side-1 and Side-2 of the Answer Sheet.
Use of pencil is strictly prohibited.
9. No candidates is allowed to carry any textual material, printed or written, bits of papers, pager, mobile phone, any
electronic device, etc., except the Admit Card inside the examination hall/room.
10. Rough work is to be done on the space provided for this purpose in the Test Booklet only. This space is given at the
bottom of each page and at the end of the booklet.
11. On completion of the test, the candidate must hand over the Answer Sheet to the Invigilator on duty in the Room / Hall.
However, the candidates are allowed to take away this Test Booklet with them.
12. The CODE for this Booklet is A. Make sure that the CODE printed on Side-2 of the Answer Sheet and also tally the serial
number of the Test Booklet and Answer Sheet are the same as that on this booklet. In case of discrepancy, the candidate
should immediately report the matter to the invigilator for replacement of both the Test Booklet and the Answer Sheet.
13. Do not fold or make any stray marks on the Answer Sheet.

Name of the Candidate (in Capital letters):


Roll Number : in figures 
in words
Examination Centre Number: 
Name of Examination Centre (in Capital letters):
Candidate’s Signature: Invigilator’s Signature:
74 Chemistry
Read the following instructions carefully:
1. The candidates should fill in the required particulars on the Test Booklet and Answer Sheet (Side-1) with
Blue/Black Ball Point Pen.
2. For writing/marking particulars on Side-2 of the Answer Sheet, use Blue/Black Ball Point Pen only.
3. The candidates should not write their Roll Numbers anywhere else (except in the specified space) on the Test
Booklet/Answer Sheet.
4. Out of the four options given for each question, only one option is the correct answer.
5. For each incorrect response, one-fourth (¼) of the total marks allotted to the question would be deducted from
the total score. No deduction from the total score, however, will be made if no response is indicated for an item
in the Answer Sheet.
6. Handle the Test Booklet and Answer Sheet with care, as under no circumstances (except for discrepancy in
Test Booklet Code and Answer Sheet Code), another set will be provided.
7. The candidates are not allowed to do any rough work or writing work on the Answer Sheet. All
calculations/writing work are to be done in the space provided for this purpose in the Test Booklet itself,
marked ‘Space for Rough Work’. This space is given at the bottom of each page and at the end of the booklet.
8. On completion of the test, the candidates must hand over the Answer Sheet to the Invigilator on duty in the
Room/Hall. However, the candidates are allowed to take away this Test Booklet with them.
9. Each candidate must show on demand his/her Admit Card to the Invigilator.
10. No candidate, without special permission of the Superintendent or Invigilator, should leave his/her seat.
11. The candidates should not leave the Examination Hall without handing over their Answer Sheet to the
Invigilator on duty and sign the Attendance Sheet again. Cases where a candidate has not signed the Attendance
Sheet a second time will be deemed not to have handed over the Answer Sheet and dealt with as an unfair
means case. The candidates are also required to put their left hand THUMB impression in the space
provided in the Attendance Sheet.
12. Use of Electronic/Manual Calculator and any Electronic Item like mobile phone, pager etc. is prohibited
13. The candidates are governed by all Rules and Regulations of the JAB/Board with regard to their conduct in the
Examination Hall. All cases of unfair means will be dealt with as per Rules and Regulations of the JAB/Board.
14. No part of the Test Booklet and Answer Sheet shall be detached under any circumstances.
15. Candidates are not allowed to carry any textual material, printed or written, bits of papers, pager,
mobile phone, electronic device or any other material except the Admit Card inside the examination
hall/room.
Mock Test-1 75

JEE-MAIN: CHEMISTRY MOCK TEST-1


1. In a galvanic cell, the salt bridge a. forward direction because Q  K c
a. does not participate chemically in the cell reaction. b. reverse direction because Q  K c
b. stops the diffusion of ions from one electrode to another.
c. forward direction because Q  K c
c. is necessary for the occurrence of the cell reaction
d. ensures mixing of the two electrolytic solutions d. reverse direction because Q  K c

2. Sodium metal crystallizes in a body centred cubic lattice 8. Two Faraday of electricity is passed through a solution of
with a unit cell edge of 4.29 Å. The radius of sodium atom CuSO4. The mass of copper deposited at the cathode is:
is approximately a. 0 g b. 63.5 g
a. 1.86 Å b. 3.22 Å c. 2 g d. 127 g
c. 5.72 Å d. 0.93 Å 9. What is the half life period of a radioactive substance if
3. Which of the following is the energy of a possible excited 87.5% of any given amount of the substance disintegrates
state of hydrogen? in 40 minutes?
a. 13.6 eV b.  6.8 eV a. 160 min b. 10 min
c.  3.4 eV d.  6.8 eV c. 20 min d. 13 min 20 sec
10. 3 g of activated charcoal was added to 50 mL of acetic
4. The number of atoms in 100 g of an fcc crystal with
acid solution (0.06 N) in a flask. After an hour it was
–3
density =10.0 g cm and cell edge equal to 200 pm is filtered and the strength of the filtrate was found to be
equal to 0.042 N. The amount of acetic acid adsorbed (per gram of
a. 5  10 24 b. 5  10 25 c. 6  10 23 d. 2  10 25 charcoal) is :
5. The following reaction is performed at 298 K. a. 18 mg b. 36 mg
c. 42 mg d. 54 mg
2NO(g)  O2 (g) 2NO2 (g)
11. The ionic radii (in Å) of N 3 , O 2 and F are respectively:
The standard free energy of formation of NO (g) is 86.6
a. 1.36, 1.40 and 1.71 b. 1.36, 1.71 and 1.40
kJ/mol at 298 K. What is the standard free energy of
c. 1.71, 1.40 and 1.36 d. 1.71, 1.36 and 1.40
formation of NO 2 (g) at 298 K? (Kp  1.6 10 )
12
12. In the context of the Hall-Heroult process for the
a. R(298) n (1.6  10 )  86600
12 extraction of A , which of the following statements is
b. 86600  R(298) n (1.6  1012 ) false?
a. CO and CO 2 are produced in this process
n (1.6  1012 )
c. 86600  b. A 2 O3 is mixed with CaF2 which lowers the melting
R(298)
point of the mixture and brings conductivity
d. 0.5[2  86600  R(298) n (1.6 1012 )]
c. A 3 is reduced at the cathode to form A
6. The vapour pressure of acetone at 20C is 185 torr. When d. Na 3A F6 serves as the electrolyte
1.2 g of a non-volatile substance was dissolved in 100 g of
13. From the following statements regarding H 2 O 2 , choose
acetone at 20  C, its vapour pressure was 183 torr. The
the incorrect statement
molar mass (g mol–1) of the substance is:
a. It can act only as an oxidising agent
a. 32 b. 64 b. It decomposes on exposure to light
c. 128 d. 488 c. It has to be stored in plastic or wax lined glass bottles in
7. The standard Gibbs energy change at 300 K for the dark
d. It has to be kept away from dust
reaction 2A B  C is 2494.2 J. At a given time, the
14. Which one of the following alkaline earth metal sulphates
composition of the reaction mixture is [A]  12 , [B]  2 and has its hydration enthalpy greater than its lattice enthalpy?
[C]  12 . The reaction proceeds in the: a. CaSO 4 b. BeSO 4

[ R  8.314 J / K / mol, e  2718 ] c. BaSO 4 d. SrSO 4


76 Chemistry
15. Which among the following is the most reactive? 21. Which compound would give 5-keto-2-methyl hexanal
a. Cl 2 b. Br2 c. I 2 d. ICl upon ozonolysis?
16. Which one has the highest boiling point? CH 3
a. He b. Ne c. Kr d. Xe CH 3
17. The number of geometric isomers that can exist for square a. CH 3 b.
planar [Pt(Cl)(py) (NH 3 )(NH 2 OH)] is (py = pyridine): CH 3
a. 2 b. 3 c. 4 d. 6 CH 3
18. The color of KMnO 4 is due to CH 3
a. M  L charge transfer transition c. d. H3C
b. d  d transition CH 3
c. L  M charge transfer transition
22. The synthesis of alkyl fluorides is best accomplished by :
d.    * transition a. Free radical fluorination
19. In Carius method of estimation of halogens, 250 mg of an b. Sandmeyer’s reaction
organic compound gave 141 mg of AgBr. The percentage c. Finkelstein reaction
of bromine in the compound is: (Atomic mass d. Swarts reaction
Ag  108, Br  80)
23. In the following sequence of reactions:
a. 24 b. 36 c. 48 d. 60
Toluene 
KMnO 2
A 
SOCl2
 B 
H 2 / Pd
BaSO 4
 C, the
20. In the reaction shown below, the major product(s) formed
is/are product C is:
H a. C6 H5COOH b. C6 H5CH3
|
N CH3 c. C6 H5CH 2OH d. C6 H 5CHO
O 24. In the reaction
a. NH3
NH 2

NaNO 2 / HCl
0  5 C
 D 
CuCN / KCN

 E  N2 The product E
O
NH 2
CH3
is
H
b. |
N CH 3 + CH 3 COOH

O O COOH
H CH3
|
N CH 3 a. b.

O CH3
H
| + H 2O CN
c. N CH 3

O O c.

d. H CH
3
N H 3CH 3COO  CH3
25. Which polymer is used in the manufacture of paints and
H
| lacquers?
d. + H 2O
N CH 3 a. Bakelite b. Glyptal
c. Polypropene d. Poly vinyl chloride
O O
Mock Test-1 77
26. Which of the vitamins given below is water soluble? Assertion and Reason
a. Vitamin C b. Vitamin D
c. Vitamin E d. Vitamin K Note: Read the Assertion (A) and Reason (R) carefully to mark
27. Which of the following compounds is not colored yellow? the correct option out of the options given below:
a. Zn 2 [Fe(CN)6 ] b. K 3 [Co(NO 2 )6 ] a. If both assertion and reason are true and the reason is the
c. (NH 4 )3 [As(Mo3O10 ) 4 ] d. BaCrO 4 correct explanation of the assertion.
b. If both assertion and reason are true but reason is not the
28. Match the statement of Column with those in Column II:
Column I Column II correct explanation of the assertion.
c. If assertion is true but reason is false.
(A) C12 H22 O11  H 2 O H 1. Pseudo first
order d. If the assertion and reason both are false.
C 6 H12 O 6  C 6 H12 O 6 e. If assertion is false but reason is true
(B) CH 3 COOC 2 H 5 HOH
 2. Zero order 29. Assertion: Enthalpy and entropy of any elementary
H  or OH 

CH 3 COOH  C 2 H 5 OH substance in the standard state are taken as zero.


Reason: At zero degree absolute, the constituent particles
(C) H 2  Cl 2 
hv
 2HCl 3. Second order
become completely motionless.
(D) CH3Cl  OH 
 CH3OH  Cl 4. First order
30. Assertion: Molecularity has no meaning for a complex
a. A1; B3, 4; C2; D3 reaction.
b. A2; B4; C3; D1
Reason: The overall molecularity of a complex reaction is
c. A1; B3, 2; C2; D4
equal to the molecularity of the slowest step.
d. A4; B1; C3; D2
Space for rough work
78 Chemistry
JEE ADVANCE PAPER-I

Time 3 Hours. Max. Marks 264 (88 for Chemistry)


Read The Instructions Carefully

Question Paper Format and Marking Scheme:


1. The question paper has three parts: Physics, Chemistry and Mathematics. Each part has three sections.

2. Section 1 contains 8 questions. The answer to each question is a single digit integer ranging from 0 to 9 (both inclusive).
Marking Scheme: +4 for correct answer and 0 in all other cases.

3. Section 2 contains 10 multiple choice questions with one or more than one correct option.
Marking Scheme: +4 for correct answer, 0 if not attempted and –2 in all other cases.

4. Section 3 contains 2 “match the following” type questions and you will have to match entries in Column I with the entries
in Column II.
Marking Scheme: for each entry in Column I, +2 for correct answer, 0 if not attempted and – 1 in all other cases.

NOTE: It’s the mock test as per previous year’s papers but sometimes IIT changes the test paper pattern and
marking scheme too.

SECTION 1 (Maximum Marks: 32) 4. The total number of lone pairs of electrons in N 2 O 3 is
 This section contains EIGHT questions.
5. For the octahedral complexes of Fe3 in SCN 
 The answer to each question is a SINGLE DIGIT INTEGER
ranging from 0 to 9, both inclusive. (thiocyanato-S) and in CN  ligand environments, the
 For each question, darken the bubble corresponding to the correct difference between the spin-only magnetic moments in
integer in the ORS. Bohr magnetons (When approximated to the nearest
 Marking scheme: integer) is [Atomic number of Fe = 26]
+4 If the bubble corresponding to the answer is darkened.
0 In all other cases.
6. Among the triatomic molecules/ions, BeCl 2 , N 3 , N 2 O,
NO 2 , O 3 ,SCl 2 , ICl 2 , I3 and XeF2 , the total number of
1. If the freezing point of a 0.01 molal aqueous solution of a
linear molecule(s)/ion(s) where the hybridization of the
cobalt (III) chloride-ammonia complex (which behaves as
central atom does not have contribution from the d-
a strong electrolyte) is 0.0558C, the number of
orbital(s) is
chloride(s) in the coordination sphere of the complex is
[Atomic number: S = 16, Cl = 17, I = 53 and Xe = 54]
[ K f of water = 1.86 K kg mol–1]
2. The total number of stereoisomers that can exist for M is 7. Not considering the electronic spin, the degeneracy of the
H 3C CH 3 second excited state (n = 3) of H atom is 9, while the
degeneracy of the second excited state of H  is

8. All the energy released from the reaction X  Y.  r G 0


H 3C
O
M  193 kJ mol 1 is used for oxidizing M  as M   M3
3. The number of resonance structures for N is
2e , E 0  0.25V. Under standard conditions, the
OH
NaOH
 N number of moles of M  oxidized when one mole of X is
converted to Y is [F  96500 C mol –1 ]
Mock Test-1 79
SECTION 2 (Maximum Marks: 40) CH 3 CH 3
 This section contains TEN questions. c. d.
H 3C Br H 3C Br
 Each question has FOUR options (a), (b), (c) and (d). ONE OR
MORE THAN ONE of these four option(s) is (are) correct. 13. The correct statement(s) for orthoboric acid is/are
 For each question, darken the bubble(s) corresponding to all the a. It behaves as a weak acid in water due to self ionisation
correct option(s) in the ORS.
b. Acidity of its aqueous solution increases upon addition
 Marking scheme:
of ethylene glycol
+4 If only the bubble(s) corresponding to all the correct
c. It has a three dimensional structure due to hydrogen
option(s) is(are) darkened.
0 If none of the bubbles is darkened
bonding.
–2 In all other cases d. It is a weak electrolyte in water
14. The major product of the reaction is
9. 
For the reaction: I  ClO  H 2SO 4  
 HSO  I 2
3

4
H 3C CO 2 H
The correct statement(s) in the balanced equation is/are 
NaNO2 , aqueous HCl
0 C

a. Stoichiometric coefficient of HSO4 is 6 CH 3 NH 2

b. Iodide is oxidised H 3C NH 2 H3C CO2 H


c. Sulphur is reduced a. b.
d. H 2 O is one of the products. CH 3 OH CH3 OH
H 3C CO 2 H H 3C NH 2
10. Compound(s) that on hydrogenation produce(s) optically
inactive compound(s) is (are) c. d.
CH 3 OH CH 3 OH
H Br
H Br 15. The reactivity of compound Z with different halogens
a. H3C CH3 b. H 2 C CH 3
under appropriate conditions is given below:
H Br OH mono halo substituted derivative when X 2  I 2
H 3C X2
di halo substituted derivative when X 2  Br2
CH 3 Br H
c. CH 3 d. H 3C CH 3 tri halo substituted derivative when X 2  Cl 2
C(CH 3 )3
Z
11. The major product of the following reaction is The observed pattern of electrophilic substitution can be
O explained by
a. the steric effect of the halogen
C H 3 i.  
KOH , H O
2
ii. H , heat
 b. the steric effect of the tert-butyl group
O c. the electronic effect of the phenolic group
CH 3 CH 3
d. the electronic effect of the tert-butyl group
O O
a. b. 16. An ideal gas in thermally insulated vessel at internal
pressure  P1 , volume  V1 and absolute temperature
 T1 expands irreversibly against zero external presssure,
O O
CH 3 as shown in the diagram. The final internal pressure,
c. CH 3 d. volume and absolute temperature of the gas are P2 , V2
and T2 , respectively. For this expansion.
12. In the following reaction, the major product is
CH 3
CH 2 
1 equivalent HBr

H 3C Pext  0 Pext  0

CH 3 P1 , V2 ,T2
CH 3 P1 , V2 ,T2
CH 3
a. H 3C b. H 3C
Br Br Thermal insulation
80 Chemistry
a. q  0 b. T2  T1  For each entry in Column I, darken the bubbles of all the
matching entries. For example, if entry (A) in Column I, matches
c. P2 V2  P1V1 d. P2 V2  P1V1
with entries (2), (3) and (4), then darken these three bubbles in the
17. Fe3 is reduced to Fe2  by using ORS. Similarly, for entries (B), (C) and (D).
a. H 2 O2 in presence of NaOH  Marking scheme:
b. Na 2 O 2 in water For each entry in Column I
+2 If only the bubble(s) corresponding to all the correct match(es)
c. H 2 O2 in presence of H 2SO 4 is(are) darkened
d. Na 2 O 2 in presence of H 2SO 4 0 If none of the bubbles is darkened
–1 In all other cases
18. The % yield of ammonia as a function of time in the
reaction N 2 (g)  3H 2 (g) 2NH 3 (g), H  0 at (P, T1 ) 19. Different possible thermal decomposition pathways for
peroxyesters are shown below. Match each pathway from
is given below:
Column -I with an appropriate structure from Column –II is:
P

 CO 
 R  R O
T1 1

O Q

 CO 
 R  R O 

O 1

time R O R R  X  carbonyl compound 


(Peroxyester) R
If this reaction is conducted at (P, T2 ), with T2  T1 , the 
 CO 
 RCO2  R O 
 CO 

1 1

% yield of ammonia as a function of time is represented R  X  carbonyl compound 


S
by   RCO 2  R O 
T2 
 CO 
 R  R O
1
T1 T1
% Yield % Yield
a. b. Column I Column II
(A) Pathway A 1. O
time time O
C6 H5 CH 2 O CH 3
T1 T2

% Yield T2 % Yield T1
(B) Pathway B 2. O
c. d. O
C6 H 5 O CH 3
time time
(C) Pathway C 3. O
O CH 3
SECTION 3 (Maximum Marks: 16)
C6 H5 CH 2 O
 This section contains TWO questions. CH 2 C6 H5 CH 3
 Each question contains two columns, Column I and Column II
 Column I has four entries (A), (B), (C) and (D) (D) Pathway D 4. O
 Column II has five entries (1), (2), (3), (4) and (5)
O CH 3
C6 H 5 O
 Match the entries in Column I with the entries in Column II
C6 H5 CH 3
 One or more entries in Column I may match with one or more
entries in Column II a. →1; B→3; C→4; D→2
 The ORS contains a 4  5 matrix whose layout will be similar to b. →2; B→4; C→4; D→1
the one shown below: c. →1; B→1; C→2; D→4
(A) (1) (2) (3) (4) (5) d. →3.; B→2; C→1; D→3
(B) (1) (2) (3) (4) (5)
20. Match the orbital overlap figures shown in Column -I with
(C) (1) (2) (3) (4) (5)
(D) (1) (2) (3) (4) (5)
the description given in Column -II and select the correct
Mock Test-1 81
answer using the code given below the lists. 3. p-d  bonding
(en H 2 NCH 2 CH 2 NH 2 ; atomic numbers : (C)
Ti  22, Cr  24, Co  27; Pt  78)
Column I Column II
4. d-d  antibonding
1. p-d  antibonding
(A) (D)

a. →2; B→3; C→1; D→4


2. d-d  bonding b. →2; B→4; C→4; D→1
(B) c. →1; B→1; C→2; D→4
d. →3.; B→2; C→1; D→3

Space for rough work


82 Chemistry

JEE ADVANCE PAPER-II

Time 3 Hours. Max. Marks 240 (80 for Chemistry)


Read The Instructions Carefully

Question Paper Format and Marking Scheme:


1. The question paper has three parts: Physics, Chemistry and Mathematics. Each part has three sections.

2. Section 1 contains 8 questions. The answer to each question is a single digit integer ranging from 0 to 9 (both inclusive).
Marking Scheme: +4 for correct answer and 0 in all other cases.

3. Section 2 contains 8 multiple choice questions with one or more than one correct option.
Marking Scheme: +4 for correct answer, 0 if not attempted and –2 in all other cases.

4. Section 3 contains 2 “paragraph” type questions. Each paragraph describes an experiment, a situation or a problem. Two
multiple choice questions will be asked based on this paragraph. One or more than one option can be correct.
Marking Scheme: +4 for correct answer, 0 if not attempted and – 2 in all other cases.

NOTE: It’s the mock test as per previous year’s papers but sometimes IIT changes the test paper pattern and
marking scheme too.

SECTION 1 (Maximum Marks: 32) CHCl2


II H2O
 →
This section contains EIGHT questions. 100 ° C

The answer to each question is a SINGLE DIGIT INTEGER


COCl
ranging from 0 to 9, both inclusive. III H2

Pd − BaSO 4

For each question, darken the bubble corresponding to the correct
integer in the ORS.
Marking scheme: CO2 Me
IV DIBAL-H
+4 If the bubble corresponding to the answer is darkened. 
Toluene, −78° C H 2 O

0 In all other cases.

1. In dilute aqueous H 2SO 4 , the complex diaquodioxalato- 4. The crystal of a solid is square packing of identical
spheres in each layer and spheres of one layer are placed
ferrate (II) is oxidized by MnO −4 . For this reaction, the
just above the voids made by spheres in previous layer.
ratio of the rate of change of [H + ] to the rate of change of The packing efficiency of such crystal (in %) is
[MnO −4 ] is [π = 3.15, 2 = 1.4]
2. The number of hydroxyl group(s) in Q is

+
H aqueous dilute KMnO4 (excess)
H  
heat
→P  0°C
→Q
HO
H3C CH3 5. Among the complex ions, [Co(NH 2 - CH 2 - CH 2 -NH2)2
3. Among the following, the number of reaction(s) that Cl 2 ]+ , [CrCl 2 (C 2 O 4 ) 2 ]3– , [Fe(H 2 O) 4 (OH) 2 ]+ , [Fe(NH3)2]
produce(s) benzaldehyde is
(CN) 4 ]− , [Co(NH 2 − CH 2 − CH 2 − NH 2 ) 2 (NH 3 )Cl]2 + and

I CO, HCl [Co(NH 3 ) 4 (H 2 O)Cl]2+ , the number of complex ion(s) that



Anhydrous AlCl3 / CuCl

show(s) cis-trans isomerism is
Mock Test-1 83
6. Three moles of B2 H 6 are completely reacted with H
|
methanol. The number of moles of boron containing O
O H3C
product formed is CH3
CH3 O
7. The molar conductivity of a solution of a weak acid HX O H
a. b.
(0.01 M) is 10 times smaller than the molar conductivity
of a solution of a weak acid HY (0.10 M). If X0   Y0  , H
|
the difference in their pK a values, pK a (HX) pK a (HY), O
O CH2
is (consider degree of ionisation of both acids to be << 1)
CH2 O H
8. A closed vessel with rigid walls contains 1 mol of 238
U H
92 c. d.
and 1 mol of air at 298 K. Considering complete decay of
238 206
92 U to 82 Pb, the ratio of the final pressure to the initial 12. In the following reactions, the major product W is
OH
pressure of the system at 298 K is NH2 NaOH
 
NaNO 2 , HCl
0 C
 V 
W
SECTION 2 (Maximum Marks: 32)
 This section contains EIGHT questions.
 Each question has FOUR options (a), (b), (c) and (d). ONE OR
MORE THAN ONE of these four option(s) is (are) correct. a.
 For each question, darken the bubble(s) corresponding to all the N N OH
correct option(s) in the ORS.
 Marking scheme:
+4 If only the bubble(s) corresponding to all the correct option(s)
is(are) darkened. OH
0 If none of the bubbles is darkened
b.
–2 In all other cases NN
9. Which of the following compounds will exhibit
geometrical isomerism?
a. 1-Phenyl-2-butene c.
b. 3-Phenyl-1-butene OH
c. 2-Phenyl-1-butene NN
d. 1,1-Diphenyl-2-propane
10. In the following reactions, the product S is
H 3C OH
d.
 i. O3
ii. Zn,H 2 O
 R 
NH3
S
N N
H 3C H 3C N
N
a. b.
13. The correct statement(s) regarding, (i) HClO, (ii) HClO 2 ,
N (iii) HClO3 and (iv) HClO 4 , is (are)
N
c. d. a. The number of Cl = O bonds in (ii) and (iii) together is
H 3C H 3C two
b. The number of lone pairs of electrons on Cl in (ii) and
11. The major product U in the following reactions is
(iii) together is three
CH2 CH CH3 , H
T radical initiator, O2
U c. The hybridisation of Cl in (iv) is sp3
high pressure, heat
d. Amongst (i) to (iv), the strongest acid is (i)
84 Chemistry
14. The pair(s) of ions where BOTH the ions are precipitated undergoes transition to a state S2 . The state S2 has one radial
upon passing H 2S gas in presence of dilute HCl, is(are) node and its energy is equal to the ground state energy of the
2
a. Ba , Zn 2 3
b. Bi , Fe 3 hydrogen atom.
2 2
c. Cu , Pb d. Hg 2 , Bi3 17. Energy of the state S1 in units of the hydrogen atom

15. Under hydrolytic conditions, the compounds used for ground state energy is :
preparation of linear polymer and for chain termination, a. 0.75 b. 1.50
respectively, are c. 2.25 d. 4.50
18. The orbital angular momentum quantum number of the
a. CH3SiCl3 and Si(CH 3 ) 4
state S2 is :
b. (CH3 ) 2 SiCl2 and (CH3 )3 SiCl
a. 0 b. 1
c. (CH3 ) 2 SiCl2 and (CH 3 )3 SiCl3 c. 2 d. 3
d. SiCl 4 and (CH3 )3 SiCl
Paragraph for Question No. 19 to 20
16. When O 2 is adsorbed on a metallic surface, electron In the following reactions
transfer occurs from the metal to O 2 . The TRUE Pd  BaSO4
C8H 6   C8 H8  i. B2 H 2
X
H2 ii. H 2O2 . NaOH, H 2O
statement(s) regarding this adsorption is(are) H2O
a. O 2 is physisorbed HgSO 4 , H 2SO4

b. heat is released C8 H8O 


i. EtMgBr, H 2O
Y
ii. H  , heat
c. occupancy of  2p
*
of O 2 is increased
19. Compound X is
d. bond length of O 2 is increased O OH

SECTION 3 (Maximum Marks: 16) CH3 CH3


a. b.
 This section contains TWO questions.
 Based on each paragraph, there will be TWO questions OH
 Each question has FOUR options (A), (B), (C) and (D). ONE OR CHO
c. d.
MORE THAN ONE of these four option(s) is(are) correct.
 For each question, darken the bubble(s) corresponding to all the
correct option(s) in the ORS.
20. The major compound Y is
 Marking scheme:
 +4 If only the bubble(s) corresponding to all the correct option(s) a. CH3 b. CH3
is(are) darkened.
 0 If none of the bubbles is darkened
CH3
 –2 In all other cases CH2
Paragraph for Question Nos. 17 to 18 CH3
CH3
The hydrogen-like species Li 2+ is in a spherically symmetric c. d.
state S1 with one radial node. Upon absorbing light the ion
Space for rough work
Mock Test-1 85
+2
ANSWER & SOLUTIONS 8. (b) Cu + 2e 
→ Cu (s)−

JEE-Main 2 mol 1 mol = 63.5


1. 2. 3. 4. 5. 6. 7. 8. 9. 10. 9. (d) t = 40 min
a a c a d b b b d a
2.303 a
11. 12. 13. 14. 15. 16. 17. 18. 19. 20. k= log
t a−x
c d a b d d b c a a
21. 22. 23. 24. 25. 26. 27. 28. 29. 30.
2.303 a
k= log
b d d c b a a a c b
40 a − 0.875 a
2.303 a
1. (a) = log
40 0.125 a
2. (a) For BCC unit 6 cell, 3a = 4r 2.303
= log 8 = 0.051 min
3 3 40
r= a= × 4.29 Å = 1.85 Å
4 4 0.693 0.693
t1/ 2 = = = 13.58 min
k 0.051
3. (c) Energy in 1st excited state = −3.4 eV
10. (a) Amount of acetic acid adsorbed
Z × Mw
4. (a) ρ = 3 eff −30 [For fcc, Zeff = 4 unit cell] (0.06 − 0.042) × 50 ×10−3 × 60
a ×10 × N A = = 16 ×10−3 = 18 mg
3
ρ × a 3 × 10−30 × N A
∴ Mw = 11. (c) Ionic Radii order: N3− > O2− > F−
Zeff
12. (d)
(10.0 g cm –3 × (200 pm)3 × 10−30 cm 3 × 6 × 10 23 atoms)
=
4 13. (a) It can act as an oxidising as well as reducing agent.
−1
Thus, 12gmol contains = N A atoms = 6 × 10 atoms 23
14. (b) BaSO 4 is least soluble, BeSO 4 is most soluble.
6 × 10 23 15. (d) The interhalogen compounds are generally more reactive
∴ 100 g contains = × 100 = 5 × 10 24 atoms
12 than halogens (except F2).
5. (d) ∆GRxn = 2∆GNO2 − 2∆G NO 16. (d) Xe has the highest boiling point.

∆G NO2 = ∆G NO + 12 ∆G Rx 17. (b) No. of Geometrical isomers of


[Pt(Cl)(py)(NH 3 )(NH 2 OH)]+ = 3
= ∆G NO + 12 (− RT ln e K p )
18. (c)
= 0.5[2 × 86600 − R(298) ln (1.6 × 1012 )]
80 141 mg
P0 − P n1 19. (a) % of Bromine = × ×100 = 24%
6. (b) = Xsolute = 188 250 mg
P0 n1 + n 2
20. (a) Only amines undergo acetylation and not acid amides.
185 − 183 2 1.2 / M
= = O
185 185 1.2 + 100
NH C CH 3
M 58
NH 2
⇒ M = 64 O O
C C + CH 3 COOH
7. (b) ∆G = −RT ln e K c H3C O OH
C NH 2
2494.2 = 8.314 × 300 ln e K c ⇒ K c = e −1 C NH 2 O
1 21. (b)
K c = e −1 = = 0.36
2.718 CH3 CH3
(B)(C) 2 × 12 O
Q= = =4 O3
[A]2 [1/ 2]2 
Zn / H 2 O
→ CHO
Q > K c , i.e. backward reaction. CH3 CH3
5-keto-2-methyl hexanal
86 Chemistry
Total no. of stereoisomers of M = 2
22. (d) R  I  AgF 
 R  F  AgI (Swarts Reaction)
H 3C CH 3
23. (d) CH 3 COOH


KMnO 2

SOCl2

H 3C O
Toluene Benzoic acid

COCl CHO 3. (9)


HO

H 2 / Pd
BaSO4
 
NaOH
N
Benzoyl chloride Benzaldehyde
O–
24. (c)

NH3 N Cl CN N is
O– O O


NaNO2 / HCl
05 C

CuCN / KCN

 N2
1 2 3
CH3 CH3 CH3 O O –
O
25. (b) Glyptal is used in the manufacture of paints and
lacquers. 4 5 6
26. (a) Vitamin ‘B’ and ‘C’ are water soluble. O O O–

27. (a) Zn 2 [Fe(CN)6 ] is bluish white ppt.


7 8 9
28. (a) A1; B3, 4; C2; D3
(A) Inversion of cane sugar is pseudo first order reaction. O:
+
(B) Hydrolysis of ester in the presence of acid is first order 4. (8) :O  N N  O :O  N –– N
while in the presence of base is second order reaction. O :O:
(C) Photochemical reactions are of zero order. Total no. of lone pairs = 8
(D) SN2 reactions are of second order.
5. (4) [Fe(SCN) 6 ]3 and [Fe(CN) 6 ]3
29. (c) Enthalpy is zero but entropy is not zero. Vibrational
In both the cases the electronic configuration of Fe3 will
motion exists even at absolute zero.
be 1s 2 , 2s 2 , 2p 6 ,3s 2 ,3p 6 ,3d 5 .
30. (b) Molecularity of a reaction can be defined only for an
elementary reaction because complex reaction does not Since SCN is a weak field ligand and CN is a strong field
take place in one single step and it is almost impossible ligand, the pairing will occur only in case of [Fe(CN)6 ]3 .
for all the total molecules of the reactants to be in a state
of encounter simultaneously. (high spin)
Case-1 3d 5
(no pairing)
JEE Advance Paper -I
1. 2. 3. 4. 5. 6. 7. 8. 9. 10. (low spin)
Case-2 (pairing)
1 2 9 8 4 4 3 4 a,b,d b,d
11. 12. 13. 14. 15. 16. 17. 18. 19. 20.
a d b,d c a,b,c a,b,c a,b b a a Case – 1   n(n  2)  5(5  2)  35  5.91 BM

1. (1) T f  iK f m Case – 2   n(n  2)  1(1  2)  3  1.73 BM

0.0558  i  1.86  0.01  i  3


Difference in spin only magnetic moment
 5.91  1.73  4.18  4
 Complex is [Co(NH 3 )5 Cl]Cl2 .
6. (4) BeCl 2 , N 3 , N 2 O, NO 2 , O 3 ,SCl 2 , ICl 2 , I3 , XeF2
2. (2) Bridging does not allow the other 2 variants to exist.
Mock Test-1 87
BeC 2 
 sp 
 linear (2) H 3 C  CH — C — CH 2 — CH 3 
 H 2 /Pt


N 
 sp 
3  linear H Br
H 3 C — CH — C — CH 2 — CH 3
N 2 O 
 sp 
 linear

H Br
NO 2 
 sp 
 linear Optically inactive

O 3 
 sp 2 
 linear
(3) H 2 C  C — C — CH 3  
H 2 /Pt
SCl 2 
 sp 3 
 linear
H2C H Br H
I 3 
 sp 3 d 
 linear
H 3 C — C — C — CH3
ICl 2 
 sp 3 d 
 linear H2C H Br
XeF2 
 sp 3d 
 linear Optically inactive
So, among the following only four (4) has linear shape and no
d-orbital is involved in hybridization. (4) H2 C  CH —C — CH2 —CH3 
H /Pt
 2

7. (3) Single electron species don’t follow the (n  ) rule but H Br H3C — CH2 — C—CH2 —CH3
multi electron species do. H Br
Ground state of H  1s2 Optically inactive

First excited state of H   1s1 , 2s1 O



11. (a) :OH

Second excited state of H   1s1 , 2s 0 , 2p1 CH 2
CH 3
O O
Px Py Pz H2O

(3 degenerate orbitals) 
CH CH
3
8.  Y  r G  193KJ / mol
(4) X  0
O

 M 3   2e  E  0.25V
0
M  
CH 3 CH 3
OH
G for the this reaction is
0
O O
H+ ,H2O/Δ
G   nFE  2  ( 0.25)  96500  48250 J / mol
0 0  

48.25 KJ/mol
So the number of moles of M  oxidized using CH 3
12. (d) |
X   Y will be H   Br 
H 2 C  C — CH  CH 2  
193
  4 moles
48.25
CH 3
  |
9. (a, b, d) 6I  ClO  6H 2SO4 
3  H 3 C — C — CH  CH 2 

Cl   6HSO 4  3I 2  3H 2 O
H3C 
:  H3C
10. (b, d) C  CH — CH2 
Br
 C  CH — CH2 — Br
H3C H3C
CH 3
(1) H 3C — CH  CH — C 13. (b, d) (a) H 3 BO 3 is a weak monobasic Lewis acid.
H Br CH 3 H3 BO3  H  OH B(OH)4  H  . . .(i)

 H 3C — CH 2 — CH 2 — C
H 2 /Pt
(b) Equilibrium (i) is shifted in forward direction by the
H Br addition of syn-diols like ethylene glycol which forms a
Optically active stable complex with B(OH) 4 .
88 Chemistry
forward reaction would be greater than at lower temperature
H O that is why the percentage yield of NH3 too would be more
O H O H
H O initially.
B 19. (a) →1; B→3; C→4; D→2
O
H O O H
H O O
O H
(A) H5C6CH2 O CH3

O
O C6 H5  CH2  CO2  CH2O
B  4H 2 O O CH 3
O
O CH 3
O
(B) H 5 C6 CH 2 O
(stable complex) CH 2 C6 H 5
(c) It has a planar sheet like structure due to hydrogen
bonding. O
|
(d) H3BO3 is a weak electrolyte in water. C6 H5 — C H 2  CO2  Ph — CH 2 — C — CH3
|
14. (c) CH3
OH :OH
| |
H3C C  O H3C C  O 
 Ph — C H 2  CH3 — CO — CH3
CH —CH2 —CH  
NaNO2 / HCl
 CH —CH2 —HC
H3C
NH2
H3C
 O CH 3
N N
O CH 3
H
H3C H3C

O (C) C6 H 5 O
CH —CH2 —CH —COOH 
H2O
 CH —CH2 —HC C== O C6 H 5
H3C H3C
OH
O
15. (a, b, c) OH |
I C6 H5 — CO 2  CH3  C — CH3
|
I2
Rxn (i)
 CH3
CMe 3
OH OH 
 Ph  CH 3 — CO — Ph  CH 3
Br O

Br2
Rxn (ii)
 O
CMe 3 CMe 3 C6 H 5 O CH 3
(D)
Br
OH C6 H 5  CO 2  CH 3O 
 C6 H 5  CO2
Cl Cl
20. (a)→2; B→3; C→1; D→4

Cl2
Rxn (iii)
CMe 3
Cl (A) 
 d-d  bonding
16. (a, b, c) Since container is thermally insulated so, q = 0,
and it is a case of free expansion therefore W = 0 and
E 0. So, T1 T2 . Also, P1V1 P2 V2 . (B) 
 p-d  bonding
3 2
17. (a, b) 2Fe H 2 O2 2OH 2Fe 2H2 O O2
Na 2 O2  H 2 O 
 H 2 O 2  NaOH
(C) 
 p-d  antibonding
18. (b) N 2 (g)  2H 2 (g)
N 2 (g)  2H 2 (g) 2NH 3 (g); H  0
(D) 
 d-d  antibonding
Increasing the temperature lowers equilibrium yield of
ammonia. However, at higher temperature the initial rate of
Mock Test-1 89
JEE Advance Paper-II 1
is shared in second layer
1. 2. 3. 4. 5. 6. 7. 8. 9. 10. 2
8 4 4 75 5 6 3 9 a a 1
 Z   4  0.5  1
11. 12. 13. 14. 15. 16. 17. 18. 19. 20. 8
b a b,c c,d b b,c,d a b c d (2r 2 )  h2  x 2 fi h = 2r
Area = 4r 2 [ a 2 = (2r ) 2 fi a = 2r ]
1. (8) [Fe(C 2 O 4 )(H 2 O)]2   MnO 24   8H  

Mn 2   Fe3  4CO 2  6H 2 O Vol  4r 2  2r  4 2r 3
4
So, the ratio of rate of change of 1  r 3
3 
[H  ] to that of rate of change of [MnO4 ] is 8. D 3
  0.75  75%
4 2r 3 2
2. (4) 5. (5) [Co(en) 2 Cl 2 ] 
 will show cis-trans isomerism
[CrCl 2 (C 2 O 4 ) 2 ]3 
 will show cis-trans isomerism

H 
H

 [Fe(H 2 O) 4 (OH) 2 ] 
 will show cis-trans isomerism
+
HO [Fe(CN) 4 (NH 3 ) 2 ] 
 will show cis-trans isomerism


 [Co(en) 2 (NH 3 )Cl]2 
 will show cis-trans isomerism
[Co(NH 3 ) 4 (H 2 O)Cl]2 
 will not show cis-trans
(P) isomerism (although it will show geometrical isomerism)
aqueous dilute KMnO4 6. (6) B 2 H 6  6MeOH  2B(OMe)3  6H 2
(excess) 0C
OH 1 mole of B2 H 6 reacts with 6 moles of MeOH to give 2
moles of B(OMe)3
3 moles of B2 H 6 will react with 18 moles of MeOH to
HO OH
HO (Q) give 6 moles of B(OMe)3 .
[H  ] [X  ]
3. (4) 7. (3) HX H  X Ka 
[HX]
CHO
I CO, HCl
 [H  ] [Y  ]
Anhydrous AlCl3 / CuCl HY H  Y Ka 
[HY]

CHCl2 CHO  m for HX   m1  m for HY   m 2


II  
 H2O
100  C 1
 m1  m Ka  C 2
10 2
III COCl CHO 2
 H2
  m 
Pd  BaSO 4 Ka1  C1   0 1 
  m1 
IV CO2 Me CHO 2
BIBAL  H
   m 
Toluene, 78 C H 2 O Ka 2  C2   0 2 
  m2 
2
4. (75) Ka1 C1  m1 
2r   
h
Ka 2 C2  m 2 
2
0.01  1 
   0.001
x 0.1  10 
pKa1  pKa 2  3

1 8. (9) In conversion of 238


92 U to 206
82 Pb, 8 -
Z  1 [ th is shared in first layer and
8 particles and 6β particles are ejected.
90 Chemistry
The number of gaseous moles initially  1 mol 12. (a)  –
NH 2 N2 Cl
The number of gaseous moles finally  1  8 mol; (1 mol 
NaNO3, HCl
βNapthol/NaOH
 
0C
from air and 8 mol of 2 He 4 ) So the ratio  9 /1  9
9. (a) Ph  CH 2 CH 3 (V)
C  C
H H N  N  Ph
cis
OH
Ph  CH 2 H
C  C
H trans
CH 3
(W)
10. (a)
O 13. (b, c) H — O — Cl
||
H 3C H 3C C—H .. ..
 (i) O3
 . .  O
(I) H — O — Cl (II) H — O — Cl  O
(ii) Zn.H 2 O
CH 2 — C — H ||
R || O
O
O
||
O (III) H — O — Cl  O (IV)
|| ||
H 3C C—H O

NH3
 
14. (c, d) Cu 2 , Pb 2 , Hg 2 , Bi 3 give ppt. with H 2S in presence
CH 2 — CH — NH3
|| of dilute HCl.
O
CH3 CH3
| |
15. (b) Cl — Si — Cl 
H O
 HO — Si — OH 

O
2
OH

| |
H 3C NH H 3C NH2 CH3 CH3

OH CH CH 3 CH3
CH 2 OH | |
2H 2 O H — O — Si — O — Si ——O — H
| |
CH 3 CH 3 n
H 3C
N
Me3SiCl, H 2 O

(S) CH 3 CH 3
Me | | Me
O
|| Me Si — O — Si — O — Si ——O — Si Me
H 3C C—H Me | | Me
CH 3 CH 3 n
..
CH 2 — CH — NH 2
|| 16. (b, c, d) * Adsorption of O 2 on metal surface is
OH
exothermic.
11. (b) 
* During electron transfer from metal to O 2 electron
 H3C — CH — CH3 

occupies  2p
*
orbital of O2 .
CH3
| * Due to electron transfer to O 2 the bond order of O 2
C — CH3 decreases hence bond length increases.

O2
 |
O
O For Question Nos. 17 to 18
U
CH H E1H  32 9
H 3C CH3 17. (c) E S1    2.25  E1H .
22 4
T
Mock Test-1 91
18. (b) S2 is 3p  l = 1 O
||
For Question Nos. 19 to 20 C — CH3 CH2  CH2  OH
C8 H 6 
  double bond equivalent
6 (i) EtMgBr (X)
 8 1 6 (ii) H 2 O
2
C  CH CH  CH2 OH CH 3
| |
H  / heat
Ph — C — CH3   Ph — C  CH — CH 3
| (Y)
(1) B2 H8 Et
HgSO 4 , H 2SO 4 , H 2O (2) H 2O 2 , NaOH, H 2O
19. (c)
20. (d)

  
92 Chemistry
JEE-MAIN: CHEMISTRY MOCK TEST-2
1. 2 g of oxygen contain same number of atoms as contained a. More cis-2-pentene is formed
by b. Equilibrium is shifted in the forward direction
a. 0.5 g hydrogen b. 4.0 g sulphur c. Equilibrium remains unaffected
c. 7.0 g nitrogen d. 2.3 g sodium d. Additional trans-2-pentene is formed
2. The axial angles in triclinic crystal system are 10. The dissociation constant of CH 3 COOH is 1.8 105.
a.       90 b.     90,   90
The hydrolysis constant for 0.1 M sodium acetate is
c.       90 d.       90
a. 5.56  10 4 b. 5.56 1010
3. A sample of drinking water was found to be severely c. 1.8  105 d. 1.8  109
contaminated with chloroform CHCl3 , supposed to be a
11. The half-life of a first order reaction having rate constant
carcinogen. The level of contamination was 15 ppm (by
mass). Determine the molality of chloroform in the water K  1.7 105 s 1 is
sample. a. 12.1 h b. 9.7 h
a. 2.12  104 mol kg 1 b. 1.26  10 4 mol kg 1 c. 11.3 h d. 1.8 h

c. 0.12  10 4 mol kg 1 d. 5.36  10 4 mol kg 1 12. Which characteristic is true in respect of colloidal particle
a. They always have two phases
4. The temperature at which hydrogen molecules will have b. They are only in liquid state
the same root mean square velocity as oxygen molecules c. They can't be electrolysed
have at 27C is d. They are only hydrophilic
a. 25C b. 7.93C
13. The heat of transition (H t ) of graphite into diamond
c. 248C d. 127C
would be, where
5. An electron makes a transition from orbit n = 4 to the C(graphite)  O 2 (g) 
 CO2 (g); H  x kJ
orbit n = 2 of a hydrogen atom. The wave number of the
emitted radiations (R = Rydberg's constant) will be C(diamond)  C2 (g) 
 CO 2 (g) ; H  y kJ
16 2R 3R 4R a. (x  y) kJ mol 1 b. (x  y) kJ mol 1
a. b. c. d.
3R 16 16 16 c. (y  x) kJ mol 1 d. None of these
6. Select the compound in which chlorine is assigned the 14. All the nuclei from the initial element to the final element
oxidation number +5 constitute a series which is called
a. HClO 4 b. HClO 2 a. g-series b. b-series
c. HClO3 d. HCl c. b-g series d. Disintegration series

7. The molar conductivity is maximum for the solution of 15. The following compound will undergo electrophilic
concentration substitution more readily than benzene
a. 0.001 M b. 0.005 M a. Nitrobenzene b. Benzoic acid
c. 0.002 M d. 0.004 M c. Benzaldehyde d. Phenol

8. The number and type of bonds between two carbon atoms 16. Cis and trans 2-butene are
in calcium carbide are a. Conformational isomers
a. One sigma, one pi b. One sigma, two pi b. Optical isomers
c.Two sigma, one pi d. Two singma, two pi c. Position isomers
d. Geometrical isomers
9. The standard state gibbs free energy change for the given
17. Electrolysis of a concentrated solution of sodium fumarate
isomerization reaction cis-2-pentene trans-2-pentene gives
is –3.67 kJ / mol at 400K. If more trans-2-pentene is added a. Ethylene b. Ethane
to the reaction vessel, then c. Acetylene d. Vinyl alcohol
Mock Test-2 93
18. Identify X and Y in the following sequence 24. The correct statement in respect of protein haemoglobin is
C 2 H 5 Br 
X
 product 
Y
 C3 H 7 NH 2 that it
a. Acts as an oxygen carrier in the blood
a. X  KCN, Y  LiAlH 4
b. Forms antibodies and offers resistance to diseases
b. X  KCN, Y  H 3O  c. Functions as a catalyst for biological reactions
c. X  CH 3Cl, Y  AlCl3 / HCl d. Maintains blood sugar level
d. X  CH 3 NH 2 , Y  HNO 2 25. Which one of the following is known as broad spectrum
antibiotics
19. Consider the following alcohols
a. Streptomycine b. Ampicillin
(i) 1-Phenyl-1-propanol c. Chloramphenicol d. Penicillin G
(ii) 3-Phenyl-1-propanol
26. The element or elements whose position is anomalous in
(iii) 1-Phenyl-2-propanol
the periodic table is
The correct sequence of increasing order of reactivity of
a. Halogens b. Fe, Co and Ni
these alcohol in their reaction with HBr is
a. (i), (ii), (iii) b. (ii), (i), (iii) c. Inert gases d. Hydrogen
c. (i), (iii), (ii) d. (ii), (iii), (i) 27. Flux added in the extraction of iron is
20. Reaction between diethyl cadmium and acetyl chloride a. Silica b. Felspar
leads to the formation of c. Limestone d. Flint
a. dimethyl ketone b. ethylmethyl ketone 28. The correct order of the increasing ionic character is
c. diethyl ketone d. acetaldehyde a. BeCl 2  MgCl 2  CaCl 2  BaCl 2

21. Which of the following is basic? b. BeCl 2  MgCl 2  BaCl 2  CaCl 2


a. CH 3CH 2 OH b. H 2O2 c. BeCl 2  BaCl 2  MgCl 2  CaCl 2
c. HOCH 2 CH 2 OH d. CH3COOH d. BaCl 2  CaCl 2  MgCl 2  BeCl 2

29. Nesseler's reagent is


22. CH3CN 
Na, C2 H5OH
 X. The compound X is:
a. K 2 HgI 4 b. K 2 HgI 4  KOH
a. CH 3CH 2 NO 2 b. CH3CH 2COOH
c. K 2 HgI 2  KOH d. K 2 HgI 4  Hg
c. C6 H 5 N(CH 3 ) 2 d. C6 H 5CONH 2
30. The primary valence of the metal ion in the co-ordination
23. Which of the following is a synthetic polymer compound K 2  Ni  CN 4  is
a. Rubber b. Perspex
c. Protein d. Cellulose a. Four b. Zero c. Two d. Six

Space for rough work


94 Chemistry
JEE ADVANCE PAPER-I
SECTION 1 Contains 8 Questions SECTION 2 Contains 10 Multiple Choice Questions
The answer to each question is a single digit integer ranging from 0 to With one or more than one correct option
9 (both inclusive). 9. Sodium metal crystallises in a body centred cubic lattice
1. At 400 K, the root mean square (rms) speed of a gas X with a unit cell edge of 4.29 Å. The radius of sodium atom
(molecular weight = 40) is equal to the most probable speed is approximately
a. 1.86 Å b. 3.22 Å c. 5.72 Å d. 0.93 Å
of gas Y at 60 K. The molecular weight of the gas Y is
10. 18 g glucose (C6 H I2 P6 ) is added to 178.2 g water. The vapor
2. Based on VSEPR theory, the number of 90 degree
pressure of water (in torr) for this aqueous solution is :
F−Br−F angles in BrF5 is
a. 7.6 b. 76.0 c. 752.4 d. 759.0
3. 20% surface sites have adsorbed N 2 . On heating N 2 gas 11. For one mole of a van der Waals gas when b = 0 and T =
evolved from sites and were collected at 0.001 atm and 300 K, the PV vs. 1/V plot is shown below. The value of
298 K in a container of volume is 2.46 cm3. Density of the van der Waals constant a (atm. liter2 mol2) is.
surface sites is 6.023 1014 / cm2 and surface area is 1000

PV (liter-atm mol–1)
24.6 (Graph not to scale)
cm2, find out the no. of surface sites occupied per 23.1
21.6
molecule of N 2 . 20.1

4. The total number of contributing structure showing


hyperconjugation (involving C–H bonds) for the 0 2.0 3.0
following carbocation is 1
(mol liter 1 )
V
H3C  CH 2 CH 3
a. 1.0 b. 4.5 c. 1.5 d. 3.0

12. A piston filled with 0.04 mol of an ideal gas expands


reversibly from 50.0 mL to 375 mL at a constant
temperature of 37.0C. As it does so, it absorbs 208 J of
5. The volume (in mL) of 0.1 M AgNO3 required for heat. The values of q and w for the process will be
complete precipitation of chloride ions present in 30 mL (R  8.314 J / mol K) (ln 7.5  2.01)
of 0.01 M solution of [Cr(H 2 O)5 Cl]Cl2 , as silver chloride a. q  208 J, w  208 J b. q  208 J, w  208 J
is close to c. q  208 J, w  208 J d. q  208 J, w  208 J

6. Among PbS, CuS, HgS, MnS, Ag 2S, NiS, CoS, Bi 2S3 and 13. The first ionisation potential of Na is 5.1 eV. The value of
electron gain enthalpy of Na  will be
SnS2 , the total number of black coloured sulphides is
a. –2.55 eV b. 5.1eV
7. 29.2% (w/w) HCl stock solution has a density of 1.25 g c. 10.2 eV d. 2.55 eV
1 1
mL . The molecular weight of HCl is 36.5 g mol . The
14. In a galvanic cell, the salt bridge
volume (mL) of stock solution required to prepare a 200 a. does not participate chemically in the cell reaction.
mL solution of 0.4 M HCl is b. stops the diffusion of ions from one electrode to another.
8. The maximum number of isomers (including c. is necessary for the occurrence of the cell reaction
d. ensures mixing of the two electrolytic solutions
stereoisomers) that are possible on monochlorination of
the following compound is  
15. Stability of the species of Li2 , Li2 and Li2 increase in
CH 3 the order of
C a. Li 2  Li 2  Li 2 b. Li 2  Li 2  Li 2
CH 3 CH 2 CH 2 CH 3 c. Li 2  Li 2  Li 2 d. Li 2  Li 2  Li 2
H
Mock Test-2 95
16. How many litres of water must be added to 1 litre of an (C) CO 2 (P  1 atm, T  273K) 3. P  nRT
aqueous solution of HCl with a pH of 1 to create an
(D) Real gas with very large 4. P(V  nb)  nRT
aqueous solution with pH of 2?
molar volume
a. 0.1 L b. 0.9 L c. 2.0 L d. 9.0 L
a. A  2; B  3; C  2; D  1, 4
17. NiCl2 {P(C2 H 5 ) 2 (C6 H 5 )}2 exhibits temperature dependent
b. A  1, 3; B  1; C  2; D  4
magnetic behavior (paramagnetic / diamagnetic). The c. A  3; B  4; C  3, 4; D  1, 2
coordination geometries of Ni2+ in the paramagnetic and
d. A  1, 2; B  3; C  1, 2; D  1, 4
diamagnetic states are respectively.
a. tetrahedral and tetrahedral 20. The standard reduction potential data at 25ºC is given
b. square planar and square planar below:
c. tetrahedral and square planar E (Fe 3  , Fe 2  )   0.77 V; E (Fe 2  , Fe)   0.44V
d. square planar and tetrahedral
E (Cu 2  , Cu)   0.34V; E (Cu  , Cu)   0.52V
18. Four successive members of the first row transition elements
E[O 2 (g)  4H   4e  
 2H 2 O]   1.23V;
are listed below with atomic numbers. Which one of them is
expected to have the highest E 0M3 / M 2 value? E[O 2 (g)  2H 2 O  4e  
 4OH  ]   0.40V

a. Cr(Z  24) b. Mn(Z  25) E (Cr 3  , Cr)   0.74V; E (Cr 2  , Cr)   0.91 V
c. Fe(Z  26) d. Co(Z  27) Match E of the redox pair in Column I with the values
given in Column II.
SECTION 3 Contains 2 Match The Following Type Questions Column I Column II
You will have to match entries in Column I with the entries in 3
(A) E (Fe , Fe) 1. – 0.18 V
Column II.
(B) E(4H 2 O 4   4OH  ) 2. – 0.4 V
19. Match gases under specified conditions listed in Column-I
with their properties/laws in Column-II. (C) E(Cu 2   Cu 
 2Cu  ) 3. – 0.04 V
Column I Column II (D) E (Cr 3  , Cr 2  ) 4. – 0.83 V
(A) Hydrogen gas (P  200 atm, 1. Compressibility
a. A  2; B  3; C  1; D  4
T  273K) factor  1
b. A  1; B  2; C  3; D  4
(B) Hydrogen gas (P 0, 2. Attractive forces
c. A  3; B  4 ; C  1; D  2
T  273K) are dominant
d. A  2; B  3; C  1; D  4
Space for rough work
96 Chemistry
JEE ADVANCE PAPER-II
SECTION 1 Contains 8 Questions.
10. Mixture(s) showing positive deviation from Raoult’s law
The answer to each question is a single digit integer ranging from 0 to at 35ºC is(are)
9 (both inclusive). a. carbon, tetrahedral + methanol
1. The maximum number of electrons that can have principal b. carbon disulphide + acetone
quantum number, n = 3, and spin quantum number, c. benzene + toluene
1 d. phenol + aniline
ms   , is
2 11. For a gaseous state, if most probable speed is denoted by
2. The dissociation constant of a substituted benzoic acid at C*, average speed by C and mean square speed by C,
25ºC is 1.0 × 10–4. The pH of a 0.01M solution of its then for a large number of molecules the ratio of these
sodium salt is speeds are
3. The total number of cyclic structural as well as stereo a. C* : C : C  1.225 :1.128 :1
isomers possible for a compound with the molecular b. C* : C : C  1.128 :1.1225 :1
formula C5 H10 is c. C* : C : C  1:1.228 :1.225
d. C* : C : C  1:1.225 :1.128
4. To an evacuated vessel with movable piston under
external pressure of 1 atm, 0.1 mol of He and 1.0 mol. of 12. The species having bond order different from that in CO is
an unknown compound (vapour pressure 0.68 atm. At a. NO– b. NO+ c. CN– d. N2
0ºC) are introduced. Considering the ideal gas behaviour, 13. Which of the following is the wrong statement?
the total volume (in litre) of the gases at 0ºC is close to a. ONCF and ONO  are not isoelectronic
5. The coordination number of Al in the crystalline state of b. O3 molecules is bent
AlCl3 is c. Ozone is violet–black in solid state
6. Among the following, the number of compounds than can d. Ozone is diamagnetic gas
react with PCl5 to give POCl3 is 14. The qualitative sketches I, II and III given below show the
variation of surface tension with molar concentration of
O 2 , CO 2 , SO 2 , H 2 O, H 2SO 4 , P4 O10
three different aqueous solutions of KCl, CH 3 OH and
7. If the freezing point of a 0.01 molal aqueous solution of a
CH 3 (CH 2 )11 OSO 3 Na  at room temperature. The correct
cobalt (III) chloride-ammonia complex (which behaves as
assignment of the sketches is
a strong electrolyte) is –0.0558ºC, the number of
chloride(s) in the coordination sphere of the complex is
[Kf of water = 1.86 K kg mol–1] II
Surface tension
Surface tension

III
Surface tension

I
8. A student performs a titration with different burettes and
finds titre values of 25.2 mL, 25.25 mL and 25.0 mL. The
number of significant figures in the average titer value is

SECTION 2 Contains 8 Multiple Choice Questions Concentration Concentration Concentration


With one or more than one correct option

9. The correct statement(s) regarding defects in solids is(are) a. I : KCl II : CH 3 OH III : CH 3 (CH 2 )11 OSO 3 Na 
a. Frenkel defect is usually favoured by a very small b. I : CH 3 (CH 2 )11 OSO 3 Na  II : CH 3 OH III : KCl
difference in the sizes of cation and anion
c. I : KCl II : CH 3 (CH 2 )11 OSO 3 Na  III : CH 3 OH
b. Frenkel defect is a dislocation defect
c. Trapping of an electron in the lattice leads to the d. I : CH 3 OH II : KCl III: CH 3 (CH 2 )11 OSO 3 Na 
formation of F-centre 15. Using the data provided, calculate the multiple bond
d. Schottky defects have no effect on the physical energy (kJ mol–1) of CC bond in C2H2. That energy is
properties of solids (take the bond energy of a C—H bond as 350 kJ mol–1)
Mock Test-2 97
1 9
2C(s) + H 2 (g) 
 C 2 H 2 (g);  H  225 kJ mol 3
18. The solubility product (K sp ; mol dm ) of MX2 at 298 K
2C(s) 
 2C(g) based on the information available for the given
H  1410 kJ mol 1 concentration cell is (take 2.303 × R × 298 / F = 0.059 V)
H 2 (g) 
 2H(g) a. 11015 b. 4  1015

 H  330 kJ mol 1
c. 11012 d. 4  1012

a. 1165 b. 837
Paragraph for Question Nos. 19 to 20
c. 865 d. 815
Chemical reactions involve interaction of atoms and molecules.
16. A solution of (–)-1-chloro-1-phenylethane in toluene A large number of atoms/molecules (approximately
racemises slowly in the presence of a small amount of
6.023  10 23 ) are present in a few grams of any chemical
SbCl5 , due to the formation of
compound varying with their atomic/molecular masses. To
a. carbanion b. carbene
handle such large numbers conveniently, the mole concept was
c. carbocation d. free radical
introduced. This concept has implications in diverse areas such
SECTION 3 Contains 2 Paragraph Type Questions as analytical chemistry, biochemistry, electrochemistry and
Each paragraph describes an experiment, a situation or a problem. radiochemistry. The following example illustrates a typical
Two multiple choice questions will be asked based on this paragraph. case, involving chemical/electrochemical reaction, which
One or more than one option can be correct. requires a clear understanding of the mole concept.
Paragraph for Question No. 17 to 18 A 4.0 molar aqueous solution of NaCl is prepared and 500mL
of this solution is electrolysed. This leads to the evolution of
The electrochemical cell shown below is a concentration cell.
chlorine gas at one of the electrodes (atomic mass:
M|M 2  (saturated solution of sparingly soluble salt, Na  23, Hg  200, 1 Faraday  96500 coulombs)
2 3
MX 2 ) || M (0.001 mol dm ) | M. The emf of the cell depends
19. The total number of moles of chlorine gas evolved is
on the difference in concentration of M 2  ions at the two a. 0.5 b. 1.0
electrodes. The emf of the cell at 298 K is 0.059 V. c. 2.0 d. 3.0
17. The value of  G (kJ mol 1 ) for the given cell is
20. If the cathode is a Hg electrode, the maximum weight (g)
(take 1 F = 96500 C mol 1 ) of amalgam formed from this solution is
a. –5.7 b. 5.7 a. 200 b. 225
c. 11.4 d. –11.4 c. 400 d. 446

Space for rough work


98 Chemistry
ANSWER & SOLUTIONS T 400
⇒ =
JEE-Main 2 32
400 × 2
1. 2. 3. 4. 5. 6. 7. 8. 9. 10. ∴ T= = 25 K or T = 25 − 273 = −248°C
b c b c c c a b a b 32
11. 12. 13. 14. 15. 16. 17. 18. 19. 20.
1 1 1  1 1  3R
c a b d d d c a d b 5. (c) Wave number =R 2 − 2  =R −  =
λ  n1 n 2   4 16  16
21. 22. 23. 24. 25. 26. 27. 28. 29. 30.
a a b a c d c a b c ∗
6. (c) HClO3
⇒ 1 + x − 2 × 3 = 0 ; x = 6 − 1 = +5
2 × 6 × 1023
1. (b) Number of atoms in 2 g of oxygen = 1
16 7. (a) Since molar conductance ∝
Molarity
(1) 6 ×10 × 0.5 = 3×10
23 23

8. (b) 1σ and 2π
6 ×1023 × 4
(2) = 7.5 ×1022
32 9. (a) Equilibrium shifts backward by Le-chatelier’s
principle.
6 ×1023 × 7
(3) = 1.5 × 1023
28 K w 1× 10−14
10. (b) K b = = = 5.56 × 10−10
6 ×10 × 2.3
23 K a 1.8 × 10−5
(4) = 6 ×1022
23
11. (c) K = 1.7 ×10−5 s−1
∴ The correct answer is (b).
0.693 0.693
t1/ 2 = = × 105 = 11.32h
2. (c) The axial angles in triclinic crystal system are different K 1.7
and none is perpendicular to any of the others i.e.,
12. (a) Dispersion medium and dispersed phase are phases of
α ≠ β ≠ γ ≠ 90°.
colloid.
3. (b) Level of contamination = 15 ppm
13. (b) Graphite → diamond ∆H t = ( x − y ) kJ mol −1 .
= 15 parts in 106 parts Mass % of CHCl3
14. (d) Definition of disintegration series.
15 15. (d) Phenol will undergo electrophilic substitution more
= 6 ×100 = 1.5 ×10−3
10 readily than benzene.
Thus, the water sample contains 16. (d) Cis and trans 2-butene are geometrical isomers.
1.5 ×10−3% (by mass) of CHCl3 CHCOONa CH
17. (c) || + 2H 2 O 
Electrolysis
→ |||
15g CHCOONa CH
Amount of CHCl3 = = 0.126mol
119.5g mol−1 Acetylene

Mass of water (solvent) = 10 6 g = 10 3 kg + 2CO 2 + 2KOH + H 2

So, molality of CHCl3 in solution 18. (a) C2 H5 Br 


KCN(X)
→ C2 H5CN 
LiAlH4 (Y)

0.126 mol C2 H5CH 2 NH 2 (C3H 7 NH 2 )
= = 1.26 × 10 −4 mol kg −1 X = KCN,Y = LiAlH 4
103 kg
19. (d) HBr reacts with alcohols through the formation of
3RT carbocation. The stable is the carbocation formed, more is
4. (c) r.m.s. (H 2 ) = the reactivity of alcohol with HBr. The carbocation
2
formed are
3R × 400 CH+CH 2CH3
r.m.s. (O 2 ) =
32 CH2CH2CH+2
( T = 273 + 127 = 400 K) CH2 − CH+ − CH3

3 RT 3R × 300
=
2 32 (1) (2) (3)
Mock Test-2 99
CH 2CH 3 2. (0)
20. (b) Cd  2CH3COCl 

CH 2CH 3
2CH3COCH 2CH3  CaCl2 Br
F F

21. (a) Ethanol is the weakest acid among these, hence it is F F


most basic. F
All four planar bonds (F−Br−F) will reduce from 90° to
22. (a) CH3CN  4H 
Na, C2 H5OH
 CH3CH 2 NH 2 (X)
84.8° after p  bp repulsion.
23. (b) Perspex is a synthesised polymer. So among the following only four (4) has linear shape and
2
24. (a) Four Fe ions of each haemoglobin can bind with 4 no d-orbital is involved in hybridization.
molecules of O 2 and it is carried as oxyhaemoglobin. 3. (2) PN2  0.001 atm, T  298 K, V  2.46 cm2
Hb 4  4O2 
 Hb 4 O8 By ideal gas, PV  nRT
PV 0.001 2.46  103
25. (c) Chloramphenicol is broad spectrum antibiotic used in n N2    1.0  107
the treatment of typhoid, dysentry, acute fever. RT 0.0821 298
Now molecules of
26. (d) It shows similarities with both alkali metals as well as
N 2  6.023  1023 1107  6.023 1016
halogens.
Now total surface sites available
27. (c) Impurities of SiO 2 is present in iron ore so basic flux
 6.023 1014 1000  6.023 1017
CaCO3 is added.  Surface site used to adsorb
CaO  SiO2 
 CaSiO3 N2 
20
 6.023  1017  12.04  1016
Flux Impurity Slag 100
 Sites occupied per molecule of
28. (a) BeCl 2  MgCl 2  CaCl 2  BaCl 2
As we go down the group I.E. decreases. Hence ionic 12.04  1016
N2  2
character increases. 6.02  1016

29. (b) 2KI  HgI 2 


 K 2 Hgl 4  KOH 4. (6) 6  H  atoms are there
Nessler's reagent
5. (6) Number of ionisable Cl in [Cr(H 2 O)5 Cl]Cl 2 is 2
30. (c) Primary valencies are also known as oxidation state.
 Millimoles of
K 2 [ Ni (CN) 4 ], 2  x  4  0  x  2
Cl  30  0.01 2  0.6
 Millimoles of Ag  required  0.6
JEE Advance Paper -I
 0.6  0.1 V
1. 2. 3. 4. 5. 6. 7. 8. 9. 10.
 V  6 ml
4 0 2 6 6 6 8 8 a c
11. 12. 13. 14. 15. 16. 17. 18. 19. 20. 6. (6) Black coloured sulphides
c a b a b d c d d c {PbS, CuS, HgS, Ag 2S, NiS, CoS}
1. (4) Vrms ( Xgas )( 400 K )  Vmp ( Ygas )( 60K ) * Bi2S3 in its crystalline form is dark brown but Bi 2S3

M.W.(X gas)  40;M.W.(Ygas)  x precipitate obtained is black in colour.

3RT1 2RT1 7. (8) Molarity



M1 M2
n W W  d sol W 100  d so ln
      1000
400  3 2  60 V M w  v M w  Wsol Wsol M w  100

40 x W
%  d  10
120 W
 30  , x4
x Mw
100 Chemistry
29.2 ×1.25 ×10 1
= = 10 M M1V1 = M 2 V2 PV = −a ×   + RT
36.5 V
⇒ 10 × V1 = 0.4 × 200 y = mx + C fi Slope = – a
0.4 × 200 y2 − y1 20.1 − 21.6
⇒ V= = 8ml Slope = = 1.5
10 x 2 − x1 3−2
Cl 12. (a) q = + 208 J, (as it absorb heat)
Br CH 3 v 
8. (8) w Re w = −2.303 nRT log10  2 
Br Cl  v1 
CH 3  375 
= −2.303 × (0.04 × 8.314 × (310) log10   = −208 J
CH 3  50 
13. (b) E.A = Ionisation potential
CH 3 CH 2 C*− CHCH 3 Two Enantiomeric pairs = 4
* ∴ EA of Na + = −5.1eV
H
14. (a)
CH 3
15. (b) B.O. of Li +2 = 0.5 , B.O. of Li −2 = 0.5
CH 3 CH 2 C − CH 2 CH 3 1
Hence stability order = Li −2 < Li +2 < Li 2
H
0.1
CH 2 Cl 16. (d) 0.1× 1 = (1 + v) × 0.01 ⇒ 1 + v =
0.01
CH 3 CH 2 C*− CH 2 CH 3 1 ⇒ 1 + v = 10 ⇒ v = 10 − 1 = 9L

H 17. (c) [NiCl2 P(C 2 H 5 ) 2 (C6 H 5 ) 2 ]+2


Total = 2 + 4 + 1 + 1 = 8 Ni +2 (28) : [Ar]4s°3d 8
9. (a) For BCC 6 unit cell, 3a = 4r
In strong ligand 3d8
3 3
r= a= × 4.29 Å = 1.85 Å
4 4
18
10. (c) Moles of glucose = = 0.1
180 In weak ligand 3d8
178.2
Moles of water = = 9.9
18 Hybridisation is dsp2 (diamagnetic)
⇒ n Total = 10
Square planar
∆P 0.1
⇒ =
P° 10
Hybridisation is sp3 (paramagnetic)
⇒ ∆P = 0.01P°
= 0.01× 760 = 7.6 torr tetrahedral
PS = 760 − 7.6 = 752.4 torr 18. (d) Factual
11. (c) b = 0, T = 300 K, n = 1
19. (d) A → 1, 2 ; B → 3 ; C → 1, 2 ; D → 1, 4
 an 2 
P + 2  (V − nb) = RT (A) Z =
PVm
at high pressure and low temperature.
 V  RT
 a   an 2 
 P + 2  (V) = RT Equation  P + 2  (V − nb) = nRT reduces to
 V   V 
a P(V − nb) = nRT.
⇒ PV + = RT
V
Mock Test-2 101
(B) For hydrogen gas value of Z = 1 at P = 0 and it increases 3. (7) Cyclic C5 H10
continuously on increasing pressure.
(C) CO2 molecules have larger attractive forces, under normal
conditions.
PVm For 3rd structure 2 cis-trans and 1 optical isomer are
(D) Z  , at very large molar volume Z  1.
RT possible. Total 7 isomers.

20. (c) A  3; B  4 ; C  1 ; D  2 4. (7) Let unknown is X.


(A) G o
 G o
 G o p He  p total  Px  (1  0.68) atm  0.32 atm
Fe3 / Fe Fe3 / Fe2 Fe2 / Fe
RT
 3  FEo(Fe3 / Fe)   1 FE (Fe
o
3
/ Fe2 )
 (2  FEoFe2 / Fe ) Now p He  n He
V
 E oFe3 / Fe   0.04 V RT 0.10  0.082  273
 v  =7
(B) O 2 (g)  2H 2 O  4e  
 4OH E   0.40 V . . .(i) p He 0.32

 O 2 (g)  4H   4e  E   1.23 V . . .(ii)


2H 2 O  5. (6) Coordination number of Al is 6. It exists in ccp lattice
with 6 coordinate layer structure.
So 4H 2 O 4H   4OH . . .(iii)
rd
6. (5)
E For III reduction  0.40  1.23   0.83 V.
7. (1) Tf = iK f m
(C) G o(Cu2 / Cu)  1 FECu
o
2
/ Cu 
 (1 F  ECu
o

/ Cu
)
0.0558  i  1.86  0.01
2  FE oCu 2 / Cu  1 FE oCu 2 / Cu   (1 F  E Cu
o

/ Cu
) i3
 E o
  0.18 V.  Complex is [Co(NH 3 )5 Cl]Cl2
Cu 2 / Cu

(D) G oCr3 / Cr 2  G oCr3 / Cr  G oCr / Cr 2 8. (3)

1 F  E oCr3 / Cr 2  3  F  E oCr 3 / Cr  (2  F  E oCr / Cr2 ) 9. (b, c)


10. (a, b)
 E oCr 3 / Cr 2   0.4 V.
(a) CCl 4  CH 3 OH  Positive deviation from Raoult’s
law
JEE Advance Paper -II
O
1. 2. 3. 4. 5. 6. 7. 8. 9. 10.
9 8 7 7 6 5 1 3 b, c a, b C
11. 12. 13. 14. 15. 16. 17. 18. 19. 20.
(b) CS2  CH 3 CH 3
c a a d d c d b b d
 Positive deviation from Raoult’s law
1. (9) Number of orbital for n  3 is  n  9 2
(c) C 6 H 6  C 7 H 8  Ideal solution
1
Number of electrons for n  3 and m s   9 OH NH 2
2
2. (8) K a (C6 H 5COOH) = 1 × 10–4
(d) +
pH of 0.01M C6 H 5COONa
C6 H5COO  H 2 C6 H5COOH OH 1  Negative deviation from Raoult’s law.
0.01(1 h) 0.01h 0.01h

2RT 8RT 3RT


K w 0.01h 2 11. (c) C*: C : C   
Kh   M M M
Ka 1 h
8
1014 10 2 h 2 2:  3
 (1–h 1) 3.14
104 1 h
1 : 1.128 : 1.225
[OH  ]  0.01h  0.01 104  106
12. (a) NO  (16 electron system)
[H  ]  10 8 pH  8
Bond order =2.
102 Chemistry
 
NO , CN and N2 are isoelectronic with CO therefore all 16. (c) Racemises slowly due to formation of intermediate
have same bond order (=3). carbocation.

13. (a) ONCF and ONO– are isoelectronic in nature. For Question Nos. 17 to 18
17. (d) G  nFE cell
14. (d) Impurities affect surface tension appreciably. It is
 2  96500  0.059
observed that impurities which tend to concentrate on
surface of liquids, compared to its bulk lower the surface  11.387 kJ mol1
tension.  11.4 kJ
Substances like detergents, soaps [CH 3 (CH 2 )11 SO 3 Na ] 0.0591 C
18. (b) E  log10 3
decreases the surface tension sharply. 2 10
Those like alcohol (e.g., –CH 3 OH, C 2 H 5 OH) lower the 0.0591  C 
0.059  log10  3 
surface tension slightly. This can also be related to the fact 2  10 
that CH 3 OH has smaller dielectric constant. Dielectric C
10 2  C  [M 2  ]  10 5 M.
constant is directly proportional to surface tension. So, on 10 3
adding CH 3 OH in water, overall dielectric constant K sp  [M 2 ] [X  ]2  4s3 = 4(10 -5 )3 = 4 ¥ 10-15
decreases and surface tension decreases.
For Question Nos. 19 to 20
Inorganic impurities present in bulk of a liquid such as
19. (b) NaCl   Na   Cl 
KCl tend to increase the surface tension of water.
At anode: 2Cl  
 Cl 2
15. (d) 2C(s)  H 2 (g) 
 C 2 H 2 (g) (H  C  C  H)
Moles of Cl  2 in 500 ml.
 BE(H2) + BE(H2) + Hsub (C) – BE (C – H) × 2 +
Therefore 1 mole of Cl2 evolves.
BE(C  C ) Hrxn
 330  1410  [350  2  x]  225  x= 815 20. (d) Na—Hg (amalgam) formed = 2 moles at cathode.

  
Mock Test-3 103
JEE-MAIN: CHEMISTRY MOCK TEST-3
1. The volume of 1.0 g of hydrogen in litres at N.T.P. is 10. Ammonium cyanide is salt of NH4OH(Kb = 1.8 × 10–5)
a. 2.24 b. 22.4 c. 1.12 d. 11.2 and HCN (Kb = 4.0 × 10–10). The hydrolysis constant of
2. Which set of characteristics of ZnS crystal is correct? 0.1 M NH 4 CN at 25ºC is
a. Coordination number (4: 4) : ccp; Zn2+ ion in the alternate a. 1.4 b. 7.2  10 15
tetrahedral voids
b. Coordination number (6: 6); hcp; Zn2+ ion in all tetrahedral c. 7.2  101 d. 1.4  106
voids
11. For the reaction A + B  C it is found that doubling the
c. Coordination number (6: 4) : hcp; Zn2+ ion in all octahedral
concentration of A increases the rate by 4 times, and
voids
doubling the concentration of B doubles the reaction rate.
d. Coordination number (4: 4); ccp; Zn2+ ion in all tetrahedral
voids What is the overall order of the reaction?
a. 4 b. 3/2
3. PtCl4.6H2O can exist as a hydrated complex. 1 molal
c. 3 d. 1
aqueous solution has depression in freezing point of 3.72º
Assume 100% ionisation and K f (H 2O)  1.86 mol1 kg, 12. The coagulation power of an electrolyte for arsenious
then complex is sulphide decreases in the order
a. [Pt(H2O)6 ]Cl4 b. [Pt(H 2 O) 4 Cl2 ]Cl 2 2H 2O a. Na + , Al+3 , Ba +2 b. PO 4-3 , SO 4-2 , Cl -
c. [Pt(H 2O)3 Cl3 ],Cl.3H 2O d. [Pt(H 2O 4 )Cl 4 ]4H 2 O c. Al+3 , Ba +2 , Na + d. None of these
4. A weather balloon filled with hydrogen gas at 1 atm and 13. Correct relationship between heat of fusion (H fus ), heat
27ºC has volume equal to 12000 litres. On ascending, it
of vaporisation (H vap ) and heat of sublimation (Hsub ) is
reaches a place where temperature is –23ºC and pressure
0.5 atm. The volume of the balloon is a. H fus  H vap  H sub
a. 24000 L b. 12000 L
b. H vap  H fus  H sub
c. 10000 L d. 20000 L
c. H sub  H vap  H fus
5. In Bohr model of the hydrogen atom, the lowest orbit
corresponds to d. H sub  H vap  H fus
a. Infinite energy b. The maximum energy
14. The number of neutrons in the parent nucleus which gives
c. The minimum energy d. Zero energy
N14 on - emission is
6. When KMnO 4 is reduced with oxalic acid in acidic
a. 7 b. 14 c. 6 d. 8
solution, the oxidation number of M n changes from
a. 7 to 4 b. 6 to 4 15. Which represents nucleophilic aromatic substitution
c. 7 to 2 d. 4 to 2 reaction?
7. The unit of molar conductivity is a. Reaction of benzene with Cl 2 in sunlight
a. –1cm–2 mol–1 b.  cm–2 mol–1 b. Benzyl bromide hydrolysis
c. –1cm2 mol–1 d.  cm2 mol–1 c. Reaction of NaOH with dinitrofluorobenzene
8. In a double bond connecting two atoms, there is a sharing of d. Sulphonation of benzene
a. 2 electrons b. 1 electron
16. Which one of the following is the chiral molecule?
c. 4 electrons d. All electrons
a. CH3Cl b. CH 2 Cl2
9. In a reversible reaction, the catalyst
c. CHBr3 d. CHClBrI
a. Increases the activation energy of the backward reaction
b. Increases the activation energy of the forward reaction 17. n-Propyl chloride and benzene react in the presence of
c. Decreases the activation energy of both, forward and anhydrous AlCl3 to form
backward reaction a. ethyl benzene b. methyl benzene
d. Decreases the activation energy of forward reaction
c. n-propyl benzene d. iso-propyl benzene
104 Chemistry
18. 1-chlorobutane reacts with alcoholic KOH to form 24. The waxes are long chain compounds of fatty acids, which
a. 1-butene b. 2-butane belong to the class of
c. 1-butanol d. 2-butanol a. Esters b. Ethers
19. An alcohol having molecular formula C5H11OH on c. Alcohols d. Acetic acid
dehydration gives an alkene, which on oxidation yield a 25. Which of the following is a local anaesthetic?
mixture of ketone and an acid. The alcohol is a. Diazepam b. Procaine
a. CH3CH2CH(OH)CH2CH3 b. CH 3CHCH 2 CH 2 CH 3 c. Mescaline d. None of these
|
OH 26. The electronic configuration of the element which is just
c. (CH3 )2 CHCH(OH)CH3 d. (CH3 )2 CCH2OH above the element with atomic number 43 in the same
periodic group is
20. m-chlorobenzaldehyde on reaction with conc. KOH at
a. 1s 2 2s 2 2p 6 3s 2 3p 6 3d 5 4s 2
room temperature gives
a. potassium m-chlorobenzate and m-chlorobenzyl alcohol b. 1s 2 2s 2 2p 6 3s 2 3p 6 3d10 4s 2 4p5
b. m-hydroxy benzaldehyde and m-chlorobenzyle alcohol c. 1s 2 2s 2 2p 6 3s 2 3p 6 3d 6 4s1
c. m-chlorobenzyl alcohol and m-hydroxy benzyle alcohol
d. 1s 2 2s 2 2p 6 3s 2 3p 6 3d10 4s1 4p 6
d. potassium m-chlorobenzoate and m-hydroxy bezaldehyde
21. Acetamide is treated separately with the following 27. Complex is formed in the extraction of
reagents. Which one of these would give methyl amine? a. Na b. Cu c. Ag d. Fe
a. PCl5 b. NaOH  Br2 28. MgCl 2 .6H 2O when heated gives
c. Sodalime d. Hot conc. H 2SO 4 a. Magnesium oxychloride b. Magnesium dichloride
22. On treating aniline with nitrous acid and HCl at 0–5ºC c. Magnesium oxide d. Magnesium chloride
gives 29. Acidified potassium dichromate on reacting with a
a. An alcohol sulphite is reduced to
b. Diazonium salt a. CrO 2Cl2 b. CrO 24
c. Nitro aniline
c. Cr 3 d. Cr 2
d. Aniline hydrogen chloride
23. ‘Rayon’ is 30. In the extraction of which of the following, complex ion
a. Natural silk b. Artificial silk forms?
c. Natural plastic or rubber d. Synthetic plastic a. Cu b. Ag c. Fe d. Na

Space for rough work


Mock Test-3 105
JEE ADVANCE PAPER-I
SECTION 1 Contains 8 Questions occupied by aluminium ions and n fraction of tetrahedral
The answer to each question is a single digit integer ranging from 0 to holes occupied by magnesium ions, m and n, respectively,
9 (both inclusive). are
1. The molar conductivity of a solution of a weak acid HX 1 1 1
a. , b. 1,
(0.01 M) is 10 times smaller than the molar conductivity 2 8 4
of a solution of a weak acid HY (0.10 M). If  X0   Y0 ,   1 1 1 1
c. , d. ,
the difference in their pK a values, pK a (HX) pK a (HY), 2 2 4 8
is (consider degree of ionization of both acids to be << 1) 10. A monatomic ideal gas undergoes a process in which the
ratio of P to V at any instant is constant and equals to 1.
2. The total number of  and  particles emitted in the What is the molar heat capacity of the gas?
238
nuclear reaction 92 U  82
214
Pb is 4R 3R 5R
a. b. c. d. 0
3. The number of hydroxyl group(s) in Q is 2 2 2


11. The given graphrepresents the variation of Z
H 
H

heat
 P 
aqueous dilute KMnO4 (excess)
0C
Q PV
(compressibility factor  ) versus P, for three real
HO nRT
H 3C CH3 gases A, B and C. Identify the only incorrect statement.
C
4. The total number of basic groups in the following form of A
A
lysine is Ideal gas
 B
1
H 3 N  CH 2  CH 2  CH 2  CH 2 O C
CH  C Z
B
H2 N O 

0 P (atm)
5. In 1 L saturated solution of AgCl [Ksp(AgCl)=1.6×10–10],
0.1 mol of CuCl [K sp (CuCl)  1.0  106 ] is added. The a. For the gas A, a = 0 and its dependence on P is linear at
all pressure
resultant concentration of Ag  in the solution is b. For the gas B, b = 0 and its dependence on P is linear at
x
1.6  10 . The value of “x” is all pressure
c. For the gas C, which is typical real gas for which
6. EDTA4– is ethylenediaminetetraacetate ion. The total
neither a nor b  0. By knowing the minima and the
number of N – Co – O bond angles in [Co(EDTA)1–]
complex ion is point of intersection, with Z = 1, a and b can be
calculated
7. Reaction of Br2 with Na2CO3 in aqueous solution gives
d. At high pressure, the slope is positive for all real gases
sodium bromide and sodium bromated with evolution of
CO2 gas. The number of sodium bromide molecules 12. For the reaction: I   ClO3  H 2SO 4 
 Cl HSO 4 I 2
involved in the balanced chemical equation is The correct statement(s) in the balanced equation is/are
8. A decapeptide (Mol. Wt. 796) on complete hydrolysis a. Stoichiometric coefficient of HSO 4 is 6
gives glycine (Mol. Wt.75), alanine and phenylanine. b. Iodide is oxidised
Glycine contributes 47.0% to the total weight of the c. Sulphur is reduced
hydrolysed products. The number of glycine units present d. H2O is one of the products.
in the decapeptide is 13. The rate of a reaction doubles when its temperature
changes from 300 K to 310 K. Activation of such a
SECTION 2 Contains 10 Multiple Choice Questions reaction will be (R  8.314 JK 1 and log 2  0.301)
With one or more than one correct option
a. 53.6 kJ mol1 b. 48.6 kJ mol1
9. If the unit cell of a mineral has cubic close packed (ccp) array
c. 58.6 kJ mol1 d. 60.5 kJ mol1
of oxygen atoms with m fraction of octahedral holes
106 Chemistry
14. The freezing point (in ºC) of a solution containing 0.1 g of Match the following:
K3[Fe(CN)6](Mol. Wt. 329) in 100 g of water (Kf = 1.86K Column I Column II
kg mol–1) is (A) Vn/Kn = ? 1. 0
a. −2.3 × 10−2 b. −5.7 × 10−2 (B) If radius of nth orbit 2. –1
c. −5.7 × 10−3 d. −1.2 × 10−2 x
∝E ,x=?
n

15. The carboxyl functional group (—COOH) is present in (C) Angular momentum in 3. –2
a. picric acid b. barbituric acid lowest orbital
c. ascorbic acid d. aspirin 1 4. 1
(D) ∝ Zy , y = ?
16. Which compound would give 5-keto-2-methyl hexanal rn
upon ozonolysis? a. A → 3 ; B → 2 ; C → 1 ; D → 4
CH3 b. A → 1 ; B → 2 ; C → 3 ; D → 4
CH3 c. A → 4 ; B → 2 ; C → 1 ; D → 3
a. b.
CH3 d. A → 3 ; B → 4 ; C → 1 ; D → 2
CH3 20. An aqueous solution of X is added slowly to an aqueous
CH3 solution of Y as shown in Column - I. The variation in
CH3
conductivity of these reactions in Column - II.
c. d. H 3C
Column I Column II
CH3
(A) (C 2 H 5 )3 N + CH 3 COOH 1. Conductivity
X Y
decreases and
17. For the process H 2 O(l) 
→ H 2 O(g) at T=100ºC and 1
then increases
atmosphere pressure, the correct choice is
(B) KI(0.1M)+ AgNO3 (0.01 M) 2. Conductivity
a. ∆Ssystem > 0 and ∆Ssurrounding > 0 X Y
decreases and
b. ∆Ssystem > 0 and ∆Ssurrounding < 0 then does not
c. ∆Ssystem < 0 and ∆Ssurrounding > 0 change much
(C) CH 3 COOH + KOH 3. Conductivity
d. ∆Ssystem < 0 and ∆Ssurrounding < 0 X Y
increases and
18. Which of the following exists as covalent crystals in the then does not
solid state? change much
a. Iodine b. Silicon (D) NaOH + HI 4. Conductivity
X Y
c. Sulphur d. Phosphorus does not
change much
SECTION 3 Contains 2 Match The Following Type Questions
and then
You will have to match entries in Column I with the entries in increases
Column II.
a. A → 1 ; B → 2 ; C → 3 ; D → 4
19. According to Bohr’s theory,
b. A → 2 ; B → 4 ; C → 3 ; D → 1
E n = Total energy K n = Kinetic energy
c. A → 3 ; B → 4 ; C → 2 ; D → 1
Vn = Potential energy rn = Radius of nth orbit
d. A → 3 ; B → 4 ; D → 1 ; D → 2
Space for rough work
Mock Test-3 107
JEE ADVANCE PAPER-II
SECTION 1 Contains 8 Questions a. The number of the nearest neighbours of an atom
The answer to each question is a single digit integer ranging from 0 to present in the topmost layer is 12
9 (both inclusive). b. The efficiency of atom packing is 74%
1. Amongst the following, the total number of compounds c. The number of octahedral and tetrahedral voids per
whose aqueous solution turns red litmus paper blue is atom are 1 and 2, respectively
KCN K 2SO 4 (NH 4 ) 2 C 2O 4 d. The unit cell edge length is 2 2 times the radius of the
NaCl Zn(NO3 ) 2 FeCl3 atom
K 2CO3 NH 4 NO3 LiCN 10. The standard Gibbs energy change at 300 K for the
2. When the following aldohexose exists in its D– reaction 2A B  C is 2494.2 J. At a given time, the
configuration, the total number of stereoisomers in its composition of the reaction mixture is [A]  12 , [B]  2 and
pyranose form is
[C]  12 . The reaction proceeds in the:
CHO
| [ R  8.314 J / K / mol, e  2718 ]
CH 2
| a. forward direction because Q  K c
CHOH
| b. reverse direction because Q  K c
CHOH
| c. forward direction because Q  K c
CHOH d. reverse direction because Q  K c
|
CH 2 OH 11. A compound MpXq has cubic close packing (ccp)
3. The number of resonance structures for N is arrangement of X. Its unit cell structure is shown below.
The empirical formula of the compound is
4. The concentration of R in the reaction RP was
measured as a function of time and the following data is
obtained:
[R] (molar) 1.0 0.75 0.40 0.10 M=
X=
t (min). 0.0 0.05 0.12 0.18
The order of the reaction is
5. The oxidation number of Mn in the product of alkaline
oxidative fusion of MnO2 is
a. MX b. MX 2
6. The total number of diprotic acids among the following is
H 3 PO 4 H 2SO 4 H 3 PO 3 H 2CO3 H 2S2 O 7 c. M 2 X d. M 5 X14

H3BO3 H 3 PO 2 H 2 CrO 4 H 2SO 3 12. Which of the following is the energy of a possible excited
7. The number of neutrons emitted when 235
U undergoes state of hydrogen?
92
142 90
a. 13.6 eV b.  6.8 eV
controlled nuclear fission to 54 Xe and 38 Sr is
c.  3.4 eV d.  6.8 eV
8. In dilute aqueous H 2SO 4 , the complex diaquodioxa-
13. Assuming 2s-2p mixing is not operative, the paramagnetic
latoferrate (II) is oxidised by MnO 4 . For this reaction,
species among the following is
the ratio of the rate of change of [H  ] to the rate of a. Be2 b. B2
change of [MnO 4 ] is c. C2 d. N2+
14. In allene(C3H4), the type(s) of hybridization of the carbon
SECTION 2 Contains 8 Multiple Choice Questions
atoms is (are)
With one or more than one correct option
a. sp and sp3 b. sp and sp 2
9. The correct statement(s) for cubic close packed (cep) three
dimensional structure is (are) c. only sp 2 d. sp 2 and sp3
108 Chemistry
15. Isomers of hexane, based on their branching, can be L = 24 cm
divided into three distinct classes as shown in the figure

I. and Cotton wool d Initial formation of Cotton wool


Soaked in X The product Soaked in Y

and
17. The value of d in cm (shown in the figure), as estimated
II.
from Graham’s law, is
a. 8 b. 12 c. 16 d. 20
II. 18. The experimental value of d is found to be smaller than
the estimate obtained using Graham’s law. This is due to
The correct order of their boiling point is a. larger mean free path for X as compared to that of Y
a. I > II > III b. III > II > I b. larger mean free path for Y as compared to that of X
c. II > III > I d. III > I > II c. increased collision frequency of Y with the inert gas as
CH3 compared to that of X with the inert gas
d. increased collision frequency of X with the inert gas as
  N (isomeric products)
Cl 2 , h v
compared to that of Y with the inert gas
16. H3C
CH3 Paragraph for Question Nos. 19 to 20
C5H11Cl 
fractional distillation
 M(isomeric products) When 100 mL of 1.0 M HCl was mixed with 100 mL of 1.0 M
What are N and M ? NaOH in an insulated beaker at constant pressure, a
a. 6, 6 b. 6, 4 temperature increase of 5.7ºC was measured for the beaker and
c. 4, 4 d. 3, 3 its contents (Expt. 1). Because the enthalpy of neutralization of
SECTION 3 Contains 2 Paragraph Type Questions a strong acid with a strong base is a constant (–57.0 kJ mol 1 ),
Each paragraph describes an experiment, a situation or a problem. this experiment could be used to measure the calorimeter
Two multiple choice questions will be asked based on this paragraph. constant. In a second experiment (Expt. 2), 100 mL of 2.0 M
One or more than one option can be correct. acetic acid (K a  2.0  10 5 ) was mixed with 100 mL of 1.0 M
Paragraph for Question Nos. 17 to 18 NaOH (under identical conditions to Expt. 1) where a
X and Y are two volatile liquids with molar weights of 10g temperature rise of 5.6C was measured. (Consider heat
mol–1 and 40g mol–1 respectively. Two cotton plugs, one capacity of all solutions as 4.2 J g 1 K 1 and density of all
soaked in X and the other soaked in Y, are simultaneously
solutions as 1.0 g mL1 )
placed at the ends of a tube of length L = 24 cm, as shown in
the figure. The tube is filled with an inert gas at 1 atmosphere 19. Enthalpy of dissociation (in kJ mol 1 ) of acetic acid
pressure and a temperature of 300 K. Vapours of X and Y react obtained from the Expt. 2 is
to form a product which is first observed at a distance d cm a. 1.0 b. 10.0 c. 24.5 d. 51.4
from the plug soaked in X. Take X and Y to have equal 20. The pH of the solution after Expt. 2 is
molecular diameters and assume ideal behaviour for the inert a. 2.8 b. 4.7 c. 5.0 d. 7.0
gas and the two vapours.
Space for rough work
Mock Test-3 109
ANSWER & SOLUTIONS 8. (c) In a double bond connecting two atoms sharing of 4
JEE-Main electrons take place as in H2C = CH2.

1. 2. 3. 4. 5. 6. 7. 8. 9. 10. 9. (c) Decreases the activation energy of both forward and


d a c d c c c c c b backward reaction.
11. 12. 13. 14. 15. 16. 17. 18. 19. 20. KW
c c c d b,c d c a c a
10. (b) K h =
Ka × Kb
21. 22. 23. 24. 25. 26. 27. 28. 29. 30.
b d b a b a c c c b
1 × 10 −4
= = 1.4
4.0 × 10 −4 × 1.8 ×10 −5
1. (d) 2 g of hydrogen occupy volume = 22.4 L
11. (c) A + B → C On doubling the concentration of A rate of
22.4 ×1
1 g of hydrogen occupies volume = = 11.2 L reaction increases by four times. Rate ∝[A]2. However on
2
doubling the concentration of B, rate of reaction increases
2. (a) ZnS has zinc blende type structure (i.e., ccp structure). two times. Rate ∝[B]
The S2– ions are present at the corners of the cube and at the Thus, overall order of reaction = 2 + 1 = 3
centre of each face. Zinc ions occupy half of the tetrahedral
12. (c) According to Hardy-Schulze rule.
sites. Each zinc ion is surrounded by four sulphide ions
which are disposed towards the corner of regular tetrahedral. 13. (c) Heats of combustion are always exothermic except
Similarly, S2– ions surrounded by four Zn2+ ions. oxidation of N as,
3. (c) (∆Tf )cal = K f × m N 2 + 12 O 2 
→ N 2 O ; ∆H = + ve

(∆Tf )cal = 1.86 ×1 = 1.86 → 2NO ; ∆H = + ve


N 2 + O 2 

(∆Tf )obs 3.72 β


14. (d) 6 X14  → 6+1 N14
i= = = 2 = 1 + (n − 1)α .
(∆Tf )cal 1.86
in 6 X14 no. of neutrons 14 – 6 = 8.
Hence,α = 1,∴ n = 2
∴ Two species will be produced from single species which 15. (b, c)
is only possible for [Pt(H 2 O)3 Cl3 ],Cl3H
Cl3H22OO NO F O2 OH
OH − +

→ Na +
4. (d) P1 = 1 atm, V1 = 12000 L,
T1 = 27 + 273 = 300 K NO NO
Di nitro fluoro benzene
Dinitrofluorobenzene Di nitro phenol
Dinitro phenol
P2 = 0.5 atm, V2 = ?
H
T2 = −23 + 273 = 250 K |
16. (d) I — C* — Br
P1 V1 P2 V2 |
=
T1 T2 Cl
P1 V1 T2 1 × 12000 × 250 A carbon atom which is attached to four different atoms
or V2 = = = 20, 000 L or groups is called a chiral or asymmetric carbon atom.
P2 T1 0.5 × 300
such a carbon atom is often marked by an asterisk.
5. (c) In hydrogen atom, the lowest orbit (n = 1) corresponds
CH 2 CH 2CH 3
to minimum energy (– 13.6 eV).
+7 +2 AlCl3
COOH 17. (c) + CH 3 CH 3 CH 2 Cl  →
6. 5 |
(c) + 2KMnO4 + 3H2SO4  → K2SO4 + 2MnSO4 +10CO2 + 8H2O
COOH
18. (a) CH 3CH 2 CH 2CH 2 − Cl + KOH(alc.)  →
K 2SO 4 + 2MnSO 4 + 10CO 2 + 8H 2 O
CH 3CH 2 − CH = CH 2 + KCl + H 2 O
In this reaction oxidation state of Mn change from +7 to +2. 1-butene

1
7. (c) Molar conductivity = CH 3 — CH — CH — CH 3 Dehydration
ρM 19. (c) | |  →
So, its unit will be Ω–1cm2 mol–1 CH 3 OH
110 Chemistry
CH 3 — C  CH — CH 3 Oxidation
CH 3 — C  O 28. (c) MgCl 2 .6H 2O  heat
 MgO  5H 2O  2HCl
| |
CH 3 CH 3
29. (c) Cr2 O72  8H   2SO32 
 2Cr 3  3SO24  4H 2O
O C — CH 3
| 30. (b) Ag 2S  4NaCN 2Na  Ag  CN 2   Na 2S
OH
Sodium dicyno argentate
20. (a)
2Na [Ag(CN) 2 ]  Zn 
 Na 2 [Zn (CN) 4 ] 2Ag
Cl Cl Cl Sodium tetracynozincate (ppt)

 KOH 
  JEE Advance Paper-I
CHO COOK CH2OH 1. 2. 3. 4. 5. 6. 7. 8. 9. 10.
Cannizzaro's
reaction 3 8 4 2 7 8 5 6 a a
 11. 12. 13. 14. 15. 16. 17. 18. 19. 20.
O O
|| | b a, b, d a a d b b b a c
Ph — C — H  HO  
 Ph — C — H
| [H  ][X  ]
OH 1. (3) HX H  X Ka 
[HX]
I
O [H  ][Y  ]
HY H  Y Ka 
|| [HY]
21. (b) CH 3 — C — NH 2  Br2  NaOH 
 CH 3 NH 2
 m for HX   m1  m for HY   m2
“Hofmann’s bromamide reaction”
1
22. (d)  m1  m Ka  C 2
NH2 N 2 Cl 10 2
2 2
 m   m 
 HONO  
HCl
 Ka1  C1   0 1  Ka 2  C2   0 2 
  m1    m2 
2
NH2 Ka1 C1  m1 
  
Ka 2 C2  m 2 
ææ
Æ O O
2
0.01  1 
   0.001  pKa1  pKa 2  3
23. (b) ‘Rayon’ is man-made fibre which consists of purified 0.1  10 
cellulose in the form of long threads. Rayon resembles 6 2 
2. (8) 92 U 238   80 X 214   82 Pb 214
silk in appearance. Hence called as artificial silk.
(6 , 2  ), total 8 particles.
24. (a) Waxes are esters of higher fatty acids.
25. (b) The anaesthetics produce temporary insensitibility to 3. (4)
the vital function of all type of cells, specially of nervous 
H 
H


system and are used during surgical operations. +
These are classified as HO
(a) General anasthetic – produces unconsciousness all over 

the body e.g. N 2 O, Cyclopropane, chloroform
(b) Local anasthetic – affect only the part of body e.g.
(P)
Xylocaine, Procain etc.
aqueous dilute KMnO4
(excess) 0C
26. (a) 25 Mn  3d 4s .
5 2

OH
27. (c) Hydrometallurgy
Ag 2S  4NaCN 
 2Na[Ag(CN)2 ]  Na 2S HO OH
2Na[Ag(CN)2 ]  Zn 
 Na 2 [Zn(CN)4 ]  2Ag HO (Q)
Mock Test-3 111
O 10. (a)
||
(2) —C — O and — NH 2 are basic groups in lysine.

4. 11. (b)

5. (7) Let the solubility of AgCl is x mol litre 1 12. (a, b, d) ClO3  6I  6H 2SO4 

AgCl Ag   Cl and that of CuCl is y mol litre 1 3I2  Cl  6HSO4  3H 2O
x x

CuCl Cu   Cl Ea  T2  T1 
y y 13. (a) 0.3010   
2.303R  T1T2 
 K sp of AgCl  [Ag  ] [Cl1 ]
Ea  310  300 
 0.3010 
1.6  1010  x(x  y) . . .(i) 2.303  8.314  10 3  310  300 
Similarly K sp of CuCl  [Cu  ] [Cl ] E a  53.6 kJ mol
1.6  106  y(x  y) . . .(ii)  3K   [Fe(CN)6 ]3 , i  4
14. (a) K 3 [Fe(CN)6 ] 
On solving (i) and (ii) [Ag  ]  1.6  107 m 1000 0.1 1000
Tf  K f  i    1.86  4  
 x7 M W 329 100
6. (8) Total no. of N – Co – O bond angles is 8.
 2.3  102 fi Tf  2.3 102
O
15. (d) O
||
O O — C — CH 3
O
COOH
N O
Co (Aspirin)
N O
16. (b) CH 3 CH 3
O O
O
O3
Zn / H 2 O
 CHO
O
CH 3 CH 3
5-keto-2-methyl hexanal
7. (5) 3Br2  3CO32  
 5Br   BrO3  3CO 2
17. (b) At 100C and 1 atmosphere pressure
8. (6) Let number of glycine units  n
Mass of decaeptide = 796 H 2 O( ) H 2 O(g) is at equilibrium. For equilibrium
Mass of H 2 O needed = 162 g, Total mass = 958 g Stotal  0 and Ssystem  Ssurrounding  0
47 Ssystem  0 and Ssurrounding  0
958   75  n 
100
18. (b) Silicon exists as covalent crystal in the solid state.
958  47
 n 6 19. (a) A  3; B  2; C 1; D 4
100  75
9. (a) In ccp lattice: 20. (c) A  3; B 4; C  2; D 1
Number of O atoms 
4 (A)  (C 2 H 5 )3 NH  CH 3 COO 
(C 2 H 5 )3  N  CH 3 COOH 
X Y

Number of Octahedral voids 


4 Initially conductivity increases due to ion formation after
that it becomes practically constant because X alone
Number of tetrahedral voids 
8
cannot form ions. Hence (3) is the correct match.
Number of Al3  4  m (B) KI(0.1 M)  A gNO3 (0.01 M) 
 AgI   KNO3
Number of Mg 2   8  n X Y

Number of ions in the solution remains constant until all


Due to charge neutrality 4(2)  4m(3)  8n( 2)  0
the AgNO3 precipitated as AgI. Thereafter conductance
1 1 increases due to increase in number of ions. Hence (4) is
 m and n 
2 8 the correct match.
112 Chemistry
(C) Initially conductance decreases due to the decrease in the 5. (6) 2MnO2 + 4KOH + O2 → 2K 2 MnO4 + 2H 2O O.S. of Mn = +6i
(Potassiummanganate)
number of OH ions thereafter it slowly increases due to
the increase in number of H2 ++ions.
2MnO 4KOH + O2 (2)
Hence → is2K
the2 MnO 4 + 2H 2O
correct O.S. of Mn = +6in K 2 MnO4
match.
6. (6) H 2SO 4 , H 2 CO3 , H 2S2 O7 , H 2 CrO 4 , H3PO3, H22SO
H 3 PO3 , H SO33
(D) Initially it decreases due to decrease in H+ ions and then
increases due to the increase in OH ions. Hence (1) is the 7. (3) 92 U 235 → 54 Xe142 + 38Sr 90 + 30 n1
correct match.
 
2.303 Ê a0 ˆ 2.303  a 
JEE Advance Paper -II 8. (9) K = log Á ˜ , K= log  0 
t Ë a0 - x ¯ t1/8  1 a0 
1. 2. 3. 4. 5. 6. 7. 8. 9. 10. 8 
3 8 9 5 6 6 3 9 b,c,d b
11. 12. 13. 14. 15. 16. 17. 18. 19. 20.
9. (b, c, d) (a) For any atom in topmost layer, coordination
b c c b b b c d a b number is not 12 since there is no layer above topmost
layer, (b) Fact, (c) Fact, (D) 2 a = 4R
1. (3) K CN , K 2CO3 ,- LiCN are basic in nature and their
aqueous solution turns red litmus paper blue. So, a = 2 2 R
2. (8) 10. (b) ∆G = − RT ln e K c
CH 2 OH
2494.2 = 8.314 × 300 ln e K c
O OH
H −1
H HO ⇒ Kc = e
H
OH H 1
K c = e −1 = = 0.36
2.718
Total No. of stereoisomers = 24 = 16 which contains 8D –
(B)(C) 2 × 12
Configuration and 8 – L Configuration. Q= 2
= =4
[A] [1/ 2]2
3. (9) OH
NaOH
→ N Q > K c , i.e. backward reaction.

1 1
HO 11. (b) X : 8 × + 6 × =4

→NNaOH 8 2
1
N is M= × 4 + 1 = 2; M 2 X 4 = MX 2
4
O–
12. (c) Energy in 1st excited state = −3.4 eV

13. (c) Assuming that no 2s-2p mixing takes place


O– O O
(a) Be 2 → σ 1s 2 , σ *1s 2 , σ 2s 2 , σ * 2s 2 (diamagnetic)
(b) B2 → σ 1s 2 , σ *1s 2 , σ 2s 2 , σ * 2s 2 , σ 2p z2 , ππ 2p
0

1 2 3
x
2p 0
(diamagnetic)
y

O O– O (diamagnetic)
2 π 2p π *2p 1 0
(c) C2 → σ 1s , σ *1s , σ 2s , σ * 2s , σ 2p z , π 2p1x , π *2p0x , σ * 2p z (param
2 2 2 2 0
y y
4 5 6
π 2p1x π *2p 0x
σ 1s 2 , σ *1s 2 , σ 2s 2 , σO* 2s 2 , σ 2p z2 , , σ * 2p 0z (paramagnetic)

O C2 O ,
π 2p1y π *2p 0y

7 8 9
(d) N 2 → σ 1s 2 , σ *1s 2 , σ 2s 2 , σ * 2s 2 , σ 2p z2 , π ,π , σ * 2p 0z (diam
π π

, σ *1s 2 , σ 2s 2 , σ * 2s 2 , σ 2p z2 , ππ 22 pp x2 , π *2 p 0x
2
4.(5) From two data, (forNzero order
σ 1s kinetics)
2
, σ * 2p 0z (diamagnetic)
2 π *2 p 0y
x 0.25 y
KI = = =5
t 0.05 H H
x 0.60 14. (b) C == C == C (allene)
⇒ K II = = =5 H sp 2 sp sp 2 H
t 0.12
Mock Test-3 113
15. (b) III > II > I More the branching in an alkane, lesser Energy evolved due to neutralization of HCl and NaOH
will be the surface area, lesser will be the boiling point.  0.1 57  5.7 kJ  5700 Joule
CH3 CH3 Energy used to increase temperature of solution
16. (b)
Cl  200  4.2  5.7  4788 Joule
* *
H3C H3C
CH3 Energy used to increase temperature of calorimeter
CH3 1,d
1,d  5700  4788  912 Joule
CH3 CH3
ms. t  912
CH3 m.s  5.7  912
H3C Cl H3C
CH2Cl ms  160 Joule / C [Calorimeter constant]
Md, 1 cannot be separated by fractional distillation. Energy evolved by neutralization of CH 3COOH and
For Question Nos. 17 to 18 NaOH  200  4.2  5.6  160  5.6  5600 Joule
rX d 40 So energy used in dissociation of 0.1 mole
17. (c)   2
rY 24  d 10 CH 3COOH  5700  5600  100 Joule
d  48  2d Enthalpy of dissociation  1 kJ / mole
3d  48 1100 1
 d  16 cm 20. (b) CH3COOH  
200 2
18. (d) As the collision frequency increases then molecular 1100 1
CH3CONa  
speed decreases than that expected. 200 2
[salt]
For Question Nos. 19 to 20 pH  pK a  log
[acid]
19. (a) HCl  NaOH 
 NaCl  H 2O
1/ 2
n  100  1  100 m mole  0.1 mole pH  5  log 2  log  pH= 4.7
1/ 2

  
114 Chemistry
JEE-MAIN: CHEMISTRY MOCK TEST-4
1. The number of moles of SO 2 Cl2 in 13.5 g is a. 1.0 104 b. 1.0 1010
a. 0.1 b. 0.2 c. 1.0 1010 d. 1.0 1014
c. 0.3 d. 0.4 11. According to Arrhenius theory, the activation energy is
2. Arrangement of sulphide ions in zinc blende is a. The energy it should possess so that it can enter into an
a. simple cubic b. hcp effective collision
c. bcc d. fcc b. The energy which the molecule should possess in order
to undergo reaction
3. A pressure cooker reduces cooking is increased
c. The energy it has to acquire further so that it can enter
a. Heat is more evenly distributed
into a effective collison
b. Boiling point of water inside the cooker is increased
d. The energy gained by the molecules on colliding with
c. The high pressure tenderizes the food another molecule
d. All of the above
12. Which of the following is property of colloid?
4. Two flasks of equal volume contains SO 2 and CO 2 a. Scattering of light b. They show attraction
respectively at 25 C and 2 atm pressure. Which of the c. Dialysis d. Emulsion
following is equal in them ? 13. Which of the following is an example of endothermic
a. masses of the two gas b. number of molecules reaction?
c. rates of effusion d. molecular structure a. C 2 H 2  2H 2 
 C 2 H 6 ; E  314.0 kJ
5. The ratio of the kinetic energy to the total energy of an
b. C  O 2 
 CO 2 ; E  393.5kJ
electron in a Bohr orbit is
a. –1 b. 2 c. N 2  O 2 
 2NO; E  180.5kJ
c. 1 : 2 d. None of these d. 2H 2  O 2 
 2H 2 O; E  571.8 kJ
6. Oxygen has oxidation states of +2 in the 14. The nuclear binding energy for Ar (39.962384 amu) is:
a. H2O2 b. CO 2 (given mass of proton and neutron are 1.007825 amu and
c. H 2 O d. OF2 1.008665 amu respectively)
a. 343.81 MeV b. 0.369096 MeV
7. Given l / a  0.5 cm 1 , R  50 ohm, N  1.0. The equivalent
c. 931 MeV d. None of these
conductance of the electrolytic cell is
15. Which is an electrophile?
a. 10 ohm 1 cm 2 gm eq 1 b. 20 ohm 1 cm 2 gm eq 1
a. AlCl3 b. CN  c. NH 3 d. CH 3OH
c. 300 ohm 1 cm 2 gm eq 1 d. 100 ohm 1 cm 2 gm eq 1
16. Cyanide and isocyanide are isomers of type
8. In N 2 molecule, the atoms are bonded by a. Positional b. Functional
a. One , Two  b. One , One  c. Tautomer d. Structural
c. Two , One  d. Three  bonds 17. Acetone will be formed by the ozonolysis of
a. Butene-1 b. Butene-2
9. For the reaction H 2 (g)  I 2 (g) 2HI(g), the c. Isobutene d. Butyne-2
equilibrium constant changes with
18. Which of the following reactions gives
a. Total pressure
H 2 C  C  C  CH 2 ?
b. Catalyst
c. The amounts of H 2 and I 2 taken a. CH 2 Br  CBr  CH 2 ææææ
Zn / CH3 OH
Æ

d. Temperature b. HC  C  CH 2  COOH 


Aq.K 2 CO 3
o

40 C

10. A certain weak acid has a dissociation constant of c. CH 2 Br  C  C  CH 2 Br 


Zn
Heat
4
1.0  10 . The equilibrium constant for its reaction with a d. 2CH 2  CH  CH 2 I 

strong base is.
Mock Test-4 115
19. Which of the following does not form phenol or 25. Which of the following is molecular disease?
phenoxide? a. Allergy b. Cancer
a. C6 H 5 Cl b. C 6 H 5 COOH c. German measeles d. Sickel-cell-anaemia
c. C6 H5 N 2 Cl d. C 6 H 5SO 3 Na 26. Hydrogen can be put in halogen group because
20. Identify the final product (Z) in the following sequence of a. It has deuterium and tritium as isotopes
reactions: b. It forms hydrides like chlorides
Me 2 C  O  HCN 
 X 
H 3O 
 Y 
H 2SO4
 Z; c. It contains one electron only
Heat
d. It is light
a. (CH 3 ) 2 C(OH)COOH b. CH 2  C(CH 3 )COOH
27. Which of the following metal is extracted by
c. HOCH 2 CH(CH 3 )COOH d. CH 3CH  CHCOOH
amalgamation process?
21. When propionic acid is treated with aqueous sodium a. Tin b. Silver
bicarbonate, CO 2 is liberated. The C of CO 2 comes from? c. Copper d. Zinc
a. methyl group b. carboxylic acid group 28. Which of the following hydroxide is insoluble in water?
c. methylene group d. bicarbonate group
a. Be(OH)2 b. Mg(OH) 2
22. Aniline undergoes condensation to form Schiff base on
c. Ca(OH)2 d. Ba(OH)2
reacting with
a. Acetyl chloride b. Ammonia 29. The product of oxidation of I ion by MnO4 in alkaline
c. Acetone d. Benzaldehyde
medium is
23. The mass average molecular mass and number average a. I2 b. IO3
molecular mass of a polymer are respectively 40,000 and
30,000. The polydispersity index of polymer will be c. IO4 d. I3
a. < 1 b. > 1 c. 1 d. 0 30. Potassium ferrocyanide is a
24. Hardening of oils is caused by a. Normal salt b. Mixed salt
a. H 2 b. N 2 c. O 2 d. CO 2 c. Double salt d. Complex salt

Space for rough work


116 Chemistry
JEE ADVANCE PAPER-I
SECTION 1 Contains 8 Questions 6. CH 3 — CH 2 — CH 2 — CH 2 OH
The answer to each question is a single digit integer ranging from 0 to CH3 — CH 2 — CH — CH 3
9 (both inclusive). |
OH
1. HCl gas is passed into water, yielding a solution of enantiomeric (  )

1
density 1.095g mL and containing 30% HCl by weight. OH
|
Calculate the molarity of the solution. CH3 — C — CH3 ; CH 3 — CH — CH 2 OH
| |
2. A sample contains a mixture of NaHCO3 and Na 2 CO3 . CH3 CH 3
HCl is added to 15.0 g of the sample, yielding 11.0 g of Total number of isomers (including stereoisomers) is
NaCl. What percent of the sample is Na 2 CO3 ? 7. Total number of isomers
Reactions are: CH 3 CH 2 CH3 CH 3 CH 3
 Na 2 CO 3  2HCl 
 2NaCl  CO 2  H 2 O 
 
 NaHCO 3  HCl   NaCl  CO 2  H 2 O 
Mw of NaCl  58.5, Mw of NaHCO3  84, Mw of CH 3 CH 3 H CH 3 Mirror CH 3 H
Na 2 CO3  106g mol1
H H CH 3 H H CH 3
3. Amongst the following, the total number of compounds
meso Pair of enantiomers
soluble in aqueous NaOH is
H 3C CH 3 8. In the scheme given below, the total number of
N COOH NO 2 OH
intramolecular aldol condensation products formed from
‘Y’ is

1. O3
 Y 
1. NaOH(aq)


N 2. Zn, H2 O 2. heat
H 3C CH 3
OCH 2CH 3 OH
CH 2OH SECTION 2 Contains 10 Multiple Choice Questions
With one or more than one correct option
9. For a dilute solution containing 2.5 g of a non-volatile
CH 2CH3 COOH non-electrolyte solute in 100 g of water, the elevation in
CH 2CH3 boiling point at 1 atm pressure is Assuming concentration
of solute is 2C. Assuming concentration of solute is
much lower than the concentration of solvent, the vapour
pressure (mm of Hg) of the solution is (take
4. The total number of contributing structures showing
K b  0.76 K kg mol1 )
hyperconjugation (involving C—H bonds) for the
a. 724 b. 740
following carbocation is.
c. 736 d. 718
5. The total number(s) of stable conformers with non-zero 10. Two closed bulbs of equal volume (V) containing an ideal
dipole moment for the following compound is (are) gas initially at pressure pi and temperature T1 are
connected through a narrow tube of negligible volume as
Cl shown in the figure below. The temperature of one of the
bulbs is then I raised to T2. The final pressure pf is:
Br CH 3
T1 T1 T1 T2
Br Cl 
Pi , V Pi , V Pf ,V Pf ,V
CH 3
Mock Test-4 117
 TT   T1  17. The number of structural isomers for C6H14 is
a. p i  1 2  b. 2p i  
 T1  T2   T1  T2  a. 3 b. 4
c. 5 d. 6
 T2   TT 
c. 2p i   d. 2p i  1 2 
 T1  T2   T1  T2  18. Amongst the given options, the compound(s) in which all
the atoms are in one plane in all the possible
11. A stream of electrons from a heated filament was passed
conformations (if any), is (are)
between two charged plates kept at a potential difference
H H
V esu. If e and m are charge and mass of an electron, CC H
a. b. H  C  C — C
respectively, then the value of h/ (where  is wavelength H2C CH 2 CH 2
associated with electron wave) is given by:
c. H 2 C  C  O d. H 2 C  C  CH 2
a. meV b. 2 meV
c. meV d. 2 meV
SECTION 3 Contains 2 Matches The Following Type Questions
12. Reduction of the metal centre in aqueous permanganate You will have to match entries in Column I with the entries in
ion involves Column II.
a. 3 electrons in neutral medium
19. According to Bohr’s theory,
b. 5 electrons in neutral medium
c. 3 electrons in alkaline medium E n  Total energy K n  Kinetic energy
d. 5 electrons in acidic medium Vn  Potential energy rn  Radius of nth orbit
13. For the following electrochemical cell at 298 K, Match the following:
 4 2
Pt(s) | H 2 (g, 1 bar) H (aq, 1 M) || M (aq), M (aq) | Pt(s) Column I Column II
2
[M (aq)] (A) Vn / K n  ? 1. 0
E cell  0.092 V when  10 x.
[M 4  (aq)] (B) If radius on nth orbit 2. –1
RT E ,x ?
x
Given: E 0M4 / M2  0.151 V; 2.303  0.059 V n
F (C) Angular momentum in 3. –2
The value of x is
lowest orbital
a. –2 b. –1
1 4. 1
c. 1 d. 2 (D) n
 Zy , y  ?
r
14. Which one of the following molecules is expected
a. A  3; B  2; C  4; D  1
diamagnetic behaviour?
b. A  1; B  2; C  3; D  4
a. C2 b. N2
c. O2 d. S2 c. A  3; B  1; C  2; D  4

15. Aqueous solutions of HNO3, KOH, CH3COOH and d. A  3; B  2; C 1; D 4


CH3COONa of identical concentrations are provided. The 20. Match the thermodynamic processes given under Column
pair (s) of solutions which form a buffer upon mixing is I with the expression given under Column II:
(are) Column I Column II
a. HNO3 and CH 3COOH
(A) Freezing of water at 1. q = 0
b. KOH and CH 3COONa 273 K and 1 atm
c. HNO3 and CH 3COONa (B) Expansion of 1 mol of 2. w = 0
d. CH 3COOH and CH 3COONa an ideal gas into a
vacuum under
16. The value of log10K for a reaction A B is
isolated conditions
(Given  r H o298K  54.07 kJ mol1 , rSo298K  10 JK1mol1 (C) Mixing of equal 3. Ssys  0
1 1
and R  8.314 JK mol ; 2.303 × 8.314 × 298 = 5705 ) volumes of two ideal
a. 5 b. 10 c. 95 d. 100 gases at constant
118 Chemistry
temperature and cooling to 300 K at 1
pressure in an isolated atm
container 5. G  0
(D) Reversible heating of 4. U  0 a. A  3,5; B  1,2,4 ; C  1,2,4; D  1,2,4,5
H 2 (g) at 1 atm from b. A  1,2,4; B  1,2,4,5; C  3,4; D  1,2,4
300 K to 600 K, c. A  3,5; B  1,2,4; C  1,2,4,5; D  1,2,4
followed by reversible d. A  1,2,4; B  3,5; C  1, 4,5; D  1,2,4

Space for rough work


Mock Test-4 119
JEE ADVANCE PAPER-II
SECTION 1 Contains 8 Questions h2 h2
a. b.
The answer to each question is a single digit integer ranging from 0 to 4 2 ma 02 16 2 ma 02
9 (both inclusive).
h2 h2
1. 0.45 g of an acid (mol wt. = 90) required 20 ml of 0.5 N c. d.
32 2 ma 02 64 2 ma 02
KOH for complete neutralization. Basicity of acid is
12. The equilibrium 2Cu I Cu  Cu II in aqueous
2. The co-ordination number of copper in cuprammonium
medium at 25ºC shifts towards the left in the presence of
sulphate is
a. NO 3 b. Cl c. SCN d. CN 
3. The co-ordination number of cobalt in the complex
[Co(en) 2 Br2 ]Cl2 is 13. The bond energy (in kcal mol–1) of C – C single bond is
approximately
4. The primary valence of the metal ion in the co-ordination a. 1 b. 10 c. 100 d. 1000
compoun K 2 [Ni  CN 4 ] is
14. N 2  3H 2 2NH 3 Which is correct statement if N 2
5. The oxidation number of Cr in [Cr(NH 3 )6 ]Cl3 is is added at equilibrium condition?
6. The number of equivalent Cr  O bonds in CrO 24 is. a. The equilibrium will shift to forward direction because
according to IInd law of thermodynamics the entropy must
7. The number of the following reagents that produce ppt. increases in the direction of spontaneous reaction
with ZnSO 4 solution is. b. The condition for equilibrium is G N2  3G H2  2G NH3
NaOH, N 2 CO3 , NaCl, Na 2 HPO 4 , Na 2S, CH 3CO3 Na where G is Gibbs free energy per mole of the gaseous
species measured at that partial pressure. The condition of
8. The change in the magnetic moment value when
equilibrium is unaffected by the use of catalyst, which
2 2
Cu  H2O 4  is converted to Cu  NH3 4  is. increases the rate of both the forward and backward
reactions to the same extent
c. The catalyst will increase the rate of forward reaction
SECTION 2 Contains 8 Multiple Choice Questions
by α and that of backward reaction by .
With one or more than one correct option
d. Catalyst will not alter the rate of either of the reaction
9. A gas described by van der Waal’s equation
a. behaves similar to an ideal gas in the limit of large 15. For a first order reaction AP, the temperature (T)
molar volumes dependent rate constant (k) was found to follow the
b. behaves similar to an ideal gas in the limit of large 1
equation log k  (2000)  6.0. The pre-exponential
T
pressures
factor A and the activation energy Ea, respectively, are
c. is characterized by van der Waal’s coefficients that are
6 1
dependent on the identity of the gas but are independent of a. 1.0 10 s and 9.2 kJmol 1
1
the temperature b. 6.0s and 16.6 kJmol 1
d. has the pressure that is lower than the pressure exerted 6 1
c. 1.0 10 s and 16.6 kJmol 1
by the same gas behaving ideally
6 1
d. 1.0 10 s and 38.3 kJmol 1
10. Assuming that Hund’s rule is violated, the bond order and
magnetic nature of the diatomic molecule B 2 is 16. The initial rate of hydrolysis of methyl acetate (1 M) by a
weak acid (HA, 1M) is 1/100th of that of a strong acid
a. 1 and diamagnetic b. 0 and diamagnetic
(HX, 1M), at 25ºC. The Ka of HA is
c. 1 and paramagnetic d. 0 and paramagnetic
a. 1104 b. 1105
11. The kinetic energy of an electron in the second Bohr orbit
c. 1106 d. 1103
of a hydrogen atom is [a0 is Bohr radius].
120 Chemistry
SECTION 3 Contains 2 Paragraph Type Questions b. At the start of the reaction, dissociation of gaseous X 2
Each paragraph describes an experiment, a situation or a problem. takes place spontaneously
Two multiple choice questions will be asked based on this paragraph. c. βequlibrium = 0.7
One or more than one option can be correct.
d. K C < 1
Paragraph for Question No. 17 to 18
Paragraph for Question No. 19 to 20
Thermal decomposition of gaseous X 2 to gaseous X at 298 K
takes place according to the following equation: Tollen’s reagent is used for the detection of aldehyde when a
X 2 (g) 2X(g) solution of AgNO 3 is added to glucose with NH 4 OH then

The standard reaction Gibbs energy, ∆r G° of this reaction is gluconic acid is formed
Ag + + e − 
→ Ag ; E ored = 0.8 V
positive. At the start of the reaction, there is one mole of X 2
and no X. As the reaction proceeds, the number of moles of X C6 H12 O 6 + H 2 O 
→ Gluconic acid
formed is given by β . Thus β equilibrium is the number of moles of (C6 H12 O7 ) + 2H + + 2e− ; E ooxd = − 0.05 V
X formed at equilibrium. The reaction is carried out at a Ag(NH3 ) 2+ + e − 
→ Ag (s) + 2NH3 ; E ooxd = 0.337 V
constant total pressure of 2 bar. Consider the gases to behave RT F
ideally. (Given: R = 0.083 L bar K −1 mol −1 ) [Use 2.303 × = 0.0592 and = 38.92 at 298 K
F RT
17. The equilibrium constant K p for this reaction at 298 K, in 19. 2Ag − + C6 H12 O6 + H 2O 
→ 2Ag (s) + C6 H12 O7 + 2H +
terms of β equilibrium , is Find ln K of this reaction.
2 2
a. 66.13 b. 58.38
8βequilibrium 8βequilibrium c. 28.30 d. 46.29
a. b. 2
2 − βequilibrium 4 − βequilibrium
20. When ammonia is added to the solution, pH is raised to
2 2
4βequilibrium 4β equilibrium 11. Which half-cell reaction is affected by pH and by how
c. d.
2 − βequilibrium 2
4 − βequilibrium much?
a. E oxd will increase by a factor of 0.65 from Eooxd
18. The incorrect statement among the following, for this
b. E oxd will decrease by a factor of 0.65 from Eooxd
reaction, is
a. Decrease in the total pressure will result in formation of c. E red will increase by a factor of 0.65 from E ored
more moles of gaseous X d. E red will decrease by a factor of 0.65 from E ored

Space for rough work


Mock Test-4 121
ANSWER & SOLUTIONS 13. (a) For exothermic reactions H p < H R .
JEE-Main For endothermic reactions H p > H R .
1. 2. 3. 4. 5. 6. 7. 8. 9. 10.
a d b b a d a a d c 14. (a) 18 Ar 40 Total no of protons = 18
11. 12. 13. 14. 15. 16. 17. 18. 19. 20. Total no of neutrons = 22
c a a a a b c c b c
Mass defect = [m × p + m × n] − 39.962384
21. 22. 23. 24. 25. 26. 27. 28. 29. 30.
d d b a b b b a b d
= [1.007825 ×18 + 1.008665 × 22] − 39.962384
= [18.14085 + 22.19063] − 39.962384 = 0.369
1. (a) Molecular mass of
SO 2 Cl 2 = 32 + 2 × 16 + 35.5 × 2 = 135 Binding energy = mass defect × 931

13.5 = 0.369 × 931 = 343.62MeV


Moles = = 0.1
135 15. (a) AlCl3 is lewis acid i.e., electron deficient compound.
2−
2. (d) Arrangement of sulphide ions (S ) in zinc blende So it is electrophile.
2+ r
(ZnS) is fcc while Zn ions occupy alternate tetrahedral 16. (b) R — C ≡≡ N and R — N = C are functional isomers.
Cyanide Isocyanide
voids.
3. (b) The temperature at which a liquid boils increases with CH 3C == O
17. (c) CH 3 — C == CH 2 |
increase in pressure. | O3 CH 3
CH 3 
H2O, Zn
→ Acetone
4. (b) Equal volumes of all gases under similar conditions of
Zn
temperature and pressure contain equal number of 18. (c) CH2Br − C ≡ C − CH2Br → CH 2 = C = C = CH 2

molecules.
19. (b) Benzoic acid.
5. (a) K.E. = – (T.E.)
OH
6. (d) Oxygen have + 2 oxidation state in OF2 . | H3O+
20. (b) Me2C = O + HCN 
→ CH 3 — C — CN  →
−1 |
7. (a) l / a = 0.5 cm , R = 50 ohm CH 3
(X )
Ra 50
p= = = 100
l 0.5 OH
| H2SO4
1000 1 1000 1 1000 CH 3 — C — COOH  → CH 2 == CCOOH
Λ = k× = × = × = 10 ohm−1 cm2 gm eq−1 | |
N p N 100 1 CH3
CH 3
(Y ) (Z)
π σ
*
8. (a) N N 21. (d) CH 3 — CH 2 — COOH + Na H CO3 
→ CH 3CH 2 COONa + H 2 O
* *
π CH 3 — CH 2 — COOH + Na H CO 3 
→ CH 3CH 2 COONa + H 2 O + C O 2
9. (d) Equilibrium constant changes with temperature,
22. (d) C6H5NH2 + O = CHC6H5 
→C6H5 − N = CHC6H5 + H2O
pressure and the concentration of either reactant or Schiff 's base
product.
23. (b) Average number molecular weight Mn = 30,000
10. (c) HA : K a = 10−4
Average mass molecular weight M w = 40,000
HA
HA+ NaOH
+ NaOH NaA
NaA+H
+ 2HO O
Clearly, the reverse reaction is the hydrolysis reaction. M w 40,000
Polydispersity index (PDI) = = = 1.33
1 K 10−4 Mn 30,000
⇒ K Required = = a = −14 = 1010
K h K w 10 Ni
24. (a) Oil (unsaturated) + H 2  → Fat (saturated)
11. (c) The definition of activation energy.
25. (b) “Cancer” is known as molecular disease.
12. (a) Scattering of light is a property of colloid.
122 Chemistry
26. (b) Hydrogen, forms hydrides like halides, e.g. HCl. 4. (6) These are total 6 — H to sp carbon and they all can
2

27. (b) Cu 2 Cl 2  Ag 2S 
 Cu 2S  2AgCl participate in hyperconjugation.
2AgCl  Hg 
 Hg 2 Cl 2  2Ag H H

H
AgCl  Hg 
 Ag  HgCl

  
28. (a) The solubility of hydroxides of alkaline earth metals in
water increases on moving down the group. Three structures Two structures

29. (b) KI  MnO  


 K IO  Mn
4
 
3
2 5. (3)
6. (5)
30. (d) In K 4 Fe(CN) 6 , the species retains its identity in solid
7. (7)
as well as in solution state.
8. (1)
JEE Advance Paper -I Ws
9. (a) TL  k b  m  k b 
1. 2. 3. 4. 5. 6. 7. 8. 9. 10. M s  Wsolution
9 9 4 6 3 5 7 1 a c 2.5  1000 0.76  2.5  1000
2  0.76  Ms   9.5
11. 12. 13. 14. 15. 16. 17. 18. 19. 20. Ms 100 100  2
d a, d d c c. d b c b, c d a
Ws
% by weight 10  d 30  10 1.095 760  x ns Ws 2.5  18
1. (9) M    9M  Xs     36.0
Mw2 36.5 760 n solution Wsolv 9.5  100
M solv
2. (9) Let x be the percentage of Na 2 CO3 . Then, weight of
x  760  36  724
NaHCO3  (15  x)g
10. (c) Initial moles = final moles
11.0 g
Moles of NaCl produced   0.18 mol Pi  V Pi  V Pf  V Pf  V
58.5g   
RT1 RT1 RT2 RT1
 x 
The NaCl is produced by the reaction of   mol of Pi Pi Pf Pf 2Pi 1 1
 106      Pf   
T1 T1 T2 T1 T1  T2 T1 
(15  x)
Na 2 CO3 and mol of NaHCO 3 . Each mol of 2Pi  T  T2   T2 
84  Pf  1  Pf  2Pi   
Na 2 CO3 produces 2 mol of NaCl.
T1  T1T2   T1  T2 

2 x 15  x 11. (d) K.E.  eV


   0.188 Solve x :  13.5g Na 2 CO3 ,
106 84 h h
   2meV
NaHCO3  (15  1.35)  13.6 g 2meV 
1.35
% Na 2 CO3  100  9.0% Na 2 CO3 12. (a, d) In acidic medium
15
MnO 4  8H   5e  
 Mn 2   4H 2 O
3. (4) Aromatic alcohols and carboxylic acids form salt with
In neutral medium,
NaOH, will dissolve in aqueous NaOH.
MnO 4  2H 2 O  3e  
 MnO 2  4OH 
H 3C  CH 2 CH 3
Hence, number of electrons loose in acidic and neutral
medium are 5 and 3 respectively.
0.059 [M 2  ][H  ]2
COOH OH OH COOH 13. (d) E cell  E ocell  log10
2 [M 4  ]pH 2
0.059
0.092  0.151  log10 10x
N 2
H 3C CH 3  x2
Mock Test-4 123
14. (c) O2 is expected to be diamagnetic in nature but actually 4. (2) Primary valencies are also known as oxidation state.
it is paramagnetic.
K 2 [ Ni (CN) 4 ], 2 + x − 4 = 0 ⇒ x = +2
15. (c, d) In option (c), if HNO3 is present in limiting amount
then this mixture will be a buffer and the mixture given in 5. (3) x + 6 × (0) + 3 × (−1) = 0
option (d), contains a weak acid (CH3COOH) and its salt x – 3 = 0, x + 3, Oxidation number of Cr is = +3
with strong base NaOH, i.e. CH3COONa. 6. (4)
16. (b) ∆G ° = ∆H° − T∆S° = −54.07 × 1000 − 298 × 10 7. (4)
= −57050 J mol−1 − 57050 = −5705log10 K 8. (0)
log10 K = 10
 n 2a 
Hence (b) is correct. 9. (a, c, d)  P + 2  (V − nb) = nRT
 V 
17. (c) C6 H14
At low pressure, when the sample occupies a large
H 3C — CH 2 — CH 2 — CH 2 — CH 2 — CH 3
volume, the molecules are so far apart for most of the time
H 3 C — CH — CH 2 — CH 2 — CH 3 that the intermolecular forces play no significant role, and
|
CH 3 the gas behaves virtually perfectly.
a and b are characteristic of a gas and are independent of
H 3C — CH — CH — CH 2 — CH 3
|  n2a 
CH 3 temperature. The term  P + 2  represents the pressure
 V 
H 3C — CH — CH — CH 3
| | exerted by an ideal gas while P represents the pressure
CH 3 CH 3 exerted by a real gas.
CH3
10. (a) B2 (10) = σ1s2 σ 1s2σ 2s2σ 2s2 π 2p2
* *
| x
H 3C — C — CH 2 — CH 3 , Hence (c) is correct
| 6−4
Bond order == 1 (nature is diamagnetic as no
CH 3 2
unpaired electron)
18. (b, c)
19. (d) A → 3; B → 2; C →1; D →4 nh mv2 e2
11. (c) mvr = and = 2 ⇒ mv2 r = e2
20. (a) A → 3,5; B → 1,2,4; C → 1,2,4; D → 1,2,4,4 2π r r

e2 × 2π nh
JEE Advance Paper -II ⇒ v= ∴ (mvr = )
nh 2π
1. 2. 3. 4. 5. 6. 7. 8. 9. 10.
1 4
2 4 6 2 3 4 4 0 a, c, d a me × 4π 2
1
11. 12. 13. 14. 15. 16. 17. 18. 19. 20. ⇒ K.E. = mv = 2 2 2
2
. . .(i)
2 nh
c b,c,d c b d a b c b a
h2
WB × 1000 Expression for a 0 =
1. (2) Normality = N = 4π 2 me2
Eq.wt × V
h2
0.45 × 1000 ⇒ me2 = . . .(ii)
∴ Eq. Wt = = 45 4π 2a 0
0.5 × 20
h2 1
Molec. Wt 90 ⇒ K.E. = × 2 . . .(iii)
∴ Basicity = = =2 8ma 0π n
2 2

Eq. Wt 45
h2
2. (4) In Cuprammonium sulphate [Cu(NH3 )4 ]SO4 co- For n = 2 K.E. =
32π 2 ma 02
ordination no. of Cu is 4.
12. (c, d) Cu2+ ions will react with CN– and SCN– forming
3. (6) [Co(en)2 Br2 ]Cl2
[Cu(CN) 4 ]3 − and [Cu(SCN) 4 ]3− leading the reaction in
C.N.of Co = 2 × number of bidentate ligand
the backward direction.
+1× number of monodentate ligand = 2 × 2 + 1× 2 = 6.
124 Chemistry
2 
Cu  2CN 
 Cu(CN) 2  
 eq   2eq 
2Cu(CN) 2 
 2CuCN  (CN) 2 Px(g)  Ptotal    Ptotal 
1  eq   2  eq 
CuCN  3CN  
 [Cu(CN) 4 ]3  2 
Cu 2   4SCN  
 [Cu(SCN) 4 ]3  2 eq. 
2

  Ptotal 
Cu 2  also combines with CuCl2 which reacts with Cu to (Px) 2  2   eq. 
So Kp  
produce CuCl pushing the reaction in the backward (Px 2 )  2   eq. 
  Ptotal 
direction. (2   )
 eq 
CuCl2  Cu 
 2CuCl 
4eq.
2
 8eq2 
13. (c)  Kp   P   2 
4  eq.  4  eq 
2 total

14. (b)
18. (c) (a) Correct statement. As one decrease in pressure
2000 reaction will move in the direction where no. of gaseous
15. (d) Given, log K  6 
T molecules increases.
Ea (b) Correct statement
Since, log K  log A 
2.303RT
At the start of reaction Qp  K p so dissociation of X 2 take
1
So, A  10 sec
6
place spontaneously.
and Ea  38.3kJ / mole (c) Incorrect statement as

16. (a) Rate in weak acid 


1
(rate in strong acid) 8 eq2 8  (0.7)2
100 Kp    1, but
4  eq2 4  (0.7)2
1
 [H  ]weak acid  [H  ]strong acid (d) Correct statement.
100
As G  0 & G  RT n K p
1
 
[H ]weak acid  M  10 2 M G  1, so K p should be less than 1.
100
 C  102 So K 1
K p  K c (RT) ng. (RT  1)
 K a  104
Kp
For Question Nos. 17 to 18 Kc   Kc  Kp
RT
17. (b) Paragraph-1 X 2 (g) 2X(g)
So Kc  1
Initial mole 1 0
t  t eq. (1   ) 2
For Question Nos. 19 to 20
RT
equilibrium 19. (b) E ocell  ln K
Given 2  equilibrium So,   nF
2 1 0.0592
(0.8  0.05)   ln K
  eq  2 2.303
Total mole at equilibrium  (1   )  (1   
 2  (0.8  0.05)  22.303
ln k   58.38
0.0592
  eq. 
1  
Px 2   2 P    2   eq P    2   eq P  20. (a) On increasing concentration of NH3 the concentration
 total   total 
of H+ ion decreases. Therefore, Ered increases.
1   eq.   2   eq   2   eq
total

 2 

  
Mock Test-5 125
JEE-MAIN: CHEMISTRY MOCK TEST-5
1. One mole of potassium dichromate completely oxidises 9. The formation of nitric oxide by contact process
the following number of moles of ferrous sulphate in N 2  O2 2NO. H  43.200 kcal is favoured by
acidic medium
a. Low temperature and low pressure
a. 1 b. 3
b. Low temperature and high pressure
c. 5 d. 6
c. High temperature and high pressure
2. In a solid lattice, the cation has left a lattice site and is d. High temperature and excess reactants concentration
located at an interstitial position, the lattice defect is
a. Frenkel defect 10. The pH of an aqueous solution Mg  OH 2 is 9.0. If the
b. Schottky defect solubility product of Mg  OH 2 is 1 10 11 , what is [ Mg 2  ] ?
c. F-centre defect
a. 1  10 5 b. 1.0  104
d. Valency defect
c. 1 102 d. 0.1
3. At 300 K, the vapour pressure of an ideal solution containing
3 mole of A and 2 mole of B is 600 torr. At the same 11. If ‘I’ is the intensity of absorbed light and C is the
temperature, if 1.5 mole of A and 0.5 mole of C (non- concentration of AB for the photochemical process
volatile) are added to this solution the vapour pressure of AB  hv  AB*, the rate of formation of AB* is directly
solution increases by 30 torr. What is the value of PBo ? proportional to
a. 940 b. 405 a. C b. I
c. 90 d. None of these c. I 2 d. C.I
12. Gold number is maximum for the lyophilic sol is
4. If the root mean square speed of helium is 4.75 m s–1 at
a. Gelatin b. Haemoglobin
25C, then its speed will become 9.50 m s–1 at
c. Sodium oleate d. Potato starch
a. 100C b. 323C
13. For the allotropic change represented by equation
c. 919C d. 1192C
C(diamond)  C(graphite); the enthalpy change is
5. Which electronic level would allow the hydrogen atom to H  1.89 kJ. If 6 g of diamond and 6 g of graphite are
absorb a photon but not to emit a photon?
separately burnt to yield carbon dioxide, the heat liberated
a. 3s b. 2p in the first case is
c. 2s d. 1s a. Less than in the second case by 1.89 kJ

6. The product of oxidation of I  with M nO 4 in alkaline b. More than in the second case by 1.89 kJ

medium is c. Less than in the second case by 11.34 kJ


a. IO 3 b. I 2 d. More than in the second case by 0.945 kJ
c. IO  d. IO 4 14. The half-life of 6 C14 if its K or  is 2.31 104 is
7. The standard reduction electrode potentials of four a. 2  10 2 yrs b. 3  10 3 yrs
elements are A   0.250 V, B   0.136 V, C   0.126 V
c. 3.5  10 4 yrs d. 4  103 yrs
and D   0.402 V. The element that displaces A from its
15. Most stable carbonium ion is
compounds is
 
a. B b. C a. C 2 H 5 b. (CH 3 )3 C
c. D d. None of these  
c. (C 6 H 5 )3C d. C 6 H 5CH 2
8. The bond angle in carbon tetrachloride is approximately
a. 90 16. Which of the following compounds will exhibit cis-trans
b. 109 isomerism
c. 120 a. 2-butene b. 2-butyne
d. 180 c. 2-butanol d. Butanone
126 Chemistry
17. Acetylene gas is obtained by the electrolysis of 24. The base present in DNA, but not in RNA is
a. Sodium fumarate a. Guanine b. Adenine
b. Sodium succinate c. Uracil d. Thymine
c. Sodium maleate
25. Which of the following is an antidiabatic drug
d. Both (a) and (c)
a. Insulin b. Penicillin
18. The compound added to prevent chloroform to form
c. Chloroquine d. Aspirin
phosgene gas is
a. C2 H5OH 26. The correct sequence of elements in decreasing order of
first ionisation energy is
b. CH3COOH
a. Na  Mg  Al b. Mg  Na  Al
c. CH 3COCH 3
c. Al  Mg  Na d. Mg  Al  Na
d. CH 3 OH
19. Epoxides are 27. In the metallurgical extraction of zinc from ZnO the
a. Cyclic ethers reducing agent used is
b. Not ethers a. Carbon monoxide b. Sulphur dioxide
c. Aryl-alkyl ethers c. Carbon dioxide d. Nitric oxide
d. Ethers with another functional group
28. In the lime (kiln), the reaction CaCO3 (s) 
 CO2 (g)
20. NaOH / H  reacts with
goes to completion because
a. C6 H 5OCH 3 b. CH 3 OH
a. Of high temperature
O b. CaO is more stable than CaCO3
||
c. CH 3 — C — CH 3 d. C2 H5OH
c. CO2 escapes simultaneously

21. In the reaction C8 H 6O 4 
 X 
 NH3
d. CaO is not dissociated
The compound X is
a. Phthalic anhydride b. Phthalic acid 29. Amalgams are
c. o-xylene d. Benzoic acid a. Highly coloured alloys
b. Always solid
22. In the reaction CH 3 (CH 2 ) 4 CN   A 
Zn , HCl

c. Alloys which contain mercury as one of the contents


A   B 
HONO O
 C ; C is d. Alloys which have great resistance to abrasion
a. Pentanal b. Pentanone
30. Generally, a group of atoms can function as a ligand if
c. 2-Hexanone d. Hexanal
a. They are positively charged ions
23. A polymer containing nitrogen is b. They are free radicals
a. Bakelite b. Dacron c. They are either neutral molecules or negatively charged ions
c. Rubber d. Nylon-66 d. None of these

Space for rough work


Mock Test-5 127
JEE ADVANCE PAPER-I

SECTION 1 Contains 8 Questions 8. Hydrolysis of an alkyl halide (RX) by dilute alkali [OH  ]
The answer to each question is a single digit integer ranging from 0 to takes place simultaneously by SN 2 and SN1 pathways. A
9 (both inclusive).
1 d[R  X]
plot of  vs. [OH  ] is a straight line of
1. The change in the number of unpaired electrons when [RX] dt
2 4
 Fe  H 2 O 6  is changed into  Fe  CN 6  is the slope equal to 2  103 mol 1 L h 1 and intercept equal to
1102 h 1. Calculate the initial rate (mol L1 min 1 ) of
2. A compound of mol. wt. 180 is acetylated to give a consumption of RX when the reaction is carried out taking
compound of mol. wt. 390. The number of amino groups
1mol L1 of RX and 0.1 mol L1 of [OH  ] ions.
in the initial compound is?

3. Specific rotations of -anomer of glucose is 112  and SECTION 2 Contains 10 Multiple Choice Questions
for -anomer is + 19  . Specific rotation of equilibrium With one or more than one correct option
mixture is 52.6 . Calculate % composition of -and -
anomers in the equilibrium mixture. 9. The ionic radii (in Å) of N3 , O2 and F are
respectively:
4. Consider all possible isomeric ketones including
a. 1.36, 1.40 and 1.71
stereoisomers of MW = 100. All these isomers are
b. 1.36, 1.71 and 1.40
independently reacted with NaBH 4 (Note: Stereoisomers
c. 1.71, 1.40 and 1.36
are also reacted separately). The total number of ketones
that give a racemic product(s) is/are d. 1.71, 1.36 and 1.40

5. Amongst the following, the total number of compounds 10. From the following statements regarding H 2O 2 , choose
soluble in aqueous NaOH is the incorrect statement
a. It can act only as an oxidizing agent
H3C CH3 b. It decomposes on exposure to light
N COOH OCH2CH3 OH c. It has to be stored in plastic or wax lined glass bottles in
CH2OH3 dark
d. It has to be kept away from dust
11. AgNO3 (aq.) was added to an aqueous KCl solution
OH CH2CH3
NO2 gradually and the conductivity of the solution was
CH2OH3 COOH
measured. The plot of conductance (^) versus the volume
of AgNO3 is
N
H 3C CH3 + +
+ + +
a.  + ++ b.  +
6. The half-life period of a radioactive substance is 2 min. +
+ +
The time taken for 1 g of the substance to reduce to 0.25 g
volume volume
will be
(a) (b)

7.  f H  of hypothetical MgCl is 125 kJ mol 1 and for +


+
+
MgCl2 is 642 kJ mol 1 . The enthalpy of c.  ++ + + + + + + d.  + + + +
disproportionation of MgCl is –49x. Find the value of x.
volume volume
(c) (d)
128 Chemistry
12. The intermolecular interaction that is dependent on the –1300 kJ/mol, respectively. The standard enthalpy of
inverse cube of distance between the molecules is: combustion per gram of glucose at 25C is
a. ion-ion interaction b. ion-dipole interaction a. + 2900 kJ b. – 2900 kJ
c. London force d. hydrogen bond c. – 16.11 kJ d. +16.11 kJ
13. According to Molecular Orbital Theory 18. Which of the following compounds is not colored yellow?
a. C 22  is expected to be diamagnetic a. Zn 2 [Fe(CN) 6 ]
b. O 22  is expected to have a longer bond length than O2 b. K 3[Co(NO2 )6 ]
 
c. N and N have the same bond order
2 2
c. (NH 4 )3 [As(Mo3O10 ) 4 ]
d. He 2 has the same energy as two isolated He atoms d. BaCrO 4

14. For the first order reaction


SECTION 3 Contains 2 Match The Following Type Questions
2N 2 O5 (g) 
 4NO 2 (g)  O 2 (g)
You will have to match entries in Column I with the entries in
a. the concentration of the reactant decreases exponent- Column II.
tially with time
19. Match each of the diatomic molecules in Column I with
b. the half-life of the reaction decreases with increasing
its property/properties in Column II.
temperature
Column - I Column – II
c. the half-life of the reaction depends on the initial
(A) B2 1. Paramagnetic
concentration of the reactant
d. the reaction proceeds to 99.6% completion in eight (B) N 2 2. Undergoes oxidation
half-life duration (C) O 2 3. Undergoes reduction
15. In the reaction, P  Q 
 R  S the time taken for 75% (D) O2 4. Bond order ≥ 2
reaction of P is twice the time taken for 50% reaction of P. 5. Mixing of ‘s’ and ‘p’
The concentration of Q varies with reaction time as shown orbitals
in the figure. The overall order of the reaction is a. A  5, 3, 1, B  5, 4, C  2, 1, D  4, 1, 2
b. A  1, 3, 5, B  1, 2, 4 C  3, 2, D  1, 2,
[Q]0 c. A  1, 3, B  1, 2, 4 C  1, 2, 3 D  4, 5,
[Q] d. A1, 3, 5, B4, 5, C1, 2, D1, 2, 4
[According to MOT]
20. Match the compounds/ions in Column I with their
Time properties/reactions in Column II.
a. 2 b. 3 c. 0 d. 1 Column I Column II
16. Choose the correct reason(s) for the stability of the (A) C6H5CHO 1. gives precipitate with
lyophobic colloidal particles. 2, 4-dinitrophenyl-
a. Preferential adsorption of ions on their surface from the hydrazine
solution (B) CH3C  CH 2. gives precipitate with
b. Preferential adsorption of solvent on their surface from AgNO3
the solution (C) CN  3. is a nucleophile
c. Attraction between different particles having opposite
(D) I  4. is involved in cyano-
charges on their surface
hydrin formation
d. Potential difference between the fixed layer and the
diffused layer of opposite charges around the colloidal
a. A  1, 2, 3, B  2, C  1, 3, 4, D  2, 3
particles
b. A  1, 3, 4, B  2, 3 C  1, 2, 3, D  2
17. The standard enthalpies of formation of CO 2 (g), H2O(l ) c. A  1, 3, B  1, 4, C  1, 3, D  2, 3
and glucose(s) at 25C are –400 kJ/mol, –300 kJ/mol and d. A  2, 3, B  1, 2, C  1, 3, D  1, 2
Mock Test-5 129
JEE ADVANCE PAPER-II

SECTION 1 Contains 8 Questions 10. Among the electrolytes Na 2SO4 , CaCl2 , Al2 (SO 4 )3 and
The answer to each question is a single digit integer ranging from 0 to NH4Cl, the most effective coagulating agent for Sb2S3
9 (both inclusive).
sol is
2
1. The number of rings formed in  Ca  EDTA   is a. Na 2SO4 b. CaCl2 c. Al2 (SO4 )3 d. NH4Cl
…………… 11. The compound(s) with TWO lone pairs of electrons on the
2. The number of equivalent Cl  O bonds in Cl2O7 is central atom is(are)
a. BrF5 b. ClF3
……………
c. XeF4 d. SF4
3. In fructose, the possible optical isomers are……………
4. Oxidation of glucose is one of the most important 12. Solubility product constant (K sp ) of salts of types
reactions in a living cell. What is the number of ATP MX, MX 2 and M 3X at temperature ‘T’ are
molecules generated in cells from one molecule of
4.0  108 , 3.2  1014 and 2.7  1015 , respectively.
glucose?
3
Solubilities (mole dm ) of the salts at temperature ‘T’
5. 10 gm of a mixture of hexane and ethanol reacts with
are in the order
sodium to give 200 ml. of H 2 at 27  C and 760 mm
a. MX  MX 2  M 3 X b. M 3 X  MX 2  MX
pressure. The percentage of ethanol is……………
c. MX 2  M 3 X  MX d. MX  M 3 X  MX 2
6. The polymerisation of propene to linear polypene is
represented by the reaction 13. According to the Arrhenius equation
 CH 3  a. a high activation energy usually implies a fast reaction
 CH 3 
|  |  b. rate constant increases with increase in temperature.
n  CH  CH 2    — CH — CH 2— 
n This is due to a greater number of collisions whose energy
Where n has large integral value, the average enthalpies of exceeds the activation energy
bond dissociation for (C  C) and (C  C) at 298 K c. higher the magnitude of activation energy, stronger is
are +590 and 331kJ mol 1 , respectively. The enthalpy the temperature dependence of the rate constant
d. the pre-exponential factor is a measure of the rate at
of polymerization is 360 kJ mol1 . Find the value of n. which collisions occur, irrespective of their energy
7. In the case of a first order reaction, the time required for 14. For a linear plot of log (x/m) versus log p in a Freundlich
93.75% of reaction to take place is x times that required adsorption isotherm, which of the following statements is
for half of the reaction. Find the value of x. correct?
8. In 1 L saturated solution of AgCl (K sp of AgCl a. Both k and 1/n appear in the slope term
b. 1/n appears as the intercept
 1.6  1010 ), 0.1mol of CuCl (Ksp CuCl  1.0 106 ) is c. Only 1/n appears as the slope
added. The resultant concentration of Ag  in the solution d. log (1/n) appears as the intercept.

is 1.6 10 x. Calculate the value of x. 15. When O2 is adsorbed on a metallic surface, electron
transfer occurs from the metal to O2 . The TRUE
SECTION 2 Contains 8 Multiple Choice Questions statement(s) regarding this adsorption is(are)
With one or more than one correct option a. O2 is physisorbed
9. Which of the following atoms has the highest first b. heat is released
ionization energy? c. occupancy of  2p
*
of O2 is increased
a. Rb b. Na d. bond length of O2 is increased
c. K d. Sc
130 Chemistry
16. Methylene blue, from its aqueous solution, is adsorbed on b. Carbon dating can be used to find out the age of earth
activated charcoal at 25 °C. For this process, the correct crust and rocks
statement is c. Radioactive absorption due to cosmic radiation is equal to
a. The adsorption requires activation at 25 ° C the rate of radioactive decay, hence the carbon content
b. The adsorption is accompanied by a decrease in remains constant in living organism
enthalpy d. Carbon dating cannot be used to determine concen-
c. The adsorption increases with increase of temperature tration of 14 C in dead beings
d. The adsorption is irreversible
18. What should be the age of fossil for meaningful
SECTION 3 Contains 2 Paragraph Type Questions determination of its age?
a. 6 years
Each paragraph describes an experiment, a situation or a problem.
Two multiple choice questions will be asked based on this paragraph. b. 6000 years
One or more than one option can be correct. c. 60,000 years
d. It can be used to calculate any age
Paragraph for Question Nos. 17 to 18
Carbon–14 is used to determine the age of organic material. Paragraph for Question Nos. 19 to 20
The procedure is based on the formation of 14 C by neutron Rocket propellants consist of rocket engines powered by
capture in the upper atmosphere propellants. These are used both in space vehicles as well as in
14
7 N + 0 n1 
→ 14
6 C + 1n
1
offensive weapons such as missiles. The propellants are
14
C is absorbed by living organisms during photosynthesis. chemical substances which on ignition provide thrust for the
The 14 C content is constant in living organism once the plant or rocket to move forward. These substances are called rocket
animal dies, the uptake of carbon dioxide by it ceases and the propellants. A propellant is a combination of an oxidiser and a
level of 14 C in the dead being, falls due to the decay which fuel which when ignited undergoes combustion to release large
C 14 undergoes quantities of hot gases. The passage of hot gases through the
− nozzle of the rocket motor provides the necessary thrust for the
C  7 N + β
→ 14
14
6
rocket to move forward according to Newton's third law of
The half life period of 14 C is 5770 years. The decay constant
motion.
(λ ) can be calculated by using the following formula
19. A biliquid propellant contains
0.693 a. Liquid hydrazine
λ= . The comparison of the β - activity of the dead
t1/ 2 b. A mixture of liquid fuel and a liquid oxidiser
matter with that of the carbon still in circulation enables c. A solid rocket fuel
measurement of the period of the isolation of the material from d. A liquid fuel which can also act as an oxidiser
the living cycle. The method however, ceases to be accurate
20. A hybrid rocket propellant uses
over periods longer than 30,000 years. The proportion of 14 C
a. A liquid oxidiser and a solid fuel
12 12
to C in living matter is 1 : 10 . b. A composite solid propellant
17. Which of the following option is correct? c. A biliquid propellant
a. In living organisms, circulation of 14 C from atmosphere d. A solid, liquid and gas as a propellant
is high so the carbon content is constant in organism
Space for rough work
Mock Test-5 131
ANSWER & SOLUTIONS 11. (b) In photochemical reaction the rate of formation of
JEE-Main product is directly proportional to the intensity of
absorbed light.
1. 2. 3. 4. 5. 6. 7. 8. 9. 10.
d a c c d a c b d d 12. (d) Gold number shows the protective power of a
11. 12. 13. 14. 15. 16. 17. 18. 19. 20. lyophilic solution. Lesser the gold number, greater will be
b d d b c a d a a c the protecting power of that colloid. Gelatin is one of the
21. 22. 23. 24. 25. 26. 27. 28. 29. 30. best protective colloid. Among the given colloids, potato
a d d d a d a d c c starch has maximum gold number.

13. (d) C(graphite) 


→ C(diamond ) , ∆H = 1.9 kJ
1. (d) Cr2 O 7−− → Cr 3+ ; Fe ++ → Fe+++
n=1 C(graphite) + O 2 
→ CO 2 , ∆H = −∆H1
n=6
eq. of K 2 Cr2 O7 = eq. of FeSO 4 1× 6 = x × 1 C(diamond) + O 2 
→ CO 2 , ∆H = −∆H 2
2. (a) When an ion (generally cation due to its small size) is ( − ∆H1 ) − ( − ∆H 2 ) = 1.9 kJ or ∆H 2 = ∆H1 + 1.9
missing from its normal position and occpy an interstitial
For combustion of 6g, ∆H 2 > ∆H1 by 1.9 / 2 = 0.95 kJ.
site between the lattice points, the lattice defect obtained
0.693 0.693
is known as Frenkel defect. 14. (b) t1/ 2 = = = 0.3 ×104 yrs = 3.0 × 10 3 yrs.
k 2.31× 10−4
3. (c) P = PAo x A + PBo x B 15. (c) In the triphenyl methyl carbonium ion the π electrons
 3  o 2  o of all the three benzene rings are delocalised with the
600 = PAo   + PB   ;3PA + 2PB = 3000
o

 3+ 2  2+3 vacant p-orbital of central carbon atom. So, it is resonance


 4.5  o 2  stabilised. It is the most stable of all the carbonium ions
630 = PAo   + PB  ;4.5PA + 2PB = 4410
o o
given
 4.5 + 2 + 0.5   4.5 + 2 + 0.5 
CH3
1.5PAo = 1410; PAo = 940 and PBo = 90
|
The ion CH 3 — C+ is stabilised by hyperconjugation, a
4. (c) r.m.s. speed ∝ T |
2 CH3
4.75 298  4.75  298
= or   = second order resonance.
9.50 T  9.50  T
or T = 1192 K or T = 1192 − 273 = 919°C 16. (a)
CH 3 CH 3 CH 3 H
5. (d) When electron jumps to lower orbit photons are C == C C == C
emitted while photons are absorbed when electron jumps H H H CH 3
Cis 2 butene Trans 2 butene
to higher orbit, 1s orbital is the lower most, electron in
this orbital can absorb photons but cannot emit. Cis-trans isomerism shown by compound which have double
or triple bond by which they restrict their rotation, since 2
6. (a) 6MnO 4− + I − + 6OH − 
→ 6MnO 24− + IO 3− + 3H 2 O
butyne have no hydrogen on triple bonded carbon.
7. (c) A is displace from D because D have a E° = −0.402 V CH 3 — C ≡≡ C — CH 3 [It does not show cis-trans]
2 butyne
8. (b) CCl4 is sp3 hybridised so bond angle will be
CH — COONa CH COO−
approximately 109 ° . 17. (d) || 
→ || + 2Na +

9. (d) High temperature and excess concentration of the CH — COONa CH COO
reactant concentration. Sodium salt of maleic acid or fumaric acid
1× 10−14
CH COO− CH
10. (d) pH = 9 ∴ [H + ] = 10 −9 [OH − ] = = 10−5 || 
→ ||| + 2CO 2 + 2e −
10−9 CH COO− CH
Acetylene
Mg(OH)
Mg(OH) 2 Mg 2 + Mg
+ 2OH −2OH
18. (a) Because it float over chloroform and prevent its
K sp = [Mg 2 + ][OH − ]2 ⇒ 1 ×10 −11 = [Mg 2 + ][10 −5 ]2 oxidation.
1× 10−11 19. (a) Cyclic ethers are called epoxies. CH 2 — CH 2
⇒ [Mg 2 + ] = = 10−1 = 0.1
(10−5 )2 O
132 Chemistry
20. (c) 2CH 3  CO  CH 3  
dil NaOH
 3. The -anomer of D-glucose has a specific rotation of +112
OH O degrees in water. The β-anomer of D-glucose has a specific
| || rotation of + 19 degrees (18.7 actually, but rounding up to 19).
CH 3 — C — CH 2 — C — CH 3
| (Diacetone alchol) If the fraction of glucose present as the -anomer is xand
CH 3
the fraction present as the β-anomer is y, and the rotation of
21. (a) the mixture is +52.6 degree, we have
O X (+112.2 degree) + y(18.7) = 52.6 degree ……..(i)
||
C There is a very little of the open chain form present, so the
COOH

 fraction present as the α-anomer (x) plus the fraction present
COOH
C as the β-anomer (y) should account for all the glucose, i.e.,
|| x + y = 1 or y = 1 – x.
O
CONH 2 Putting value of y in equation (i), we get
 NH 3 

COOH
x (+112.2 degree) + (1 – x)(18.7) = 52.6 degree ……..(ii)
Solving equation (ii) for (x), we have, x = 0.36 or 36%. Thus
22. (d) CH 3 (CH 2 ) 4 CN  
Zn , HCl

(y) must be (1 – 0.36) = 0.64 or 64%.


CH 3 (CH 2 ) 4 CH 2 NH 2  
HONO
So, percentage composition of - and -anomers in the
CH 3 (CH 2 ) 4 CH 2OH 
O
 CH 3 (CH 2 ) 4 CHO equilibrium mixture is 36% and 64% respectively.
Hexanal
4. (5)
23. (d) Nylon-66- It is a polymer containing nitrogen
5. (4)
 H H O 
 | | ||  H3C CH3
 — N — (CH 2 ) 6 — N — C — (CH 2 ) 4 — C —  N OH OH COOH
 || 
 O n
Nylon-66

24. (d) Thymine is present in DNA while in RNA there is


Uracil.
N
25. (a) Insulin is an antidiabatic drug.
26. (d) Mg  Al  Na. This is due to the presence of fully H3C CH3
filled s-orbital in Mg. 6. (2) 1g 
t1/ 2
0  5g 
t1/ 2
0  25g t 25%  4min
27. (a) ZnO  CO    CO 2  Zn
28. (d) Lime stone – CaCO3, Clay – silica and alumina 7. (8) 2MgCl 
 Mg  MgCl 2 H  ?
Gypsum – CaSO44 .2H 2O  MgCl ; H1  125 kJ mol 1
Mg(s)  1/ 2Cl2 (g) 
29. (c) Amalgams are alloys which contain mercury as one of  MgCl 2 ; H 2  642 kJ mol 1
Mg(s)  Cl 2 (g) 
the contents.
H  H 2  2H1  642  (2  125)  392 kJ mol 1
30. (c) Ligand must have capacity to donate lone pair of
 49 x  392  x8
electrons to form coordinate bond.
d[RX]
JEE Advance Paper -I 8. (5)  k 2 [RX][OH  ] (by SN 2 pathway)
dt
1. 2. 3. 4. 5. 6. 7. 8. 9. 10.
k 2  rate constant of SN2 reaction
4 5 2 5 4 2 8 5 c a
d[RX]
11. 12. 13. 14. 15. 16. 17. 18. 19. 20.  k 2 [RX][OH  ]  k1[RX]
d b a, c c b,c,d b c b b a dt
1 d[RX]
1. (4)   k 2 [OH  ]  k1
[RX] dt
2. (5) Difference in mass of compound 1 d[RX]
= 390 – 180 = 210 This is the equation of a straight line for 
[RX] dt
wt. of CH 3 CO – group is = 43
vs [OH  ] plot with slope equal to k 2 and intercept equal
210
Therefore no. of  NH 2 group =  4.88  5 . to k1 .
43
Mock Test-5 133
1
From question: k 2  2  10 mol L hr , k1  1 10 hr
3 1 2 1
H combustion  (6  H f CO 2  6  H f H 2 O)  H f C6 H12 O 6
[RX]  1.0 M and [OH ]  0.1M 
 (6  400  6  300)  (1300)
d[RX]  2900 kJ / mol
Hence,  2  103  1 0.1  1 102  1
dt  2900 /180 kJ / g
 300 mol L1 hr 1  5 mol L1 min 1
 16.11 kJ / g
3 2 
9. (c) Ionic Radii order: N  O  F 18. (a) Zn 2 [Fe(CN)6 ] is bluish white ppt.
10. (a) It can act as an oxidising as well as reducing agent. 19. (d) A  1, 3, 5, B  4, 5, C  1, 2, D  1, 2, 4
[According to MOT]
+
+
 + 20. (a) A  1, 2, 3, B  2, C  1, 3, 4, D  2, 3
+++ +
11. (d) (Note: Assuming AgNO3 is ammonical)
Volume NO2
12. (b) Ion-dipole interaction
(a) PhCHO  O 2 N NH  NH 2 

13. (a,c) (a) C 22  Total no. of electrons = 14 so it is
O2N
diamagnetic
(b) O 22  Bond order = 3; O2 Bond order = 2 PhHC  N  NH NO2
(ppt.)

 Bond length in O 22  is less than bond length in O2 . PhCHO  Ag 2 O 


NH3
 PhCOO   Ag 
  ( whiteppt )
(c) Bond order of N  2.5 ; Bond order of He  1/ 2
2 2
CN
 Some energy is released during the formation of He 2 from |
two isolated He atoms. PhCHO 
KCN
 Ph — C — O 
|
H
14. (a, b, d) For first order reaction [A]  [A]0 e  kt
Hence concentration of [NO2 ] decreases exponentially, (b) CH3C  CH 
ammonical AgNO3
 CH3  C  C Ag  
(White ppt)

0.693 CN
Also, t1/ 2  , which is independent of concentration |
K (c) PhCHO 
KCN
 Ph — C — O  ;
and t1/ 2 decreases with the increase of temperature. |
H
2.303  100  AgNO3  CN  
 AgCN 
t 99.6  log  
K  0.4 
(d) AgNO3  I  
 AgI 
2.303 0.693
t 99.6  (2.4)  8   8 t1/ 2
K K
JEE Advance Paper –II
15. (d) Overall order of reaction can be decided by the data
given t 75%  2t 50% 1. 2. 3. 4. 5. 6. 7. 8. 9. 10.
5 6 8 38 10 5 4 7 d c
 It is a first order reaction with respect to P.
11. 12. 13. 14. 15. 16. 17. 18. 19. 20.
From graph [Q] is linearly decreasing with time, i.e., order
b, c d b,c,d b,c,d b b c b 2 1
of reaction with respect to Q is zero and the rate
expression is r  k[P]1[Q]0 . 1. (5)
16. (a, d) (a) Preferential adsorption of ions on surface from 2. (6)
the solution 3. (8) Fructose has three chiral centres and hence 23  8
(c) Attraction between particles having same charges on optical isomers are possible.
their surface accounts for the Brownian motion. 4. (38) C 6 H 12 O 6  6O 2  6CO 2  6H 2 O  38ATP
(d) Definition of Zeta Potential.
5. (10) 22400 ml of H 2 is produced by 46 gms of ethanol.
17. (c) Combustion of glucose
46  200 23
C6 H12 O6  6O2 
 6CO2  6H 2 O 200 ml of H 2 is produced by 
22400 56
Mock Test-1
Test Booklet code
135

A Mock Test “JEE-Main”


Do not open this Test Booklet until you are asked to do so.
Read carefully the Instructions on the Back Cover of this Test Booklet.

Important Instructions:
1. Immediately fill in the particulars on this page of the Test Booklet with Blue/Black Ball Point Pen. Use of pencil is strictly
prohibited.
2. The Answer Sheet is kept inside this Test Booklet. When you are directed to open the Test Booklet, take out the Answer
Sheet and fill in the particulars carefully.
3. The test is of 3 hours duration.
4. The Test Booklet consists of 90 questions. The maximum marks are 360.
5. There are three parts in the question paper A, B, C consisting of, Physics, Chemistry and Mathematics having 30
questions in each part of equal weightage. Each question is allotted 4 (four) marks for each correct response.
6. Candidates will be awarded marks as stated above in instruction No. 5 for correct response of each question. 1/4 (one
fourth) marks will be deducted for indicating incorrect response of each question. No deduction from the total score will
be made if no response is indicated for an item in the answer sheet.
7. There is only one correct response for each question. Filling up more than one response in each question will be treated as
wrong response and marks for wrong response will be deducted accordingly as per instruction 6 above.
8. Use Blue/Black Ball Point Pen only for writing particulars/marking responses on Side-1 and Side-2 of the Answer Sheet.
Use of pencil is strictly prohibited.
9. No candidates is allowed to carry any textual material, printed or written, bits of papers, pager, mobile phone, any
electronic device, etc., except the Admit Card inside the examination hall/room.
10. Rough work is to be done on the space provided for this purpose in the Test Booklet only. This space is given at the
bottom of each page and at the end of the booklet.
11. On completion of the test, the candidate must hand over the Answer Sheet to the Invigilator on duty in the Room/Hall.
However, the candidates are allowed to take away this Test Booklet with them.
12. The CODE for this Booklet is A. Make sure that the CODE printed on Side-2 of the Answer Sheet and also tally the serial
number of the Test Booklet and Answer Sheet are the same as that on this booklet. In case of discrepancy, the candidate
should immediately report the matter to the invigilator for replacement of both the Test Booklet and the Answer Sheet.
13. Do not fold or make any stray marks on the Answer Sheet.

Name of the Candidate (in Capital letters):


Roll Number: in figures
in words
Examination Centre Number:
Name of Examination Centre (in Capital letters):
Candidate’s Signature: Invigilator’s Signature:
136 Mathematics
Read the Following Instructions Carefully:
1. The candidates should fill in the required particulars on the Test Booklet and Answer Sheet (Side-1) with
Blue/Black Ball Point Pen.
2. For writing/marking particulars on Side-2 of the Answer Sheet, use Blue/Black Ball Point Pen only.
3. The candidates should not write their Roll Numbers anywhere else (except in the specified space) on the Test
Booklet/Answer Sheet.
4. Out of the four options given for each question, only one option is the correct answer.
5. For each incorrect response, one-fourth (¼) of the total marks allotted to the question would be deducted from
the total score. No deduction from the total score, however, will be made if no response is indicated for an item
in the Answer Sheet.
6. Handle the Test Booklet and Answer Sheet with care, as under no circumstances (except for discrepancy in
Test Booklet Code and Answer Sheet Code), another set will be provided.
7. The candidates are not allowed to do any rough work or writing work on the Answer Sheet. All
calculations/writing work are to be done in the space provided for this purpose in the Test Booklet itself,
marked ‘Space for Rough Work’. This space is given at the bottom of each page and at the end of the booklet.
8. On completion of the test, the candidates must hand over the Answer Sheet to the Invigilator on duty in the
Room/Hall. However, the candidates are allowed to take away this Test Booklet with them.
9. Each candidate must show on demand his/her Admit Card to the Invigilator.
10. No candidate, without special permission of the Superintendent or Invigilator, should leave his/her seat.
11. The candidates should not leave the Examination Hall without handing over their Answer Sheet to the
Invigilator on duty and sign the Attendance Sheet again. Cases where a candidate has not signed the
Attendance Sheet a second time will be deemed not to have handed over the Answer Sheet and dealt with as an
unfair means case. The candidates are also required to put their left hand THUMB impression in the
space provided in the Attendance Sheet.
12. Use of Electronic/Manual Calculator and any Electronic Item like mobile phone, pager etc. is prohibited.
13. The candidates are governed by all Rules and Regulations of the JAB/Board with regard to their conduct in the
Examination Hall. All cases of unfair means will be dealt with as per Rules and Regulations of the JAB/Board.
14. No part of the Test Booklet and Answer Sheet shall be detached under any circumstances.
15. Candidates are not allowed to carry any textual material, printed or written, bits of papers, pager,
mobile phone, electronic device or any other material except the Admit Card inside the examination
hall/room.
Mock Test-1 137

JEE-MAIN: MATHEMATICS MOCK TEST-1

log 2 ( x + 3)  π   3π 
1. The domain of definition of f ( x) = is: 10. sin   sin  =
x 2 + 3x + 2  10   10 
a. R − {−1, −2} b. (−2, ∞) a. 1/2 b. – ½ c. 1/4 d. 1
c. R − {−1, −2, −3} d. (−3, ∞) − {−1, −2} 11. Find real part of cosh −1 (1)
2 2 a. – 1 b. 1
2. How many roots the equation x − = 1− have
x −1 x −1 c. 0 d. None of these
a. One b. Two 12. From a 60 meter high tower angles of depression of the
c. Infinite d. None top and bottom of a house are α and β respectively. If
3. If ω is the cube root of unity, then (3 + 5ω + 3ω 2 ) 2 + 60sin ( β − α )
the height of the house is , then x =
x
+(3 + 3ω + 5ω ) = 2 2

a. sin α sin β b. cos α cos β


a. 4 b. 0
c. – 4 d. None of these c. sin α cos β d. cos α sin β

4. From the following find the correct relation log(1 + ax) − log(1 − bx)
13. The function f ( x) = is not defined
a. ( AB)′ = A′B′ b. ( AB )′ = B′A′ x
adj A at x = 0. The value which should be assigned to f at
c. A−1 = d. ( AB) −1 = A−1B −1 x = 0 so that it is continuous at x = 0, is
A
a. a − b b. a + b
If the roots of the cubic equation ax + bx + cx + d = 0
3 2
5.
c. log a + log b d. log a − log b
are in G.P., then
a. c a = b d b. ca = bd c. a b = c d d. ab = cd , at θ = 3π , is
3 3 3 3 3 3 3 3
dy
14. If x = a cos 4 θ , y = a sin 4 θ , then
dx 4
6. C1 + 2C2 + 3C3 + 4C4 + .... + nCn =
a. –1 b. 1
a. 2 n b. n. 2n c. − a 2 d. a 2
c. n. 2n−1 d. n. 2n+1
15. The minimum value of [(5 + x )(2 + x)]/[1 + x ] for non-
(log e n) 2 (log e n) 4 negative real x is
7. 1+ + +K =
2! 4! a. 12 b. 1 c. 9 d. 8
−1
a. n b. 1/ n e tan x
1 1
16. ∫ 1 + x2
dx =
c. ( n + n −1 ) d. (e n + e − n ) −1
2 2 a. log(1 + x 2 ) + c b. log etan x
+c
8. How many words can be made from the letters of the −1 −1

word INSURANCE, if all vowels come together. c. e tan x


+c d. tan−1 etan x
+c
a. 18270 b. 17280 17. The measurement of the area bounded by the co-ordinate
c. 12780 d. None of these
axes and the curve y = loge x is
9. There are four machines and it is known that exactly two a. 1 b. 2
of them are faulty. They are tested, one by one, in a c. 3 d. ∞
random order till both the faulty machines are identified.
dy
Then, the probability that only two tests are needed, is 18. The solution of the equation = e x − y + x 2 e − y is
dx
1 1
a. b. x3
3 6 a. e y = e x + +c b. e y = e x + 2 x + c
3
1 1
c. d. c. e y = e x + x 3 + c d. y = e x + c
2 4
138 Mathematics
19. The distance between 4 x + 3 y = 11 and 8 x + 6 y = 15, is 26. The sum to infinity of the given series
1 1 1 1
a.
7
b. 4 − + − + K is
2 n 2 n 2 3n 3 4 n 4
7  n +1  n 
c. d. None of these a. loge   b. loge  
10  n   n +1
20. The area of a circle whose centre is (h, k) and radius a is  n −1   n 
c. loge   d. loge  
a. π (h2 + k 2 − a2 ) b. π a 2 hk  n   n −1 

c. π a 2 d. None of these 27. The values of A and B such that the function
 π
21. The locus of the mid-point of the line segment joining the  −2sin x, x≤−
2
focus to a moving point on the parabola y 2 = 4 ax is 
 π π
another parabola with directrix f ( x) =  A sin x + B, − < x < , is continuous
 2 2
a. x = − a b. x = −
a  π
 cos x, x≥
2  2
a everywhere are
c. x = 0 d. x =
2 a. A = 0, B = 1 b. A = 1, B = 1
r r
22. If a = (2, 5) and b = (1, 4), then the vector parallel to c. A = −1, B = 1 d. A = −1, B = 0
r r
( a + b ) is
a. (3, 5) b. (1, 1)
28. ∫ x 3 3 + 5 x 4 dx =

c. (1, 3) d. (8, 5) 1
a. (3 + 5 x 4 )3/ 2 + c b. (3 + 5 x 4 )3/ 2 + c
5
23. The acute angle between the line joining the points (2, 1,
1
x −1 y z + 3 c. (3 + 5 x 4 )3/ 2 + c d. None of these
–3), (–3,1,7) and a line parallel to = = 30
3 4 5
through the point (–1, 0, 4) is 29. The equations of tangents to the circle

 7   1  x + y − 22 x − 4 y + 25 = 0 which are perpendicular to the


2 2

a. cos −1   b. cos −1  
 5 10   10  line 5 x + 12 y + 8 = 0 are

 3   1  a. 12 x − 5 y + 8 = 0, 12 x − 5 y = 252
c. cos −1   d. cos −1  
 5 10   5 10  b. 12 x − 5 y = 0, 12 x − 5 y = 252
c. 12 x − 5 y − 8 = 0,12 x − 5 y + 252 = 0
24. Which of the following is logically equivalent to
d. None of these
~ (~ p ⇒ q)
a. p ∧ q b. p ∧ ~ q 30. ~ p ∧ q is logically equivalent to
c. ~ p ∧ q d. ~ p ∧ ~ q a. p → q
b. q → p
25. The number of solutions of the system of equations
2x + y − z = 7, x − 3 y + 2z = 1, x + 4 y − 3z = 5 is c. ~ ( p → q )

a. 3 b. 2 c. 1 d. 0 d. ~ (q → p )

Space for rough work


Mock Test-1 139
JEE ADVANCE PAPER-I

Time 3 Hours. Max. Marks 264 (88 for Mathematics)


Read The Instructions Carefully

Question Paper Format and Marking Scheme:


1. The question paper has three parts: Physics, Chemistry and Mathematics. Each part has three sections.

2. Section 1 contains 8 questions. The answer to each question is a single digit integer ranging from 0 to 9 (both inclusive).
Marking Scheme: +4 for correct answer and 0 in all other cases.

3. Section 2 contains 10 multiple choice questions with one or more than one correct option.
Marking Scheme: +4 for correct answer, 0 if not attempted and –2 in all other cases.

4. Section 3 contains 2 “match the following” type questions and you will have to match entries in Column I with the entries
in Column II.
Marking Scheme: For each entry in Column I, +2 for correct answer, 0 if not attempted and –1 in all other cases.

NOTE: It’s the mock test as per previous year’s papers but sometimes IIT changes the test paper pattern and
marking scheme too.

SECTION 1 (Maximum Marks: 32) 5. Let n be the number of ways in which 5 boys and 5 girls
This section contains EIGHT questions. can stand in a queue in such a way that all the girls stand
The answer to each question is a SINGLE DIGIT INTEGER consecutively in the queue. Let m be the number of ways
ranging from 0 to 9, both inclusive. in which 5 boys and 5 girls can stand in a queue in such a
For each question, darken the bubble corresponding to the correct way that exactly four girls stand consecutively in the
integer in the ORS. m
queue. Then the value of is
Marking scheme: n
+4 If the bubble corresponding to the answer is darkened.
6. TP and TQ are any two tangents to a parabola and the
0 In all other cases.
tangent at a third point R cuts then in P' and Q', then the
1. If area enclosed between the curves y = ln( x + e) and TP ′ TQ ′
value of = must be
TP TQ
1
x = ln   and the axis of x is λ sq unit, then the value of
 y 7. Let f :R→R be a function defined by

{[0,x], xx ≤> 22, where [x] is the greatest integer less


22
6 must be
f ( x) =
2. The number of distinct solutions of the equation
5 2
xf ( x 2 )
cos 2 2 x + cos 4 x + sin 4 x + cos 6 x + sin 6 x = 2
4
in the than or equal to x. If I = ∫ 2 + f ( x + 1) dx, then the value
−1
interval [0, 2π ] is
of (4 I − 1) is
3. Let the curve C be the mirror image of the parabola
y 2 = 4 x with respect to the line x + y + 4 = 0. If A 8. A cylindrical container is to be made from certain solid
and B are the points of intersection of C with the line material with the following constraints: It has a fixed
y = −5, then the distance between A and B is inner volume of V mm3 , has a 2 mm thick solid wall and
4. The minimum number of times a fair coin needs to be is open at the top. The bottom of the container is a solid
tossed, so that the probability of getting at least two heads circular disc of thickness 2 mm and is of radius equal to
is at least 0.96 is the outer radius of the container.
140 Mathematics
If the volume of the material used to make the container is 3
13. In R , let L be a straight line passing through the origin.
minimum when the inner radius of the container is 10 Suppose that all the points on L are at a constant distance
mm, then the value of V / 250π is
from the two planes P1 : x + 2 y − z + 1 = 0 and

SECTION 2 (Maximum Marks: 40) P2 : 2 x − y + z − 1 = 0. Let M be the locus of the feet of the
This section contains TEN questions. perpendiculars drawn from the points on L to the plane
Each question has FOUR options (a), (b), (c) and (d). ONE OR P1 . Which of the following points lie(s) on M?
MORE THAN ONE of these four option(s) is (are) correct.
 5 2
For each question, darken the bubble(s) corresponding to all the a.  0, − , − 
correct option(s) in the ORS.  6 3
Marking scheme:  1 1 1
+4 If only the bubble(s) corresponding to all the correct b.  − , − , 
 6 3 6
option(s) is(are) darkened.
 5 1
0 If none of the bubbles is darkened c.  − , 0, 
–2 In all other cases  6 6
r uuur r uuur  1 2
9. Let ∆PQR be a triangle. Let a = QR, b = RP, and d.  − , 0, 
 3 3
r uuur r r rr
c = PQ. If | a | = 12, | b | = 4 3 and b.c = 24, then which
14. Let P and Q be distinct points on the parabola y 2 = 2 x
of the following is (are) true?
r r such that a circle with PQ as diameter passes through the
| c |2 r | c |2 r
a. − | a | = 12 b. − | a | = 30 vertex O of the parabola. If P lies in the first quadrant and
2 2
r r r r rr the area of the triangle ∆OPQ is 3 2, then which of the
c. | a × b + c × a | = 48 3 d. a.b = −72
following is (are) the coordinates of P?
10. Let X and Y be two arbitrary, 3×3, non-zero, skew- a. (4, 2 2) b. (9,3 2)
symmetric matrices and Z be an arbitrary 3×3, non-zero,
1 1 
symmetric matrix. Then which of the following matrices c.  ,  d. (1, 2)
4 2
is (are) skew symmetric?
15. Let y( x) be a solution of the differential equation
a. Y 3 Z 4 − Z 4Y 3 b. X 44 + Y 44
(1 + e x ) y ′ + ye x = 1. If y(0) = 2, then which of the
c. X Z − Z X
4 3 3 4
d. X 23 + Y 23
following statements is (are) true ?
11. Which of the following values of α satisfy the equation
a. y(−4) = 0
(1 + α) 2
(1 + 2α) 2
(1 + 3α) 2
b. y(−2) = 0
(2 + α) 2 (2 + 2α) 2 (2 + 3α) 2 = −648α ??
c. y( x) has a critical point in the interval ( −1, 0)
(3 + α)2 (3 + 2α)2 (3 + 3α)2
d. y( x) has no critical point in the interval ( −1, 0)
a. −4 b. 9
16. Consider the family of all circles whose centers lie on the
c. −9 d. 4
straight line y = x. If this family of circles is represented
12. In R 3 , consider the planes P1 : y = 0 and P2 : x + z = 1. by the differential equation Py ′′ + Qy ′ + 1 = 0, where P, Q
Let P3 be a plane, different from P1 and P2 , which passes
dy d2 y
are functions of x, y and y ′(here y ′ = , y ′′ = 2 ), then
through the intersection of P1 and P2 . If the distance of dx dx
the point (0, 1, 0) from P3 is 1 and the distance of a point which of the following statements is (are) true?
a. P = y + x
(α, β, γ) from P3 is 2, then which of the following
b. P = y − x
relations is (are) true?
a. 2α + β + 2γ + 2 = 0 b. 2α – β + 2γ + 4 = 0 c. P + Q = 1 − x + y + y ′ + ( y ′) 2

c. 2α + β – 2γ – 10 = 0 d. 2α – β + 2γ – 8 = 0 d. P − Q = x + y − y ′ − ( y ′) 2
Mock Test-1 141
17. Let g:R→R be a differential function with For each entry in Column I, darken the bubbles of all the
matching entries. For example, if entry (A) in Column I, matches
g (0) = 0, g ′(0) = 0 and g ′(1) ≠ 0.
with entries (2), (3) and (5), then darken these three bubbles in the
 x g ( x ), ORS. Similarly, for entries (B), (C) and (D).
x≠0
Let f ( x) =  | x | and h ( x ) = e| x| for all
Marking scheme:
 0, x=0
For each entry in Column I
x ∈ R. Let ( f o h) (x) denote f ( h ( x )) and (h o f )( x) +2 If only the bubble(s) corresponding to all the correct match(es)
denote h( f ( x)). Then which of the following is (are) is(are) darkened
0 If none of the bubbles is darkened
true?
–1 In all other cases
a. f is differentiable at x = 0
b. h is differentiable at x = 0 19. Match the Column:
c. f o h is differentiable at x = 0 Column I Column II
(A) In R 2 , if the magnitude of 1. 1
d. h o f is differentiable at x = 0
the projection vector of the
π π  vector αiˆ + β ˆj on 3iˆ + ˆj
18. Let f ( x) = sin  sin  sin x   for all x∈R and
6 2 
is 3 and if α | = 2 + 3β,
π
g ( x) = sin x for all x ∈ R. Let (f o g)(x) denote f (g(x)) then possible value(s) of | α |
2
and (g o f )(x) denote g(f (x)). Then which of the following is (are)
is (are) true? (B) Let a and b be real numbers 2. 2
such that the function
 1 1
a. Range of f is  − , 
 2 2
 1 1
f (x) = {
−3ax2 −2, x <1
bx +a2, x ≥1
is

b. Range of f o g is  − ,  differentiable for all x ∈ R.


 2 2
Then possible value(s) of a
f ( x) π
c. lim = is (are)
x→0 g ( x) 6
(C) Let ω ≠ 1 be a complex 3. 3
d. There is an x ∈ R such that ( g o f )( x) = 1
cube root of unity. If
2 4n+3
(3 − 3ω + 2ω )
SECTION 3 (Maximum Marks: 16)
+ (2 + 3ω − 3ω 2 ) 4 n + 3
This section contains TWO questions.
Each question contains two columns, Column I and Column II + ( − 3 + 2ω + 3ω 2 ) 4 n + 3 = 0,
Column I has four entries (A), (B), (C) and (D) then possible value(s) of n is
Column II has five entries (1), (2), (3), (4) and (5) (are)
Match the entries in Column I with the entries in Column II
(D) Let the harmonic mean of 4. 4
One or more entries in Column I may match with one or more
two positive real numbers a
entries in Column II
and b be 4. If q is a positive
The ORS contains a 4 × 5 matrix whose layout will be similar to
real number such that a, 5,
the one shown below:
q, b is an arithmetic
(A) (1) (2) (3) (4) (5)
(B) (1) (2) (3) (4) (5) progression, then the
(C) (1) (2) (3) (4) (5) value(s) of | q − a | is (are)
(D) (1) (2) (3) (4) (5) 5. 5
142 Mathematics
20. Match the thermodynamic processes given under Column I 2
(C) In R , let 3iˆ + ˆj , iˆ + 3 ˆj 3. 3
with the expression given under Column II:
and β iˆ + (1 − β ) ˆj be the
Column I Column II
position vectors of X, Y and
(A) In a triangle ∆XYZ , let a, b 1. 1 Z with respect of the origin
and c be the lengths of the O, respectively. If the
sides opposite to the angles distance of Z from the
X, Y and Z, respectively. If bisector of the acute angle
uuur uuur 3
2( a 2 − b 2 ) = c 2 and of OX with OY is ,
2
sin( X − Y )
λ= , then then possible value(s) of
sin Z | β | is (are)
possible values of n for (D) Suppose that F (α) denotes 4. 5
which cos(nπλ ) = 0 is (are) the area of the region
(B) In a triangle ∆XYZ , let a, b 2. 2 bounded by x = 0, x = 2,
y2 = 4x and
and c be the lengths of the
y = | αx − 1| + | αx − 2 | + αx,
sides opposite to the angles
X, Y and Z, respectively. If where α ∈ {0,1}. Then the
1 + cos 2 X − 8
value(s) of F (α) +
2,
2cos 2Y = 2sin X sin Y , 3
when α = 0 and α = 1, is
a
then possible value(s) of (are)
b 5. 6
is (are)

Space for rough work


Mock Test-1 143

JEE ADVANCE PAPER-II

Time 3 Hours. Max. Marks 240 (80 for Mathematics)


Read The Instructions Carefully

Question Paper Format and Marking Scheme:


1. The question paper has three parts: Physics, Chemistry and Mathematics. Each part has three sections.

2. Section 1 contains 8 questions. The answer to each question is a single digit integer ranging from 0 to 9 (both inclusive).
Marking Scheme: +4 for correct answer and 0 in all other cases.

3. Section 2 contains 8 multiple choice questions with one or more than one correct option.
Marking Scheme: +4 for correct answer, 0 if not attempted and –2 in all other cases.

4. Section 3 contains 2 “paragraph” type questions. Each paragraph describes an experiment, a situation or a problem. Two
multiple choice questions will be asked based on this paragraph. One or more than one option can be correct.
Marking Scheme: +4 for correct answer, 0 if not attempted and –2 in all other cases.

NOTE: It’s the mock test as per previous year’s papers but sometimes IIT changes the test paper pattern and
marking scheme too.

SECTION 1 (Maximum Marks: 32) 4. The coefficient of x9 in the expansion of (1 + x)(1 + x 2 )


This section contains EIGHT questions.
(1 + x 3 )...(1 + x100 ) is
The answer to each question is a SINGLE DIGIT INTEGER
ranging from 0 to 9, both inclusive.
5. If the normals at the four points ( x1 , y1 ),( x2 , y2 ), ( x3 , y3 )
For each question, darken the bubble corresponding to the correct
integer in the ORS. x2 y2
and ( x4 , y4 ) on the ellipse + = 1 are concurrent, then
Marking scheme: a2 b2
+4 If the bubble corresponding to the answer is darkened. 1 1 1 1
0 In all other cases. the value of ( x1 + x2 + x3 + x4 ) ×  + + +  must be
 x1 x2 x3 x4 
r r r 6. Let m and n be two positive integers greater than 1. If
1. Suppose that p, q and r are three non-coplanar vectors
r r r  ecos(α ) − e 
n

in R3. Let the components of a vector s along p, q and e m


lim   = −   then the value of is
a →0  
 α
m
r 2 n
r be 4, 3 and 5, respectively. If the components of this 
r r r r r r r r r r
vector s along (− p + q + r ), ( p − q + r ) and (− p − q + r )
9 x + 3tan − x  12 + 9 x 
2
1
7. If α = ∫ (e ) −1
 dx where tan x takes
are x, y and z, respectively, then the value of 2x + y + z is 0
 1+ x 
2

For any integer k, let α k = cos 


 kπ   kπ   3π 
2.  + i sin   , where only principal values, then the value of  loge |1 + α |
4 
is
 7   7  
12
Σ | α k +1 − α k | 8. Let f : R → R be a continuous odd function, which
i = −1. The value of the expression 3
k =1
is
Σ | α 4 k −1 − α 4 k − 2 | 1
k =1 vanishes exactly at one point and f (1) = . Suppose that
2
3. Suppose that all the terms of an arithmetic progression x
∫ t | f ( f (t))| dt
x
F ( x) = ∫ f (t ) dt for all x∈[1−1, 2] and G(x) =
(A.P.) are natural numbers. If the ratio of the sum of the −1 −1

first seven terms to the sum of the first eleven terms is 6 : F(x) 1 1
for all x ∈[−1, 2]. If lim = , then the value of f  
11 and the seventh term lies in between 130 and 140, then x→1 G(x) 14 2
the common difference of this A.P. is is
144 Mathematics
SECTION 2 (Maximum Marks: 32) 43 7
a. e12 + e22 = b. e1e2 =
This section contains EIGHT questions. 40 2 10
Each question has FOUR options (a), (b), (c) and (d). ONE OR
5 3
MORE THAN ONE of these four option(s) is (are) correct. c. | e12 − e22 | = d. e1e2 =
8 4
For each question, darken the bubble(s) corresponding to all the
correct option(s) in the ORS. 13. Consider the hyperbola H : x 2 − y 2 = 1 and a circle S with
Marking scheme: center N ( x2 , 0). Suppose that H and S touch each other at
+4 If only the bubble(s) corresponding to all the correct option(s)
a point P( x1 , y1 ) with x1 > 1 and y1 > 0. The common
is(are) darkened.
tangent to H and S at P intersects the x-axis at point M. If
0 If none of the bubbles is darkened
–2 In all other cases (l , m) is the centroix of the triangle ∆PMN, then the
correct expression(s) is (are)
3
192x 1 dl 1
9. Let f ′( x) = for all x ∈ R with f   = 0. If a. = 1 − 2 for x > 1
2 + sin π x
4
2 dx1 3x1
1
dm x1
m≤ ∫ f ( x ) dx ≤ M , then the possible values of m and M = for x1 > 1
)
b.
(
1/ 2
dx1 3 x12 − 1
are
1 1 dl 1
a. m = 13, M = 24 b. m = ,M = c. = 1 + 2 for x1 > 1
4 2 dx1 3x1
c. M = −11, M = 0 d. M = 1, M = 12
dm 1
d. = for y1 > 0
10. Let S be the set of all non-zero real numbers α such that dy1 3
the quadratic equation α x 2 − x + α = 0 has two distinct 14. The option(s) with the values of a and L that satisfy the
real roots x1 and x2 satisfying the inequality | x1 − x2 | < 1. following equation is(are)

Which of the following intervals is(are) a subset(s) of S ?
∫ (sin at + cos 4 at ) dt
6

 1 1   1  π
0
= L?
a.  − , − b.  − ∫ e (sin at + cos 4 at ) dt
1 6
 , 0
 2 5  5  0

 1   1 1 e4π − 1 e4π + 1
c.  0, ,  a. a = 2, L = b. a = 2, L =
 d.  eπ − 1 eπ + 1
 5  5 2
e4π − 1 e4π + 1
6 4 c. a = 4, L = d. a = 4, L =
11. If α = 3 sin −1   and β = 3 cos −1   , where the inverse eπ − 1 eπ + 1
 11  9
trigonometric functions take only the principal values, 15. Let f , g :[−1, 2] → R be continuous functions which are
then the correct option(s) is (are) twice differentiable on the interval (–1, 2). Let the values
a. cos β > 0 b. sin β < 0 of f and g at points –1, 0 and 2 be as given in the
c. cos(α + β ) > 0 d. cos α < 0 following table:
x = −1 x=0 x=2
12. Let E1 and E 2 be two ellipse whose centers are at the f ( x) 3 6 0
origin. The major axes of E1 and E 2 lie along the x-axis g ( x) 0 1 –1
and the y-axis, respectively. Let S be the circle In each of the intervals (–1, 0) and (0, 2) the function
x 2 + ( y − 1) 2 = 2. The straight line x + y = 3 touches the ( f − 3g )′′ never vanishes. Then the correct statements(s) is
curves S, E1 and E 2 at P, Q and R, respectively Suppose (are)
a. f ′( x) − 3 g ′( x) = 0 has exactly three solutions in
2 2
that PQ = PR = . If e1 and e2 are the eccentricities of (−1, 0) ∪ (0, 2)
3
E1 and E2 , respectively, then the correct expression(s) b. f ′( x) − 3 g ′( x) = 0 has exactly one solution in (–1, 0)
is(are)
Mock Test-1 145
c. f ′( x) − 3 g ′( x) = 0 has exactly one solution in (0, 2) 3 3
∫ x F ′( x ) dx = − 12 and ∫ F ′′( x ) dx = 40, then
2
18. If the
1 1
d. f ′( x) − 3 g ′( x) = 0 has exactly two solutions in (–1, 0) correct expression(s) is(are)
and exactly two solutions in (0, 2) a. 9 f ′(3) + f ′(1) − 32 = 0
16. Let f ( x ) = 7 tan 8 x + 7 tan 6 x − 3 tan 4 x − 3 tan 2 x for all 3
b. ∫
1
f ( x ) dx = 12
 π π
x ∈  − ,  Then the correct expression(s) is (are)? c. 9 f ′(3) − f ′(1) + 32 = 0
 2 2 3
π /4
a. ∫ xf ( x ) dx =
1 π /4
d. ∫ f ( x ) dx = − 12

1
b. f ( x ) dx = 0
0 12 0

π /4 1 π /4 Paragraph-II
c. ∫ xf ( x ) dx = d. ∫ f ( x ) dx = 1
0 6 0
Let n1 and n2 be the number of red and black balls,

SECTION 3 (Maximum Marks: 16) respectively, in box I. Let n3 and n4 be the number of red and

This section contains FOUR questions. black balls, respectively, in box II.
Each question has FOUR options (a), (b), (c) and (d). ONE OR
19. One of the two boxes, box I and box II, was selected at
MORE THAN ONE of these four option(s) is(are) correct.
random and a ball was drawn randomly out of this box. The
For each question, darken the bubble(s) corresponding to all the
ball was found to be red. If the probability that this red ball
correct option(s) in the ORS.
1
Marking scheme: was drawn from box II is , then the correct option(s) with
3
+4 If only the bubble(s) corresponding to all the correct option(s)
the possible values of n1 , n2 , n3 and n4 is (are)
is(are) darkened.
a. n1 = 3, n2 = 3, n3 = 5, n4 = 15
0 If none of the bubbles is darkened
–2 In all other cases b. n1 = 3, n2 = 6, n3 = 10, n4 = 50
c. n1 = 8, n2 = 6, n3 = 5, n4 = 20
Paragraph-I
d. n1 = 6, n2 = 12, n3 = 5, n4 = 20
Let F : R → R be a thrice differentiable function. Suppose that
1  20. A ball is drawn at random from box I and transferred to
F (1) = 0, F (3) = −4 and F ′( x) < 0 for all x ∈  , 3  . Let box II. If the probability of drawing a red ball from box I,
2 
1
f ( x ) = xF ( x ) for all x ∈ R. after this transfer, is , then the correct option(s) with
3
17. The correct statement(s) is (are)
the possible values of n1 and n2 is(are)
a. f ′(1) < 0
a. n1 = 4, n2 = 6 b. n1 = 2, n2 = 3
b. f (2) < 0
c. n1 = 10, n2 = 20 d. n1 = 3, n2 = 6
c. f ′( x) ≠ 0 for any x ∈ (1, 3)
d. f ′( x) = 0 for some x ∈ (1, 3)

Space for rough work


146 Mathematics
ANSWER & SOLUTIONS 8. (d) IUAENSRNC Obviously required number of words
JEE-Main 6!
are × 4 ! = 8640
2!
1. 2. 3. 4. 5. 6. 7. 8. 9. 10.
d d c a a c c d a c 9. (b) The probability that only two tests are needed =
11. 12. 13. 14. 15. 16. 17. 18. 19. 20. probability that the first machine tested is faulty ×
c d b a c c d a c c probability that the second machine tested is faulty
21. 22. 23. 24. 25. 26. 27. 28. 29. 30.
2 1 1
c c a d d a c c a d = × =
4 3 6
1. (d) x + 3 > 0 and x 2 + 3 x + 2 ≠ 0 π 3π
10. (c) sin sin = sin18°.sin 54°
10 10
2. (d) If x ≠ 1, multiplying each term by ( x − 1), the given
5 −1 5 +1 1
equation reduces to x( x − 1) = ( x − 1) or ( x − 1) 2 = 0 or = sin18°.cos 36° = . = .
4 4 4
x = 1, which is not possible as considering x ≠ 1. Thus
given equation has no roots.
11. (c) We know that cosh −1 x = log x + x 2 − 1( )
3. (c) (3 + 5ω + 3ω 2 )2 + (3 + 3ω + 5ω 2 )2 ∴ ( )
cosh −1 (1) = log 1 + 12 − 1 = log1 = 0.
= (3 + 3ω + 3ω + 2ω ) + (3 + 3ω + 3ω + 2ω )
2 2 2 2 2
12. (d)
(1 + ω + ω = 0, ω = 1)
2 3

α
H=60m
= (2ω )2 + (2ω 2 )2 = 4ω 2 + 4ω 4 = 4(−1) = −4.
h
4. (b) It is understandable. β
d
A H = d tan β and H − h = d tan α
5. (a) Let , A, AR be the roots of the equation
R
60 tan β
ax3 + bx2 + cx + d = 0 ⇒ =
60 − h tan α
d
then A 3 = Product of the roots = − 60sin( β − α )
a ⇒ − h = 60 tan α − 60 tan β ⇒ h=
tan β sin β
1/ 3 cos α cos β
d cos β
⇒ A = − 
a ⇒ x = cos α sin β .
Since A is a root of the equation.
13. (b) Since limit of a function is a + b as x → 0, therefore
∴ aA3 + bA2 + cA + d = 0
to be continuous at a function, its value must be a + b at
2/3 1/ 3
 d  d  d x=0
⇒ a −  + b −  + c −  +d =0
 a  a  a ⇒ f (0) = a + b.
2/3 1/ 3
d  d 
2
d d
⇒ b  = c  ⇒ b3 = c3 ⇒ b3d = c3a. 14. (a) y = a sin 4 θ
a a a2 a
dy
6. (c) Trick: Put n = 1, 2, 3,.... ⇒ = 4a sin 3 θ cosθ

S1 = 1, S 2 = 2 + 2 = 4 and x = a cos θ
4

Now by alternate (c), dx


⇒ = −4a cos3 θ sin θ
put n = 1, 2 dθ
S1 = 1.20 = 1, S 2 = 2.21 = 4 dy dy / dθ − sin 2 θ
∴ = = = − tan 2 θ
dx dx / dθ cos 2 θ
 dy  2  3π 
(log e n) 2 (log e n) 4 eloge n + e − loge n n + n −1 ∴   3π = − tan   = −1.
7. (c) 1 + + +K = = .  dx θ =  4 
2! 4! 2 2 4
Mock Test-1 147
[(5 + x )(2 + x )] 21. (c) Let P(h, k ) be the mid-point of the line segment
15. (c) Given f ( x ) =
[1 + x ] joining the focus (a,0) and a general point Q( x, y) on the
4 4
f ( x) = 1 + + (5 + x) = (6 + x) + parabola.
1+ x (1 + x )
x+a y
4 Then h = , k = ⇒ x = 2 h − a, y = 2 k .
⇒ f '( x ) = 1 − = 0; 2 2
(1 + x) 2 Put these values of x and y in y 2 = 4 ax, we get
x2 + 2 x − 3 = 0 4 k 2 = 4 a (2 h − a )
⇒ x = −3, 1
⇒ 4 k 2 = 8ah − 4 a 2 ⇒ k 2 = 2 ah − a 2
8 So, locus of P(h, k ) is y 2 = 2 ax − a 2
Now f ′′ ( x ) = ,
(1 + x)3
 a
f ′′ (−3) = −ve, ⇒ y 2 = 2a  x − 
 2
f ′′ (1) = +ve Its directrix is
Hence minimum value at x = 1 a a
x− = − ⇒ x = 0.
(5 + 1)(2 + 1) 6 × 3 2 2
f (1) = = = 9.
(1 + 1) 2 r r
22. (c) a + b = 3iˆ + 9 ˆj = 3(iˆ + 3 ˆj ). Hence it is parallel to (1, 3).
1
16. (c) Putting t = tan −1 x ⇒ dt = dx, we get 23. (a) Direction ratio of the line joining the point
1 + x2 (2, 1, − 3), (− 3, 1, 7) are (a1 , b1 , c1 )
−1
e tan x −1
⇒ (− 3 − 2, 1 − 1, 7 − (−3))
∫ 1+ x 2
dx = ∫ et dt = et + c = e tan x + c.
⇒ (− 5, 0, 10)
17. (d) Area

Direction ratio of the line parallel to line
A = ∫ log x.dx x −1 y z + 3
0
= = are
3 4 5
= ( x log x − x)∞0 = ∞
(a2 , b2 , c2 )
Y
⇒ (3, 4, 5)
Angle between two lines,
y =loge x
a1a2 + b1b2 + c1c2
(1,0)
X cos θ =
a1 + b12 + c12 a22 + b22 + c22
2

(− 5 × 3) + (0 × 4) + (10 × 5)
cosθ =
25 + 0 + 100 9 + 16 + 25
dy
18. (a) = e x − y + x 2 e − y = e − y (e x + x 2 ) 35
dx ⇒ cos θ =
25 10
⇒ e y dy = ( x 2 + e x ) dx
 7 
x3 ⇒ θ = cos −1  
Now integrating both sides, we get e = + e x + c. y
 5 10 
3
19. (c) 4x + 3 y = 11 and 4 x + 3 y =
15 24. (d) Since ~ ( p ⇒ q) ≡ p ∧ ~ q
2 ~ (~ p ⇒ q) = ~ p ∧ ~ q
15
Therefore, 11 −
D= 2 = 7 . 2 1 −1
5 10 25. (d) ∆ = 1 −3 2
1 4 −3
20. (c) Since area = π r 2 , where
r =a = 2(9 − 8) − 1(−3 − 2) − 1(4 + 3) = 7 − 7 = 0

⇒ Area = π a 2 . Hence, number of solutions is zero.


148 Mathematics
1 1 1 1 JEE Advance Paper-I
26. (a) − 2 + 3 − 4 +K
n 2n 3n 4n 1. 2. 3. 4. 5. 6. 7. 8. 9. 10.
2 3 4 4 8 4 8 5 1 0 4 a,c,d c, d
1 (1/ n) (1/ n) (1/ n)
= − + − +K 11. 12. 13. 14. 15. 16. 17. 18. 19. 20.
n 2 3 4 b,c b,d a,b a,d a,c b,c a,d a,b,c c c
 1  n +1
= log e  1 +  = log e  .
 n   n  1. (4) 7
6
27. (c) For continuity at all x ∈ R , we must have 5

 π 4
f  −  = lim − (−2sin x)
 2  x→( −π / 2) 3

= lim ( A sin x + B) 2
x →( −π / 2)+
1
⇒ 2 = −A + B . . .(i) –2 –1
O 1 2 3 4 5
π  –1
and f   = lim − ( A sin x + B )
 2  x→(π / 2) –2

= lim + (cos x) –3
x →(π / 2)
Required area = 4 × (1× 1) = 4 sq unit
⇒ 0 = A+ B . . .(ii)
From (i) and (ii), ∴ λ =4
1 1 1 1 1/ 2
A = −1 + + + +K∞
λ λ λ λ K∞ = λ 2 4 8 16
= λ 1−1/ 2 = λ = 4
and B = 1.
5
2. (8) cos 2 2 x + cos 4 x + sin 4 x + cos 6 x + sin 6 x = 2
4
28. (c) Put 3 + 5 x 4 = t ⇒ 20 x3 dx = dt , then
5
1 ⇒ cos 2 2 x − 5cos 2 x sin 2 x = 0
∫ x3 3 + 5 x 4 dx = ∫ t 1/ 2 dt
20
4
⇒ tan 2 2 x = 1, where 2 x ∈ [0, 4π ]
2 1 1
= × .t 3/ 2 + c = (3 + 5 x 4 )3/ 2 + c. Number of solutions = 8
3 20 30 3. (4) Image of y = −5 about the line x + y + 4 = 0 is
x =1
29. (a) Equation of line perpendicular to
⇒ Distance AB = 4
5x + 12 y + 8 = 0 is
4. (8) Let coin was tossed ‘n’ times
12 x − 5 y + k = 0.
1 n
Now it is a tangent to the circle, if Probability of getting at least two heads = 1 −  n + n 
2 2 
Radius of circle
 n + 1
= Distance of line from centre of circle ⇒ 1 −  n  ≥ 0.96
 2 
12(11) − 5(2) + k 2n
121 + 4 − 25 =
144 + 25 ⇒ ≥ 25 ⇒ n ≥ 8
n +1
⇒ k = 8 or −252.
5. (5) n = 6!.5! (5 girls together arranged along with 5
Hence equations of tangents are 5
boys) m = C4 .(7! − 2.6!).4!
12 x − 5 y + 8 = 0 and
(4 out of 5 girls together arranged with others – number of
12x − 5 y = 252 cases all 5 girls are together)
m 5 ⋅ 5 ⋅ 6!⋅4!
30. (d) ~ p ∧ q =~ (q → p) . =
n 6! ⋅ 5!
Mock Test-1 149
6. (1) Let Parabola be y = 4ax and coordinates of P and Q
2
10. (c,d) (Y Z − Z Y ) = ( Z ) (Y ) − (Y ) ( Z )
3 4 4 3 T T 4 T 3 T 3 T 4

on this parabola are P ≡ ( at , 2at1 ) and Q ≡ ( at , 2 at 2 ); T


1
2 2
2
= − Z 4Y 3 + Y 3 Z 4
is the point of intersection of tangents at t1 and t2. ⇒ symmetric X 44 + Y 44 is symmetric

∴ Coordinates of T ≡ {at1 , t2 , a (t1 + t2 )} X 4 Z 3 − Z 3 X 4 skew symmetric

Similarly , P′ ≡ {at3 , t1 , a (t3 + t1 )} X 23 + Y 23 skew symmetric.

Q′ ≡ {at2 , t3 , a (t2 + t3 )} (1 + α) 2 (1 + 2α) 2 (1 + 3α) 2


Let TP′ : TP = λ : 1 11. (b,c) We get 3 + 2α 3 + 4α 3 + 6α
t3 − t2 5 + 2α 5 + 4α 5 + 6α
∴ λ=
t1 − t2 = − 648α ( R3 → R3 − R2 ; R2 → R2 − R1 )
TP′ t3 − t2 α 2 − 2 4α 2 − 2 9α 2 − 2
or =
TP t1 − t 2 3 + 2α 3 + 4α 3 + 6α
TQ′ t1 − t3 2 2 2
Similarly, =
TQ t1 − t2 = − 648α ( R1 → R1 − R2 ; R3 → R3 − R2 )
TP ′ TQ ′ −2α 2 −5α 2 −9α 2 − 3
or = =1
TP TQ ⇒ −2α −2α 3 + 6α = − 648α
0
x ⋅0 1
x ⋅0 2
x ⋅1 1 0 0 2
7. (0) I = ∫ 2 + 0 dx + ∫ 2 + 1 dx + ∫ 2 + 0 dx + 0 = 4
−1 0 1 12. (b,d) Let the required plane be x + z + λ y − 1 = 0
⇒ 4I = 1 = 0
| λ − 1| 1
⇒ =1 ⇒ λ = −
8. (4) Let inner radius be r and inner length be l λ +2
2 2
πr l =V
2
⇒ P3 ≡ 2 x − y + 2 z − 2 = 0
Volume of material be M, Distance of P3 from (α , β , γ ) is 2
M = π ( r + 2) 2 (l + 2) − π r 2 l | 2α − β + 2γ − 2 |
=2
dM 4V 8V
= − 2 − 3 + 8π + 0 + 4π r 4 ×1 + 4
dr r r ⇒ 2α − β + 2λ + 4 = 0 and 2α − β + 2λ − 8 = 0
dM
= 0 when r = 10 13. (a,b) Line L will be parallel to the line of intersection of
dr
P1 and P2
⇒ V = 1000π
Let a, b and c be the direction ratios of line L
V
⇒ =4 ⇒ a + 2b − c = 0 and 2a − b + c = 0
250π
⇒ a : b : c ::1 : − 3 : − 5
r r r
9. (a,c,d) | b + c | = | a | x−0 y−0 z −0
Equation of line L is = =
r2 r2 r r r2 1 −3 −5
⇒ | b | + | c | = 2b ⋅ c = | a | Again foot of perpendicular from origin to plane P1 is
r
⇒ 48 + | c |2 + 48 = 144  1 1 1
r − ,− , 
r | c |2 r  6 3 6
⇒ |c|=4 3∴ − | a | = 12 ∴ Equation of project of line L on plane P1 is
2
r r r 1 2 1
Also, | a + b | = | c | x+ y+ z−
r2 r2 r r r2 6= 6 = 6 =k
⇒ | a | + | b | = 2a ⋅ b = | c | 1 −3 −5
r r r r r  5 2  1 1 1
⇒ a ⋅ b = − 72 a + b + c = 0 Clearly points  0, − , −  and  − , − ,  satisfy the
r r r r r r r r r r  6 3  6 3 6
⇒ a × b = c × a ⇒ | a × b + c × a | = 2| a × b | = 48 3
line of projection i.e. M
150 Mathematics

 16a 8a  17. (a,d) Differentiability of f ( x ) at x = 0


14. (a,d) P( at 2 , 2at ) ⇒ Q  2 , − 
 t t   f (0) − f (0 − δ )  0 + g (−δ )
LHD f ′(0− ) = lim   = lim =0
1 δ →0
 δ  δ → 0 δ
∆OPQ = OP ⋅ OQ
2  f (0 + δ ) − f (0)  g (δ )
RHD f ′(0+ ) = lim   = lim =0

1 a (4) 16 δ →0
 δ  δ → 0 δ
at t 2 + 4 ⋅ +4 =3 2
2 t t2 ⇒ f ( x ) is differentiable at x = 0
t − 3 2t + 4 = 0
2
Differentiabiligy of h( x) at x = 0
⇒ t = 2, 2 2 h′(0+ ) = 1, h( x) is an even function
 t2  Hence non diff. at x = 0
Hence, P ( at 2 , 2at ) = P  , t 
2  Differentiability of f ( h ( x )) at x = 0
f ( h( x )) = g (e| x| ) ∀ x ∈ R
t = 2 ⇒ P(1, 2)
f ( h(0)) − f ( h(0 − δ ))
t = 2 2 ⇒ P(4,2 2) LHD f ′( h(0 − )) = lim
δ →0 δ
dy ye x 1 g (1) − g (e ) δ
15. (a,c) + = x = lim = g ′(1)
dx 1 + e x
e +1 δ →0 δ
ex f ( h(0 + δ )) − f ( h(0))
I .F . = ∫
x
dx = eIn(1+ e ) = 1 + e x RHD f ′( h(0 + )) = lim
e 1+ e
x
δ →0 δ
δ
x+c g (e ) − g (1)
⇒ y (1 + e x ) = ∫ 1dx ⇒ y = = lim = g ′(1) Since g ′(1) ≠ 0
1 + ex δ →0 δ
y (0) = 2 ⇒ f ( h ( x )) is non diff. at x = 0
⇒ c =1 Differentiability of h ( f ( x )) at x = 0

{
x+4
⇒ y= h( f ( x )) = e
( f ( x )|
, x≠0
1 + ex 1, x=0
⇒ y (−4) = 0 h( f (0)) − h( f (0 − δ ))
LHD. h′( f (0 − δ )) = lim
(1 + e x ) − ( x + 4)e x δ →0 δ
⇒ y′ = =0
(1 + e x ) 2 1− e | g ( − δ )|
| g ( −δ ) |
= lim ⋅ =0
(1 + e ) − ( x + 4)e
x x δ →0 | g ( −δ ) | δ
Let g ( x) =
(1 + e x ) 2 π
18. (a,b,c) Given g ( x) = sin x ∀ x ∈ R
2
2−4
g (0) = 2 < 0 1 
2 f ( x) = sin  g ( g ( x)) 
 1 3 3 
2
1 +  − 1−  π π
g (−1) =  e e
<0= e >0 Also, g ( g ( g ( x))) ∈ − ,  ∀ x ∈ R
 1
2
 1
2
 2 2
 1 +   1 + 
 e  e  1 1
Hence, f ( x) and f ( g ( x)) ∈  − , 
g (0) ⋅ g ( −1) < 0. Hence g(x) has a root in between ( −1, 0)  2 2
1 
16. (b,c) Let the family of circles be x 2 + y 2 − αx − αy + c = 0 sin  g ( g ( x))  1 g ( g ( x))
lim
f ( x)
= lim  3  ⋅3
On differentiation 2 x + 2 yy ′ − α − αy ′ = 0 x →0 g ( x) x →0 1 g ( x)
g ( g ( x))
Again on differentiation and substituting ' α' we get 3
 2 x + 2 yy′  π 
2 x + 2 y ′2 + 2 yy′′ −  sin  sin x 
 y ′′ = 0 π =π
lim ⋅ 
2
 1 + y′  ⇒
x →0 6 π 6
⇒ ( y − x) y ′′ + y ′(1 + y ′ + y ′2 ) + 1 = 0 sin x
2
Mock Test-1 151
 π 1 π  1  ⇒ 2(sin( X − Y ) ⋅ sin( X + Y ) = sin ( Z ) 2

Range of g ( f ( x)) ∈  − sin   , sin  


 2 2 2  2  ⇒ 2 ⋅ sin( X − Y ) ⋅ sin( Z ) = sin 2 ( Z )
⇒ g ( f ( x)) ≠ 1. sin( X − Y ) 1
⇒ = =λ
3α + β sin Z 2
19. (c) (A) = 3
2  nπ 
⇒ cos   = 0 for n = odd integer.
 2 
3α + β = ± 2 3 . . .(i)
(B) 1 + cos 2 X − 2 cos 2Y = 2 sin X sin Y
Given α = 2 + 3β . . .(ii) sin 2 X + sin X sin Y − 2sin 2 Y = 0
From equation (i) and (ii), we get α = 2 or −1 (sin X − sin Y )(sin X + 2 sin Y ) = 0
So |α| = 1 or 2 ⇒ sin X = sin Y
sin X a
−3ax 2 − 2, x < 1 ⇒ = = 1.
(B) f ( x) =  sin Y b
 bx + a , x ≥ 1
2

(C) Here, distance of Z from bisector of


For continuity −3a − 2 = b + a 2 uuur uuur 3
OX and OY =
a2 + 3a + 2 = −b . . .(i) 2
For differentiability − 6 a = b 2 2
 1  1 9
6a = −b ⇒ β −  +β −  =
 2  2 2
a − 3a + 2 = 0
2
⇒ β = 2, −1
a = 1, 2
⇒ | β | = 2,1
(C) (3 − 3ω + 2ω 2 ) 4 n + 3 + (2 + 3ω − 3ω 2 ) 4 n + 3
(D) When α = 0
+ ( −3 + 2ω + 3ω 2 ) 4 n + 3 = 0 2
Area = 6 − ∫ 2 x dx
(3 − 3ω + 2ω ) 2 4n +3
+ (ω (2ω + 3 − 3ω )) 4n+3
2 0

8 2
+ (ω ( −3ω + 2ω + 3) 4 n + 3 = 0
2 2
= 6−
3
⇒ (3 − 3ω + 2ω 2 ) 4 n + 3 + (1 + ω 4 n + ω 8 n ) = 0
When α = 1
⇒ n ≠ 3k , k ∈ N 1 2

(D) Let a = 5 − d
Area = ∫ 0
(3 − x − 2 x ) dx + ∫ ( x + 1 − 2 x ) dx
1

1 2
q = 5+ d x2 4 x2 4
= 3x − − x3/ 2 + + x − x3/ 2
b = 5 + 2d 2 3 0
2 3 1
⇒ | q − a | = | 2d | 8
= 5− 2.
2ab 3
Given =4
a+b
ab JEE Advance Paper-II
⇒ =2
a+b
1. 2. 3. 4. 5. 6. 7. 8. 9. 10.
(5 − d )(5 + 2d ) = 2(5 − d + 5 + 2 d ) = 2(10 + d )
9 4 9 8 4 2 9 7 d a,d
25 + 10d − 5d − 2d 2 = 20 + 2d 11. 12. 13. 14. 15. 16. 17. 18. 19. 20.
2d 2 − 3d − 5 = 0 b,c,d a,b a,b,c a,c b,c a,b a,b,c c,d a,b c,d
5
d = −1, d = ⇒ | 2 d | = 2, 5 1.
r r r r
(9) s = 4 p + 3q + 5r
2
r r r r r r r r r r
c2 s = x(− p + q + r ) + y ( p − q + r ) + z (− p − q + r )
20. (c) (A) a 2 − b 2 = (given) r r r r
2 s = (− x + y − z ) p + ( x − y − z )q + ( x + y + z )r
4R2 ⇒ −x + y − z = 4
4 R 2 (sin 2 X − sin 2 Y ) = sin 2 ( Z )
2
152 Mathematics
⇒ x− y−z =3 Multiplying equation (iii) and (iv), we get
⇒ x+ y+z =5 1 1 1 1
( x1 + x2 + x3 + x4 ) ×  + + +  = 4
On solving we get x = 4, y =
9 7  x1 x2 x3 x4 
,z=−
2 2 ( ) −e
cos α n
⇒ 2x + y + z = 9 e e
6. (2) lim =−
α →0 αm 2

( ) (cosα
12 kπ π
e e (cos(α )
i i n
−1)
Σ e 7
e 7 −1
n
− 1) e
k =1
12 lim α 2n = −
2. (4)
3 i
π
=
3
=4 α →0
( cos(α n
) −1 α α ) m 2n
2
Σ ei (4 k − 2) e 7 − 1
k =1 if and only if 2n − m = 0

3. (9) Let seventh term be ‘a’ and common difference be ‘d’ 1 −1  12 + 9 x 2 


(9) α = ∫ e
(9 x +3tan x)
7.  2 
dx
Given
S7
=
6
⇒ a = 15 d
0
 1+ x 
S11 11
Put 9 x + 3tan −1 x = t
Hence, 130 < 15 d < 140 ⇒ d = 9
 3 
4. (8) x9 can be formed in 8 ways ⇒  9 + 1 + x 2  dx = dt
 
i.e., x 9 , x1+8 , x 2+ 7 , x 3+ 6 , x 4+5 , x1+ 2+6 , x1+3+5 , x 2+3+ 4 and 3π 3π
9+ 9+
coefficient in each case is 1 ⇒ α =∫ 4
e t dt = e 4
−1
0
⇒ Coefficient of x = 1 + 1 + 1 + .........+ 1 = 8
9
8 times  3π 
⇒  log e |1 + α | − 4 =9
5. (4) Let point of concurrent is (h, k )  
′ ′
Equation of normal at ( x′, y′) is, x − x2 = y1 − y2 8.
1
(7) G (1) = ∫ t | f ( f (t )) | dt = 0 f ( − x) = − f ( x )
x′ / a y′ / b −1

It is passes through (h, k ) , then 1


Given f (1) =
2
y ′ 2 {a 2 ( h − x ′) + b 2 x ′}2 = b 4 k 2 x′ 2 . . .(i)
F ( x) − F (1)
′ ′ 2 2 2
But x 2 = y 2 = 1 or y ′ 2 = b 2 ( a 2 − x ′ 2 ) . . .(ii) lim
F ( x)
= lim x −1 =
f (1)
=
1
a b a x →1 G ( x ) x →1 G ( x ) − G (1) | f ( f (1)) | 14
2
Value of y′ from equation (ii), putting in equation (i), x −1
b2 2 1/ 2 1 1
we get ( a − x ′ 2 ){a 2 h + (b 2 − a 2 ) x ′}2 = b 4 k 2 x ′2 ⇒ = ⇒ f   = 7.
a2 | f (1/ 2) | 14 2
⇒ b2 2
( a − x ′ 2 ){a 4 h 2 + (b 2 − a 2 ) 2 x ′ 2 + 2 a 2 hx ′(b 2 − a 2 )} = b 4 k 2 x ′ 2
a2 192 x 3 192 x 3
Arranging above as a fourth degree equation in x′, we get
9. (D)
3 ∫1/ 2
t dt ≤ f ( x ) ≤
2 ∫1/ 2
t dt

⇒ 3
− ( a 2 − b 2 ) 2 x ′4 + 2 ha 2 ( a 2 − b 2 ) x ′3 + x ′ 2 (K) − 2 a 4 h ( a 2 − b 2 ) x ′ + a 6 h 2 = 0 16 x 4 − 1 ≤ f ( x ) ≤ 24 x 4 −
2
Above equation being of fourth degree in x′, therefore 1 
1 1 3
roots of the above equation are x1 , x2 , x3 , x4 then ∫1/ 2
(16 x 4 − 1)dx ≤ ∫
1/ 2
f ( x)dx ≤ ∫  24 x 2 −  dx
1/ 2
 2
2ha 2 (a 2 − b 2 ) 2ha 2 26 1 39
10 ∫1/ 2
( x1 + x2 + x3 + x4 ) = − = . . .(iii) 1< ≤ f ( x ) dx ≤ < 12
−(a 2 − b 2 ) 2 (a 2 − b 2 ) 10

1 1 1 1  Σx1 x2 x3 10. (a,d) Here, 0 < ( x1 − x2 ) 2 < 1


 + + + =
x
 1 x 2 x3 x 4  x1 x2 ⋅ x3 ⋅ x4

⇒ 0 < ( x1 + x2 ) 2 − 4 x1 x2 < 1
2a h ( a − b )
4 2 2
1
⇒ 0< − 4 <1
−( a 2 − b 2 ) 2 2(a 2 − b 2 ) α2
= 6 2
= . . .(iv)
ah a2h  1 1   1 1
−( a 2 − b 2 ) 2 ⇒ α ∈ − , − ∪ , 
 2 5   5 2
Mock Test-1 153

π 3π for a = 2 as well as a = 4
11. (b,c,d) <α <π,π < β < Similarly
2 2

3π 5π
∫ π e (sin at + cos 4 at) dt = e 2π A
1 6
⇒ <α + β < 2
2 2
⇒ sin β < 0; cos α < 0 A + eπ A + e2π A + e3π A 34π −1
So, L = = π
A e −1
⇒ cos(α + β ) > 0.

12. (a,b) For the given line, point of contact for For both a = 2, 4
x2 y 2  a 2 b2 
E1 : 2
+ 2 = 1 is  ,  15. (b,c) Let H ( x ) = f ( x) − 3g ( x)
a b  3 3
H (−1) = H (0) = H (2) = 3.
x2 y 2  B2 A2 
and for E2 : 2 + 2 = 1 is  ,  Applying Rolle’s Theorem in the interval [–1, 0]
B A  3 3 
H ′( x) = f ′( x) − 3 g ′( x) = 0 for atleast one c ∈ (−1, 0)
Point of contact x + y = 3 of and circle is (1, 2)
As H ′′( x ) never vanishes in the interval
Also, general point on x + y = 3 can be taken as
⇒ Exactly one c ∈ (−1,0) for which H ′( x) = 0
 r r  2 2
1 m , 2±  where, r =
 2 2 3 Similarly, apply Rolle’s Theorem in the interval [0, 2]

1 8 5 4 ⇒ H ′( x) = 0 has exactly one solution in (0, 2)


So, required points are  ,  and  , 
3 3 3 3
16. (a,b) f ( x ) = (7 tan 6 x − 3 tan 2 x ) (tan 2 x + 1)
Comparing with points of contact of ellipse,
π /4 π /4
a 2 = 5, B 2 = 8
∫0
f ( x)dx = ∫
0
(7 tan 6 x − 3tan 2 x)sec 2 xdx
b 2 = 4, A2 = 1 π /4

7 43
⇒ ∫0
f ( x ) dx = 0
∴ e1e2 = and e12 + e22 =
40 π /4 π /4 π /4
2 10
∫ xf ( x)dx =  x ∫ f ( x) dx  − ∫  ∫ f ( x) dx  dx
0  0 0  
13. (a,b,d) Tanget at P, xx1 − yy1 = 1 intersects x-axis at
π /4 1
1  ∫0
xf ( x ) dx =
12
M  , 0
 x1 
17. (a,b,c) (a) f ′( x) = f ( x) + xF ′( x)
y1 y1 − 0
Slope of normal = − =
x1 x1 − x2 f ′(1) = F (1) + F ′(1)
⇒ x2 = 2 x1 ⇒ N ≡ (2 x1 ,0) f ′(1) = F ′(1) < 0
1
3 x1 + f ′(1) < 0
x1 y
For centroid l = ,m= 1
3 3 (b) F (2) = 2 F (2)
dl 1
= 1− 2 F ( x ) is decreasing and F (1) = 0
dx1 3x1
dm 1 dm 1 dy1 x1 Hence F (2) < 0
= , = =
dy1 3 dx1 3 dx1 3 x12 − 1 ⇒ f (2) < 0
π 1 (c) f ′( x) = F ( x) + xF ′( x)
14. (a,c) Let ∫
0
e (sin 6 at + cos 4 at) dt = A
2π F ( x) < 0 ∀ x ∈ (1, 3)
I = ∫ e1 (sin 6 + at + cos 4 at) dt
π
F ′( x) < 0 ∀ x ∈ (1, 3)
Put t = π + x
dt = dx Hence f ′( x) < 0 ∀ x ∈ (1, 3)
Mock Test-2 155
JEE-MAIN: MATHEMATICS MOCK TEST-2

1 7. If (1 + x − 2 x 2 ) 6 = 1 + a1 x + a2 x 2 + .... + a12 x12 , then the


1. If f :[1, ∞) → [2, ∞) is given by f ( x) = x + then
x expression a2 + a4 + a6 +.... + a12 has the value
f − 1( x) equals: a. 32 b. 63
x+ x −4 2
x c. 64 d. None of these
a. b.
2 1 + x2 1+ 2 1+ 2 + 3 1+ 2 + 3 + 4
8. 1+ + + +K∞ =
x− x −4 2 2! 3! 4!
c. d. 1 + x 2 − 4
2 a. e b. 3e c. e / 2 d. 3e / 2
2. Let α and β be the roots of the equation x2 + x + 1 = 0  1  1  1 
9. log e 2 + log e 1 +  + log e 1 +  + .... + log e 1 +  is
The equation whose roots are α 19 , β 7 is  2   3   n −1 
a. x − x − 1 = 0
2
b. x − x + 1 = 0
2 equal to
a. log e 1 b. log e n
c. x + x − 1 = 0 d. x + x + 1 = 0
2 2

c. log e (1 + n) d. log e (1 − n)
 2π   2π  2
3. Let ωn = cos   + i sin   , i = −1, then 10. There are 10 lamps in a hall. Each one of them can be
 n   n 
switched on independently. The number of ways in which
( x + yω3 + zω3 2 ) ( x + yω32 + zω3 ) is equal to
the hall can be illuminated is.
a. 0
a. 102 b. 1023
b. x 2 + y 2 + z 2 10
c. 2 d. 10 !
c. x 2 + y 2 + z 2 − yz − zx − xy
11. A fair coin is tossed repeatedly. If tail appears on first four
d. x 2 + y 2 + z 2 + yz + zx + xy tosses, then the probability of head appearing on fifth toss
equals
6
 2π k 2π k 
4. The value of ∑  sin
k =1 7
− i cos
7 
 is a.
1
b.
1
2 32
a. –1 b. 0
31 1
c. –i d. i c. d.
32 5
1 0 0 
tan 2 60°cosec30°
5. The inverse of the matrix  0 1 0  is 12. If x sin 45° cos 2 60° = , then x =
  sec 45° cot 2 30°
 0 0 1 
a. 2 b. 4 c. 8 d. 16
0 0 1  1 0 0 
a.  0 1 0  b.  0 1 0  13. If sin θ + sin 2θ + sin 3θ = sin α and
   
 1 0 0   0 0 1  cos θ + cos 2θ + cos 3θ = cos α ,then θ is equal to
a. α / 2 b. α c. 2α d. α / 6
0 1 0 1 0 0 
c.  0 0 1  d.  0 0 1   5 7 − 9i 
    14. Find imaginary part of sin −1  
 1 0 0   0 1 0 
 16 
a. log 2 b. − log 2
6. The sum of the first five terms of the series
1 3 c. 0 d. None of these
3 + 4 + 6 + ...... will be
2 4
15. An observer on the top of a tree, finds the angle of
9 3 depression of a car moving towards the tree to be 30°.
a. 39 b. 18
16 16 After 3 minutes this angle becomes 60°. After how much
7 9 more time, the car will reach the tree
c. 39 d. 13
16 16 a. 4 min b. 4.5 min c. 1.5 min d. 2 min
156 Mathematics
 x + x − 16 x + 20
3 2 23. The vertex of an equilateral triangle is (2,–1) and the
 ,if x ≠ 2 .
16. Let f ( x) =  ( x − 2)2 If f ( x) be equation of its base in x + 2 y = 1. The length of its sides
 k , if x = 2 is
continuous for all x, then k = a. 4 / 15 b. 2 / 15
a. 7 b. –7 c. 4 / 3 3 d. 1/ 5
c. ± 7 d. None of these
K ( x + 1) 2 ( y + 2) 2
x + e x +.... ∞ dy 24. If the equation + = 1 represents a circle,
17. If y = e x + e , then = 3 4
dx then K =
y 1 a. 3/4 b. 1 c. 4/3 d. 12
a. b.
1− y 1− y 25. The equation of the common tangent to the curves
y y y 2 = 8 x and xy = −1 is
c. d.
1+ y y −1 a. 3 y = 9 x + 2 b. y = 2 x + 1
c. 2 y = x + 8 d. y = x + 2
18. One maximum point of sin p x cos q x is
r r
a. x = tan −1 ( p / q ) b. x = tan −1 ( q / p ) 26. The vectors a and b are non-collinear. The value of x for
c. x = tan −1 ( p / q ) d. x = tan −1 ( q / p ) r r r r r r
which the vectors c = ( x − 2) a + b and d = (2 x + 1) a − b

1 are collinear, is
19. ∫ x(log x) 2
dx =
a. 1 b.
1
2
1 1
a. +c b. − +c 1
log x log x c. d. None of these
3
c. log log x + c d. − log log x + c
r r r r r r r
27. If vectors a , b , c satisfy the condition | a − c | = | b − c |,
20. Area bounded by the parabola y = 2 x and the ordinates 2
r
r r  r ar + b 
x = 1, x = 4 is then (b − a ) ⋅  c −  is equal to
 2 
a. 4 2 sq. unit b. 28 2 sq.unit a. 0 b. –1 c. 1 d. 2
3 3
56 x +1 y − 2 z + 3
c. sq. unit d. None of these 28. The angle between the straight lines = =
3 2 5 4
x −1 y + 2 z − 3
dy 1 + x 2 and = = is
21. The solution of the differential equation + = 0 is 1 2 −3
dx x
a. 45° b. 30° c. 60° d. 90°
1 x2
a. y = − tan −1 x + c b. y + log x + +c =0 29. A variable plane is at a constant distance p from the origin
2 2
and meets the axes in A, B and C. The locus of the
1 x2
c. y = tan −1 x + c d. y − log x − =c centroid of the tetrahedron OABC is
2 2
a. x −2 + y −2 + z −2 = 16 p −2 b. x −2 + y −2 + z −2 = 16 p −1
22. The solution of the
differential equation c. x −2 + y −2 + z −2 = 16 d. None of these
cos y log(sec x + tan x ) dx = cos x log (sec y + tan y )dy is
30. ~ ( p ∨ q ) is equal to
a. sec2 x + sec2 y = c b. sec x + sec y = c
a. ~ p ∨ ~ q b. ~ p ∧ ~ q
c. sec x − sec y = c d. None of these
c. ~ p ∨ q d. p ∨ ~ q
Space for rough work
Mock Test-2 157
JEE ADVANCE PAPER-I

SECTION 1 Contains 8 Questions. SECTION 2 Contains 10 Multiple Choice Questions


The answer to each question is a single digit integer ranging from 0 to With one or more than one correct option
9 (both inclusive).
9. Let f be a non-negative function defined on the interval
9 x x
1. The coefficient of x in the expansion of
(1 + x ) (1 + x 2 ) (1 + x 3 ).....(1 + x100 ) is
[0, 1]. If ∫
0
1 − ( f ′(t )) 2 dt = ∫ f (t ) dt , 0 ≤ x ≤ 1, and
0

2. The minimum value of the sum of real numbers f (0) = 0, then

a −5 , a −4 , 3a −3 , 1, a 8 and a with a > 0 is


10 1 1 1 1
a. f   < and f   >
2 2 3 3
3. The value of
1 1 1 1
b. f   > and f   >
   
2 2 3 3
1 1 1 1
6 + log 3  4− 4− 4−  is

2 3 2 3 2 3 2 3 2  1 1 1 1
  c. f   < and f   <
2 2 3 3
4. The number of distinct solutions of the equation 1 1 1 1
d. f   > and f   <
5 2 2 3 3
cos 2 2 x + cos 4 x + sin 4 x + cos6 x + sin 6 x = 2 in the
4
interval [0, 2π ] is 10. Let α and β be the roots of x 2 − 6 x − 2 = 0, with α > β . If
a10 − 2 a8
5. A cylindrical container is to be made from certain solid an = α n − β n for n ≥ 1, then the value of is
2 a9
material with the following constraints: It has a fixed
a. 1 b. 2
inner volume of V mm3 , has a 2 mm thick solid wall and
c. 3 d. 4
is open at the top. The bottom of the container is a solid
15
circular disc of thickness 2 mm and is of radius equal to 11. Let z = cosθ + i sin θ . Then the value of ∑ Im( z
m =1
2 m −1
) at
the outer radius of the container.
If the volume of the material used to make the container is
θ = 2° is
minimum when the inner radius of the container is 10 1 1
a. b.
sin 2° 3 sin 2°
V
mm, then the value of is 1 1
250π c. d.
2 sin 2° 4 sin 2°
6. Let the curve C be the mirror image of the parabola
y 2 = 4 x with respect to the line x + y + 4 = 0. If A 1 2 2 
12. If A =  2 1 −2  is a matrix satisfying the equation
and B are the points of intersection of C with the line
y = −5, then the distance between A and B is  a 2 b 
AAT = 9 I , where I is 3× 3 identity matrix, then the
7. Let f : R → R be a continuous odd function, which vanishes
ordered pair (a, b) is equal to:
1 a. (2, −1) b. (−2,1)
exactly at one point and f (1) = . Suppose that
2
x x c. (2,1) d. ( −2, −1)
F ( x ) = ∫ f (t ) dt for all x ∈[1 − 1, 2] and G(x) = ∫ t | f ( f (t)) | dt
−1 −1
13. The sum of first 20 terms of the sequence
for all x ∈[−1, 2]. If lim F ( x) = 1 , then the value of f  1  is 0.7, 0.77, 0.777, ..... is
x →1 G ( x ) 14 2
7 7
8. Consider the set of eight vectors a. (179 − 10 −20 ) b. (99 − 10 −20 )
81 9
V = {aiˆ + bjˆ + ckˆ; a, b, c ∈{−1,1}}. Three non-coplanar vectors 7 7
c. (179 + 10 −20 ) d. (99 + 10 −20 )
can be chosen from V in 2 p ways. Then p is. 81 9
158 Mathematics

14. Coefficient of x in11


the expansion of (B) Let a and b be real 2. 2
(1 + x ) (1 + ) (1 + x )
2 4 3 7 4 12
is numbers such that the
function
a. 1051 b. 1106
c. 1113 d. 1120
15. The number of seven digit integers, with sum of the digits
f (x) = { −3ax2 −2, x <1
bx + a2 , x ≥1
is

differentiable for all


equal to 10 and formed by using the digits 1, 2 and 3 only,
x ∈ R. Then possible
is
value(s) of a is (are)
a. 55 b. 66
c. 77 d. 88 (C) Let ω ≠1 be a 3. 3
complex cube root of
16. The mean of the data set comprising of 16 observations is
unity. If
16. If one of the observation valued 16 is deleted and three
new observations values 3, 4 and 5 are added to the data, (3 − 3ω + 2ω 2 ) 4 n + 3
then the mean of the resultant data, is + (2 + 3ω − 3ω 2 ) 4 n + 3
a. 16.8 b. 16.0
+ ( −3 + 2ω + 3ω 2 ) 4 n + 3 = 0,
c. 15.8 d. 14.0
then possible value(s)
π
17. Let ABC be a triangle such that ∠ACB = and let a, b of n is (are)
6
(D) Let the harmonic mean 4. 4
and c denote the lengths of the sides opposite to A, B and
of two positive real
C respectively. The value (s) of x for which
numbers a and b be 4. If
a = x 2 + x + 1, b = x 2 − 1 and c = 2 x + 1 is (are)
q is a positive real
a. − (2 + 3) b. 1 + 3 number such that a, 5,
c. 2 + 3 d. 4 3 q, b is an arithmetic
1 progression, then the
18. Let f k ( x) = (sin k x + cos k x), where x ∈ R and k ≥ 1. value(s) of | q − a | is
k
Then f 4 ( x ) − f 6 ( x ) equals: (are)
1 1 5. 5
a. b.
6 3 20. Consider the following linear equations
1 1 ax + by + cz = 0
c. d.
4 12 bx + cy + az = 0
cx + ay + bz = 0
SECTION 3 Contains 2 Match The Following Type Questions Match the conditions / expressions in Column I with
You will have to match entries in Column I with the entries in statements in Column II
Column II. Column I Column II
19. Match the Column:
(A) a + b + c ≠ 0 and 1. the equations represent
Column I Column II planes meeting only at
a + b + c = ab + bc + ca
2 2 2
2
(A) In R , if the magnitude 1. 1 a single point.
of the projection vector (B) a + b + c = 0 and 2. the equations represent
of the vector αiˆ + β ˆj a + b + c ≠ ab + bc + ca
2 2 2 the line x = y = z .

on 3iˆ + ˆj is 3 and (C) a + b + c ≠ 0 and 3. the equations represent


a + b + c ≠ ab + bc + ca
2 2 2 identical planes.
if α | = 2 + 3β, then
(D) a + b + c = 0 and 4. the equations represent
possible value(s) of
a + b + c = ab + bc + ca
2 2 2 the whole of the three
| α | is (are)
dimensional space.
Space for rough work
Mock Test-2 159
JEE ADVANCE PAPER-II

SECTION 1 Contains 8 Questions. 7. Let f :IR → IR be defined as f(x) = | x | + | x 2 − 1| . The total


The answer to each question is a single digit integer ranging from 0 to number of points at which f attains either a local
9 (both inclusive). maximum or a local minimum is
1. The number of distinct real roots of 8. The total number of distinct x∈R for which
x − 4 x + 12 x + x − 1 = 0 is
4 3 2
x x 1+ x
2 3

2. The number of all possible values of θ , where 0 < θ < π , 2x 4x 2


1 + 8 x 3 = 0 is
for which the system of equations 3x 9 x 2 1 + 27 x 3
2cos3θ 2sin3θ
( y + z ) cos3θ = ( xyz )sin 3θ x sin3θ = +
y z SECTION 2 Contains 8 Multiple Choice Questions
( xyz )sin 3θ = ( y + 2 z ) cos3θ + y sin 3θ have a solution With one or more than one correct option
( x0 , y0 , z0 ) with y0 z0 ≠ 0, is 9. If X = {4 n − 3 n − 1 : n ∈ N } and Y = {9( n − 1) : n ∈ N },
where N is the set of natural numbers, then X ∪ Y is
x 2 sin( β x )
3. Let α , β ∈ R be such that lim = 1. Then equal to:
x → 0 α x − sin x
a. N b. Y – X c. X d. Y
6 (α + β ) equals
10. A value of b for which the equations x 2 + bx − 1 = 0
2

4. The centres of two circles C1 and C2 each of unit radius x + x + b = 0,


Have one root in common is
are at a distance of 6 units from each other. Let P be the
a. − 2 b. −i 3 c. i 5 d. 2
mid-point of the line segment joining the centres of C1
11. The variance of first 50 even natural numbers is:
and C2 and C be a circle touching circles C1 and C2 833 437
a. b. 833 c. 437 d.
externally. If a common tangent to C1 and C passing 4 4
tan A cot A
through P is also a common tangent to C2 and C, then the 12. The expression + can be written as
1 − cot A 1 − tan A
radius of the circle C is
r r r
5. Suppose that p, q and r are three non-coplanar vectors in a. sin A cos A + 1 b. sec A cosec A + 1
r r r r
R3. Let the components of a vector s along p, q and r be c. tan A + cot A d. sec A + cosec A
4, 3 and 5, respectively. If the components of this vector  23  n

r r r r r r r r r r 13. The value of cot  ∑ cot −1  1 + ∑ 2k   is
s along (− p + q + r ), ( p − q + r ) and (− p − q + r ) are x, y  k =1  
 n =1
and z, respectively, then the value of 2 x + y + z is 23 25
2 2
a. b.
x y 25 23
6. Suppose that the foci of the ellipse + = 1 are
9 5 23 24
c. d.
( f1 , 0) and ( f 2 , 0) where f1 > 0 and f 2 < 0. Let P1 and 24 23
14. A man is walking towards a vertical pillar in a straight
P2 be two parabolas with a common vertex at (0, 0) and
path, at a uniform speed. At a certain point A on the path,
with foci at ( f1 , 0) and (2 f 2 , 0), respectively. Let T1 be
he observes that the angle of elevation of the top of the
a tangent to P1 which passes through (2 f 2 , 0) and T2 be pillar is 30°. After walking for 10 minutes from A in the
a tangent to P2 which passes through ( f1 , 0). The m1 is same direction, at a point R, he observes that the angle of
the slope of T1 and m2 is the slope of T2 , then the value elevation of the top of the pillar is 60°. Then the time
taken (in minutes) by him, from B to reach the pillar, is:
 1 
of  2 + m22  is a. 6 b. 10
m 
c. 20 d. 5
160 Mathematics
(1 − cos 2 x) (3 + cos x ) 18. If 2 balls are drawn (without replacement) from a
15. lim is equal to
x →0 x tan 4 x randomly selected box and one of the balls is white and
1 1 the other ball is red, the probability that these 2 balls are
a. − b. c. 1 d. 2 drawn from box B2 is
4 2
 π π 116 126 65 55
16. Let the function g : (−∞, ∞) →  − ,  be given by a. b. c. d.
 2 2 181 181 181 181
π
g (u ) = 2 tan −1 (eu ) − . Paragraph for Question No. 19 to 20
2 Suppose we define the definite integral using the following
Then, g is
d −a
b
formula ∫ f ( x )dx = ( f (a ) + f (b)), for more accurate
a. even and is strictly increasing in (0, ∞ ) 2
a

b. odd and is strictly decreasing in ( −∞ , ∞ ) c−a b−c


result for c ∈ ( a, b) F (c) = ( f ( a ) + f (c)) +
c. odd and is strictly increasing in ( −∞ , ∞ ) 2 2
d. neither even nor odd, but is strictly increasing in a+b
( f (b) + f (c)). When c = ,
( −∞ , ∞ ) 2
b−a
b

∫ f ( x)dx = 4
( f (a) + f (b) + 2 f (c)).
SECTION 3 Contains 2 Paragraph Type Questions a
π /2
Each paragraph describes an experiment, a situation or a problem.
Two multiple choice questions will be asked based on this paragraph.
19. ∫ sin x dx
0
is equal to
One or more than one option can be correct.
π π
a. (1 + 2) b. (1 + 2)
Paragraph for Question No. 17 to 18 8 4
A box B1 contains 1 white ball, 3 red balls and 2 black balls. π π
c. d.
Another box B2 contains 2 white balls, 3 red balls and 4 black 8 2 4 2
balls. A third box B3 contains 3 white balls, 4 red balls and 5 20. If f ′′( x ) < 0 ∀x ∈ (a, b) and c is a point such that
black balls. a < c < b , and (c, f (c )) is the point lying on the curve for
17. If 1 ball is drawn from each of the boxes B1 , B2 and B3 , which F ( c ) is maximum, then f ′( c ) is equal to
the probability that all 3 drawn balls are of the same
f (b ) − f ( a ) 2( f (b) − f (a ))
colour is a. b.
b−a b−a
82 90 558 566 2 f (b ) − f ( a )
a. b. c. d. c. d. 0
648 648 648 648 2b − a

Space for rough work


Mock Test-2 161
ANSWER & SOLUTIONS 5. (d) It is understandable.
JEE-Main 1 3 9 27
6. (a) Given series is 3 + 4 + 6 + ........ = 3 + + + .....
1. 2. 3. 4. 5. 6. 7. 8. 9. 10.
2 4 2 4
a d c d b a d d b b 32 33 34 35
= 3+ + + + + ..... (in G.P.)
11. 12. 13. 14. 15. 16. 17. 18. 19. 20. 2 4 8 16
a c a b c a a a b b 3
21. 22. 23. 24. 25. 26. 27. 28. 29. 30.
Here a = 3, r = , then sum of the five terms
2
b d b a d c a d a b
 3  5   5 
3   − 1 1  3 − 1
1. (a) Use the identity f ( f − 1( x)) = x a(r n − 1)  2 
S5 = =   =  32 
replace x by f − 1( x), in the given function we get r −1 3 1
−1
1 2 2
f ( f − 1( x )) = f − 1( x) + −1
f ( x)  243 − 32  211× 3 633 9
= 6  = = = 39 .
 32  16 16 16
1
⇒ x = f − 1( x) + −1
, solve to find f − 1x.
f ( x) 7. (d) (1 + x − 2 x 2 ) 6 = 1 + a1 x + a2 x 2 + .... + a12 x12 .
Putting x = 1 and x = −1 and adding the results
2. (d) Given x2 + x + 1 = 0
64 = 2(1 + a2 + a4 + ...)
1 1 1
∴ x = [−1 ± i 3] = (−1 + i 3), (−1 − i 3) = ω, ω 2 ∴ a2 + a4 + a6 + .... + a12 = 31.
2 2 2
But α 19 = ω19 = ω and β 7 = ω 14 = ω 2 . Σn n(n + 1)
8. (d) Tn = =
Hence the equation will be same. n! 2 (n)!
2π   2π  1  (n + 1)  1  n − 1 2 
3. (c) ω n = cos   + i sin   =  =  + 
 n   n  2  (n − 1)! 2  (n − 1)! (n − 1)!
2π 2π 1 i 3 1 1 2  (e + 2e) 3e
⇒ ω3 = cos + i sin =− + =ω =  + = = .
3 3 2 2 2  (n − 2)! (n − 1)! 2 2
2
 2π 2π  4π 4π 9. (b) The given series reduces to
and ω32 =  cos + i sin  = cos + i sin
 3 3  3 3 3 4  n 
log e 2 + log e   + log e   + K + log e  
1 i 3 2 3  n −1 
=− − = ω2.
2 2 = log e 2 + log e 3 − log e 2 + log e 4 − log e 3 + K
∴ ( x + yω3 + zω ) ( x + yω + zω3 )
2
3
2
3 + log e (n) − log e (n − 1) = log e n.
= ( x + yω + zω 2 ) ( x + yω 2 + zω )
10. (b) 210 − 1 = 1023, corresponds to none of the lamps is
= x + y + z − xy − yz − zx.
2 2 2
being switched on.
6
 2kπ 2kπ  11. (a) The event that the fifth toss results in a head is
4. (d) ∑  sin
k =1 7
− i cos
7 

independent the event that the first four tosses result in tails.
 6 i 27kπ  ∴ Probability of the required event = ½
6
 2 kπ 2kπ   
= ∑ −i  cos + i sin  = −i ∑ e  1 1 3.2 x
k =1  7 7   k =1  12. (c) x. . = ⇒ = 2 ⇒ x=8.
2 4 2.3 4 2
= −i{ei 2π / 7 + ei 4π / 7 + ei 6π / 7 + ei 8π / 7 + ei10π / 7 + ei12π / 7 }
13. (a) sin θ + sin 3θ + sin 2θ = sin α
 (1 − ei12π / 7 )   ei 2π / 7 − ei14π / 7 ) 
= −i  e i 2 π / 7  = −i   ⇒ 2sin 2θ cos θ + sin 2θ = sin α
 1 − ei 2π / 7   1− e
i 2π / 7
 ⇒ sin 2θ (2 cos θ + 1) = sin α . . .(i)
i 2π / 7
e − 1 Now, cos θ + cos 3θ + cos 2θ = cos α
(Q ei14π / 7 = 1) = −i  i 2π / 7 
=i
1 − e  2 cos 2θ cos θ + cos 2θ = cos α
162 Mathematics
⇒ cos 2θ (2cos θ + 1) = cos α . . .(ii) 1
19. (b) Put log x = t ⇒ dx = dt , then
From (i) and (ii), x
⇒ tan 2θ = tan α ⇒ 2θ = α ⇒ θ = α / 2 . 1 1 1 1
∫ x(log x)2
dx = ∫
t 2
dt = − + c = −
t log x
+ c.
  5 7 9 i 
14. (b) Imaginary part of sin −1  −
 16 16  
    20. (b) Y x =1 x=4

 9 y =
9 2x
= − log  + 1 +  = − log(2). D C
 16 16 
O A B X
15. (c) D′
C′
h
Required area = CDD ' C ' = 2 × ABCD
60° 30°
4 28 2
d = 2∫ 2 x 1/ 2 dx = sq. unit.
1 3
d = h cot 30 ° − h cot 60 ° and time = 3 min.
h (cot 30 o − cot 60 o ) dy 1 + x 2 1 
∴ Speed = per minute 21. (b) + = 0 ⇒ dy +  + x  dx = 0
3 dx x x 
It will travel distance h cot 60° in x2
On integrating, we get y + log x + + c = 0.
h cot 60 × 3 o
2
= 1.5 minute.
h(cot 30o − cot 60 o )
22. (d) cos y log(sec x + tan x)dx = cos x log(sec y + tan y)dy
16. (a) For continuous lim f ( x ) = f (2) = k
x→2
⇒ ∫ sec y log(sec y + tan y ) dy

x 3 + x 2 − 16 x + 20 = ∫ sec x log(sec x + tan x) dx


⇒ k = lim
x→2 ( x − 2) 2
Put log(sec x + tan x) = t and log(sec y + tan y ) = z
( x 2 − 4 x + 4) ( x + 5)
= lim = 7.
x→2 ( x − 2) 2 [log(sec x + tan x)]2 [log(sec y + tan y )]2
= + c.
2 2
17. (a) y = e x + y ⇒ log y = ( x + y ) log e
A (2, –1)
1 dy dy dy y 23. (b)
⇒ = 1+ ⇒ = .
y dx dx dx 1 − y

18. (a) Let y = sin p x.cos q x


dy 60o
= p sin p −1 x.cos x.cos q x + q cos q −1 x.(− sin x)sin p x C
dx D
x + 2y –1 = 0
dy
= p sin p −1 x.cos q +1 x − q cos q −1 x.sin p +1 x 2 − 2 −1 1
dx | AD | = =
12 + 2 2 5
dy
Put = 0, AD
dx ⇒ tan 60° =
p BD
∴ tan 2 x =
q 1/ 5
⇒ 3=
BD
p
⇒ tan x = ± 1
q ⇒ BD =
15
p
∴ Point of maxima x = tan −1 . ⇒ BC = 2 BD = 2 / 15 .
q
Mock Test-2 163
24. (a) It represents a circle, if a = b JEE Advance Paper-I
k 1 3
⇒ = ⇒ k= . 1. 2. 3. 4. 5. 6. 7. 8. 9. 10.
3 4 4 8 8 4 8 4 4 7 5 c c
25. (d) Tangent to the curve y 2 = 8 x is y = mx + 2 / m. 11. 12. 13. 14. 15. 16. 17. 18. 19. 20.
So, it must satisfy xy = −1. d d c c c d b d c a

 2 2
⇒ x  mx +  = −1⇒ mx 2 + x + 1 = 0 1. (8) x9 can be formed in 8 ways
 m m
i.e., x 9 , x1+8 , x 2+ 7 , x 3+ 6 , x 4+5 , x1+ 2+ 6 , x1+3+5 , x 2+3+ 4
Since, it has equal roots.
D=0 z and coefficient in each case is 1

⇒ Coefficient of x9 = 1 + 1 + 1 + .........+ 1 = 8
4 8 times
⇒ − 4m = 0 ⇒ m3 = 1
m2
a −5 + a −4 + 3( a −3 ) + 1 + a 8 + a10
≥ ( a −5 a −4 ( a −3 ) 3 (1) ( a 8 )( a10 ) ) = 1
1/8

⇒ m = 1 Hence, equation of common tangent is y = x + 2. 2. (8)


1+1+ 3 +1+1+1
r r r r r r (using AM ≥ GM)
26. (c) Since c = ( x − 2) a + b and d = (2 x + 1) a − b are
r r ∴ a −5 + a −4 + 3a −3 + 1 + a8 + a10 ≥ 8
collinear, therefore c = λ d
r r r r
⇒ ( x − 2) a + b = λ (2 x + 1) a − λ b 1 1 1 1
r 3. (4) 6 + log3/ 2 4− 4− 4− ...
r 3 2 3 2 3 2 3 2
or [( x − 2) − λ (2 x + 1)] a + (λ + 1)b = 0
( x − 2) − λ (2 x + 1) = 0, λ + 1 = 0 1 1
r r put x = 4 − 4− ...
(Q a , b are linearly independent) 3 2 3 2
⇒ x − 2 + 2x +1 = 0 x
x2 = 4 −
1 3 2
⇒ x= .
3 3 2 x 2 = 12 2 − x
r r r r
r r
  r  3 2 x 2 + x − 12 2 = 0
27. (a) (b − ar ) ⋅  cr − a + b  = b ⋅ cr − b ⋅  a + b  − ar ⋅ cr + a (ar + b )
r r
 2   2  2
4 2
r r r r ⇒ x= ; x = − 3/ 2 not possible
and | a − c | = | b − c | 3
r r r r
⇒ | a − c |2 = | b − c |2  1 4 2
⇒ 6 + log 3/ 2  × 

r r r 3 2 3 
a + b = 2c
r  4
r r  r ar + b  ⇒ 6 + log3/ 2   ⇒ 6 − 2 = 4
Therefore, (b − a ) ⋅  c − =0 9
 2 
5
(2 + 10 − 12) 4. (8) cos 2 2 x + cos 4 x + sin 4 x + cos 6 x + sin 6 x = 2
28. (d) θ = cos −1 = cos −1 (0) 4
4 + 25 + 16 1 + 4 + 9
5 2
⇒ θ = 90° ⇒ cos 2 x − 5cos2 x sin 2 x = 0
4
x y z 1
29. (a)Plane is + + = 1 , where p = ⇒ tan 2 2 x = 1, where 2 x ∈ [0, 4π ]
a b c Σ (1/ a )2
Number of solutions = 8
1 1 1 1
or 2
+ 2+ 2 = 2 . . .(i) 5. (4) Let inner radius be r and inner length be l
a b c p
π r 2l = V
a b c
Now according to equation, x = , y = , z = Volume of material be M
4 4 4
M = π (r + 2)2 (l + 2) − π r 2 l
Put the values of x, y, z in (i), we get the locus of the
centroid of the tetrahedron. dM 4V 8V
= − 2 − 3 + 8π + 0 + 4π r
30. (a) ~ ( p ∨ q) ≡ ~ p ∧ ~ q . dr r r
164 Mathematics
dM a10 − 2a8 (α − β ) − 2(α − β )
10 10 8 8
= 0 when r = 10 ⇒ =
dr 2a9 2(α 9 − β 9 )
⇒ V = 1000π α 8 (α 2 − 2) − β 8 ( β 2 − 2)
=

V
=4 2(α 9 − β 9 )
250π
α 8 .(6α ) − β 8 (6 β )
=
6. (4) Image of y = −5 2(α 9 − β 9 )
about the line x + y + 4 = 0 is x = 1 6 (α 9 − β 9 )
− =3
⇒ Distance AB = 4 2 2(α 9 − β 9 )

7.
1
(7) G (1) = ∫ t | f ( f (t )) | dt = 0 11. (d) X = sin θ + sin 3θ + ... + sin 29θ
−1
2(sin θ ) X = 1 − cos 2θ + cos 2θ − cos 4θ + ... + cos 28θ − cos30θ
⇒ f (− x ) = − f ( x )
1 − cos 30θ 1
1 ⇒ X = =
Given f (1) = 2sin θ 4 sin 2°
2
F ( x) − F (1) 12. (d) AAT = 9I
F ( x) x −1 f (1) 1
⇒ lim = lim = = 1 2 2  1 2 a 
x →1 G ( x ) x →1 G ( x ) − G (1) | f ( f (1)) | 14
  
x −1  2 1 −2   2 1 2  = 9 I
1/ 2 1 1  a 2 b   2 −2 b 
⇒ = ⇒ f   = 7.
| f (1/ 2) | 14 2  9 0 a + 4 + 2b  9 0 0 
⇒  2a + 2 − 2b  ⇒  0 9 0 
8. (5) Let (1,1,1), ( −1,1,1), (1, −1,1), ( −1, −1, −1) be vectors  0 9
r r r r r r r r  a + 4 + 2b 2a + 2 − 2b a 2 + 4 + b 2   0 0 9 
a , b , c , d rest of the vectors are −a, −b , −c , −d and let us
find the number of ways of selecting co-planar vectors. Equation a + 4 + 2b = 0
Observe that out of any 3 coplanar vectors two will be ⇒ a + 2b = −4 . . .(i)
collinear (anti parallel) and 2a + 2 − 2b = 0
Number of ways of selecting the anti parallel pair = 4
⇒ 2a − 2b = −2 . . .(ii)
Number of ways of selecting the third vector = 6
Total = 24 a2 + 4 + b2 = 0
Number of non co-planar selections ⇒ a2 + b2 = 5 . . .(iii)
= C3 − 24 = 32 = 2 , p = 5
8 5
Solving a = −2, b = −1
Alternate
8× 6× 4 13. (c) 0.7 + 0.77 + 0.777 + ..... + 0.777...7
Required value =
3! 7
= [0.9 + 0.99 + 0.999 + ... + 0.999...9]
∴ p=5 9
7
= [(1 − 0.1) + (1 − 0.01) + (1 − 0.001...1) + ... + (1 − 0.000...1)]
9. (c) f ′ = ± 1 − f 2
9

⇒ f ( x) = sin x or f ( x) = − sin x (not possible) 7  1 1 1 1 


=  20 −  + 2 + 3 + ... + 20 

9  10 10 10 10 
f ( x) = sin x
Also, x > sin x ∀ x > 0  1 
1 − 20  7
7 1 10 1  1020 − 1  
=  20 − .  =  20 − .  
10. (c) α is a roots of equation 9 10 1 − 1  9  9  1020  
 10 
α 2 − 6α − 2 = 0; β 2 − 6 B − 2 = 0
7   1  7 −20
α 2 − 6α − 2 = 0 180 −  1 − 20   = [179 + 10 ]
81   10   81
⇒ α 2 − 2 = 6α
Mock Test-2 165
14. (c) 2 x1 + 3 x2 + 4 x3 = 11 − x <1
(B) f ( x ) =  3ax −2 2,
2

Possibilities are (0, 1, 2); (1, 3, 0); (2, 1, 1); (4, 1, 0).  bx + a , x ≥ 1
∴ Required coefficients For continuity −3a − 2 = b + a 2
= ( 4 C0 ×7 C1 ×12 C2 ) + ( 4 C1 ×7 C3 ×12 C0 ) + ( 4 C2 ×7 C1 ×12 C1 ) + ( 4 C4 ×7 C1 × 1)
⇒ a 2 + 3a + 2 = −b . . .(i)
= (1× 7 × 66) + (4 × 35 × 1) + (6 × 7 × 12) + (1× 7)
For differentiability −6a = b
= 462 + 140 + 504 + 4 = 113.
⇒ 6 a = −b
15. (c) Coefficient of x10 in ( x + x 2 + x 3 )7
⇒ a 2 − 3a + 2 = 0
Coefficient of x in (1 + x + x )
3 2 7
⇒ a = 1, 2
Coefficient of x3 in (C) (3 − 3ω + 2ω 2 )4 n + 3 + (2 + 3ω − 3ω 2 )4 n + 3 + (−3 + 2ω + 3ω 2 ) 4n +3 = 0
−7 7 + 3+ 7
(1 − x ) (1 − x)
3 7
= C3 − 7 = C3 − 7
9
(3 − 3ω + 2ω 2 ) 4 n + 3 + (ω (2ω 2 + 3 − 3ω )) 4 n + 3 + (ω 2 (−3ω + 2ω 2 + 3) 4 n + 3 = 0

9 ×8× 7 ⇒ (3 − 3ω + 2ω 2 ) 4 n + 3 + (1 + ω 4 n + ω 8 n ) = 0
= − 7 = 77
6 ⇒ n ≠ 3k , k ∈ N
Σxi
16. (d) = 16 ⇒ Σxi = 256 (D) Let a = 5 − d
16
q = 5+d
(Σxi ) − 16 + 3 + 4 + 5 252
= = 14 b = 5 + 2d
18 18
17. (b) Using cosine rule for ∠C | q − a | = | 2d |

3 ( x 2 + x + 1) 2 + ( x 2 − 1) 2 − (2 x + 1) 2 2ab
= Given =4
2 2( x 2 + x + 1)( x 2 − 1) a+b
ab
2x2 + 2 x − 1 ⇒ =2
⇒ 3= 2 a+b
x + x +1
⇒ (5 − d )(5 + 2 d ) = 2(5 − d + 5 + 2d ) = 2(10 + d )
⇒ ( 3 − 2) x 2 + ( 3 − 2) x + ( 3 + 1) = 0
⇒ 25 + 10d − 5d − 2d 2 = 20 + 2d
(2 − 3) ± 3
⇒ x= ⇒ 2d 2 − 3d − 5 = 0
2( 3 − 2)
5
⇒ x = − (2 + 3),1 + 3 ⇒ x = 1 + 3 as ( x > 0). ⇒ d = −1, d =
2
1 1
18. (d) f k = (sin k x + cos k x) f 6 ( x) = (sin 6 x + cos6 x) ⇒ | 2 d | = 2,5
4 6
1 a b c
f 4 ( x) = (sin 4 x + cos 4 x)
4 20. ∆ = b c a = − 1 (a + b + c)[( a − b) 2 + (b − c) 2 + (c − a )2 ]
2
1 3  c a b
f 6 K = 1 − sin 2 2 x 
6 4 
(A) If a + b + c ≠ 0
1  sin 2 2 x 
f 4 ( x ) = 1 =  and a 2 + b 2 + c 2 = ab + bc + ca
4 2 
⇒ ∆ = 0 and a = b = c ≠ 0
 1 sin 2 2 x   1 sin 2 2 x  1 1 1
f 4 ( x) − f 6 ( x) =  − − − = − = ⇒ the equations represent identical planes.
4 8  6 8  4 6 12
(B) a + b + c = 0 and a 2 + b 2 + c 2 ≠ ab + bc + ca
3α + β ⇒ ∆=0
19. (A) = 3
2 ⇒ the equations have infinitely many solutions.
3α + β = ± 2 3 . . .(i) ax + by + ( a + b) z

Given α = 2 + 3β . . .(ii) bx + cy = (b + c) z

From equation (i) and (ii), we get α = 2 or − 1 ⇒ (b 2 − ac ) y = (b 2 − ac ) z ⇒ y=z

So |α| = 1 or 2 ⇒ ax + by + cy = 0 ⇒ ax = ay ⇒ x = y = z
166 Mathematics
(C) a + b + c ≠ 0 2 2 1
4. (8) cos α = sin α =
and a + b + c ≠ ab + bc + ca
2 2 2
3 3
⇒ ∆≠0 C1

⇒ the equation represent planes meeting at only one point. 2 2 C2


A P α 1
(D) a + b + c = 0 α
3 B
and a + b + c = ab + bc + ca
2 2 2

R α
⇒ a =b=c =0
⇒ the equation represent whole of the three dimensional space.
C

JEE Advance Paper -II


2 2
1. 2. 3. 4. 5. 6. 7. 8. 9. 10. tan α =
R
2 3 7 8 9 4 5 2 d b
2 2
11. 12. 13. 14. 15. 16. 17. 18. 19. 20. ⇒ R= = 8 units.
tan α
b b b d d c a d a a
r r r r
5. (9) s = 4 p + 3q + 5r
1. (2) Let f ( x ) = x 4 − 4 x 3 + 12 x 2 + x − 1 = 0 r r r r r r r r r r
s = x(− p + q + r ) + y ( p − q + r ) + z (− p − q + r )
f ′( x ) = 4 x 3 − 12 x 2 + 24 x + 1 = 4( x 3 − 3 x 2 + 6 x ) + 1 r r r r
s = (− x + y − z ) p + ( x − y − z )q + ( x + y + z )r
f ′′( x ) = 12 x 2 − 24 x + 24 = 12( x 2 − 2 x + 2)
⇒ −x + y − z = 4 ⇒ x − y − z = 3 ⇒ x + y + z = 5
f ′′( x ) has 0 rl roots f(x)has maximum 2 distinct real roots
9 7
On solving we get x = 4, y = ,z=−
as f (0) = −1. 2 2
2. (3) ( y + z ) cos 3θ − ( xyz )sin 3θ = 0 . . .(i) ⇒ 2x + y + z = 9

xyz sin 3θ = (2cos3θ ) z + (2sin 3θ ) y . . .(ii) 6. (4) The equation of P1 is y 2 − 8x = 0 and P2 is


∴ ( y + z ) cos 3θ = (2 cos 3θ ) z + (2sin 3θ ) y = ( y + 2 z ) cos 3θ + y sin 3θ y y 2 + 16x = 0
(cos 3θ − 2sin 3θ ) = z cos 3θ and y (sin θ − cos3θ ) = 0
Tangent to y 2 − 5x = 0 passes through (–4, 0)
0 ⇒ sin 3θ − cos 3θ = 0 ⇒

2
⇒ sin 3θ = cos 3θ ⇒ 0 = m1 (−4) +
m1
∴ 3θ = nπ + π / 4
1
⇒ =2
 β x β x 3 3

5 5 m12
x2  β x − + − ...... 
x sin β x
2
3! 5! Also tangent to y 2 + 16x = 0 passes through (2, 0)
3. (7) lim = lim  
x →0 α x − sin x x →0  3 5

x x
α x −  x − − − .......  ⇒ 0 = m2 × 2 −
4
 3! 5!  m2
 β 3 x2  ⇒ m22 = 2
x3  β − + ..... 
 3!  =1
= lim 1
x→0 x3 x5 ⇒ + m 22 = 4
(α − 1) x + − + ....... m12
3! 5!
⇒ α −1 = 0 7. (5) fx = x + x 2 − 1
⇒ α = 1,
 x 2 − x − 1 if x ≤ −1
Limit = 6 β = 1  2
− x − x + 1 if –1 ≤ x<0
1 = 2
⇒ β=  –x + x + 1 if 0 ≤ x < 1
6
 x 2 + x –1 if x ≥1
 1 7
⇒ 6(α + β ) = 6  1 +  = 6 × = 7
 6 6 So, total no. of local maxima and local minima is = 5
Mock Test-2 167

1 1 1+ x 3 x 1
14. (d) tan 30° = =
8. (2) x ⋅ x 0 2 6 x3 − 1 = 0
2 y+z 3
0 6 24 x3 − 2
⇒ x 3 (12 x 3 + 2) = 0
x
⇒ 6 x 6 + x3 − 5 = 0
⇒ 6 x 6 + 6 x 3 − 5 x3 − 5 = 0
60 30
⇒ (6 x 3 − 5)( x 3 + 1) = 0
y B z A
5
⇒ x = − 1, x =
3 3
⇒ 3x = y + z
6
5
1/ 3 x
⇒ x = −1, x =   ⇒ tan 60° = = 3
6 y
So, two solution ⇒ x = 3y = y + z
9. (d) x = 4n − 3n − 1, n ∈ N 3y = y + z
x = (1 + 3) n − 3n − 1, n ∈ N ⇒ 2y = z
⇒ X = 0,9,54, ... y = 9(n − 1), n ∈ N for 2y distance time
⇒ y = 0,9,18, ... ⇒ x ∪ y = y. = 10 min.
So for y dist time = 5 min.
10. (b) x 2 + bx − 1 = 0
x2 + x + b = 0 . . .(i) (1 − cos 2 x) (3 + cos x )
15. (d) lim
x →0 x tan 4 x
Common root is (b − 1)x − 1 − b = 0
b +1 (2sin 2 x )(3 + cos x)
⇒ x= This value of x satisfies equation (i) = lim
b −1 x →0  tan 4 x 
x  × 4x
(b + 1)2 b + 1  4x 
⇒ + +b =0
(b −1)2 b −1 2sin 2 x (3 + cos x ) 2
= lim = (3 + 1) = 2
x →0 4x2 4
⇒ b = 3i, − 3i, 0.
π
16. (c) g (u ) = 2 tan −1 (eu ) −
11. (b) Variance =
∑ x − ( x) 2
1
2
2
n = 2 tan −1 e u − tan −1 e u − cot −1 e u = tan −1 e u − cot − 1 e u
2
 2 + 4 + 6 + ... + 100   2 + 4 + ... + 100 
2 2 2 2
g ( − x) = − g ( x )
⇒ σ2 − 
 50   50 
⇒ g ( x) is odd and g ′( x ) > 0
⇒ σ 2 = 3434 − 2601 = 833
⇒ increasing.
 2 1 
2
12. (b) Exp. = tan A + 1
=
1
tan A −
tan A − 1 tan A − tan 2 A tan A − 1  tan A  17. (a) P (required) = P (all are white) + P (all are red) + P
(all are black)
tan 2 A + tan A + 1
= = tan A + cot A + 1 = sec A. cosec A + 1
tan A 1 2 3 3 3 4 2 4 5 5
= × × + × × + × × ×
 23
 6 9 12 6 9 12 6 9 12 12
13. (b) cot  ∑ cot −1 ( n 2 + n + 1) 
 n =1  6 36 40 82
= + + = .
648 648 648 648
 23  n +1− n  
cot  ∑ tan −1  
 n =1  1 + n(n + 1)   18. (d) Let A : one ball is white and other is red
  23   25 E1 : Both balls are from box B1
⇒ cot  tan −1   = .
  325   23 E2 : Both balls are from box B2
Mock Test-3 169
JEE-MAIN: MATHEMATICS MOCK TEST -3

 −1 x < 0 1 7
 a. b.
1. Let g ( x) = 1 + x − [ x] and f ( x) =  0 x = 0. Then for all 2 15
 1 x >1
 c.
2
d.
1
x, f {g ( x)} is equal to: 15 3

a. x b. 1 c. f ( x) d. g ( x ) 3π 1 − cos α 1 + cos α
10. If π < α < , then + =
2 1 + cos α 1 − cos α
2. If x = 6 + 6 + 6 + ....to ∞ , then 2 2
a. b. −
a. x is an irrational number b. 2 < x < 3 sin α sin α
c. x = 3 d. None of these 1 1
c. d. −
sin α sin α
3. If z + z −1 = 1, then z100 + z −100 is equal to
a. i b. – I c. 1 d. – 1  3 i
11. Find real part of cos −1 
 2 + 2 
 2 −3  
4. The inverse of   is
 −4 2  a. π / 3 b. π / 4

−1  2 3  −1  3 2   3 −1 
c. log 
 2 
a. b. d. None of these
8  4 2  8  2 4   
1  2 3 1  3 2 12. A house of height 100 metres subtends a right angle at the
c. d.
8  4 2  8  2 4  window of an opposite house. If the height of the window
be 64 metres, then the distance between the two houses is
5. The sum of 100 terms of the series .9 + .09 + .009......... ill be
100 100
a. 48 m b. 36 m c. 54 m d. 72 m
 1  1
a. 1 −   b. 1 +    sin x
 10   10   + cos x, when x≠0
106 100
13. If f ( x) =  x then
 1  1  2, when x=0
c. 1 −   d. 1 +  
 10   10 
a. lim f ( x) ≠ 2 b. lim f ( x) = 0
x →0 + x →0 −
6. If n is an integer greater than 1, then a − C1 ( a − 1) +n n
c. f ( x ) is continuous at x = 0 d. None of these
C2 (a − 2) +.... + ( −1) n (a − n) =
dy
a. a b. 0 14. If x y = e x − y , then =
dx
c. a 2 d. 2 n
a. log x.[log(ex)]−2 b. log x.[log(ex)]2
2 6 12 20
7. The sum of + + + +K is c. log x.(log x)2 d. None of these
1! 2! 3! 4!
3e 15. 20 is divided into two parts so that product of cube of one
a. b. e
2 quantity and square of the other quantity is maximum. The
c. 2e d. 3e parts are
a. 10, 10 b. 16, 4
8. If the letters of the word KRISNA are arranged in all
c. 8, 12 d. 12, 8
possible ways and these words are written out as in a
dictionary, then the rank of the word KRISNA is 1
a. 324 b. 341
16. ∫ x
tan 4 x sec 2 x dx =

c. 359 d. None of these 1 5


a. 2 tan 5 x + c b. tan x + c
9. Seven white balls and three black balls are randomly 5
placed in a row. The probability that no two black balls 2 5
c. tan x + c d. None of these
are placed adjacently, equals 5
170 Mathematics
17. The area bounded by the straight lines x = 0, x = 2 and the 24. ~ ( p ∧ q) is equal to
curves y = 2 , y = 2 x − x is
x 2
a. ~ p ∨ ~ q b. ~ p ∧ ~ q
4 1 3 4 c. ~ p ∧ q d. p ∧ ~ q
a. − b. +
3 log 2 log 2 3
25. The value of b and c for which the identity
4 3 4 f ( x + 1) − f ( x) = 8 x + 3 is satisfied, where
c. −1 d. −
log 2 log 2 3
f ( x) = bx 2 + cx + d , are
dy a. b = 2, c = 1 b. b = 4, c = −1
18. The solution of = e x (sin x + cos x ) is
dx
c. b = −1, c = 4 d. None of these.
a. y = e x (sin x − cos x) + c
26. If H is the harmonic mean between p and q, then the
b. y = e x (cos x − sin x) + c
H H
c. y = e x sin x + c value of + is
p q
d. y = e x cos x + c
pq
a. 2 b.
19. The product of the perpendiculars drawn from the points p+q
x y p+q
( ± a 2 − b 2 ,0) on the line cos θ + sin θ = 1 , is c. d. None of these
a b pq
2 2
a. a b. b
27. Six cards and six envelopes are numbered 1, 2, 3, 4, 5, 6
c. a + b 2 2
d. a 2 − b 2
and cards are to be placed in envelopes so that each
20. Area of the circle in which a chord of length 2 makes envelope contains exactly one card and no card is placed
π in the enveloped bearing the same number and moreover
an angle at the centre is
2 the card numbered 1 is always placed in envelope
π numbered 2. Then, the number of ways it can be done is
a. b. 2π
2 a. 264 b. 265
π c. 53 d. 67
c. π d.
4 28. ABCD is a rectangular field. A vertical lamp post of height
21. The focal chord to y = 16 x is
2
tangent to 12m stands at the corner A. If the angle of elevation of its
top from B is 60° and from C is 45°, then the area of the
( x − 6) 2 + y 2 = 2, then the possible values of the slope of
field is
this chord, are
a. 48 2 sq. m b. 48 3 sq. m
a. {−1,1} b. {−2, 2}
c. 48 sq. m d. 12 2 sq. m
c. {−2,1/ 2} d. {2, −1/ 2}
r r r r r r r 29. The area bounded by the curves y = x , 2 y + 3 = x and
22. If a , b , c are non-zero vectors such that a ⋅ b = a ⋅ c , then
x-axis in the 1st quadrant is:
which statement is true
r r r r r a. 9 sq unit
a. b = c b. a ⊥ (b − c ) b. 27/4 sq unit
r r r r r
c. b = c or a ⊥ (b − c ) d. None of these c. 36 sq unit
d. 18 sq unit
23. The point of intersection of the lines
30. If the line 2x + 6 y = 2 touches the hyperbola
x−5 y −7 z + 2 x +3 y −3 z −6
= = , = = is
x 2 − 2 y 2 = 4, then the point of contact is:
3 −1 1 −36 2 4
5 10
a. 21, , b. ( 2,10, 4) a. (−2, 6) b. (−5, 2 6)
3 3
1 1 
c. ( −3,3,6) d. (5,7, − 2) c.  ,  d. (4, − 6)
2 6
Mock Test-3 171
JEE ADVANCE PAPER-I

SECTION 1 Contains 8 Questions. 8. Let n ≥ 2 be an integer. Take n distinct points on a circle


The answer to each question is a single digit integer ranging from 0 to and join each pair of points by a line segment. Colour the
9 (both inclusive). line segment joining every pair of adjacent points by blue
and the rest by red. If the number of red and blue line
1. If z is any complex number satisfying | z − 3 − 2i |≤ 2, then
segments are equal, then the value of n is
the minimum value of | 2z − 6 + 5i | is
SECTION 2 Contains 10 Multiple Choice Questions
2. Let M be a 3 × 3 matrix satisfying
With one or more than one correct option
0   −1 1 1
M 1  =  2  , M  −1 =  1  ,
   
9. Let A and B be two sets containing four and two elements
0   3   0   −1 respectively. Then the number of subsets of the set A × B ,
1  0  each having at least three elements is:
and M 1 =  0  . a. 219 b. 256
1 12  c. 275 d. 510

Then the sum of the diagonal entries of M is 10. The real number k for which the equation,
2 x + 3 x + k = 0 has two distinct real roots in [0, 1]
3
3. Suppose that all the terms of an arithmetic progression
a. lies between 1 and 2 b. lies between 2 and 3
(A.P.) are natural numbers. If the ratio of the sum of the
c. lies between –1 and 0 d. does not exist
first seven terms to the sum of the first eleven terms is
6 : 11 and the seventh term lies in between 130 and 140, 11. If z is a complex number of unit modulus and argument
then the common difference of this A.P. is  1+ z 
θ , then arg   equals
4. A pack contains n cards numbered from 1 to n. Two  1+ z 
π
consecutive numbered cards are removed from the pack a. −θ b. −θ
and the sum of the numbers on the remaining cards is 2
1224. If the smaller of the numbers on the removed cards c. θ d. π − θ
is k, then k − 20 = _____
 5 a −b 
5. Consider a triangle ABC and let a, b and c denote the 12. If A =   and A adj A = A A , then 5a + b is equal
T

3 2
lengths of the sides opposite to vertices A, B and C
to:
respectively. Suppose a = 6, b = 10 and the area of the
a. –1 b. 5
triangle is 15 3. If ∠ACB is obtuse and if r denotes the c. 4 d. 13
radius of the in circle of the triangle, then r 2 is equal to
13. Three positive numbers form an increasing G.P. If the
6. Let m and n be two positive integers greater than 1. If
middle term in this G.P. is doubled, the new numbers are
 e cos(α ) − e 
n
e m
lim   = −   then the value of is in A.P. Then the common ratio of the G.P. is :
a →0  
 α
m
 2 n
a. 2+ 3 b. 3 + 2
7. Let S be the focus of the parabola y2 = 8x and let PQ be
c. 2 − 3 d. 2 + 3
the common chord of the circle x2 + y2 – 2x –4y = 0 and
the given parabola. The area of the triangle PQS is.
14. If the number of terms in the expansion of
Q n
 2 4
1 − + 2  , x ≠ 0, is 28, then the sum of coefficients
S  x x 
P (2,0) of all the terms in this expansion, is:
a. 64 b. 2187
c. 243 d. 729
172 Mathematics
15. Let Tn be the number of all possible triangles formed by SECTION 3 Contains 2 Match The Following Type Questions

joining vertices of a n-sides regular polygon. If You will have to match entries in Column I with the entries in
Column II.
Tn+1 − Tn = 1 then the value of n is
a. 7 b. 5 19. Match the statements given in Column I with the values
c. 10 d. 8 given in Column II.
16. A computer producing factory has only two plants T1 and Column I Column II
r ˆ
T2 . Plant T1 produces 20% and plant T2 produces 80% (A) If a = j + 3kˆ = −ˆj + 3kˆ 1. π
r 6
of the total computers produced. 7% of computers and c = 2 3kˆ form a
produced in the factory turn out to be defective. It is triangle, then the internal
known that P(computer turns out to be defective given angle of the triangle
that it is produced in plant T1 ) = 10 P (computer turns out r r
between a and b is
to be defective given that it is produced in Plant T2 ), b

∫ (f (x) − 3x)dx = a − b2 ,
2
where P(E) denotes the probability of an event E. A (B) If 2.
a
3
computer produced in the factory is randomly selected and
it does not turn out to be defective. Then the probability π 
then the value of f   is
that it is produced in plant T2 is 6

36 47 π2 π
a. b. (C) The value of 3.
73 79 ln 3 3
5/6
78 75
c.
93
d.
83 ∫ sec(π x) dx is
7/6

17. A bird is sitting on the top of a vertical pole 20 m high and (D) The maximum value 4. π
is elevation from a point O on the ground is 45° . It flies  1 
Arg   for
off horizontally straight away from the point O. After one 1− z 
second, the elevation of the bird from O is reduced to
| z |= 1, z ≠ 1 f is given by
30°. Then the speed (in m/s) of the bird is:
π
a. 40( 2 − 1) 5.
2
b. 40( 3 − 2)
20. Consider the lines given by L1 : x + 3 y − 5 = 0,
c. 20 2
L2 : 3 x − ky − 1 = 0, L3 : 5 x + 2 y − 12 = 0 Match the
d. 20( 3 − 1) Statements /Expressions in Column I with the Statements/
Expressions in Column II
18. Let f ( x) be differentiable on the interval (0, ∞) such that
Column I Column II
t 2 f ( x) − x 2 f (t ) (A) L1 , L2 , L3 are 1. k = −9
f (1) = 1, and lim = 1 for each x > 0.
t →x t−x concurrent, if
Then f ( x) is
(B) One of L1 , L2 , L3 is 6
2
2. k=−
1 2x parallel to at least of 5
a. +
3x 3 the other two, if
1 4x2 (C) L1 , L2 , L3 form a 5
b. − + 3. k =
3x 3 6
triangle, if
1 2
c. − + (D) L1 , L2 , L3 do not form 4. k =5
x x2
1 a triangle, if
d.
x
Mock Test-3 173
JEE ADVANCE PAPER-II

SECTION 1 Contains 8 Questions. 8. If the normals of the parabola y 2 = 4 x drawn at the end
The answer to each question is a single digit integer ranging from 0 to points of its latus rectum are tangents to the circle
9 (both inclusive).
( x − 3) 2 + ( y + 2) 2 = r 2 , then the value of r 2 is
1. For a point P in the plane, let d1 ( P ) and d 2 ( P) be the
distances of the point P from the lines x − y = 0 and SECTION 2 Contains 8 Multiple Choice Questions
With one or more than one correct option
x+ y =0 respectively. The area of the region R
1
consisting of all points P lying in the first quadrant of the
9. Let p = lim (1 + tan 2 x ) 2 x then log p is equal to:
x→0+
plane and satisfying 2 ≤ d1 ( P ) + d 2 ( P) ≤ 4, is _______
1 1
a. 2 b. 1 c. d.
2. The slope of the tangent to the curve 2 4
( y − x5 )2 = x(1 + x2 )2 at the point (1, 3) is 10. Let f ( x) be a non-constant twice differentiable function
3. Let (x, y, z) be points with integer coordinates satisfying defined on (−∞, ∞) such that f ( x) = f (1 − x) and
the system of homogeneous equations: 3x − y − z = 0 , 1
f ′   = 0. Then
−3 x + z = 0 −3x + 2 y + z = 0. 4
Then the number of such points for which a. f ′′( x) vanishes at least twice on [0, 1]
x + y + z ≤ 100 is
2 2 2
1
b. f ′   = 0
2
 −1  sin θ  π π 1/ 2
4. Let f (θ ) = sin  tan    , where − < θ < .  1
  cos 2θ  4 4 c. ∫
−1/ 2
f  x +  sin x dx = 0
 2
d 1/ 2 1
Then the value of (f (θ )) is f ( t ) esin π t dt =
d(tan θ ) d. ∫
0

1/ 2
f (1 − 1)esin π t dt

5. Two parallel chords of a circle of radius 2 are at a distance


π /2
3 + 1 apart. If the chords subtend at the centre, angles of ∫ (2 cos ec x)
17
11. The following integral dx is equal to
π /4
π 2π
and , where k > 0, then the value of [k] is [Note: [k] log(1+ 2 )
k k
denotes the largest integer less than or equal to k].
a. ∫
0
2(eu + e − u )16 du

log(1+ 2 )
b
6. Let a, b, c be positive integers such that
a
is an integer. b. ∫
0
(eu + e −u )17 du

If a, b, c are in geometric progression and the arithmetic log(1+ 2 )

a 2 + a − 14
c. ∫ (eu − e −u )17 du
mean of a, b, c is b + 2, then the value of is 0
a +1 log(1+ 2 )

7. Let ω = eiπ / 3 , and a, b, c, x, y, z be non-zero complex d. ∫


0
2(eu − e −u )16 du
numbers such that
a+b+c = x x2 − 1
12. ∫x 3
2 x4 − 2x2 + 1
dx is equal to
a + bω + cω 2 = y
2x4 − 2x2 + 1 2 x4 − 2 x2 + 1
a + bω 2 + cω = z. a. +c b. +c
x2 x3
| x |2 + | y |2 + | z |2
Then the value of is 2 x4 − 2 x2 + 1 2 x4 − 2 x2 + 1
| a |2 + | b |2 + | c |2 c. +c d. +c
x 2 x2
174 Mathematics
13. If y ( x) satisfies the differential equation SECTION 3 Contains 2 Paragraph Type Questions
y '− y tan x = 2 x sec x and y(0) = 0, then. Each paragraph describes an experiment, a situation or a problem.
Two multiple choice questions will be asked based on this paragraph.
π  π
2
a. y   = One or more than one option can be correct.
4 8 2
π  π Tangents are drawn from the point P(3, 4) to the ellipse
2
b. y '   =
 4  18 x2 y 2
+ = 1 touching the ellipse at points A and B.
π  π
2 9 4
c. y   =
3 9
17. The coordinates of A and B are
 π  4π 2π
2
d. y '   = + a. (3, 0) and (0, 2)
3 3 3 3
 8 2 161   9 8
b.  − ,  and  − , 
 5 15   5 5
14. Given an isosceles triangle, whose one angle is 120° and
radius of its incircle = 3. Then the area of the triangle in  8 2 161 
c.  − ,  and (0, 2)
sq. units is  5 15 
a. 7 + 12 3  9 8
d. (3, 0) and  − , 
 5 5
b. 12 − 7 3
18. The equation of the locus of the point whose distances
c. 12 + 7 3
from the point P and the line AB are equal, is
d. 4π a. 9 x 2 + y 2 − 6 xy –54 x –62 y + 241 = 0
b. x 2 + 9 y 2 + 6 xy –54 x + 62 y – 241 = 0
15. The circle passing through the point (–1, 0) and touching
c. 9 x 2 + 9 y 2 –6 xy –54 x –62 y – 241 = 0
the y-axis at (0, 2) also passes through the point
d. x 2 + y 2 – 2 xy + 27 x + 31 y –120 = 0
 3 
a.  − , 0 
 2  Paragraph for Question No. 19 to 20
 5  Let S = S1 ∩ S 2 ∩ S3 , where S1 = { z ∈ C :| z | < 4},
b.  − , 2 
 2   
 z − 1 + 3i 
 3 5 S2 =  z ∈ C : Im   > 0
c.  − ,    1 − 3i  
 2 2
and S3 = { z ∈ C : Re Z > 0}.
d. (–4, 0)
19. Area of S =
16 The normal at a point P on the ellipse x 2 + 4y 2 = 16 meets 10π 20π
a. b.
3 3
the x-axis at Q. If M is the mid-point of the line segment
PQ, then the locus of M intersects the latus rectums of the 16π 32π
c. d.
given ellipse at the points 3 3

 3 5 2 20. min |1 − 3i − z |=
a.  ± , ±  z∈S
 2 7
 2− 3
a.
 3 5 19  2
b.  ± ,± 
 2 4 
 2+ 3
b.
 1 2
c.  ±2 3, ± 
 7 3− 3
c.
 4 3 2
d.  ±2 3, ± 
 7  3+ 3
 d.
2
Mock Test-3 175
ANSWER & SOLUTIONS 2 6 12 20
7. (d) Let S = + + + + K and let
JEE-Main 1! 2! 3! 4!
S1 = 2 + 6 + 12 + 20 + K + Tn
1. 2. 3. 4. 5. 6. 7. 8. 9. 10.
b c d a a b d a b b S1 = 2 + 6 + 12 + KTn −1 + Tn
11. 12. 13. 14. 15. 16. 17. 18. 19. 20.
0 = 2 + 4 + 6 + 8 + K upto n terms − Tn
b a c a d c d c b c
21. 22. 23. 24. 25. 26. 27. 28. 29. 30. Tn = 2 + 4 + 6 + 8 + .......upto n terms
a c a a b a c a a d
n
⇒ Tn = [2 × 2 + (n − 1) 2] = n(2 + n − 1) = n(n + 1)
2
 −1 g ( x) < 0
 ∴ nth term of given series
1. (b) f ( g ( x)) = 0 g ( x) = 0
1 n(n + 1) n(n + 1) 1 2
 g ( x) > 0 Tn = or Tn = or Tn = +
n! n(n − 1)! (n − 2)! (n − 1)!
Since g ( x) ≥ 1 > 0
∞ ∞
1 1
Hence g ( g ( x)) = 1 Now, sum = ∑ + 2∑ = e + 2e = 3e.
n =1 ( n − 2)! n =1 ( n − 1)!

2. (c) x = 6 + x , x > 0 ⇒ x 2 = 6 + x, x > 0 8. (a) Words starting from A are 5 ! = 120


⇒ 2
x − x − 6 = 0, x > 0 Words starting from I are 5 ! = 120
⇒ x = 3, x > 0. Words starting from KA are 4 ! = 24
−1
Words starting from KI are 4 ! = 24
3. (d) z + z = 1 ⇒ z 2 − z + 1 = 0 Words starting from KN are 4 ! = 24
z = −ω or −ω
2
⇒ Words starting from KRA are 3 ! = 6
For z = − ω , z100 + z −100 = (−ω )100 + (−ω ) −100 Words starting from KRIA are 2 ! = 2
1 Words starting from KRIN are 2 ! = 2
=ω+ = ω + ω 2 = −1 Words starting from KRISA are 1 ! = 1
ω
Words starting from KRISNA are 1 ! = 1
For z = −ω 2 , z100 + z −100 = (−ω 2 )100 + (−ω 2 ) −100
Hence rank of the word KRISNA is 324.
1 1
= ω 200 + = ω2 + = ω2 + ω 9. (b) The number of ways of placing 3 black balls without
ω 200 ω2
any restriction is 10 C3 . Since, we have total 10 places of
= −1.
4. (a) Let The matrix of cofactors of the elements of A viz. putting 10 balls in a row. Now the number of ways in
−( −4)   2 4  which no two black balls put together is equal to the
 c11 c12   2
 = = number of ways of choosing 3 places marked '− ' out of
c21 c22   −( −3) 2   3 2 
eight places.
∴ adjA = transpose of the matrix of cofactors of elements
−W − W − W − W − W − W − W −
2 3 This can be done in 8 C3 ways.
of A =  
4 2 8
C3 8× 7 × 6 7
∴ A(adj A) =| A | I . ∴ Required probability = = =
10
C3 10 × 9 × 8 15
9 1
5. (a) Series is a G.P. with a = 0.9 = and r = = 0.1 1 − cos α 1 + cos α 1 − cos α + 1 + cos α
10 10 10. (b) + =
1 + cos α 1 − cos α 1 − cos 2 α
 1 
 1 − r100  9  1 − 10100  1 2 2  3π 
∴ S100 = a =   = 1 − 100 . = = ,  since π < α < .
 1 − r  10  1 − 1  10 ± sin α − sin α  2 
 10 
11. (b) Q Expression cos −1 (cos θ + i sin θ )
6. (b) L.H.S. = a[C0 − C1 + C2 − C3 + ...( −1) .Cn ] n

π
+[C1 − 2C2 + 3C3 − .... + ( −1) n −1 n.Cn ] = a.0 + 0 = 0 = sin −1 sin θ − i log( sin θ + 1 + sin θ ), where θ =
6
176 Mathematics
 3 i 1 π  b a 2 − b 2 cos θ + 0 − ab  −b a 2 − b 2 cos θ − ab 
∴ Real part of cos −1  +  = sin −1

= . 19. (b)   
 2 2 2 4  b 2 cos 2 θ + a 2 sin 2 θ  b 2 cos 2 θ + a 2 sin 2 θ 
  
12. (a) 64 cot θ = d −[b 2 ( a 2 − b 2 ) cos 2 θ − a 2b 2 ]
=
Also (100 − 64) tan θ = d or (64)(36) = d , 2
(b 2 cos 2 θ + a 2 sin 2 θ )
∴ d = 8 × 6 = 48 m. b 2 [a 2 − a 2 cos 2 θ + b 2 cos 2 θ ]
=
b 2 cos 2 θ + a 2 sin 2 θ
13. (c) f (0+) = f (0−) = 2 and f (0) = 2
b 2 [a 2 sin 2 θ + b 2 cos2 θ ]
Hence f ( x) is continuous at x = 0. = = b2
b 2 cos 2 θ + a 2 sin 2 θ
14. (a) x y = e x− y
⇒ y log x = x − y 20. (c)
O
x
⇒ y=
1 + log x 45°
A C B
dy
⇒ = log x(1 + log x) −2 = log x[log ex ]−2 . AB = 2
dx
Let AB be the chord of length 2 , O be centre of the circle
15. (d) Let x + y = 20 ⇒ y = 20 − x
and let OC be the perpendicular from O on AB. Then
and x 3 . y 2 = z ⇒ z = x 3 . y 2
2 1
AC = BC = =
z = x 3 (20 − x) 2 2 2
⇒ z = 400 x 3 + x5 − 40 x 4 1
In ∆ OBC , OB = BC cosec 45° = . 2 =1
dz 2
= 1200 x 2 + 5 x 4 − 160 x 3
dx ∴ Area of the circle = π (OB ) 2 = π
dz
Now = 0, then x = 12, 20 y
dx
21. (a) Tangent as
d 2z  d 2z  A focal chord
Now = 2400 x + 16 x 3
− 480 x 2
;  2 = −ive 2
dx 2  dx  x =12 P θ C(6,0)
x'
θ x
Hence x = 12 is the point of maxima (4,0)
2
∴ x = 12, y = 8. B

1
16. (c) ∫ x
tan 4 x .sec2 x dx y'
Here, the focal chord of y 2 = 16 x is tangent to circle
2
sec x
Put tan x = t ⇒ dx = dt , ( x − 6) 2 + y 2 = 2.
2 x
⇒ focus of parabola as (a,0) ie, (4,0)
then it reduces to
2 2 Now, tangents are drawn from (4,0) to ( x − 6) 2 + y 2 = 2.
2 ∫ t 4 dt = (tan x )5 + c = tan 5 x + c.
5 5 Since, PA is tangent to circle.
2 AC BC 2
17. (d) Required area = ∫
0
[2 x − (2 x − x 2 )] dx ∴ tan θ = slope of tangent =
AP
2
=
= 1, or
BP
= −1
2
 2x x3  4 8 1 3 4 ∴ Slope of focal chord as tangent to circle = ±1.
= − x2 +  = −4+ − = − .
 log 2 3 0 log 2 3 log 2 log 2 3 r r r r r r r r r r r
22. (c) a ⋅ b = a ⋅ c ⇒ a ⋅ b − a ⋅ c = 0 ⇒ a ⋅ (b − c ) = 0
dy r r r r r r r r
18. (c) Given equation = e x (sin x + cos x ) ⇒ Either b − c = 0 or a = 0 ⇒ b = c or a ⊥ (b − c ).
dx
⇒ dy = e x (sin x + cos x) dx 23. (a) Given lines are,
x−5 y−7 z+ 2
On integrating, we get y = e x sin x + c. = = = r1 , (say)
3 −1 1
Mock Test-3 177
x+3 y−3 z−6 Let AE is a vertical lamp-post. Given, AE = 12m
and = = = r2 , (say)
−36 2 4 AE
tan 45° =
∴ x = 3r1 + 5 = −36r2 − 3 , AC
y = −r1 + 7 = 3 + 2r2 and z = r1 − 2 = 4r2 + 6 AC = AE = 12m
m
AE
5
On solving, we get x = 21, y = , z =
10 tan 60° =
3 3 AB
Trick: Check through options. AE
AB = =4 3
3
24. (a) ~ ( p ∧ q) ≡ ~ p ∨ ~ q .
BC = AC 2 − AB 2 = 144 − 48 = 96 = 4 6
25. (b) From the given identity
Area = = AB × BC = 4 3 × 4 6 = 48 2 sq
sq.cm.
. cm.
b( x + 1) 2 + c ( x + 1) + d − (bx 2 + cx + d ) = 8 x + 3
y
⇒ 2bx + b + c = 8 x + 3 29. (a)
⇒ b = 4, c = −1 B y= x
2 pq
26. (a) As given H = x−3
p+q y=
2
x' x
H H 2q 2p 2( p + q ) O A(3,0) (9,0)
∴ + = + = =2.
p q p+q p+q p+q y'

27. (c) Plan: A square matrix M is invertible if f det(M) or To find the area between the curves, y = x , 2 y + 3 = x
| M |≠ 0. and x-axis in the Ist quadrant (we can plot the above
a b condition as);
Let, M =  
b c  Area of shaded portion OABO
9 9
 a  b  9  x −39  x3 / 2  1  x 2 
(a) Given that   =   ⇒ a = b = c = α (let) =∫ x dx − ∫   dx =   −  − 3x 
b   c 
0 3
 2   3 / 2 0 2  2 3
α α  2  1  81  9 
⇒ M =  ⇒ | M |= 0 =  ⋅ 27  −  − 27  −  − 9  
α α   3  2  2   2 
⇒ M is non-invertible. 1
= 18 − (18) = 9 sq unit
(b) Given that [b c] = [a b] 2
⇒ a = b = c = α (let) 30. (d) The equation of tangent at ( x1 , y1 ) is xx1 − 2 yy1 = 4,
Again | M |= 0 which is same as 2 x + 6 y = 2
⇒ M is non-invertible. x1 2y 4
∴ =− 1 =
 a 0 2 6 2
(c) As given M =   ⇒ | M |= ac ≠ 0
0 c  ⇒ x1 = 4 and y1 = − 6
( Q a and c are non-zero)
⇒ M is invertible. JEE Advance Paper-I
a b
(s) M =   ⇒ | M |= ac − b 2 ≠ 0 1. 2. 3. 4. 5. 6. 7. 8. 9. 10.
b c 
5 9 9 5 3 2 4 5 a d
Q ac is not equal to square of an integer.
11. 12. 13. 14. 15. 16. 17. 18. 19. 20.
∴ M is invertible.
c b d d b c d a c a
D C
28. (a)  5
E 450 1. (5) | 2z − 6 + 5i |= 2 z −  3 − 
 2
 5  5
600
≥ 2 3 −  3 − i  (corresponding Pt A) = 2   = 5
B  2  2
A
178 Mathematics

a b c 7. (4) Solving y = 8x and x + y − 2 x − 4 y = 0


2 2 2

2. (9) Let M = d e f  Simultaneously, we get (2, 4) and (0, 0)


g h i  Focus is (2, 0)
1
0   −1 ∴ Are = × 2 × 4 = 4sq. units.
M 1  =  2  ⇒ b = −1, e = 2, h = 3
2
8. (5) Number of red lines = n C2 − n
0   3 
Number of blue lines = n
1 1 n
C2 − n = n
M  −1 =  1  ⇒ a = 0, d = 3, g = 2
Hence,

 0   −1 ⇒
n
C2 = 2n

1  0  n(n − 1)
= 2n
M 1 =  1  ⇒ g + h + i = 12 ⇒ i = 7 2
1 12  ⇒ n −1 = 4
⇒ n = 5.
∴ Sum of diagonal elements = 9.
9. (a) Set A has 4 elements
3. (9) Let seventh term be ‘a’ and common difference be‘d’
Set B has 2 elements
S7 6 ∴ Number of elements in set ( A × B) = 4 × 2 = 8
Given =
S11 11
∴ Total number of subsets of ( A × B) = 28 = 256
⇒ a = 15d
Hence, 130 < 15 d < 140 ⇒ d = 9 Number of subsets having 0 elements = 8C0 = 1
4. (5) Clearly, 1 + 2 + 3 + K + n − 2 ≤ 1224 ≤ 3 + 4 + K n Number of subsets having 1 element each = 8C1 = 8
(n − 2) (n − 1) (n − 2) ∴ Number of subsets having 2 elements each
⇒ ≤ 1224 ≤ (3 + n)
2 2 8! 8× 7
= 8C2 = = = 28
⇒ n 2 − 3n − 2446 ≤ 0 and n 2 + n − 2454 ≥ 0 2!6! 2
⇒ 49 < n < 51 Number of subsets having at least 3 elements
⇒ n = 50 = 256 − 1 − 8 − 28 = 256 − 37 = 219
n(n + 1) 10. (d) f ( x) = 2 x3 + 3x + k
∴ − (2k + 1) = 1224
2 f ′( x) = 6 x 2 + 3
⇒ k = 25 ⇒ k − 20 = 5
1
1 ⇒ f ′( x) = 0 ⇒ x 2 = −
5. (3) ∆ = ab sin C 2
2 Not possible. As condition for two distinct real root is
2∆ 2 ×15 3 3 f (α ) f ( β ) = 0
⇒ sin C = = = ⇒ C = 120°
ab 6 ×10 2 (where are α , β roots of f ′( x) = 0)
⇒ c= a 2 + b 2 − 2 ab cos C 11. (c) Let z = ω
= 6 + 10 − 2 × 6 × 10 × cos120 ° = 14
2 2
1+ z 1+ ω −ω 2
Now = = =ω
∆ 225 × 3 1+ z 1+ ω2 −ω
r= ⇒ r = = 3.
2

s  10 + 14 
6 + 1+ z
  ∴ arg = arg ω = θ (put z = cosθ + i sin θ )
 2  1+ z
12. (b) | A | I = AAT
( )
cos α n
e −e e
6. (2) lim =− 1 0  5a −b   5a 3 
α →0 αm 2 ⇒ (10a + 3b)  =  
 0 1   3 2   −b 2 

lim
(
e e (cos(α )
n
−1)
) (cos α n
− 1)
α 2n = −
e ⇒ 25a 2 + b2 = 10a + 3b & 15a − 2b = 0 & 10a + 3b = 13
α →0
( cos(α n
)
) −1 α α m 2n
2
3.15a
⇒ 10a + = 13 ⇒ 65a = 2 ×13
if and only if 2n − m = 0 2
Mock Test-3 179
2 ⇒ OB − OA = 20( 3 − 1)
⇒ a= ⇒ 5a = 2
5 Hence distance covered in one second by the bird is
⇒ 2a = 6 ⇒ b = 3 AB = 20( 3 − 1)
∴ 5a + b = 5
Thus speed of bird
13. (d) Let the numbers be a, ar, ar2 is G.P. = 20( 3 –1)
− 1) m/s
/
2 a + ar 2
Given a, 2ar, ar are in A.P. the 2 ar = ( a ≠ 0)
2 t 2 f ( x) − x 2 f (t )
18. (a) lim =1
which gives r = 2 + 3, as the G.P. is an increasing G.P. t →x t−x
⇒ x 2 f ′( x ) − 2 x f ( x ) + 1 = 0
14. (d) Theoretically the number of terms are 2N + 1 (i.e. odd)
But As the number of terms being odd hence considering ⇒ f ( x ) = cx 2 + 1
that number clubbing of terms is done hence the solutions 3x
follows: Also f (1) = 1
n+2
Number of terms = C2 = 28
⇒ c=2
∴ n=6 3
Sum of coefficient = 3n = 36 = 729 2 2 1
Hence f ( x ) = x + .
3 3x
Put x = 1
r r
n +1 19. (A) a − b = 1 + 3 = 2
15. (b) C3 − C3 = 10
n
r
r
| a |= b, | b |= 2
⇒ On solving n = 5
2 1
16. (c) Let x = P (computer turns out to be defective given cos θ = =
2× 2 2
that it is produced in plate T2 ),
π 2π 2π
7 1 4 θ= , but its as its opposite to side of maximum
⇒ = (10 x) + x 3 3 3
100 5 5
length.
⇒ 7 = 200 x + 80 x b

∫ (f (x) − 3x) dx = a − b2
2
7 (B)
⇒ x= a
280
−a 2 + b 2
b
3 2
P (produced in T2 / not defective)
∫ f (x)dx = 2
(b − a 2 ) + a 2 − b 2 =
2
P( A ∩ B) a
= ⇒ f (x) = x.
P(B)

π 2  ln (sec π x + tan π x) 7 / 6 
5/ 6
4 4  273 
(C)  
5(1 − x) 5  280  ln 3  π 
⇒ =  
1 4 1  280 − 70  4  273 
(1 − 10 x) + (1 − x) +
5 5 5  280  5  280  π  5π 5π 7π 7π 
=  ln sec + tan − ln sec + tan  = π.
ln 3  6 6 6 6 
4 × 273 2 × 273 546 78
⇒ = = =
210 + 4 × 273 105 + 2 × 273 651 93 1 1
(D) Let u = ⇒ z = 1−
17. (d) Q P
1− z u
1
| z |= 1 ⇒ 1 − =1
2
⇒ | u − 1|=| u |
B A O ∴ locus of u is perpendicular bisector of line segment
Here, AP = QB = 20m joining 0 and 1
∠POA = 45°, ∠QOB = 30° π
⇒ maximum arg u approaches but will not attain.
⇒ OA = 20; OB = 20 3 2
180 Mathematics
20. x + 3 y − 5 = 0 and 5 x + 2 y − 12 = 0 intersect at (2, 1)  −1 sin θ 
4. (1) f (θ ) = sin  tan 
Hence 6 − k − 1 = 0 k = 5  2cos θ 
1 3  
for L1 , L2 to be parallel = ⇒ k = −9 sin θ
3 −k = sin  sin −1 
 sin θ + cos 2θ
2

3 −k −6
for L2 , L3 to be parallel = ⇒k= . sin θ sin θ sin θ
5 2 5 = = = = tan θ
−6 sin θ + (cos θ − sin θ )
2 2 2 | cos θ | cos θ
for k ≠ 5, − 9, they will form triangle
5 d d(tan θ )
−6 ⇒ f (θ ) = =1
for k = 5, k = −9, they will not form triangle d(tan θ ) d(tan θ )
5
π π
5 (2) 2cos + 2cos = 3 +1
JEE Advance Paper-II 2k k
1. 2. 3. 4. 5. 6. 7. 8. 9. 10. π π 3 +1
⇒ cos + cos =
6 8 7 1 5 4 3 2 3 All 2k k 2
11. 12. 13. 14. 15. 16. 17. 18. 19. 20. π θ 3 +1
a d a,d c d a d a b c
Let = 0, cos θ + cos =
k 2 2
1. (6) θ θ 3 +1
y=x ⇒ 2 cos 2 − 1 + cos =
2 2 2
θ 3 +3
P(α , β ) ⇒ cos = t ⇒ 2t 2 + t − =0
2 2
x= 2 x=2 2 −1 ± 1 + 4(3 + 3) −1 ± (2 3 + 1) −2 − 2 3 3
⇒ t= = = ,
2 ≤ d1 ( p ) + d 2 ( p ) ≤ 4 4 4 4 2
For P (α , β ), α > β θ 3
Q t ∈ [ −1,1],cos =
2 2
⇒ 2 2 ≤ 2α ≤ 4 2
θ π
2 ≤α ≤ 2 2 ⇒ = ⇒ k = 3.
2 6
⇒ Area of region = ((2 2) − ( 2) ) 2 2

b c
= 8 − 2 = 6 sq.. units 6. (4) = = (integer) b 2 = ac
a b
 dy  b2 a +b+c
2. (8) 2( y − x 5 )  − 5 x 4  ⇒ c= ⇒ =b+2
 dx  a 3
= 1(1 + x 2 )2 + ( x)(2(1 + x 2 )(2 x)) a + b + c = 3b + 6
Now put x = 1, y = 3 ⇒ a − 2b + c = 6
dy b2
and = m. 2(3 − 1)( m − 5) = 1(4) + (1)(4)(2) a − 2b + =6
dx a
12 2b b 2 6
⇒ m−5 = ⇒ 1− + =
4 a a2 a
dy 2
⇒ m = 5+3 = 8 ⇒ = m = 8. b  6
dx  − 1 = a = 6 only
 a  a
3. (7) 3x − y − z = 0 −3 x + z = 0 7. (3) The expression may not attain integral value for all a, b, c
⇒ y = 0 and z = 3 x If we consider a = b = c, then x = 3a
⇒ x 2 + y 2 + z 2 = x 2 + z 2 = x 2 + 9 x 2 = 10 x 2 ≤ 100 y = a(1 + ω + ω 2 ) = a(1 + i 3)
⇒ x2 ≤ 10 ⇒ x = 0, ± 1, ± 2, ± 3 z = a(1 + ω 2 + ω ) = a(1 + i 3)
There are such seven points. ∴ | x |2 + | y |2 + | z |2 = 9 | a |2 +4 | a |2 +4 | a |2 = 17 | a |2
Mock Test-3 181
8. (2) Equation of normals are x + y = 3 and x − y = 3. (e − e )
u −u
⇒ − ∫ (eu + e − u )17 du
⇒ Distance from (3, −2) on both normals is ‘r’ 2cosec x cot x
ln(1+ 2 )
|3−2−3|
0


2
=r = −2 ∫ (eu + e − u )16 du = ∫ 2(eu + e − u )16 du
ln(1+ 2 ) 0

⇒ r = 2.
2
 1 1 
 3 − 5  dx
12. (d) ∫ 
1 x x 
9 (3) P = lim+ (1 + tan 2 x − 1) 2x
then log p =
x →∞ 2 1
2− 2 + 4
(1+ tan 2 x −1 )
1
lim
(tan x )2 1 x x
lim 2x
x →0+ 2( x ) 2
P=e x→0+
=e = e2 2 1
Let 2 − + =z
1
1 x2 x4
⇒ log P = log e = 2
2 1 dz
4∫ z

10. (a,b,c,d) 1
⇒ × z +c
2
1/4 3/4
1 2 1
1/2 ⇒ 2− 2 + 4 +c
2 x x
13. (a,d) y '− y tan x = 2 x sec x
f ( x) = f (1 − x)
I.F. = e∫
tan x dx
= elog cos x = cos x
Put x = 1/ 2 + x
∴ y cos x = ∫ 2 x sec x.cos x dx
1  1 
⇒ f  + x = f  − x
2  2  ⇒ y cos x = x 2 + c
Hence f ( x + 1/ 2) is an even function or f ( x + 1/ 2)sin x ⇒ y ⋅ cos x = x 2 (Q y (0) = 0)
an odd function.
⇒ y = x 2 sec x
Also, f '( x) = − f '(1 − x) and for x = 1/ 2,
π  π  π  4π 2π
2 2

We have f '(1/ 2) = 0. ∴ y  = and y'   = +


4 8 2 3 3 3 3
1 0

Also,
1/ 2
∫ f (1 − t )esin π t dt = − ∫ f ( y)esinπ t dy
1/ 2 14. (c) ∆ =
3 2
b . . .(i)
4
(obtained by putting, 1 − t = y ).
sin120° sin 30°
Also =
Since f ′(1/ 4) = 0, f ′(3/ 4) = 0. Also f ′(1/ 2) = 0 a b
⇒ f ′( x) = 0 atleast twice in [0, 1] (Rolle’s Theorem) ⇒ a = 3b and ∆ = 3s and s =
1
(a + 2b)
2
π
2 3
⇒ ∆= (a + 2b)
∫ (2cosec x) . . .(ii)
17
11. (a) dx
π
2
From equation (i) and (ii), we get ∆ = (12 + 7 3)
4

π
Let eu + e− u = 2cosec x, x = 15. (d) Circle touching y-axis at (0, 2) is
4
π (x − 0) 2 + (y − 2) 2 + λ x = 0
⇒ u = ln(1 + 2), x =
2 passes through (–1, 0)
⇒ u=0 ∴ 1+ 4 − λ = 0 ⇒ λ = 5
⇒ cosec x + cot x = eu and cosec x − cot x = e − u ∴ x 2 + y 2 + 5x − 4y + 4 = 0
eu − e −u Put y = 0 ⇒ x = −1, − 4
⇒ cot x (eu − e − u ) dx = −2cosec x cot xdx
2 ∴ Circle passes through (–4, 0)
182 Mathematics
16. (a) Any point on the line can be taken as 18. (a) Locus is parabola
Q ≡ {(1 − 3µ ), ( µ − 1), (5µ + 2)} 3x 4 y
Equation of AB Is + =1
9 4
PQ = {−3µ − 2, µ − 3, 5 µ − 4}
x
Now 1(−3µ − 2) − 4( µ − 3) + 3(5µ − 4) = 0 ⇒ + y = 1 ⇒ x + 3y − 3 = 0
3
⇒ − 3 µ − 2 − 4 µ + 12 + 15 µ − 12 = 0
( x + 3 y − 3) 2
( x − 3)2 + ( y − 4) 2 =
8 µ = 2 ⇒ µ = 1/ 4 10
10 x + 90 – 60 x + 10 y + 160 – 80 y = x 2 + 9 y 2 + 9 + 6 xy – 6 x – 18
2 2
17. (d) y = mx + 9 m 2 + 4
10 x 2 + 90 – 60 x + 10 y 2 + 160 – 80 y = x 2 + 9 y 2 + 9 + 6 xy – 6 x – 18 y
4 − 3m = 9m 2 + 4
⇒ 9 x 2 + y 2 – 6 xy – 54 x – 62 y + 241 = 0.
12 1
16 + 9m 2 − 24m = 9m 2 + 4 ⇒ m = =
24 2 19. (b) y + 3x = 0

1
Equation is y − 4 = ( x − 3)
2 60°
2y − 8 = x − 3 60°
x 2 + y 2 < 16
⇒ x − 2y + 5 = 0
Let B = (α , β )
Area of region S1 ∩ S 2 ∩ S3 = shaded area
xα y β
⇒ + −1 = 0
9 4 π × 42 42 × π
= +
α / 9 β / 4 −1 4 6
⇒ = = ⇒
1 −2 5  1 1  20π
= 42 π  +  =

9
α =− , β =
8 4 6 3
5 5 20. (c) Distance of (1, –3) from y + 3 x = 0
 9 8
⇒ B ≡  − , . −3 + 3 × 1 3 − 3
 5 5 > >
2 2
Mock Test-4 183
JEE-MAIN: MATHEMATICS MOCK TEST-4
1. The number of solutions of log 4( x − 1) = log 2( x − 3) is: a. occurrence of E ⇒ occurrence of F
a. 3 b. 1 c. 2 d. 0 b. occurrence of F ⇒ occurrence of E
c. non-occurrence of E ⇒ non-occurrence of F
2. The real roots of the equation x 2 + 5 | x | + 4 = 0 are
d. None of the above implication holds
a. – 1, 4 b. 1, 4
π  π 
c. – 4, 4 d. None of these 10. tan  + θ  − tan  − θ  =
4  4 
1+ 3 i a. 2 tan 2θ b. 2 cot 2θ c. tan 2θ d. cot 2θ
3. If is a root of equation x 4 − x 3 + x − 1 = 0 then
2
11. tanh( x + y ) equals
its real roots are
a. 1, 1 b. – 1, – 1 c. 1, – 1 d. 1, 2 tanh x + tanh y tanh x + tanh y
a. b.
1 − tanh x tanh y 1 + tanh x tanh y
 1 0 0
tanh x − tanh y tanh x − tanh y
4. Let A =  5 2 0  , then the adjoint of A is c. d.
1 − tanh x tanh y 1 + tanh x tanh y
 −1 6 1 
12. The length of the shadow of a pole inclined at 10° to the
 2 −5 32   −1 0 0 
a.  0 1 −6  b.  −5 −2 0 
vertical towards the sun is 2.05 metres, when the elevation
of the sun is 38°. The length of the pole is
 0 0 2   1 −6 1 
2.05 sin 38° 2.05 sin 42°
a. b.
 −1 0 0  sin 42 ° sin 38°
c.  −5 −2 0  d. None of these 2.05 cos 38°
c. d. None of these
 1 −6 −1 cos 42 °

 2 1
5. The value of 0.234 is  x sin , when x≠0
13. If f ( x) =  x , then
232 232  0, when x=0
a. b.
990 9990 a. f (0 + 0) = 1 b. f (0 − 0) = 1
232 232 c. f is continuous at x = 0 d. None of these
c. d.
990 9909
( x − a)( x − b) dy
6. The sum of the coefficients of even power of x in the 14. If y = , then =
( x − c)( x − d ) dx
expansion of (1 + x + x 2 + x 3 )5 is
y 1 1 1 1 
a. 256 b. 128 c. 512 d. 64 a. + − −
2  x − a x − b x − c x − d 
1 − 2 x + 3x 2
7. In the expansion of , the coefficient of x5  1 1 1 1 
ex b. y  + − −
 x − a x − b x − c x − d 
will be
1 1 1 1 1 
71 71 c. + − −
a. b. − 2  x − a x − b x − c x − d 
120 120
d. None of these
31 31
c. d. −
40 40 x2 − 1
15. If f ( x) = , for every real number x, then the
8. The total number of seven digit numbers the sum of x2 + 1
whose digits is even is minimum value of f
a. 9000000 b. 4500000 a. Does not exist because f is unbounded
c. 8100000 d. None of these b. Is not attained even though f is bounded
9. If E and F are events with P ( E ) ≤ P ( F ) and P ( E ∩ F ) > 0, c. Is equal to 1
then d. Is equal to –1
184 Mathematics
x
a 23. A line makes the same angle θ , with each of the x and
16. ∫ 1 − a2x
dx =
z-axis. If the angle β , which it makes with y-axis is such
1 that sin 2 β = 3sin 2 θ , then cos 2 θ equals
a. sin −1 a x + c −1
b. sin a + c x

log a
3 2
1 a. b.
c. cos −1 a x + c d. cos −1 a x + c 5 3
log a
1
c. d. None of these
17. The area bounded by the circle x 2 + y 2 = 4, line x = 3 y 5
and x-axis lying in the first quadrant, is 24. (~ (~ p)) ∧ q is equal to
π π π
a. b. c. d. π a. ~ p ∧ q b. p ∧ q c. p ∧ ~ q d. ~ p ∧ ~ q
2 4 3

dy 25. If α and β are the roots of the equation


18. The general solution of x 2 = 2 is
dx x2 + 6 x + λ = 0 and 3α + 2β = −20, then λ =
2 2 3
a. y = c + b. y = c − c. y = 2 cx d. y = c − a. –8 b. –16 c. 16 d. 8
x x x2
26. If a, b, c, d are in H.P., then ab + bc + cd is equal to
19. The length of perpendicular from the point
a. 3ad b. (a + b)(c + d )
( a cos α , a sin α ) upon the straight line
c. 3ac d. None of these
y = x tan α + c, c > 0 is
a. c cosα b. c sin 2 α c. c sec 2 α d. c cos 2 α 27. If y = 3 x + 6 x 2 + 10 x 3 + ...., then the value of x in terms of
y is
20. The circle passing through point of intersection of the
a. 1 − (1 − y ) −1/ 3 b. 1 − (1 + y )1/ 3
circle S = 0 and the line P = 0 is
a. S + λ P = 0 c. 1 + (1 + y ) −1/ 3 d. 1 − (1 + y ) −1/ 3
b. S − λ P = 0 and λ S + P = 0 28. The number of arrangements of the letters of the word
c. P − λ S = 0 BANANA in which the two N’s do not appear adjacently,
d. All of these is
21. Axis of a parabola is y = x and vertex and focus are at a a. 40 b. 60
c. 80 d. 100
distance 2 and 2 2 respectively from the origin. Then
equation of the parabola is- 29. Differential coefficient of sin −1 x w.r.t cos −1 1 − x 2 is
a. ( x − y ) 2 = 8( x + y − 2) b. ( x + y ) 2 = 2 ( x + y − 2) 1
a. 1 b.
1 + x2
c. ( x − y ) 2 = 4 ( x + y − 2) d. ( x + y ) 2 = 2 ( x − y + 2)
c. 2 d. None of these
r r r r
22. If a and b be unlike vectors, then a ⋅ b = 30. The interval in which the function x 2 e − x is non decreasing, is
r r r r a. (−∞, 2]
a. | a | | b | b. − | a | | b | b. [0, 2]
c. 0 d. None of these c. [2, ∞) d. None of these

Space for rough work


Mock Test-4 185
JEE ADVANCE PAPER-I

SECTION 1 Contains 8 Questions. SECTION 2 Contains 10 Multiple Choice Questions


The answer to each question is a single digit integer ranging from 0 to With one or more than one correct option
9 (both inclusive).
dy
9. If y = sec(tan −1 x), then at x = 1 is equal to
 kπ   kπ  dx
1. For any integer k, let α k = cos   + i sin   , where
 7   7  1 1
a. b. c. 1 d. 2
12 2 2
Σ | α k +1 − α k |
i = −1. The value of the expression k =1
is 1
3 10. For function f(x) = x cos , x ≥ 1,
Σ | α 4 k −1 − α 4 k − 2 | x
k =1
a. for atleast one x in interval [1, ∞),f (x + 2) − f (x) < 2
2. Let m be the smallest positive integer such that the
b. lim f '(x) = 1
x →∞
coefficient of x 2 in the expansion of
c. for all x in the interval [1, ∞),f (x + 2) − f (x) > 2
(1 + x) 2 + (1 + x)3 + ... + (1 + x)49 + (1 + mx)50 is (3n + 1) 51C3
d. f’ (x) is strcily decreasing in the interval [1, ∞)
for some positive integer n. Then the value of n is
x
11. Let f ( x ) = for n ≥ 2 and
3. The minimum number of times a fair coin needs to be (1 + x n )1/ n

∫x
n− 2
tossed, so that the probability of getting at least two heads g ( x) = ( f o f oKo f )( x). Then g ( x) dx equals
144244 3
f occurs n times
is at least 0.96 is
1 1
1 1− 1 1−
4. For any real number x, let |x| denote the largest integer less a. (1 + nx n ) n + K b. (1 + nx n ) n + K
n(n − 1) n −1
than or equal to x. Let f be a real valued function defined 1 1
1 1+ 1 1+
Ï x - [ x] if [ x] is odd c. (1 + nx n ) n + K d. (1 + nx n ) n + K
on the interval [−10, 10] by f (x) = Ì n(n + 1) n +1
Ó1 + [ x] - x if [ x] is even
12. Let f :[0, 2] → R be a function which is continuous on
Then the value of π
2 10

10 −∫10
f ( x)cosπ x dx is [0, 2] and is differentiable on (0, 2) with f (0) = 1. Let
x2
1
 d2  F ( x) = ∫ f ( t ) dt for x ∈ [0,2]. If F ′( x ) = f ′( x ) for all
5. The value of ∫ 4 x3  2 (1 − x 2 )5  dx is ________ 0
0  dx 
x ∈ (0, 2), then F (2) equals
6. Let f be a real-valued differentiable function on R (the set a. e2 − 1 b. e4 − 1 c. e − 1 d. e4
of all real numbers) such that f(1) = 1. If the y-intercept of
13. The function y = f ( x ) is the solution of the differential
the tangent at any point P(x, y) on the curve y = f(x) is
dy
dy xy
xy xx 4 + 2xx
equal to the cube of the abscissa of P, then the value of equation ++ 2 == in (–1,1) satisfying
dx x − 11 1 − x2
f (−3) is equal to
3
7. The line 2x + y = 1 is tangent to the hyperbola 2
f (0) = 0. Then
2 2
∫ f ( x ) dx is
3
x y
− = 1. If this line passes through the point of 2
a 2 b2
π 3 π 3 π 3 π 3
intersection of the nearest directrix and the x-axis, then the a. − b. − c. − d. −
3 2 3 4 6 4 6 2
eccentricity of the hyperbola is 14. Let O (0, 0), P (3, 4), Q (6, 0) be the vertices of the triangle
r r r
8. If a , b and c are unit vectors satisfying OPQ. The point R inside the triangle OPQ is such that the
r r2 r r2 r r2 r r r triangles OPR, PQR, OQR are of equal area. The
a − b + b − c + c − a = 9, then 2a + 5b + 5c is. coordinates of R are
186 Mathematics
4   2 19. Match the statements given in Column I with the
a.  , 3  b.  3, 
3   3 interval/union of intervals given in Column II
 4 4 2 Column I Column II
c.  3,  d.  , 
 3 3 3   2iz   1. (−∞, − 1) ∪ (1, ∞)
(A) The set Re    : z is
  1 − z 
2
x2 y 2
15. Tangents are drawn to the hyperbola − = 1, parallel
9 4 a complex number,
to the straight lien 2 x − y = 1. The points of contact of the | z |= 1, z ≠ ±1 is
tangents on the hyperbola are. (B) The domain of the 2. (−∞, 0) ∪ (0, ∞)
−1
function f (x) = sin
 9 1   9 1 
a.  ,  b.  − ,  x−2
2 2 2  2 2 2  8(3) 
 2( x −1) 
is
c. 3 3, −2 2 d. −3 3, 2 2 1− 3 
(C) If 3. [2, ∞)
16. If the vectors AB = 3iˆ + 4kˆ and AC = 5iˆ − 2 ˆj + 4kˆ are the
1 tan θ 1
sides of a triangles ABC , then the length of the median f (θ ) = − tan θ 1 tan θ ,
through A is −1 − tan θ 1
then the set
a. 18 b. 72
 π
c. 33 d. 45 f (θ ) : 0 ≤ θ <  is
 2
17. In R3 , consider the planes P1 : y = 0 and P2 : x + z = 1. Let (D) If f (x) = x 3 / 2 (3x − 10), 4. (−∞, − 1] ∪ [1, ∞)
P3 be a plane, different from P1 and P2 , which passes x ≥ 0, then f(x) is
through the intersection of P1 and P2 . If the distance of the increasing in

point (0, 1, 0) from P3 is 1 and the distance of a point 5. (−∞, 0] ∪ [2, ∞)

(α, β, γ) from P3 is 2, then which of the following x2 − 6 x + 5


20. Let f ( x) = Match the conditions/expressions
relations is (are) true? x2 − 5 x + 6
in Column I with statements in Column II
a. 2α + β + 2γ + 2 = 0
Column I Column II
b. 2α – β + 2γ + 4 = 0 (A) If −1 < x < 1, then 1. 0 < f ( x) < 1
c. 2α + β – 2γ – 10 = 0 f ( x) satisfies
d. 2α – β + 2γ – 8 = 0 (B) If 1 < x < 2, then 2. f ( x) < 0
18. The negation of ~ s ∨ (~ r ∧ s ) is equivalent to f ( x) satisfies
a. s ∧ ~ r b. s ∧ ( r ∧ ~ s ) (C) If 3 < x < 5, then 3. f ( x) > 0
c. s ∨ ( r ∨ ~ s ) d. s ∧ r
f ( x) satisfies
SECTION 3 Contains 2 Match The Following Type Questions (D) If x > 5, then f ( x) 4. f ( x ) < 1
You will have to match entries in Column I with the entries in satisfies
Column II.

Space for rough work


Mock Test-4 187
JEE ADVANCE PAPER-II

SECTION 1 Contains 8 Questions. SECTION 2 Contains 8 Multiple Choice Questions


The answer to each question is a single digit integer ranging from 0 to With one or more than one correct option
9 (both inclusive).
9. Let A and B be two sets containing 2 elements and 4
1. The least period of the function elements respectively. The number of subsets of A × B
 π [ x]  πx  π [ x]  having 3 or more elements is
sin   + cos   + tan   is λ, then the value a. 256 b. 220
 12   4   3 
of 201λ must be (where [⋅] denotes the greatest integer c. 219 d. 211
function) 10. The real number k for which the equation,

 ax − b  dy 2 x + 3 x + k = 0 has two distinct real roots in [0, 1]


3

2. If y = tan −1   then the value of (2008) a. lies between 1 and 2


 bx + a  dx x =−1
b. lies between 2 and 3
must be
c. lies between –1 and 0
3. The indicated horse power I of an engines is calculated
d. does not exist
PLAN π
from the formula I= where A = d 2.
33000 4 11. A complex number z is said to be unimodular if | z |= 1.
Assuming that error of 10% may have been made in Suppose z1 and z2 are complex numbers such that
measuring P, L, N and d. If the greatest possible in I is z1 − 2 z 2
λ % then λ must be is unimodular and z2 is not unimodular. Then
2 − z1 z 2
 2x + 2   2x + 2  the point z1 lies on a
∫ sin
−1
4. If   dx = ( x + 1) tan −1  
   3 
 (4 x + 8 x + 13) 
2
a. straight line parallel to x-axis
b. straight line parallel to y-axis
+ λ ln(4 x 2 + 8 x + 13) + c, then the value of − 4λ must be
c. circle of radius 2
5. A particle moves in a straight line with a velocity given
d. circle of radius 2
dx
by = ( x + 1) (x is the distance travelled). If the time
dt 1 4 4
taken by a particle to traverse a distance of 99 m is λ. 12. If the adjoint of a 3 × 3 matrix P is 2 1 7  , then the
1 1 3
then the value of 20 λ log10 e must be
6. If a triangle has its orthocenter at (1, 1) and circum possible value(s) of the determinant of P is (are)
a. –2 b. –1
3 3
centre at  ,  and if centroid and nine point centre are c. 1 d. 2
2 4
(α , β ) and (γ ,δ ) respectively, then the value of 13. If the sum of the first ten term of the series
2 2 2 2
6α + 12β + 4γ + 8δ must be  3  2  1
+  4 4  + ..., is 16 m,
1  +  2  +  3  + 4
2
then
 5  5  5  5 5
7. If the radius of a circle which passes through the point
m is equal to:
(2, 0) and whose centre is the limit of the point of
a. 102 b. 101
intersection of the lines 3 x + 5 y = 1 and
c. 100 d. 99
λ
(2 + c) x + 5c 2 y = 1 as c → 1 is , then the value of 14. Six cards and six envelopes are numbered 1, 2, 3, 4, 5, 6
25 and cards are to be placed in envelopes so that each
λ must be envelope contains exactly one card and no card is placed
in the envelope bearing the same number and moreover
8. A, B, C, D are any four points in the space. If the card numbered 1 is always placed in envelope
uuur uuur uuur uuur uuur uuur
| AB × CD + BC × AD + CA × BD | = λ (area of ∆ABC ) numbered 2. Then the number of ways it can be done is
then the value of 125λ must be a. 264 b. 265 c. 53 d. 67
188 Mathematics
15. Let θ ,ϕ ∈ 0,2π be such that 2cos θ 1 − sin ϕ number of such n-digit
θ θ integers ending with digit 1 and c n = the number of such

= sin 2 θ  tan + cot  cos ϕ − 1 tan 2π − θ > 0 and
 2 2 n–digit integers ending with digit 0.
17. Which of the following is correct?
3
−1 < sin θ < − . Then ϕ cannot satisfy: a. a17 = a16 + a15 b. c17 ≠ c16 + c15
2
c. b17 ≠ b16 + c16 d. a17 = c17 + b16
π
a. 0 < ϕ <
2 18. The value of b6 is
π 4π a. 7 b. 8
b. < ϕ <
2 3 c. 9 d. 11
4π 3π
c. <ϕ <
3 2 Paragraph for Question No. 19 to 20
3π Box 1 contains three cards bearing numbers 1, 2, 3; box 2
d. < ϕ < 2π contains five cards bearing numbers 1, 2, 3, 4, 5: and box 3
2
1/ 2 contains seven cards bearing numbers 1, 2, 3, 4, 5, 6, 7. A card
16. If 0 < x < 1, then 1 + x 2 {x cos(cot −1 x ) + sin(cot −1 x)}2 − 1
is drawn from each of the boxes/ Let xi be the number on the
is equal to
card drawn from the i th box, i = 1, 2, 3.
x
a. b. x
1 + x2 19. The probability that x1 + x2 , x3 is odd, is
c. x 1 + x 2 d. 1 + x2 29 53
a. b.
105 105
SECTION 3 Contains 2 Paragraph Type Questions 57 1
c. d.
Each paragraph describes an experiment, a situation or a problem. 105 2
Two multiple choice questions will be asked based on this paragraph.
20. The probability that x1 , x2 , x3 are in an arithmetic
One or more than one option can be correct.
progression, is
Paragraph for Question No. 17 to 18 9 10
a. b.
Let a n denote the number of all n-digit positive integers formed 105 105
by the digits 0, 1 or 11 7
c. d.
both such that no consecutive digits in them are 0. Let b n = the 105 105

Space for rough work


Mock Test-4 189
ANSWER & SOLUTIONS 6. (c) (1 + x + x + x ) = (1 + x ) (1 + x )
2 3 5 5 2 5

JEE-Main = (1 + 5 x + 10 x 2 + 10 x 3 + 5 x 4 + x 5 )
1. 2. 3. 4. 5. 6. 7. 8. 9. 10. ×(1 + 5 x 2 + 10 x 4 + 10 x 6 + 5 x 8 + x10 )
b d c d a c b b d a Therefore the required sum of coefficients
11. 12. 13. 14. 15. 16. 17. 18. 19. 20.
= (1 + 10 + 5).25 = 16 × 32 = 512
b a c a d a c b a d
21. 22. 23. 24. 25. 26. 27. 28. 29. 30. Note: 2 n = 25 = Sum of all the binomial coefficients in
a b a b b a d a a b the 2nd bracket in which all the powers of x are even.

 x x2 x3 
log( x − 1) log( x − 3) 7. (b) (1 − 2 x + 3 x 2 )e − x = (1− 2x + 3x2 ) 1− + − + 
1. (b) Given =  1! 2! 3! 
2 log 2 log 2
 1 1  1
⇒ ( x − 1) = ( x − 3) 2 ∴ The coefficient of x5 = 1 −  + (−2)   + 3  − 
 5 !   4 !  3 !
⇒ x = 2,5
1 1 1 71
But at x = 2, given log is not defined. =− − − =− .
120 12 2 120
2. (d) x 2 + 5 | x | + 4 = 0
8. (b) Suppose x1 x2 x3 x4 x5 x6 x7 represents a seven digit number.
⇒ | x |2 +5 | x | + 4 = 0
Then x1 takes the value 1, 2, 3, ,9 and x2 , x3 , x7 all
⇒ | x | = −1, −4, which is not possible. Hence, the given
take values 0, 1, 2, 3, , 9. If we keep x1 , x2 , , x6 fixed,
equation has no real root.
then the sum x1 + x2 + + x6 is either even or odd. Since
3. (c) x − x + x − 1 = 0
4 3
x7 takes 10 values 0, 1, 2, ,9, five of the numbers so
⇒ x ( x − 1) + 1( x − 1) = 0
3
formed will be even and 5 odd.
Hence the required number of numbers
x − 1 = 0 or x 3 + 1 = 0
= 9 .10 .10.10 .10.10 . 5 = 4500000.
1 + 3i 1 − 3i
⇒ x = 1, − 1, , 9. (d) It is given that P( E ) ≤ P( F ) ⇒ E ⊆ F
2 2
So its real roots are 1 and −1. and P( E ∩ F ) > 0 ⇒ E ⊂ F . . .(ii)
(a) Occurrence of E ⇒ occurrence of F [from Eq.(i)]
 1 0 0 (b) Occurrence of F ⇒ occurrence of E [from Eq.(ii)]
4. (d) A =  5 2 0  (c) Non-occurrence of E ⇒ non-occurrence of F [from
 −1 6 1  Eq.(i)]
 2 −5 32   2 0 0
T
π  π  1 + tan θ 1 − tan θ
10. (a) tan  + θ  − tan  − θ  = −
⇒ adj ( A) =  0 1 −6  =  −5 1 0  .
  4  4  1 − tan θ 1 + tan θ
4 tan θ  2 tan θ 
 0 0 2   32 −6 2  = = 2  = 2 tan 2θ .
1 − tan θ
2
 1 − tan θ 
2

5. (a) 0.234 = 0.2343434..... 11. (b) It is understandable.


0.2 + 0.034 + 0.00034 + 0.0000034 + ... 12. (a) P
Q
34 34 34
= 0.2 + + + + .....∞ l
1000 100000 10000000
10°
2  1 1 1 
= + 34  3 + 5 + 7 + ........∞  38°
10 10 10 10  S 2.05 m O
2  1/10  2 3
1 100 sin 38 o
sin( SPO )
= + 34   = + 34 × × =
10  1 − 1/1000  10 1000 99 l 2.05
2 34 232 sin(180o − 38 o − 90 o − 10 o ) 2.05 sin 38 o
= + = . = ⇒ l=
10 990 990 2.05 sin 42 o
190 Mathematics
1 3 x 2
13. (c) lim+ f ( x) = x 2 sin , 17. (c) Required area = ∫ dx + ∫ 4 − x 2 dx
0 3
x →0 x 3
1 3 2
but −1 ≤ sin ≤ 1 and x → 0 1  x2  x 4 x
x =   + 4 − x 2 + sin −1 
3  2  0  2 2 2
Therefore, lim+ f ( x) = 0 = lim− f ( x) = f (0)
3
x →0 x →0
3  3 2π  π Y
Hence f ( x) is continuous at x = 0. = + π − − = . x= 3y
2  2 3  3
 ( x − a )( x − b)  Trick: Area of sector made
14. (a) y =  
 ( x − c )( x − d ) 
(2,0)
θ cR2 X
by an arc =
1 2
⇒ log y = [log( x − a ) + log( x − b) − log( x − c) − log( x − d )]
2 π 4 π
= . = .
Differentiating w.r.t. x we get 6 2 3
1 dy 1  1 1 1 1  dy 2 2
=  + − −  18. (b) = 2 ⇒ dy = 2 dx, Now integrate it.
y dx 2  ( x − a) ( x − b) ( x − c) ( x − d )  dx x x
dy y  1 1 1 1  19. (a) Here, equation of line is y = x tan α + c, c > 0
Thus =  + − − .
dx 2  ( x − a) ( x − b) ( x − c) ( x − d )  Length of the perpendicular drawn on line from point
(a cos α , a sin α )
x2 −1 x2 + 1 − 2 2
15. (d) f ( x) = = = 1− 2 − a sin α + a cos α tan α + c c
x2 + 1 x2 + 1 x +1 p= ; p= = c cos α
1 + tan α 2 sec α
∴ f ( x) < 1∀x
2 20. (d) It is a fundamental concept.
and ≥ − 1 as ≤2
x +1
2
21. (a) y
y=x
∴ −1 ≤ f ( x ) < 1
P F
Hence f ( x) has minimum value –1 and also there is no (2,2)
N
maximum value. M
(1,1) V
( x + 1)2 x − ( x − 1)2 x
2 2
4x x+y–2=0
Alternate: f ′( x) = = 2 x' x
( x 2 + 1) 2 ( x + 1) 2 O

f ′( x) = 0 ⇒ x = 0
y'
( x + 1) 4 − 4 x.2( x + 1)2 x
2 2 2
Since, distance of vertex from origin is 2 and focus is
f ′′ ( x) =
( x 2 + 1) 4
2 2.
( x 2 + 1)4 − 16 x( x) −12 x 2 + 4 ∴ V(1, 1) and F(2, 2) (ie, lying on y = x)
= =
( x 2 + 1)3 ( x 2 + 1)3 where, length of latusrectum = 4a = 4 2 (Q a = 2)
∴ f ′′(0) > 0 ∴ By definition of parabola PM 2 = (4 a )( PN )
∴ There is only one critical point having minima. Where, PN is length of perpendicular upon x + y − 2 = 0
Hence f ( x) has least value at x = 0. (ie, tangent at vertex).
−1 ( x − y )2  x+ y−2
f min = f (0) = = −1. ⇒ = 4 2 
1 2  2 

16. (a) Put a x = t ⇒ a x log e a dx = dt , then ⇒ ( x − y ) 2 = 8( x + y − 2)


r r r r
ax 1 dt 22. (b) a ⋅ b = − | a | | b | , (Q cos θ = −1)
∫ 1 − a2x
dx =
log e a ∫ 1− t2
23. (a) Here, l = cosθ , m = cos β , n = cos θ , (Q l = n)
1 sin −1 (a x )
= sin −1 (t ) + c = + c. Now, l 2 + m 2 + n 2 = 1
log e a log e a
Mock Test-4 191
⇒ 2cos θ + cos β = 1
2 2 JEE Advance Paper-I
⇒ 2 cos θ = sin β Given, sin β = 3sin θ
2 2 2 2
1. 2. 3. 4. 5. 6. 7. 8. 9. 10.
4 5 3 4 2 9 2 3 a b,c,d
⇒ 2 cos 2 θ = 3sin 2 θ ⇒ 5cos 2 θ = 3,
11. 12. 13. 14. 15. 16. 17. 18. 19. 20.
3
∴ cos 2 θ = . a b b c a,b c b,d d b a
5
kπ π
24. (b) (~ (~ p)) ∧ q = p ∧ q . 12 i i
Σ e 7
e 7 −1
k =1
12
25. (b) α + β = −6 . . .(i) 1. (4) π
= =4
3 i 3
αβ = λ . . .(ii) Σ ei (4 k − 2) e 7 − 1
k =1

and given 3α + 2β = −20 . . .(iii)


Solving (i) and (iii), we get β = 2, α = −8 2. (5) Coeff. x 2

Substituting these values in (ii), we get λ = −16 . ⇒


2
C2 + 3C2 + 4C2 + ....... + 49C2 + 50C2 m2 = (3n + 1) 51C3

26. (a) Since a , b, c, d are in H.P., therefore b is the H.M. of ⇒


3
C3 + 3C2 + 4C2 + ....... + 49C2 + 50C2 m2 = (3n + 1) 51C3
2ac ⇒
n
Cr + nCr −1 = n+1Cr ⇒ 50
C3 + 50C2 ⋅ m2 = (3n + 1) 51C3
a and c i.e. b = and c is the H.M. of b and d
a+c 51 50
2bd 2ac 2bd ⇒
50
C3 + 50C2 + (m2 − 1) 50C2 = 3n ⋅ C2 + 51C3
i.e. c = , ∴ ( a + c)(b + d ) = . 3
b+d b c

51
C3 + (m2 − 1) 50C2 = 51n ⋅ 50C2 + 51C3
⇒ ab + ad + bc + cd = 4ad ⇒ ab + bc + cd = 3ad .
1 1 1 ⇒ m2 − 1 = 51n m2 = 51n + 1
Trick : Check for a = 1, b = , c = , d = .
2 3 4 Min value of m2 for 51n + 1 is integer for n = 5.
27. (d) We have y = 3 x + 6 x 2 + 10 x 3 + .... 3. (8) Let coin was tossed ‘n’ times
∴ 1 + y = 1 + 3 x + 6 x 2 + 10 x 3 + ... 1 n
Probability of getting at least two heads = 1 −  n + n 
⇒ 1 + y = (1 − x ) −3
⇒ 1 − x = (1 + y ) −1/ 3
⇒ x = 1 − (1 + y ) −1/ 3
 2 2 
28. (a) Total number of arrangements of word BANANA  n + 1
⇒ 1 −  n  ≥ 0.96
6!  2 
= = 60
3!2! 2n
⇒ ≥ 25 ⇒ n ≥ 8
The number of arrangements of words BANANA in n +1
5!
which two N’s appear adjacently = = 20 4. (4)
3! 1
Required number of arrangements = 60 – 20 = 40
2 3
29. (a) Let y1 = sin −1 x and y 2 = cos −1 1 − x 2 –2 –1 1

Differentiating w.r.t. x of y1 and y 2 , we get  x − 1, 1 ≤ x < 2


f ( x) =  f (x) is periodic with period 2
dy1 1 1 − x , 0 ≤ x < 1
=
dx 1 − x2 10

dy2 1 1(−2 x) 1 dy
∴ I= ∫
−10
f ( x ) cos π x dx
=− = ⇒ 2 = 1.
dx 1 − (1 − x ) 2 1 − x
2
1− x 2 dy1 10 2
= 2 ∫ f ( x ) cos π x dx = 2 × 5∫ f ( x ) cos π x dx
30. (b) Let y = f ( x ) = x 2e − x
0 0

 1
 2
dy 10 =  ∫ (1 − x ) cos π xdx + ∫ ( x − 1) cos π x dx  = 10( I1 + I 2 )
⇒ = 2 xe− x − x 2e− x = e− x (2 x − x 2 ) 0 
dx 1

Hence f '( x) ≥ 0 for every x ∈ [0, 2], therefore it is non-


2
I 2 = ∫ ( x − 1) cos π x dx put x − 1 = t
decreasing in [0, 2]. 1
192 Mathematics
2
1
1= a m −b
2 2
I 2 = − ∫ t cos π t dt
0 1 = 4a2 − b2
1 1
I1 = − ∫ (1 − x ) cos π x dx = − ∫ x cos(π x ) dx 4e 2
1= − b2
0 0 4
1
 1
  sin π x cos π x  b2 = e2 = 1. Also, b 2 = a 2 ( e1 − 1)
∴ I = 10  −2∫ x cos π x dx  = −20  x +
 0   π π 2  0 ∴ a = 1, e = 2

 1 1  40 π 2
r r2 r r2 r r2
= −20  − 2 − 2  = 2 ∴ I = 4 8. (3) a − b + b − c + c − a = 9
 π π  π 10
r r r r r r
1 ⇒ 2 − 2a ⋅b + 2 − 2 b ⋅c + 2 − 2 c ⋅ a = 9
d2
5. (2) ∫ 4 x 3 (1 − x 2 ) 5 dx
dx 2 3 r r r r r r
− = a ⋅b + b ⋅c + c ⋅a
I II
0 ⇒ . . .(i)
1 1
2
 d  d r r r
=  4 x 3 (1 − x 2 )5  − ∫12 x 2 (1 − x 2 )5 dx Now, | a + b + c |2 ≥ 0
 dx 0 0 dx
r r r r r r
1  1
1
 ⇒ 1+1+1+ 2 a ⋅b + b ⋅c + c ⋅ a ≥ 0
=  4 x 3 × 5(1 − x 2 )4 (−2 x )  − 12   x 2 (1 − x 2 )5  − ∫ 2 x (1 − x 2 )5 dx 
0

0
 r r r r r r 3
a ⋅b + b ⋅c + c ⋅ a ≥ −
0
⇒ . . .(ii)
1 2
1
 (1 − x 2 )6 
= 0 − 0 − 12[0 − 0] + 12∫ 2 x(1 − x 2 )5 dx = 12 ×  −  Equation (i) and (ii) are simultaneously true
0  6 0
r r r r r r 1
 1 If a ⋅ b = b ⋅ c = c ⋅ a ≥ −
= 12 0 +  = 2 2
 6 r r r 2
Now, 2a + 5b + 5c
6. (9) y − y1 = m ( x − x1 )
r r r r r r
= 4 + 25 + 25 + 20 a ⋅ b + 50b ⋅ c + 20( a ⋅ c )
Put x = 0, to get y intercept y1 = mx1 = x1
3

r r r
dy = 54 − 10 − 25 − 10 = 9 ⇒ 2a + 5b + 5c = 3
y1 − x1 = x13
dx
dy 9 (a) y = sec(tan −1 x)
x − y = − x3
dx dy 1
⇒ = sec(tan −1 x ).tan (tan −1 x ).
dy y dx 1 + x2
− = − x2
dx x  dy  2 1
1
⇒  dx  = 1 + 1 =
f dx 1   x =1 2
e x
= e− ln x =
x 1
10. (b,c,d) For f (x) = x cos   , x ≥ 1
1 1 x
y × = ∫ − x 2 × dx
x x
1 1 1
y y x2 f '(x) = cos   + sin   → 1forx → ∞
= −∫ xdx ⇒ = − + c x x x
x x 2
1 1 1 1 1 1
x3 3 also f '(x) = 2
sin   − 2 sin   − 3 cos  
⇒ f ( x) = − + x ∴ f (−3) = 9. x x x x x x
2 2
1 1
a  = cos   < 0 forx ≥ 1
(2) Substituting  , 0  in y = −2x + 1
3
7. x x
e 
⇒ f '(x) is decreasing for [1, ∞)
2a
0=− +1 ⇒ f '(x + 2) < f '(x). Also,
e
2a
=1 a =
e  1 1
lim f (x + 2) − f (x) = lim (x + 2)cos − x cos  = 2
e 2 x →∞ x →∞
 x + 2 x 
Also, 1 = a2 m2 − b2 ∴ f (x + 2) − f (x) > 2∀x ≥ 1
Mock Test-4 193
f ( x) x 14. (c) Since, ∆ is isosceles, hence centroid is the desired
11. (a) Here ff ( x ) = =
[1 + f ( x ) n ]1/ n (1 + 2 x n )1/ x point.
x
⇒ fff ( x ) = (3, 4)
(1 + 3 x n )1/ n
x
⇒ g ( x ) = ( f o f oK o f )( x ) =
n times (1 + nx n )1/ n
xn−1dx
Hence I = ∫ x n−2 g ( x) dx = ∫
(1 + nx n )1/ n (0, 0) (6, 0)

d 15. (a,b) Slope of tangent = m = 2 Equation of tangent in


(1 + nx n )
1 n 2 x n −1dx 1 dx slope form is
= 2∫ = 2 ∫ dx
n (1 + nx ) n 1/ n
n (1 + nx n )1/ n
y = mx ± a 2 m 2 − b 2 , y = 2 x ± 4 2
1
1 1−
∴ I= (1 + nx ) n n
+K  ma 2 −b 2 
n(n − 1) and point of contact is  − , 
 c c 
12. (b) F (0) = 0
 2×9 4   9 1 
F ′( x ) = 2 xf ( x ) = f ′( x ) ≡ − ,−  ≡ ± ,± 
 ±4 2 ±4 2   2 2 2
2
+c
f ( x) = e x
R Q
2
f ( x ) = e x (Q f (0) = 1) 16. (c)
T
x2
(α , β , γ )
F ( x ) = ∫ e dx x
S P
0
2
F ( x) = e x − 1 (Q F (0) = 0) ⇒ F (2) = e 4 − 1 iˆ ˆj kˆ
1 uuur uuur 1
Area of base (PQRS) = | PR × SQ |= 3 1 −2
dy x x4 + 2x 2 2
13. (b) + 2 y= 1 −3 −4
dx x − 1 1 − x2
1
This is a linear differential equation = | −10iˆ + 10 ˆj − 10kˆ |= 5 | iˆ − ˆj + kˆ |= 5 3
x 1 2
∫ x2 −1dx ln| x 2 −1|
I.F. = e = e2 = 1 − x2 1− 2 + 3 2
Height = proj. of PT on iˆ − ˆj + kˆ = =
x ( x 3 + 2) 3 3
⇒ Solution is y 1 − x = ∫ 2
⋅ 1 − x dx
2

1 − x2  2 
Volume = (5 3)   = 10 cu. units
x5  3
or y 1 − x 2 = ∫ ( x 4 + 2 x )dx = + x 2 + c
5
17. (b, d) Let the required plane be x + z + λ y − 1 = 0
f (0) = 0 ⇒ c = 0
| λ −1 | 1
x5 ⇒ =1 ⇒ λ = −
⇒ f ( x) 1 − x 2 = + x2 λ +22 2
5
3/2 3/2
x2 ⇒ P3 ≡ 2 x − y + 2 z − 2 = 0
Now, ∫ f ( x )dx = ∫ 1 − x2
dx (Using property)
| 2α − β + 2γ − 2 |
− 3/2 − 3/2
Distance of P3 from (α , β , γ ) is 2 =2
π /3 4 ×1 + 4
sin θ
3/2 2 2
x
= 2 ∫
0 1− x 2
dx = 2 ∫
0
cos θ
cos θ dθ ⇒ 2α − β + 2λ + 4 = 0 and 2α − β + 2λ − 8 = 0

(Taking x = sin θ ) 18. (d) ~ [~ s ∨ (~ r ∧ s )]


π /3 π /3
θ sin 2θ  = ~ (~ s )∧ ~ (~ r ∧ s )
= 2 ∫ sin 2 θ dθ =  −
2 4  0
0 = s ∧ (r ∨ ~ s)
π   3 π 3 = (s ∧ r ) ∨ (s ∧ ~ s)
= 2   − 2   = − .
6  8  3 4
= s∧r∨ F = s∧r
194 Mathematics
2i ( x + iy ) 2i ( x + iy ) JEE Advance Paper-II
19. (A) z = =
1 − ( x + iy ) 2 1 − ( x 2 − y 2 + 2ixy )
1. 2. 3. 4. 5. 6. 7. 8. 9. 10.
Using 1 − x 2 = y 2 4824 1004 50 3 40 30 1601 500 c d
2ix − 2 y 1 11. 12. 13. 14. 15. 16. 17. 18. 19. 20.
Z= =− .
2 y 2 − 2ixy y c a,d b c a,c,d c a b b c

1 1
Q −1 ≤ y ≤ 1 ⇒ − ≤ −1 or − ≥ 1.  π [ x]  2π
y y 1. (4824) The period of sin   is = 24 as
 12  π /12
8.3x − 2
(B) For domain −1 ≤ ≤1  π [ x + 24]   π ([ x] + 24) 
1 − 32( x −1) sin   = sin  
 12   12 
3x − 3x − 2
⇒ −1 ≤ ≤ 1.  π [ x]   π [ x] 
1 − 32 x − 2 = sin  2π +  = sin  
 12   12 
3 x − 3x − 2
Case (i): −1 ≤ 0  π [ x] 
1 − 32 x − 2 Similarly the period of tan   is 3 and the period of
 3 
(3x − 1)(3x − 2 − 1)
⇒ ≥0
(32 x − 2 − 1) π x  2π
cos   is =8
⇒ x ∈ ( −∞, 0] ∪ (1, ∞ ).
 4  π /4
Hence, the period of the given function
3 x − 3x − 2 λ = LCM of 24, 8, 3 = 24
Case (ii): +1 ≥ 0
1 − 32 x − 2
∴ 201 λ = 201× 24 = 4824
(3x − 2 − 1)(3x + 1)
⇒ ≥0  b 
(3x.3x − 2 − 1)  x− a 
 ax − b  −1 −1
⇒ x ∈ ( −∞, 1) ∪ [2, ∞ ). 2. (1004) Q y = tan   = tan  
 bx + a  1+ b ⋅ x 
So, x ∈ (−∞, 0] ∪ [2, ∞)  a 
(C) R1 → R1 + R3 b
= tan −1 x − tan −1  
a
0 0 2
dy 1
f (θ ) = − tan θ 1 tan θ = 2(tan 2 θ + 1) = 2sec 2 θ . ∴ = −0
dx 1 + x 2
−1 − tan θ 1
dy 1 1 1
∴ = = =
3 1/ 2 15 1/ 2 dx 1 + (−1) 1 + 1 2
2
(D) f ′( x) = ( x) (3x − 10) + ( x) × 3 = ( x) ( x − 2)
3/ 2 x = −1
2 2
dy 1
Increasing, when x ≥ 2. ⇒ (2008) = 2008 × = 1004
dx x = −1 2
( x − 1)( x − 5)
20. f ( x) = . The graph of f ( x) is shown π 
( x − 2)( x − 3) PL  d 2  N
3. (50) I =
PLAN
= 4 
y
33000 33000
∆I ∆P ∆L 2∆d ∆N
y= ∴ = + + +
I P L d N
⇒  ∆I   ∆P   ∆L   ∆d   ∆N 
1 2 3 5 x  ×100  =  ×100  +  ×100  + 2  ×100  +  ×100 
 I   P   L   d   N 
= 10% + 10% + 2 ×10% + 10% = 50%
(A) If −1 < x < 1 ⇒ 0 < f ( x) < 1 ∴ λ = 50
(B) If 1 < x < 2 ⇒ f ( x) < 0  2x + 2 
4. (3) Let I = ∫ sin −1   dx
(C) If 3 < x < 5 ⇒ f ( x) < 0  (4 x 2 + 8 x + 13) 
 
(D) If x > 5 ⇒ f ( x) < 1
Mock Test-4 195
 2x + 2  7. (1601) Solving the equation (2 + c ) x + 5c y = 1 and
2

I = ∫ sin −1   dx
 (2 x + 2)2 + 32  3x + 5 y = 1,
 
Put 2 x + 2 = 3tan θ  1 − 3x 
(2 + c) x + 5c 2   = 1, then, (2 + c ) x + c (1 − 3 x ) = 1
2

∴ 2 dx = 3sec θ dθ
2  5 
1 − c2
3sec2 θ dθ 3 ∴ x=
Then I = ∫ θ ⋅ = {θ ⋅ tan θ − ln secθ } + c 2 + c − 3c 2
2 2
(1 + c )(1 − c ) 1+ c
or x = =
3  2 x + 2   2 x + 2  
2
−1  2 x + 2  (3c + 2)(1 − c ) 3c + 2
=   ⋅ tan   − ln 1 +   + c
2  3   3   3   1+ c 2
  ∴ x = lim ⇒ x=
3c + 2
c →1 5
 2x + 2  3
= ( x + 1) tan −1   − ln(4 x + 8 x + 13) + c
2
1 − 3 x 1 − (6 / 5) 1
 3  4 ∴ y= = =−
5 5 25
3
Hence λ = − 2 1 
4 Therefore the centre of the required circle is  , − 
 5 25 
Then, −4λ = 3
but circle passes through (2,0)
dx 2   1
2

2
5. (40) = x +1
dt ∴ Radius of the required circle =  − 2 +  − − 0
 5   25 
dx
⇒ = dt
x +1 64 1 1601 1601 λ
= + = = =
⇒ ln( x + 1) = t + c 25 625 625 25 25
∴ λ = 1601
Putting t = 0, x = 0 we get c = 0
r r r r
⇒ t = ln( x + 1) 8. (500) Let PV of A, B, C and D be a , b , c and 0
uuur uuur r r r r r r r
For x = 99, t = ln100 = 2log e 10 ∴ AB × CD = (b − a ) × − c = − b × c + a × c
uuur uuur r r r r r r r
∴ 20λ log10 e = 20 × 2log e 10 × log10 e = 40 BC × AD = ( c − b ) × − a = − c × a + b × a
uuur uuur r r r r r r r
6. (30) Since, centroid divides the orthocenter and circum and CA × BD = ( a − c ) × −b = − a × b + c × b
center in the ratio 2 : 1 (internally) and if centroid Adding all we get
uuur uuur uuur uuur uuur uuur r r r r r r
2 1 3 2 AB × CD + BC × AD + CA × BD = − 2 ( a × b + b × c + c × a )
G ( x, y ), then O (1, 1) G ( x, y ) C ,  uuur uuur uuur uuur uuur uuur
2 4 r r r r r r
∴ | AB × CD + BC × AD + CA × BD | = 2 | a × b + b × c + c × a |
3 3 r r r r uuur uuur
2 × + 1× 1 2 × + 1× 1 = 2 | ( c − a ) × (b − a ) | = 2 | AC × AB |
2 4 4 5
x= = and y = =
2 +1 3 2 +1 6 1 uuur uuur
= 4 ⋅ | AC × AB | = 4 (Area of ∆ABC )
4 5 2
∴ Centroid is  ,  and nine point centre is the mid point ∴ λ=4
3 6
of orthocenter and circumcentre. Then, 125 λ = 500
 1 + ( 3/ 2 ) 1 + ( 3/ 4 )  9. (c) A = [ x, y ] B = {a, b, c, d}
∴ Nine point centre is  , ,
 2 2  A × B Having 2 × 4 = 8 elements
5 7 Total subsets of A × B is 28 = 256
i.e.,  , 
4 8 ∴ Total no. of subsets of A × B having 3 or more elements


4
a= ,β =
5 5
and γ = , δ =
7  
3 6 4 8 = 256 −  1 + 8 + 8 C2 
 null set single ton set subset having 
∴ 6α + 12β + 4γ + 8δ  2 elements 
4 5 5 7 = 256 − 1 − 5 − 28 = 219
= 6× + 12 × + 4 × + 8 × = 8 + 10 + 5 + 7 = 30
3 6 4 8
196 Mathematics

10. (d) f ( x) = 2 x + 3x + k 3
 
 1 
f ′( x) = 6 x 2 + 3 f ′( x) = 0 Also, 2cosθ (1 − sin φ ) = sin θ 
2
 cos φ − 1
 sin cos θ
θ 
1  2 2 
⇒ x2 = −
2 ⇒ 2cos θ − 2 cos θ sin φ = 2sin θ cos φ − 1
Not possible. As condition for two distinct real root is ⇒ 1 + 2 cos θ = 2sin(θ + φ )
f (α ) f ( β ) = 0 (where are α , β roots of f ′( x) = 0) 1
⇒ sin(θ + φ ) = + cosθ
z1 − 2 z 2 2
= 1 ⇒ | z1 − 2 z2 | = | 2 − z1 z2 |
2 2
11. (c)
2 − z1 z 2 1 π 4π
⇒ < sin(θ + φ ) < 1 ⇒ < φ <
2 2 3
⇒ ( z1 − 2 z2 )( z1 − 2 z2 ) = (2 − z1 z2 )(2 − z1 z2 )
1/ 2
⇒ z1 z1 + 4 z2 z2 = 4 + z1 z1 z2 z2 16. (c) 1 + x 2  ( x cos cot −1 x + sin cot −1 x ) 2 − 1
⇒ 4 + | z1 |2 | z2 |2 − 4 | z2 |2 − | z1 |2 = 0  2

1/ 2
x 1 
= 1 + x  x cos cos −1
2
+ sin sin −1 
 − 1
⇒ (| z1 |2 −1) ⋅ ( | z2 |2 − 4) = 0  1 + x2 1 + x2  
 
But | z2 | ≠ 1, ∴ | z2 | = 2 1/ 2
 x 2 1 
2

Hence, z lies on a circle of radius 2 centered at origin. = 1 + x 2  +  − 1
 1 + x 2
1 + x2  
1 4 4   
12. (a,d) adj P =  2 1 7  ⇒ | adj P |= 4 = 1 + x2 ( x2 + 1 − 1)1/ 2 = x 1 + x 2 .
1 1 3 
17. (a) a1 = 1 a2 = 2 a3 = 3 a4 = 5
We know, | adj P | = | P |n −1 (where n is order of matrix)
an is following Fibonacci series. Hence a17 = a16 + a15 .
⇒ | adj P | = | P |2 ⇒ 4 = | P |2 ⇒ | P | = ± 2
18. (b) 1 1
2 2 2 2 2
 8   12   16   20   24  Total no of ways exactly two consecutive two is = 5 ways
13. (b)   +   +   +   +   + ...
5  5   5   5   5  Total no of ways exactly three consecutive two is = 2
ways
82 122 162 202 242 (4n + 4) 2
= 2
+ 2 + 2 + 2 + 2 + .... Tn = Total no of ways exactly four consecutive two is = 1 ways
5 5 5 5 5 52
10
So, b6 = 5 + 2 + 1 = 8
1 16 10 2
Sn =
52
∑16(n + 1)
n =1
2
= ∑ (n + 2n + 1)
25 n =1 19. (b) Case (i): One odd, 2 even
16 10 × 11× 21 2 × 10 × 11  16 16 Total number of ways = 2×2×3+1×3×3+1×2×4 = 29.
=  + + 10  = × 505 = m Case (ii): AII 3 odd
25  6 2  25 5
Number of ways = 2×3×4 = 24
⇒ m = 101
Favourable ways = 53
14. (c) Number of required ways
53 53
= 5!− {4 ⋅ 4}!− 4 C2 ⋅ 3!+ 4 C3 ⋅ 2!− 1} = 53. Required probability = = .
3 × 5 × 7 105
15. (a,c,d,) Conditions: 20. (c) Here 2 x2 = x1 + x3
3 ⇒ x1 + x3 = even
− tan(θ ) > 0 ⇒ tan θ < 0 and −1 < sin θ < −
2 Hence number of favourable ways
 3π 5π  1 = 2C1 ⋅4 C2 +1 C 1 ⋅3 C1 = 11.
∴ θ ∈  ,  ⇒ 0 < cosθ <
 2 3  2
Mock Test-5 197
JEE-MAIN: MATHEMATICS MOCK TEST-5
1. The domain of definition of the function y ( x ) given by 10. sin(π + θ ) sin(π − θ ) cosec2θ =
the equation 2 x + 2 y = 2 is: a. 1 b. –1 c. sin θ d. − sin θ
a. 0 < x ≤ 1 b. x ≤ x ≤ 1
11. sinh 2 x equals
c. −∞ < x ≤ 0 d. −∞ < x < 1.
a. cosh 2 x − 1 b. cosh 2 x + 1
2. A real root of the equation log 4 {log 2 ( x + 8 − x )} = 0 is 1 1
c. (cosh 2 x − 1) d. (cosh 2 x + 1)
a. 1 b. 2 c. 3 d. 4 2 2

3. cosh(α + i β ) − cosh(α − i β ) is equal to 12. A tower subtends an angle α at a point A in the plane of
a. 2 sinh α sinh β b. 2 cosh α cosh β its base and the angle of depression of the foot of the
tower at a point l meters just above A is β . The height of
c. 2i sinh α sin β d. 2 cosh α cos β
the tower is
 0 −4 1 a. l tan β cot α b. l tan α cot β
4. Matrix  4 0 −5  is
 c. l tan α tan β d. l cot α cot β
 −1 5 0 
13. The value of k so that the function
a. Orthogonal b. Idempotent
c. Skew-symmetric d. Symmetric k (2 x − x 2 ), when x < 0
f ( x) =  is continuous at x = 0, is
20  cos x, when x ≥ 0
an +1 1
5. For a sequence < an >, a1 = 2 and
an
= . Then
3
∑a
r =1
r is a. 1 b. 2
c. 4 d. None of these
20  1 
a. [4 + 19 × 3] b. 3 1 − 20  dy
2  3  14. If y = (1 + x) x , then =
dx
c. 2(1 − 320 ) d. None of these
 x 
a. (1 + x) x  + log ex 
6. Coefficients of x r [0 ≤ r ≤ ( n − 1)] in the expansion of 1 + x 
( x + 3) n −1 + ( x + 3) n − 2 ( x + 2) x
b. + log(1 + x)
+ ( x + 3) n −3
( x + 2) + ... + ( x + 2)
2 n −1 1+ x

a. Cr (3 − 2 n )
n r n
Cr (3n− r − 2 n − r )  x 
b. c. (1 + x) x  + log(1 + x) 
1 + x 
c. n Cr (3r + 2 n− r ) d. None of these
d. None of these
2 3 4
7. 1+ + + + K∞ = 15.
x
is maxima at
3! 5! 7! 1 + x tan x
a. e b. 2e c. e/2 d. e/3 a. x = sin x b. x = cos x
8. The number of ways in which the following prizes be π
c. x = d. x = tan x
given to a class of 20 boys, first and second Mathematics, 3
first and second Physics, first Chemistry and first English 1
is. 16. ∫ x 1 + log x
dx =
a. 204 ×192 b. 203 ×193
2
c. 202 × 194 d. None of these a. (1 + log x )3/ 2 + c b. (1 + log x)3/ 2 + c
3
9. If the integers m and n are chosen at random between 1 c. 2 1 + log x + c d. 1 + log x + c
and 100, then the probability that a number of the form
17. The area of the triangle formed by the tangent to the
7 m + 7 n is divisible by 5, equals
hyperbola xy = a 2 and co-ordinate axes is
1 1 1 1
a. b. c. d. a. a 2 b. 2a 2 c. 3a 2 d. 4a 2
4 7 8 49
198 Mathematics
dy 24. ~ ( p ∨ (~ q )) is equal to
18. The solution of = x log x is
dx a. ~ p ∨ q
x2 b. (~ p ) ∧ q
a. y = x 2 log x − + c
2
c. ~ p ∨ ~ p
x2
b. y = log x − x 2 + c d. ~ p ∧ ~ q
2
25. If the sum of the roots of the equation λ x + 2x + 3λ = 0
2
1 1
c. y = x 2 + x2 log x + c
2 2 be equal to their product, then λ =
d. None of these a. 4
19. The distance of point (–2, 3) from the line x − y = 5 is b. –4
c. 6
a. 5 2
d. None of these
b. 2 5
26. If a , b, c and u , v , w are the complex numbers representing
c. 3 5
the vertices of two triangles such that c = (1 − r )a + rb and
d. 5 3 w = (1 − r )u + rv, where r is a complex number, than the
20. If the coordinates of one end of the diameter of the circle two triangles
x + y − 8x − 4 y + c = 0 are (–3, 2), then the coordinates
2 2 a. have the same area
b. are similar
of other end are
c. are congruent
a. (5, 3) b. (6, 2)
d. None of these
c. (1, –8) d. (11, 2)
21. Let ( x, y) by any point on the parabola y 2 = 4 x. Let P be 2 3 1 4
27. The rank of the matrix, A =  0 1 2 − 1 is
the point that divides the line segment from (0, 0) to 
 0 −2 −4 2 
( x, y) in the ratio 1: 3. Then, the locus of P is
a. 2 b. 3
a. x 2 = y c. 1 d. Indeterminate
b. y 2 = 2 x 28. The coefficient of xn in the expansion of
c. y 2 = x (1 + x + x + ....) is
2 −n

d. x = 2 y
2
a. 1
r r r r r r b. (–1)n
22. If a , b , c are unit vectors such that a + b + c = 0, then c. n
r r r r r r
a ⋅b + b ⋅c + c ⋅ a = d. n + 1
a. 1 29. Three boys and two girls stand in a queue. The probability
b. 3 that the number of boys ahead of every girl is atleast one
c. – 3/2 more than the number of girls ahead of her, is
d. 3/2 a. 1/2
b. 1/3
23. The direction cosines of three lines passing through the
c. 2/3
origin are l1 , m1 , n1 ; l2 , m2 , n2 and l3 , m3 , n3 . The lines will
d. 3 4
be coplanar, if
l1 n1 m1 l1 m2 n3 30. The imaginary part of tan −1 (cos θ + i sin θ ) is
a. l2 n2 m2 = 0 b. l2 m3 n1 = 0 a. tanh −1 (sin θ )
l3 n3 m3 l3 m1 n2 b. tanh −1 (∞)
c. l1l2l3 + m1m2 m3 + n1n2 n3 = 0 1
c. tanh −1 (sin θ )
d. None of these 2
d. None of these
Mock Test-5 199
JEE ADVANCE PAPER-I

SECTION 1 Contains 8 Questions.  y2 y2 


7. If f  2 x 2 + , 2 x 2 −  = xy , then the value of
The answer to each question is a single digit integer ranging from 0 to  8 8 
9 (both inclusive).
f (60, 48) + f (80, 48) + f (13, 5) must be
1. A variable plane is at a constant distance p form the origin
8. If x = a (cos t + t sin t ); y = a (sin t − t cos t ), then the value
and meets the axes in A, B and C. If the locus of the
centroid of the tetrahedron OABC is d2y
of 120) must be
x −2 + y −2 + z −2 = λ p −2 then the value of 160 λ must be dx 2 t =π / 3

2. The normal to the parabola y 2 = 8 x at the point (2, 4) SECTION 2 Contains 10 Multiple Choice Questions
meets it again at (18, –12). If length of normal chord is λ , With one or more than one correct option
then the value of λ must be
2
9. Let A and B be two sets containing four and two elements
3. An electric component manufactured by ‘RASU respectively. Then the number of subsets of the set A × B ,
electronics’ is tested for its defectiveness by a each having at least three elements is:
sophisticated testing device. Let A denote the event “the a. 219 b. 256
device is defective” and B the event “the testing device c. 275 d. 510
reveals the component to be defective”. Suppose 10. The quadratic equation p( x) = 0 with real coefficients
P( A) = α and P( B / A) = P( B '/ A ') = 1 − α , where 0 < α <
< 1.
1. has purely imaginary roots. Then the equation
If the probability that the component is not defective, p( p( x)) − 0 has
given that the testing device reveals it to be defective, is
a. only purely imaginary roots
λ, then the value of 2008 λ must be b. all real roots
4. The coefficient of x 50 in the polynomials after parenthesis c. two real and two purely imaginary roots
have been removed and like terms have been collected in d. neither real nor purely imaginary roots

the expansion (1 + x )1000 + x (1 + x )999 + x2 (1 + x )998 + ... + x100011. A value of θ for which 2 + 3i sin θ is purely imaginary, is:
1 − 2i sin θ
λ!
(1 + x ) 998 + ... + x1000 is , then the value of λ + 2µ +3v must be
µ ! v! π π
a. b.
3 6
(v > µ )
 3  1 
c. sin −1  d. sin −1 
5. The value of  4   3

 
1 −1  7π  −1  2π  −1  5π   −π  
 216 sin  sin  + 27 cos  cos  + 28 tan  tan + cot −1ω cos
200Let
12. 
be a complex cube root of unity with ω ≠ 1 and
π  6   3   4   4  i+ j
P[ Pij ] be a n × n matrix with pij = ω . Then when
 2π  −1  5π  −1  −π  
7 cos −1  cos  + 28 tan  tan  + 200 cot  cos  P 2 ≠ 0, n =
 3   4   4 
a. 57 b. 55
must be c. 58 d. 56
r r r
6. Let a, b and c be three non-coplanar unit vectors such 13. If m is the A.M. of two distinct real numbers l and
π n ( l, n > 1) and G1, G2 and G3 are three geometric means
that the angle between every pair of them is . If
3 between l and n, then G14 + 2G24 + G34 equals,
r r r r r r r
a × b + b × c = pa + qb + rc , where p, q and r are a. 4l 2 mn b. 4l m2 n
p 2 + 2q 2 + r 2 c. 4l mn2 d. 4l 2 m2 n 2
scalars, then the value of is _______
q2
200 Mathematics
4n k ( k +1) SECTION 3 Contains 2 Match The Following Type Questions
14. Let Sn = ∑ (−1) 2
k 2 . Then S n can take value(s)
k =1
You will have to match entries in Column I with the entries in
Column II.
a. 1056 b. 1088 c. 1120 d. 1332
15. If 3x = 4 x −1 , then x = 19. Match the statements given in Column I with the
interval/union of intervals given in Column II
2 log 3 2 2
a. b. Column I Column II
2 log 3 2 − 1 2 − log 2 3
  2iz   1. (−∞, − 1) ∪ (1, ∞)
1 2 log 2 3 (A)The set Re  2 
: z is a
c. d.   1 − z 
1 − log 4 3 2 log 2 3 − 1
complex number,
| z |= 1, z ≠ ±1 is
16. The number of integers greater than 6,000 that can be
formed, using the digits 3, 5, 6, 7 and 8, without (B) The domain of the function 2. (−∞, 0) ∪ (0, ∞)
x −2
repetition, is:  8(3) 
f (x) = sin −1  2( x −1)  is
a. 216 b. 192 c. 120 d. 72  1− 3 
17. A ship is fitted with three engines E1 , E2 and E3 . The 1 tan θ 1 3. [2, ∞ )
engines function independently of each other with (C)If f (θ ) = − tan θ 1 tan θ ,
1 1 1 −1 − tan θ 1
respective probabilities , and . For the ship to be
2 4 4 then the set
operational at least two of its engines must function. Let X  π
denote the event that the ship is operational and let f (θ ) : 0 ≤ θ <  is
 2
X 1 , X 2 and X 3 denote respectively the events that the (D)If f (x) = x 3 / 2 (3x − 10), 4. (−∞, − 1] ∪ [1, ∞)
engines E1 , E2 And E3 are functioning. Which of the x ≥ 0, then f(x) is
following is (are) true? increasing in
a. P[ X 1c | X ] =
3 5. (−∞, 0] ∪ [2, ∞ )
16
b. P [Exactly two engines of the ship are functioning 20. Match the statements/expressions in Column I with the
values given in Column II.
7
[X ] = Column I Column II
8
(A) The number of solutions of the 1. 1
5
c. P[ X | X 2 ] = equation xesin x − cos x = 0 in the
16
 π
7 interval  0, 
d. P[ X | X 1 ] = 2 
16
(B) Value (s) of k for which the plane 2. 2
7 kx + 4 y + z = 0, 4 x + ky + 2 z = 0 and
18. Let PQR be a triangle of area ∆ with a = 2, b = and
2
2 x + 2 y + z = 0 intersect in a
5
c = , where a, b and c are the lengths of the sides of the straight line
2 (C) Value (s) of k for which 3. 3
triangle opposite to the angles at P, Q and R respectively. | x − 1 | + | x − 2 | + | x + 1 | x + 2 |= 4 k
2 sin P − sin 2 P has integer solution (s)
Then equals
2 sin P + sin 2 P (D) If y′y + 1 and y (0) = 1 then value 4. 4
2 2
3 45  3   45  (s) of y (ln2)
a. b. c.   d.  
4∆ 4∆  4∆   4∆  5. 5

Space for rough work


Mock Test-5 201
JEE ADVANCE PAPER-II

SECTION 1 Contains 8 Questions.  2x  1


9. Let tan −1 y = tan −1 x + tan −1   , where | x |< .
 1− x
2
The answer to each question is a single digit integer ranging from 0 to  3
9 (both inclusive). Then a value of y is
1. If the approximate value of log10 (4.04) 0 abcdef, It is 3 x − x3 3x + x 3
a. b.
given that log 4 4 = 0.6021 and log10 e = 0.4343, then the 1 − 3x 2 1 − 3x 2

value of abcd must be


3 x − x3 3x + x 3
c. d.
1 + 3x 2 1 + 3x 2
 tan x − 1 
2. If ∫ ( tan x + cot x )dx = a tan −1   + c, then the
 b tan x  10. If the angles of elevation of the top of a tower from three
value of a 4 + b 5 must be collinear points A, B and C, on a line leading to the foot
of the lower, are 30°, 45° and 60° respectively, then the
d3y d2y ratio, AB : BC is
3. The solution of the differential equation 3
−8 2 = 0
dx dx a. 3 :1 b. 3: 2
1
satisfying y (0) = , y1 (0) = 0and y2 (0) = 1is c. 1: 3 d. 2 : 3
8

e8 x − 8 x + 7
y1 = , then the numerical value of λ must be ( x − 1) n
λ 11. Let g ( x) = ; 0 < x < 2, m and n are
log cos m ( x − 1)
4. If m1 and m2 are the roots of the equation integers, m ≠ 0, n > 0, and let p be the left hand
x + ( 3 + 2) x + ( 3 − 1) = 0 and if area of the triangle derivative of | x − 1| at x = 1. If lim+ g ( x) = p, then
2

x →1
formed by the lines y = m1 x, y = m2 x2 , and y = c is a. n = 1, m = 1 b. n = 1, m = −1
(a + b)c , then the value of 2008 (a + b ) must be
2 2 2
c. n = 2, m = 2 d. n > 2, m = n

5. Tangents are drawn form P(6, 8) to the circle x 2 + y 2 = r 2 , 12. Let g ( x) = log( f ( x)) where f ( x ) is a twice differentiable
then the radius of the circle such that the area of the ∆ formed positive function on (0, ∞) such that f ( x + 1) = x f ( x).
by tangents and chore of contact is maximum must be
Then, for N = 1, 2, 3, ...,
6. ‘P’ is any arbitrary point on the circum circle of the  1 1 1 
a. −4 1 + + + ... + 2
equilateral triangle of side length 26 unit, then the value  9 25 (2 N − 1) 
uuur uuur uuur
of | PA |2 + | PB |2 + | PC |2 must be  1 1 1 
b. 4 1 + + + ... + 2
 9 25 (2 N − 1) 
x + 4 y + 6 z −1
7. The lines = = and 3 x − 2 y + z + 5 = 0  1 1 1 
3 5 −2 c. −4 1 + + + ... + 2
 9 25 (2 N + 1) 
= 2 x + 3 y + 4 z − k are coplanar for k is equal to
 1 1 1 
d. 4 1 + + + ... + 2
8. If a circle cuts a rectangular hyperbola xy = c in A, B, C 2
 9 25 (2 N + 1) 
and D are the parameters of these four points be t1, t2 , t3 ex e− x
13. Let I = ∫ dx , J = ∫ e−4 x + e −2 x + 1 dx. Then for
and t 4 respectively, then the value of 16t1t 2 t3t 4 must be e 4 x + e2 x + 1
an arbitrary constant C , the value of J − I equals
SECTION 2 Contains 8 Multiple Choice Questions 1  e4 x − e2 x + 1 
a. log  4 x 2 x +C
With one or more than one correct option 2  e + e +1 
202 Mathematics

1  e + e +1
2x x 17. The unit vector perpendicular to both L1 and L2 is
b. log  2 x x +C
2  e − e +1  −iˆ + 7 ˆj + 7kˆ
a.
1  e2 x − e x + 1  99
c. log  2 x +C
 e + e +1
x
2 −iˆ − 7 ˆj + 5kˆ
b.
1  e4 x + e2 x + 1  5 3
d. log  4 x 2 x +C
2  e − e +1  −iˆ + 7 ˆj + 5kˆ
c.
π 5 3
x x
14. The integral ∫ 1 + 4sin 2
2
− 4sin dx equals:
2 7iˆ − 7 ˆj − kˆ
0 d.
a. π − 4 99
2π 18. The distance of the point (1, 1, 1) from the plane passing
b. −4−4 3
3 through the point (−1, −2, −1) and whose normal is
c. 4 3 − 4 perpendicular to both the lines L1 and L2 is
π 2
d. 4 3 − 4 − a.
3 75
15. Let the population of rabbits surviving at a time t be 7
b.
government by the differential equation 75
dp (t ) 1 13
= p (t ) − 200. If ( p )0 = 100, then (p)t equals: c.
dt 2 75
a. 400 − 300et / 2 23
d.
−t / 2 75
b. 300 − 200e
c. 600 − 500et / 2
Paragraph for Question No. 19 to 20
d. 400 − 300e−t / 2
Let ABCD be a square of side length 2 units, C2 is the circle
16. A straight line L through the point (3, –2) is inclined at an through vertices, A, B, C, D and C1 is the circle touching all the
angle 60° to the line 3x + y = 1. If L also intersects the sides of the square ABCD. L is a line through A.
x-axis, then the equation of L is 19. If P is a point on C1 and Q in another point on C 2 , then
a. y + 3x + 2 − 3 3 = 0 PA2 + PB 2 + PC 2 + PD 2
is equal to
b. y − 3x + 2 + 3 3 = 0 QA2 + QB 2 + QC 2 + QD 2
a. 0.75
c. 3y − x + 3 + 2 3 = 0 b. 1.25
d. 3y + x − 3 + 2 3 = 0 c. 1
d. 0.5
SECTION 3 Contains 2 Paragraph Type Questions 20. A line M through A is drawn parallel to BD. Point S
Each paragraph describes an experiment, a situation or a problem. moves such that its distances from the line BD and the
Two multiple choice questions will be asked based on this paragraph. vertex A are equal. If locus of S cuts M at T2 and T3 and
One or more than one option can be correct. AC at T1, then area of ∆ T1 T2 T3 is
Paragraph for Question No. 17 to 18 1
a. sq. unit
Consider the line 2
x +1 y + 2 z +1 2
L1 : = = , b. sq. unit
3 1 2 3
x −2 y +2 z −3 c. 1 sq. unit
L2 : = =
1 2 3 d. 2 sq. unit
Mock Test-5 203
ANSWER & SOLUTIONS 1 2 3 4 n
7. (c) S = + + + +K + +K
JEE-Main 1! 3 ! 5 ! 7 ! (2n − 1) !
1 2n 1 (2n − 1) + 1
1. 2. 3. 4. 5. 6. 7. 8. 9. 10. Here Tn = . =
d a c c b b c a a b 2 (2n − 1) ! 2 (2n − 1) !
11. 12. 13. 14. 15. 16. 17. 18. 19. 20. 1 1 1 
c b d c b c b d a d =  + 
2  (2n − 2) ! (2n − 1) !
21. 22. 23. 24. 25. 26. 27. 28. 29. 30.
c c a b d b a b a c 1  e + e−1 e − e−1  e
⇒ S = ∑ Tn =  + = .
2 2 2  2
1. (d) 2 y = 2 − 2 x
Trick: The sum of this series upto 4 terms is 1.359…and
⇒ y = log 2 2 − 2 x for domain 2 − 2 x ⇒ x < 1
this is value of e/2 approximately.
2. {
(a) log 4 log 2 ( x + 8 − x ) = 0 } 8. (a) Four first prizes can be given in 204 ways since first
⇒ 40 = log ( 2 x+8 − x ) prize of Mathematics can be given in 20 ways, first prize
of Physics also in 20 ways, similarly first prizes of
⇒ 21 = x + 8 − x Chemistry and English can be given in 20 ways each.
(Note that a boy can stand first in all the four subjects).
⇒ 4 = x + 8 + x − 2 x 2 + 8x
Then two second prizes can be given in 192 ways since a
⇒ 2 x + 8x = 2x + 4
2
boy cannot get both the first and second prizes. Hence the
⇒ x 2 + 8x = x2 + 4 + 4 x required number of ways
⇒ 4x = 4 = 204 ×192.
⇒ x = 1.
9. (a) 71 = 7, 7 2 = 49, 7 3 = 34, 7 4 = 2401,.....
3. (c) cosh(α + i β ) − cosh(α − i β ) Therefore, for 7 r , r ∈ N the number ends at unit place 7,
= cosh α cosh (i β ) + sinh α sinh(i β ) 9, 3, 1, 7, …..
− cosh α cosh (i β ) + sinh α sinh (i β ) ∴ 7 m + 7 n will be divisible by 5 if it end at 5 or 0.
= 2 sinh α sinh i β = 2i sinh α sin β . But it cannot end at 5. Also for end at 0. For this m and n
should be as follows
4. (c) It is skew-symmetric. m n
1 1 4r 4r − 2
5. (b) The sequence is a G.P. with common ratio .
3 2 4r − 1 4r − 3
a(1 − r n ) 2[1 − (1/ 3) 20 ] 3 4r − 2 4r
Now from ,
1− r 1 − (1/ 3) 4 4 r − 3 4r − 1
 1 
= 3 1 − 20  . For any given value of m, there will be 25 values of n.
 3 
Hence, the probability of the required event is
6. (b) We have ( x + 3) n −1 + ( x + 3) n− 2 ( x + 2) + 100 × 25 1
=
( x + 3) n −3
( x + 2) + .... + ( x + 2)
2 n −1 100 × 100 4
Note: Power of prime numbers have cyclic numbers in
( x + 3) − ( x + 2)n
n
= = ( x + 3) n − ( x + 2)n their unit place.
( x + 3) − ( x + 2)
xn − an 10. (b) sin(π + θ ) sin(π − θ )cosec 2θ
(Q = x n−1 + x n − 2 a1 + x n−3a 2 + .... + a n −1 )
x−a 1
= − sin θ sin θ = −1 .
r
Therefore coefficient of x in the given expression sin 2 θ
= Coefficient of x r in [( x + 3) n − ( x + 2) n ] 1
11. (c) sinh 2 x = (cosh 2 x − 1)
= nCr 3n − r − nCr 2 n− r = nCr (3n − r − 2 n − r ) 2
204 Mathematics
T 1
16. (c) Put t = 1 + log x ⇒ dt = dx, then
12. (b) x
dx dt
P H
∫ x 1 + log x
= ∫ 1/ 2 = 2t 1/ 2 + c = 2(1 + log x)1/ 2 + c.
t
l
α β 17. (b) Y
A O
From figure, we can deduce H = l tan α cot β .
2a
13. (d) f (0−) = lim k (2 x − x 2 ) = 00;
x →0 − X
2a
f (0 + ) = lim cos x = 1 (–a,–a)
x →0+

∴ f (0) = cos x = 1
Hence no value of k can make f (0−) = 1.
a2
Given xy = a 2 or y = . . .(i)
14. (d) y = (1 + x) x x
There are two points on the curve (a, a),(– a,– a)
Taking log on both sides, log y = x log(1 + x)
The equation of the line at (a, a) is,
Differentiating w.r.t. x, we get
−a  2 
 dy 
 ( x − a ) =  x 2 
1 dy 1 y−a = ( x − a)
= log(1 + x) + x  dx (a, a)
y dx (1 + x)   (a,a )

dy  x  y − a = −( x − a) therefore, equation of the tangent at


Thus = (1 + x) x  + log(1 + x) 
dx 1 + x  ( a , a ) is x + y = 2 a. The interception of line x + y = 2 a
with x-axis is 2a and with y-axis is 2a.
x
15. (b) If is maxima, then its reciprocal 1
1 + x tan x ∴ Required area = × 2a × 2a = 2a 2 .
2
1 + x tan x
will be minima.
x dy
18. (d) = x log x
1 + x tan x 1 dx
Let y = = + tan x
x x ⇒ dy = x log xdx
dy 1

dx
= − 2 + sec 2 x,
x
⇒ ∫ dy =∫ x log xdx

d2y 2 x2 x2
= + 2sec x sec x tan x ⇒ y= log x − + c.
dx 2 x 3 2 4
dy 1
On putting = 0, − 2 + sec 2 x = 0 19. (a) Distance of point (–2, 3) from the line x − y = 5 is
dx x
1 −2 − 3 − 5 −10
⇒ sec 2 x = 2 = = 5 2.
x 2 2
⇒ x 2 = cos 2 x 20. (d) Obviously the centre of the circle is (4, 2) which
⇒ x = cos x should be the middle point of the ends of diameter.
d2y 2 Hence the other end is (11, 2).
∴ = + 2sec 2 x tan x
dx 2 cos 3 x y
= 2sec x(sec x + tan x), which is positive.
2 21. (c) (x,y)Q
3
1 + x tan x P(h,k)
At x = cos x , is minimum. 1
x x
(0,0) O
x
So will be maximum.
1 + x tan x y2 = 4x
Mock Test-5 205
x+0 y+0 a u 1
By section formula, h = ,k = Consider b v 1
4 4
c w 1
∴ x = 4h, y = 4k
Applying R3 → R3 − {(1 − r ) R1 + rR2 }
Substituting in y 2 = 4 x , (4 k ) 2 = 4(4 h ) ⇒ k 2 = h
a u 1
Or y 2 = x is required locus. = b v 1
r r r c − (1 − r ) a − rb w − (1 − r ) u − rv 1 − (1 − r ) − r
22. (c) Squaring ( a + b + c ) = 0,
r r r r r r r r r a u 1
we get a 2 + b 2 + c 2 + 2a ⋅ b + 2b ⋅ c + 2c ⋅ a = 0 = b v 1 [from Eq. (i)]= 0
r r r r r r r r r 0 0 0
⇒ | a |2 + | b |2 + | c |2 +2(a ⋅ b + b ⋅ c + c ⋅ a ) = 0
r r r r r r Hence, two triangle are similar.
⇒ 2( a ⋅ b + b ⋅ c + c ⋅ a ) = −3
r r r r r r 3 2 3 1 4
⇒ a ⋅b + b ⋅c + c ⋅ a = − 27. (a) Given A =  0 1 2 − 1 , ( R2 → 2 R2 + R3 )
2 
 0 −2 −4 2 
23. (a) Here, three given lines are coplanar if they have
common perpendicular 2 3 1 4

A = 0 0 0 0 
Let d.c.'s of common perpendicular be l, m, n
 0 −2 −4 2 
⇒ ll1 + mm1 + nn1 = 0 . . .(i)
ll2 + mm2 + nn2 = 0 . . .(ii) Since every minor of order 3 in A is 0 and there exists a
2 3 
and ll3 + mm3 + nn3 = 0 . . .(iii) minor order 3 i.e.   in A which is non-zero. Thus,
0 −2 
Solving (ii) and (iii), we get
rank = 2.
l m n
= = =k
m2 n3 − n2 m3 n2l3 − n3l2 l2 m3 − l3 m2 28. (b) We have, (1 + x + x 2 + ...) − n = [(1 − x) −1 ]− n = (1 − x) n
⇒ l = k ( m2 n3 − n2 m3 ), m = k (n2l3 − n3l2 ), n = k (l2 m3 − l3m2 ) = nC0 − nC1 x + nC2 x 2 + ... + (−1) n nCn .x n
Substituting in (i), we get
Coefficient of x n is (−1)n nCn = (−1)n .
l1 (m2 n3 − n2 m3 ) + m1 (n2l3 − n3l2 ) + n1 (l2 m3 − l3m2 ) = 0
l1 m1 n1 29. (a) Total number of ways to arrange 3 boys and 2 girls are
5!. According to given condition, following cases may
⇒ l2 m2 n2 = 0
arise.
l3 m3 n3
B G G B B
l1 n1 m1 G G B B B
⇒ − l2 n2 m2 = 0 G B G B B
l3 n3 m3 G B B G B
B G B G B
24. (b) ~ ( p ∨ (~ q)) ≡ ~ p ∧ ~ (~ q) ≡ (~ p) ∧ q . So, number of favourable ways = 5 ¥ 3! ¥ 2! = 60
60 1
25. (d) Under condition, ∴ Required probability = =
120 2
2
− =3
λ 30. (c) We know that

⇒ λ=−
2 π i  1 + sin θ 
tan −1 (cos θ + i sin θ ) = + log  ,
3 4 4  1 − sin θ 
26. (b) Since a, b, c and u, v, w are the vertices of two (cos θ ) > 0
triangles.
then the Imaginary part of tan −1 (cosθ + i sin θ ) be
Also, c = (1 − r ) a + rb
1  1 1+ x 
and w = (1 − r ) u + rv . . .(i) tanh −1 (sin θ ) , Q tanh −1 x = log  
2  2  1− x 
206 Mathematics
JEE Advance Paper-I ⇒ y + 8 y − 48 = 0
2

1. 2. 3. 4. 5. 6. 7. 8. 9. 10. ⇒ ( y + 12)( y − 4) = 0
2560 3 1004 3954 139 4 112 2880 a d ∴ y = −12 and y = 4
11. 12. 13. 14. 15. 16. 17. 18. 19. 20.
Then x = 18 and x = 2
d bcd b ad abc b bd c b a
Hence, point of intersection of normal and parabola are
1. (2560) Let the equation of the variable plane be (18, − 12) and (2, 4) therefore normal meets the parabola
 x  y z at (18, − 12) and length of normal chord is distance
  +   +  =1 . . .(i)
a b c between their points
Given that the plane is at a distance p form (0, 0, 0)
= PQ = (18, − 12) 2 + ( − 12 − 4) 2 = 16 2
1
∴ p= = λ (given)
 1   1  2  1 2 
2

  +   +    ∴ λ2 = 5/ 2 ( 2.5 ≈ 3 ) .
 a   b   c  
3. (1004) Given that P( A) = α , P( B / A) = P( B′ / A′) = 1 − α
1 1 1 1
or 2
= 2+ 2+ 2 . . .(ii) Thus, P ( A′) = 1 − P( A) = 1 − α
p a b c
Also, the plane (i) meets the axes in A, B and C. So the and P ( B / A′) = 1 − P ( B′ / A′) = 1 − (1 − α ) = α . . .(i)
coordinates of O, A, B and C are (0, 0, 0), (a, 0, 0), (0, b, P ( A′ ∩ B) P( B) − P( A ∩ B )
∴ P( A′ / B) = =
0) and (0, 0, c) respectively P( B) P( B)
Let ( x, y , z ) be the centroid of the tetrahedron OABC, P ( B) − P ( A) P( B / A)
=
1 1 P( B)
then x = (0 + a + 0) = a
4 4
 P( A ∩ B) 
1 1 Q P( B / A) = 
Similarly y = b and z = c  P( A) 
4 4
or a = 4 x, b = 4 y, c = 4 z P( B) − α (1 − α )
= . . .(ii)
P( B)
Substituting these values in equation (ii), we get
But P ( B ) = P( A) ⋅ P ( B / A) + P ( A′) ⋅ P( B / A′)
1 1 1 1
= + +
p 16 x 16 y 16 z 2
2 2 2 α ⋅ (1 − α ) + (1 − α ) ⋅ α {from equation (i)}
Or x −2 + y −2 + z −2 = 16 p −2 = 2α (1 − α ) . . .(iii)
∴ λ = 16 Putting the value of P( B) from equation (iii) in (ii), then
⇒ 160λ = 160 × 16 = 2560 2α (1 − α ) − α (1 − α ) α (1 − α ) 1
P ( A′ / B ) = = = =λ
2α (1 − α ) 2α (1 − α ) 2
2. (3) Comparing the given parabola (i.e., y 2 = 8xx) with
(given)
y 2 = 4 ax 1
∴ 2008λ = 2008 × = 1004
∴ 4a = 8 2
∴ a=2 4. (3954) Using the formula for the sum of a geometric
progression, we find
Since, normal at ( x1 , y1 ) to the parabola y 2 = 4 ax is
y (1 + x)1000 + x(1 + x)999 + x 2 (1 + x)998 + K + x1000
y − y1 = 1 ( x − x1 )
2a   x 1001  x1001
(1 + x)1000 1 −    (1 + x)1000 −
Here, x1 = 2 and y1 = 4   1 + x   (1 + x )
= = = (1 + x)1001 − x1001
  x  x +1− x
4
Equation of normal is y − 4 = − ( x − 2) 1 −   (1 + x)
∴   1 + x 
4
1001!
⇒ y − 4 = −x + 2 Hence, the coefficient of x 50 = 1001C50 =
50!951!
⇒ x+ y −6 = 0 . . . (i) ∴ λ = 1001, µ = 50 and v = 951
Solve equation (i) and y = 8 x then y = 8(6 − y )
2 2
∴ λ + 2 µ + 3v = 1001 + 100 + 2853 = 3954
Mock Test-5 207
 7π  7π π 9. (a) Set A has 4 elements
5. (139) Q sin −1  sin  =π − =− ,
 6  6 6 Set B has 2 elements
 2π  2π
∴ Number of elements in set ( A × B ) = 4 × 2 = 8
cos −1  cos = ,
 3  3 ∴ Total number of subsets of ( A × B) = 28 = 256
 5π  5π π Number of subsets having 0 elements = 8C0 = 1
tan −1  tan = −π =
 4  4 4
Number of subsets having 1 element each = 8C1 = 8
  π 
−1 π 3π
And cot cot  −   = π − = ∴ Number of subsets having 2 elements each
  4  4 4
8! 8× 7
Hence, required value is = 8C2 = = = 28
2!6! 2
1 π 2π π 3π 
 216 × − + 27 × + 28 × + 200 ×  Number of subsets having at least 3 elements
π 6 3 4 4 
= 256 − 1 − 8 − 28 = 256 − 37 = 219
= −36 + 18 + 7 + 150 = 139
r r r 10. (d) P ( x ) = ax 2 + b with a, b of same sign.
6. (4) | a | = | b | = | c | = 1
P ( P ( x )) = a ( x 2 + b ) 2 + b
r r r r r r r
a × b + b × c = pa + qb + rc If x ∈ R or ix ∈ R
r r r r r r r r r r 1 ⇒ x2 ∈ R
a ⋅ (b × c ) = p + q (a ⋅ b ) + r (a ⋅ c ) and [a b c ] =
2 ⇒ P ( x) ∈ R
q r r r r ⇒ P( P( x)) ≠ 0
p + + = [a b c ] . . .(i)
2 2
Hence real or purely imaginary number cannot satisfy
p r
+q+ =0 . . .(ii) P ( P ( x)) = 0.
2 2
p q r r r 2 + 3i sin θ 1 + 2i sin θ
+ + r = [a b c ] . . .(iii) 11. (d) ×
2 2 1 − 2i sin θ 1 + 2i sin θ
⇒ p = r = −q
2 − 6sin 2 θ = 0 (For purely imaginary)
⇒ p + 2q + r
2 2 2
1
=4 sin 2 θ =
q2 3
 y2 y2  1 1
7. (112) ∴ f  2 x 2 + , 2 x 2 −  = xy sin θ = , θ = sin −1
 8 8  3 3
2 2
 y2   y2  ω 2 ω3 ω4 K ω n+2 
=  2x2 +  −  2x −
2

 8   8   3 
ω ω4 ω5 K ω n+3 
12. (b,c,d) P = 
∴ f (60, 48) + f (80, 48) + f (13, 5) M M M M M 
 n+ 2 n+3 2n + 4

= (60)2 − (48)2 + (80)2 − (48)2 + (13)2 − (5)2 = 36 + 64 + 12 ω ω K K ω 

= 112 ω 4 + ω 6 K ω5 + ω7 + ω9 K K 
 5 
ω + ω7 + ω9 K K K K
P2 =  
dy dy / dt a (cos t − t (− sin t ) − cos t ) M M M K 
8. (2880) = = = tan t  n+ 4 n+6

dx dx / dt a(− sin t + t cos t + sin t ) ω + ω K K K ω 2n + 4 +ω 2n+6
K

d2y d d dt 1 sec3 t P 2 = Null matrix if n is a multiple of 3


⇒ 2
= (tan t ) = (tan t ) ⋅ = sec 2 t =
dx dx dt dx at cos t at 13. (b) Given m is A. M. between l and n
d2y 8 24 ⇒ 2m = l + n
= =
dx t =π / 3 π a / 3 π a
2
Given l, G1 , G2 , G3 , n in G.P.
1/ 4
d2y n n
∴ 120 π a = 120 × 24 = 2880 r =  ⇒ r4 =
dx 2 t =π / 3 l l
208 Mathematics
∴ G1 = l, G2 = lr , G3 = lr 2 3
17. (b,d)(a)  X 1c 
1 1 1
× ×
2 4 4 1
P 1 1 3 1 1 1 1 1 3 1 1 1 =
So, G + 2G + G = l r [1 + 2r + r ]
1
4 4
2
4
3
4 4 4 8
 X  × × + × × + × × + × × 8
2 4 4 4 4 2 2 4 4 2 4 4
 n  n n 
2
8 1 1 1
= l4 ⋅   1 + 2   +    − × ×
 l    l   l   (b) P  Exactly two enginges arefunctioning  = 32 4 4 2 = 7
  8
 X  8
2
 n ( n + l) 2 32
= nl3 1 +  = nl3 = nl(2m)2 = 4lm2 n
 l l2 1 1 3 1 1 1 1
 X   × + × + × 
4 2 4 2 4 2 4 5
k ( k +1) ( n −1)
(c) P   = =
 
4n
14. (a,d) Sn = ∑ (−1) k 2 ∑ ((4r + 4) 2 + (4r + 3) 2 − (4r + 2) 2 − (4r + 1) 2 ) X
 2 1 1 1 1 3 1 1 1 3 8
 × + × + × + × 
2

k =1 r =0 4 2 4 2 4 2 4 2 4
( n −1)
= ∑ (2(8r + 6) + 2(8r + 4)) 11 1 1 3 1 3
 × + × + × 
r =0  X  2 4 4 4 4 4 4 7
( n −1) (d) P   = =
X
 1 1  3 3 1 1 1 3 1 3  16
= ∑ (32r + 20)
r =0
 × + × + × + × 
2 4 4 4 4 4 4 4 4
= 16(n − 1)n + 20n 18. (c) P
1056 for n = 8
= 4n (4 n + 1) = 
1332 for n = 9
5/2 7/2
15. (a,b,c) log 2 3 = ( x − 1) log 2 4 = 2( x − 1)
x

R Q
B1 A3 2sin P − 2sin P cos P

π /6 2sin P + 2sin P cos P
B2 A2
O
1 − cos P 3
B3 A1 ⇒ =
1 + cos P 32
x = −1/ 2 x = 1/ 2 29
cos P =
⇒ x log 2 3 = 2 x − 2 35
2 1 5 7 8 6
⇒ x= ∆= × × ×
2 − log 2 3 2 2 2 35
2
Rearranging, we get  3 
−2 2 log 3 2 ⇒ ∆= 6 ⇒  
x= =  4∆ 
2−
1 2 log 3 2 − 1
log 3 2 2i(x + iy) 2i(x + iy)
19. (A) z = =
1 1 − (x + iy) 2
1 − (x 2 − y 2 + 2ixy)
log 3 4 log 4 3 1
Rearranging again, x = = = . Using 1 − x 2 = y 2
log 3 4 − 1 1
−1 1 − log 4 3
log 4 3 2ix − 2y 1
Z= =− .
2y − 2ixy
2
y
16. (b) Case-1 Any 5 – digit number > 6000 is all 5-digits
1 1
number Q −1 ≤ y ≤ 1 ⇒ − ≤ −1 or − ≥ 1.
y y
Total number > 6000 using 5-digits = 5! = 120
(B) For domain
Case-2: Using 4-digits
8.3x − 2
−1 ≤ ≤1
1 − 32( x −1)
3x − 3x − 2
Can be 6.7 or 8 4 ways 3 ways 2 ways ⇒ −1 ≤ ≤ 1.
i.e., 3 ways Total number = 3 × 4 × 3 × 2 = 72 1 − 32 x − 2

Total ways = 120 + 72 = 192 3 x − 3x − 2


Case (i): −1 ≤ 0
1 − 32x − 2
Mock Test-5 209
x −2
(3 − 1)(3 − 1)
x JEE Advance Paper-II
⇒ ≥0
(32x − 2 − 1) 1. 2. 3. 4. 5. 6. 7. 8. 9. 10.

⇒ x ∈ (−∞, 0] ∪ (1, ∞). 1 36 64 5522 5 1352 4 16 a a


11. 12. 13. 14. 15. 16. 17. 18. 19. 20.
3x − 3x − 2
Case (ii): +1 ≥ 0 c a c d a b b c a c
1 − 32x − 2
(3x − 2 − 1)(3x + 1) 1. (1) Let f ( x ) = log10 x
⇒ ≥0
(3x .3x − 2 − 1) ⇒
1
f ′( x) = ⋅ log10 e
x
⇒ x ∈ ( −∞, 1) ∪ [2, ∞ ).
Let x = 4, δ x = 0.04
So, x ∈ (−∞, 0] ∪ [2, ∞ ).
∴ f ( x + δ x) = f ( x) + δ x ⋅ f ′( x)
(C) R 1 → R1 + R 3 1
⇒ log10 ( x + δ x ) = log10 x + (0.04) × log10 e
0 0 2 x
f (θ ) = − tan θ 1 tan θ ⇒ 1
log10 4.04 = log10 4 + (0.04) × × log10 e
−1 − tan θ 1 x
= 0.6021 + (0.01)(0.4343)
= 2(tan 2 θ + 1) = 2 sec 2 θ .
= 0.6021 + 0.004343 = 0.606443 ≈ 1
3 15
(D) f ′(x) = (x)1/ 2 (3x − 10) + (x)3 / 2 × 3 = (x)1/ 2 (x − 2)
2 2 2. (36) Let I = ∫ ( tan x + cot x ) dx
Increasing, when x ≥ 2. (sin x + cos x )
=∫ dx
20. (A) f '( x) > 0, ∀x ∈ (0, π / 2) (sin x cos x )

f (0) < 0 and f (π / 2) > 0 Put sin x − cos x = t


⇒ 1 − sin 2 x = t 2
So, one solution.
(B) Let (a, b, c) is direction ratio of the intersected line, then ∴ (cos x + sin x) dx = dt
ak + 4b + c = 0 Then, I = dt  t 
∫ 1− t 
= 2 sin −1 t = 2 tan −1 
2
 1− t
2
+c

4a + kb + 2c = 0  
 2 
a b c
= = 2
8 − k 4 − 2 k k − 16  sin x − cos x  −1  tan x − 1 
= 2 tan −1   + c = 2 tan  +c
We must have  sin 2 x   2 tan x 
2(8 − k ) + 2(4 − 2 k ) + ( k 2 − 16) = 0 We, get a = 2, b = 2 Then,
⇒ k = 2, 4. ⇒ a 4 + b 5 = 4 + 32 = 36
(C) Let f ( x) =| x + 2 | + | x + 1| + | x − 1| + | x − 2 |
d 3 y / dx 3
⇒ k can take value 2, 3, 4, 5. 3. (64) We have, =8
d 2 y / dx 2
−4x 4x d2y
⇒ ln = 8x + c
dx 2
4 − 2x 2x − 4
6 Or ln y2 = 8 x + c

−2 −1 1 2
Putting x = 0 we have,
c = log y2 (0) = ln1 = 0
dy ∴ ln y2 = 8 x
(D) ∫ y + 1 = ∫ dx
⇒ y2 = e8 x i.e.,
⇒ f ( x ) = 2e x − 1
e8 x
⇒ f (ln 2) = 3 y1 = +D
8
210 Mathematics
1
Again putting x = 0, then y1 (0) = + D  r4 
8 Then QR = 2 ⋅ QM = 2 {(OQ)2 − (OM ) 2 } = 2  r 2 − 
 100 
1
⇒ 0= +D P(6, 8)
8
1
∴ D=−
8 Q
M
e8 x 1 e8 x x
⇒ y1 = − ⇒ y= − +E r R
8 8 64 8 r
O
1 1 (0, 0)
Putting x = 0, we have y (0) = −0+ E =
64 8
1 1 7 1
∴ E= − = Area of ∆QPR = ⋅ QR ⋅ PM
8 64 64 2
e8 x x 7 (e8 x − 8 x + 7)  r 4  100 − r
2
Hence, y = − + = ⇒ λ = 64 ∴
1
∆ (say) = ⋅ 2  r 2 −
64 8 64 64 ⋅
2  100  10
4. (5522) Since m1 and m2 are the roots of the equation r 2 (100 − r 2 )3
∴ ∆2 = = z (say)
x 2 + ( 3 + 2) x + ( 3 − 1) = 0 1000
Then m1 + m2 = − ( 3 + 2), m1 m2 = ( 3 − 1) dz 1
∴ = {r 2 ⋅ 3(100 − r 2 ) 2 ⋅ (−2r ) + (100 − r 2 )3 ⋅ 2r}
dr 1000
∴ m1 − m2 = (m1 + m2 )2 − 4m1m2 = (3 + 4 + 4 3 − 4 3 + 4) = 11
2r (100 − r 2 )2
and coordinates of the vertices of the given triangle are = {100 − r 2 − 3r 2 }
1000
(0, 0) (c / m1 , c ) and (c / m2 , c)
dz
Hence, the required area of triangle For maximum or minimum = 0, then we get
dr

=
1 c c
×c − ×c r = 5, ( r ≠ 10 as P is outside the circle)
2 m1 m2
d 2z
And = −ve
1  1 1  1 2 m2 − m1 1 11 dr 2 r =5
= c2  − = c = c2
2  m1 m2  2 m1m2 2 ( 3 − 1) ∴ Area of triangle is also maximum at r = 5.
1 11( 3 + 1)  33 + 11  2 r r r r
= c2 =   c 6. (1352) Let PV of P, A, B and C are p , a , b and c
2 ( 3 − 1)( 3 + 1)  4  r
respectively and O (0) be the circumcircle of the
33 11
On comparing, a = ,b= equilateral triangle ABC.
4 4
r r r
a+b +c
or a =
11
,b=
33 Then, =0 . . .(i)
4 4 3
r r r r 26
33 11 44 11 and | p | = | a | = | b | = | c |= unit . . .(ii)
∴ a 2 + b2 = + = =
16 16 16 4 3
uuur r r r r
11 Now, | PA |2 = | a − p |2 = a 2 + p 2 − 2a ⋅ p
⇒ 2008( a 2 + b 2 ) = 2008 × = 502 ×11 = 5522
4 uuur r r
Similarly | PB |2 = b 2 + p 2 − 2b ⋅ p and
5. (5) Equation of chord of contact (QR ) is 6 x + 8 y − r 2 = 0 uuur r r
| PC |2 = c 2 + p 2 − 2c ⋅ p
6 ⋅ 6 + 8⋅8 − r2 100 − r 2 uuur r r r r
PM = = and ∴ Σ | PA |2 = a 2 + b 2 + c 2 + 3 p 2 − 2 p ( a + b + c )
(62 + 82 ) 10
= 6 p 2 − 0 [From equation (i) and (ii)]
0 + 0 − r2 r2
OM = = (26) 2
=6 = 2(26) 2 = 1352
(6 + 8 )
2 2 10 3
Mock Test-5 211
7. (4) Any point on the first line in symmetrical form is 11. (c) From graph, p = −1
(3r − 4, 5r − 6, − 2r + 1). If the lines are coplanar, this
point must lie on both the planes which determine the
second line.
–x+1 x–1
⇒ 3(3r − 4) − 2(5r − 6) − 2r + 1 + 5 = 0 . . .(i)
and 2(3r − 4) + 3(5r − 6) + 4(−2r + 1) − k = 0 . . .(ii)
0 1
From equation (i), we get r = 2
⇒ lim g ( x) = −1 ⇒ lim g (1 + h ) = −1
x →1+
Now substituting r = 2 equation (ii), then k = 4 h→0

 hn 
8. (16) Let the equation of circle ⇒ lim   = −1
h →0 log cos m h
 
x 2 + y 2 + 2 gx + 2 fy + k = 0 . . .(i)
 n ⋅ h n−1  n  h n−1 
and the equation of the rectangular hyperbola is ⇒ lim 
h →0 m ⋅ ( − tan h)
 = −   lim   = −1, which holds if
   m  h→0  tan h 
xy = c 2 . . .(ii)
n = m = 2.
c
Put x = ct and y = in equation (i) 12. (a) g ( x + 1) = log( f ( x + 1) = log x + log( f ( x)) = log x + g ( x)
t
2 ⇒ g ( x + 1) − g ( x) = log x
Then c 2 t 2 + c2 + 2 gct + 2 fc + k = 0
t t 1
⇒ g ′′( x + 1) − g ′′( x) = −
⇒ c 2t 4 + 2 gct 3 + kt 2 + 2 fct + c 2 = 0 x2
This equation being fourth degree in t. Let the roots be  1 1
g ′′ 1 +  − g ′′   = −4
t1 , t2 , t3 , t4 then t1t2t3t4 = 1  2 2
∴ 16 t1t2t3t4 = 16 ×1 = 16  1  1 4
g ′′  2 +  − g ′′ 1 +  = − ..........
 2x   2  2 9
−1
 x + 1 − x2
−1
  1  1 4
9. (a) tan ( y ) = tan   g ′′  N +  − g ′′  N −  = −
 1 − x.2 x2   2  2 (2 N − 1)2
 1− x 
Summing up all terms
 3x − x3  3x − x3  1 1  1 1 
⇒ tan −1 ( y) = tan −1  2  ⇒ y = Hence, g ′′  N +  − g ′′   = −4 1 + + ... + .
2 
 1 − 3x  1 − 3x 2  2 2  9 (2 N − 1) 
h 1
10. (a) = e x (e 2 x − 1) ( Z 2 − 1)
AQ 3 13. (c) J − I = ∫ dx = ∫ 4 dz
e + e +1
4x 2x
z + z2 +1
P
 1
1 − 2  dz 1  e x + e − x − 1 
where z = e x = ∫ 
z 
= ln  x − x +c
 1
2
2  e + e +1 
 z +  −1
h  z
1  e2 x − e x + 1 
30° 45° 60° ∴ J − I = ln  2 x x  + c.
A B C Q 2  e + e +1
π π
⇒ AQ = 3h x x x
14. (d) I = ∫ 1 + 4sin 2 − 4sin I = ∫ 1 − 2sin dx
2 2 2
Similarly, BQ = h 0 0

π
⇒ h 3
 x 
π
π
CQ = I = ∫  1 − 2sin dx + ∫ −  1 − 2sin dx
3  2   2
0 π

AB AQ − BQ ( 3 − 1)h
3
3
∴ = = = π
BC BQ − CQ  h  1  x3  x
π

h −  1 =  x + 4 cos  +  − x − 4 cos 
 3  2 0  2 π
3
212 Mathematics

π  3   π  3 18. (c) Plane is given by −( x + 1) − 7( y + 2) + 5( z + 1) = 0


1= + 4  − 1 −  π −  − 4  0 − 
3  2   3  2  ⇒ x + 7 y − 5z + 10 = 0
π 2π π 1 + 7 − 5 + 10 13
I= +2 3 −4− +2 3 = 4 3−4− ⇒ distance = = .
3 3 3 75 75

15. (a) Rearranging the equation we get, 19. (a) Let A, B, C and D be the complex numbers,
dp (t ) 1 2, − 2, 2i
= dt . . .(i)
p (t ) − 400 2
and − 2i respectively.
Integrating (1) on both sides we get
p ( t ) = 400 + k e t / 2 , PA2 + PB 2 + PC 2 + PD 2

QA2 + QB 2 + QC 2 + QD 2
where k is a constant of integration.
Using p (0) = 100, we get k = −300 | z1 − 2 |2 + | z1 − 2 |2 + | z1 − 2i |2 + | z1 − 2i |2 | z1 |2 +2 3
= = =
| z2 + 2 |2 + | z2 − 2 |2 + | z2 − 2i |2 + | z2 − 2i |2 | z1 | +2 4
2

∴ the relation is P (t ) = 400 − 300 e1/ 2


20. (c) AG = 2
16. (b) Inclination of line 3x + y = 1 is 150°
1
Inclination of line L = 150° ± 60° = 210°, 90° ∴ AT1 = T1G = [as A is the foucs,
2
Slope of line
T1 is the vertex and BD is the directrix of parabola].
1
L = tan 210° = tan 30° = Also T2T3 is latus return
3
Equation of = Line L
M D C
1
⇒ y+2= ( x − 3) T1 G
3 T2
A B
⇒ 3y − x + 3 + 2 3 = 0

i j k
17. (b) 3 1 2 = −iˆ − 7 ˆj + 5kˆ 1
∴ T2T3 = 4 ×
1 2 3 2
1 1 4
−iˆ − 7 ˆj + 5kˆ ∴ Area of ∆ T1 T2T3 = × × =1
Hence unit vector will be . 2 2 2
5 3

You might also like